Você está na página 1de 84

TOPNOTCH MEDICAL BOARD PREP OBSTETRICS-GYNECOLOGY SUPEREXAM

For inquiries visit www.topnotchboardprep.com.ph or email us at topnotchmedicalboardprep@gmail.com


DEAR TOPNOTCH FRIENDS:

PLEASE FOLLOW THESE INSTRUCTIONS:

1. These questions are previous diagnostic, midterm, and finals exams of Topnotch, almost all of them made by Topnotch Board Exam Topnotchers.
2. Answer this Topnotch Superexam seriously 100-items at a time. Cover the “Explanations” Column. Do not immediately look at the answers from the
answer key. That’s not the correct way of answering sample exams. You need to treat these MCQs as exercises and not as handouts.
3. Time yourself. 1.5 hours per 100-item block.
4. After answering each 100-item block, refer to the Topnotch Answer Key for the correct answers. Please be careful of “frameshift mutations” when
checking your answers – check every 10 items. (the format of the answer key was designed for you to practice against “frameshift mutations”)
5. The Topnotch Superexams are EXERCISES for the actual med boards. They will not appear verbatim in your future exams. More than knowing what’s
the correct answer, it’s more important for you to:
a. Know why the other choices are wrong
b. Know why the other choices were included in the first place
c. Know the explanation to the correct answer
6. Sharpen your mind by answering the Topnotch Superexams. Most of these questions based on past feedback are more difficult than the actual questions
in the med boards. In these exams made by Board Exam Topnotchers, if you’re getting a score of 60/100 , that’s already a good score. More than 80/100
is outstanding.

Ite QUESTION EXPLANATION AUTHOR TOPNOTCH
m # EXAM
1 A 29 year old woman underwent vaginal delivery. The other structures are located too deep to be KRISTEL TANHUI DIAGNOSTIC
A midline episiotomy most likely damages which of injured in a midline episiotomy. (TOP 3 - AUG EXAM - MARCH
the following? 2015 MED 2016
A. Superficial transverse perineal muscle The muscles attached to the perineal body are the BOARDS;
B. Levator ani muscle superficial and deep transverse perineal muscle, TOPNOTCH MD
C. Puborectalis muscle bulbospongiosus and external anal sphincter. FROM LA SALLE)
D. Pubococcygeus muscle
E. All of the above Source: Moore Clinically oriented anatomy 4th ed
Chapter 3 (Ebook)
2 A 25 year old primipara gives birth to a neonate This case describes fetal hydantoin syndrome. It is KRISTEL TANHUI DIAGNOSTIC
with IUGR, microcephaly, craniofacial due to ingestion of the antiseizure drug phenytoin. (TOP 3 - AUG EXAM - MARCH
dysmorphism, hypopastic nails and distal 2015 MED 2016
phalanges. Which of the following teratogen may Ethanol – Fetal alcohol syndrome BOARDS;
be responsible for this? Lithium – Ebstein anomaly TOPNOTCH MD
A. Ethanol Phenytoin – Fetal hydantoin syndrome FROM LA SALLE)
B. Lithium Thalidomide – phocomelia
C. Phenytoin Methimazole – aplasia cutis congenita
D. Thalidomide
E. Methimazole Source: Topnotch handout on pharmacology
3 A 30 year old female seeks consult for a lump on This is a case of Bartholin’s gland abscess. The KRISTEL TANHUI DIAGNOSTIC
her vulva with 2 days duration of pain. There was bartholin’s glands are located in the 4 and 8 o’clock (TOP 3 - AUG EXAM - MARCH
no associated fever, chills, nausea or vomiting. On position of the vaginal orifice. They are normally 2015 MED 2016
PE, a 3x3cm fluctuant tender mass is palpated on nonpalpable. Consider taking a biopsy in an older BOARDS;
the 8 o’clock position of her vulva. How should she patient to rule out adenocarcinoma. TOPNOTCH MD
be managed? FROM LA SALLE)
A. Incision and drainage Source: Topnotch handout on Obstetrics and
B. Marsupialization Gynecology
C. Consider broad spectrum antibiotics
D. A and B
E. All of the above

4 A 26 year old female complains of amenorrhea for Naegele’s rule KRISTEL TANHUI DIAGNOSTIC
2 months. Urine pregnancy test was positive. If her First day of LMP minus 3 months, plus 7 days, plus (TOP 3 - AUG EXAM - MARCH
LMP was July 20, 2015, when is her EDC? 1 year 2015 MED 2016
A. March 27, 2016 BOARDS;
B. April 13, 2016 Source: Topnotch handout on Obstetrics and TOPNOTCH MD
C. April 27, 2016 Gynecology FROM LA SALLE)
D. May 13, 2016
E. June 13, 2016
5 A 25 yr old female presents with vaginal spotting This is a case of ectopic pregnancy. The absence of a KRISTEL TANHUI DIAGNOSTIC
and RLQ pain. A TVS is done which shows empty gestational sac inside the uterus with TVS when the (TOP 3 - AUG EXAM - MARCH
uterus. Serum HCG was 4000mIU/ml. Which of the HCG level is >1500mIU/mL is 100% accurate in 2015 MED 2016
following is not a risk factor for her condition? excluding a lie uterine pregnancy. An investigation BOARDS;
A. PID must be carried out to rule out ectopic pregnancy. TOPNOTCH MD
B. Previous similar diagnosis FROM LA SALLE)
C. IUD use Source: Williams Obstetrics 23RD ed p 244
D. Congenital malformation of the uterus
E. None of the above
6 Which of the following is not true regarding HCG? Alpha subunit is similar with alpha subunit of LH, KRISTEL TANHUI DIAGNOSTIC
A. Peaks at 10 wks gestation FSH and TSH (TOP 3 - AUG EXAM - MARCH
B. Nadirs at 14-16 wks Beta subunits are unique: this is the one tested for 2015 MED 2016
C. Increases by more than half every 48 hours prior in urine pregnancy tests BOARDS;
to 10 weeks. TOPNOTCH MD
D. Urine tests are based on antibody specificity to Source: Williams Obstetrics 23RD ed p63 FROM LA SALLE)
the alpha subunit of HCG.
E. Also elevated in H mole, choriocarcinoma, germ
cell tumors, large cell carcinoma of the lung
7 At 16 weeks AOG, a 38 year old G2P1 (1001) Quad screen interpretation KRISTEL TANHUI DIAGNOSTIC
undergoes a quad screen which revealed increased (TOP 3 - AUG EXAM - MARCH
AFP, normal estriol, HCG and inhibin A. What is the Trisomy 21 – Increased HCG + inhibin, decreased 2015 MED 2016
most likely concern for the fetus? afp + estriol BOARDS;
A. Nothing. Everything is normal (My mnemonic is down syndrome = H and I are HI) TOPNOTCH MD
B. Trisomy 21 Trisomy 18 – all are low FROM LA SALLE)
C. Trisomy 18
D. Ectopic pregnancy Causes of increased AFP – NTDs, abdominal wall
E. Neural tube defects defects, underestimation of gestational age, fetal
death, placental abnormalities, multiple gestation
TOPNOTCH MEDICAL BOARD PREP OBSTETRICS-GYNECOLOGY SUPEREXAM Page 1 of 84
For inquiries visit www.topnotchboardprep.com.ph or email us at topnotchmedicalboardprep@gmail.com
TOPNOTCH MEDICAL BOARD PREP OBSTETRICS-GYNECOLOGY SUPEREXAM
For inquiries visit www.topnotchboardprep.com.ph or email us at topnotchmedicalboardprep@gmail.com
Ite QUESTION EXPLANATION AUTHOR TOPNOTCH
m # EXAM

Source: Williams Obstetrics 23RD ed p290

8 A 32 year old G2P1 (1001) seeks consult at 6 400 micrograms/day ideally started 3 months prior KRISTEL TANHUI DIAGNOSTIC
weeks AOG for initial obstetric visit. Her previous to conception can help prevent NTDs. For those (TOP 3 - AUG EXAM - MARCH
child had a spina bifida. How much folate should be with previous children with NTDs, a higher dose of 2015 MED 2016
given to prevent NTD’s in her current pregnancy? 4 milligrams/day should be given. BOARDS;
A. There is insufficient evidence that folate can TOPNOTCH MD
prevent NTDs Source: Williams Obstetrics 23RD ed 205 FROM LA SALLE)
B. 40 micrograms/day
C. 400 micrograms/day
D. 4 milligrams/day
E. 40 milligrams/day
9 Engagement is the entrance of a particular Engagement occurs when the biparietal diameter KRISTEL TANHUI DIAGNOSTIC
diameter of the fetal head in occiput position into (the widest transverse diameter of the fetal head in (TOP 3 - AUG EXAM - MARCH
the smallest diameter of the maternal pelvis. This occiput position) reaches the level of the ischial 2015 MED 2016
diameter of the fetal head is the? spines. BOARDS;
A. Suboccipitofrontal diameter TOPNOTCH MD
B. Suboccipitobregmatic diameter Source: Williams Obstetrics 23RD ed p378 FROM LA SALLE)
C. Submentobregmatic diameter
D. Occipitofrontal diameter
E. Biparietal diameter
10 A 32 year old G3P2 2002 with history of 2 This is not arrest in descent. Arrest in descent KRISTEL TANHUI DIAGNOSTIC
uncomplicated NSD is admitted for labor at 39 occurs in the second stage of labor. Patient is still in (TOP 3 - AUG EXAM - MARCH
weeks AOG . 2 hours prior, her IE was 5cm/60% the 1st stage of labor. 2015 MED 2016
effaced/-2 station. Her IE now is 6cm/60% BOARDS;
effaced/-2 station. What abnormal labor pattern is Since the patient is a multipara, she should dilate TOPNOTCH MD
she exhibiting? 1.5cm/hr and should at least already be 8cm. The FROM LA SALLE)
A. She is not exhibiting any abnormal labor pattern next step in management is to determine if there
B. Prolonged latent phase are adequate contractions and to rule out a
C. Protracted active phase dilation fetopelvic disproportion. (Note: not cephalopelvic
D. Arrest in descent disproportion. We already know her pelvis is
E. Arrest in dilation adequate because she delivered 2 VSDs already.)

Source: Williams Obstetrics 23RD ed p465
11 Which of the following may be given postpartum Ergonovine and methylergonovine are 5HT2 KRISTEL TANHUI DIAGNOSTIC
after expulsion of placenta to prevent postpartum receptor antagonist that are uteroselective and (TOP 3 - AUG EXAM - MARCH
hemorrhage? used for postpartum hemorrhage. 2015 MED 2016
A. Ergonovine BOARDS;
B. Ergotamine Ergotamine, methysergide are also 5HT2 receptor TOPNOTCH MD
C. Methysergide antagonist that are vasoselective which are used for FROM LA SALLE)
D. A and B migraine treatment.
E. All of the above
Topnotch handout in pharmacology (SIMILAR TO
PREVIOUS BOARD EXAM CONCEPT/PRINCIPLE
12 In which part of the uterus is the incision made for Source: Williams Obstetrics 23RD ed p550 KRISTEL TANHUI DIAGNOSTIC
a low transverse cesarean section? (TOP 3 - AUG EXAM - MARCH
A. Fallopian tube 2015 MED 2016
B. Fundus BOARDS;
C. Body TOPNOTCH MD
D. Cervix FROM LA SALLE)
E. Lower uterine segment
13 A 22 year G3P3(3003), 3 weeks postpartum seeks Combined OCPs may reduce the amount of breast KRISTEL TANHUI DIAGNOSTIC
advice regarding contraception. Which postpartum milk. (TOP 3 - AUG EXAM - MARCH
contraception is ideal for the patient if she is IUDs are not used while the uterus is undergoing 2015 MED 2016
breastfeeding? involution due to risk of expulsion and uterine BOARDS;
A. Combined OCP perforation. TOPNOTCH MD
B. Progestin only pills Withdrawal is not effective. FROM LA SALLE)
C. IUD
D. Withdrawal method Source: Williams Obstetrics 23RD ed 652
E. All of the above
14 A neonate is born to a 25 year old G1P1 1001 who Give both HBIG and vaccine. The infant may be KRISTEL TANHUI DIAGNOSTIC
has active chronic hepatitis B infection. How breastfed if given prophylaxis and vaccinated. (TOP 3 - AUG EXAM - MARCH
should the neonate be managed? 2015 MED 2016
A. Give anti-HBs IgG only BOARDS;
B. Give Hepatitis B vaccine only TOPNOTCH MD
C. Give Both HBIG and vaccine at birth FROM LA SALLE)
D. Give HBIG then start vaccine after 4 weeks
E. Give Acyclovir + Lamivudine
15 Patient presents at 7 weeks AOG with vaginal Management for this patient is expectant or D and KRISTEL TANHUI DIAGNOSTIC
bleeding and denies passage of meaty material. On C. (TOP 3 - AUG EXAM - MARCH
PE, the cervical OS is open at 5cm. US shows 2015 MED 2016
cardiac activity. What is the diagnosis? Source: Williams Obstetrics 23RD ed Chapter 9 BOARDS;
A. Threatened abortion Abortion TOPNOTCH MD
B. Inevitable abortion FROM LA SALLE)
C. Incomplete abortion
D. Complete abortion
E. Missed abortion

TOPNOTCH MEDICAL BOARD PREP OBSTETRICS-GYNECOLOGY SUPEREXAM Page 2 of 84


For inquiries visit www.topnotchboardprep.com.ph or email us at topnotchmedicalboardprep@gmail.com
TOPNOTCH MEDICAL BOARD PREP OBSTETRICS-GYNECOLOGY SUPEREXAM
For inquiries visit www.topnotchboardprep.com.ph or email us at topnotchmedicalboardprep@gmail.com
Ite QUESTION EXPLANATION AUTHOR TOPNOTCH
m # EXAM
16 A 60 year old postmenopausal female comes in Nulliparity is a risk factor for endometrial CA. KRISTEL TANHUI DIAGNOSTIC
with 1 month history of spotting not associated Multiparity is protective. (TOP 3 - AUG EXAM - MARCH
with pain. An ultrasound showed thickened 2015 MED 2016
endometrium. And biopsy confirmed malignancy. Source: Topnotch handout on Obstetrics and BOARDS;
Which of the following is not a risk factor? Gynecology TOPNOTCH MD
A. PCOS FROM LA SALLE)
B. Multiparity
C. Early menarche
D. Obesity
E. Liver disease
17 What is the least invasive definitive treatment for Hysterectomy is the definitive treatment for KRISTEL TANHUI DIAGNOSTIC
adenomyosis? adenomyosis. (TOP 3 - AUG EXAM - MARCH
A. GnRH antagonist 2015 MED 2016
B. NSAIDs Source: Topnotch handout on Obstetrics and BOARDS;
C. OCPs Gynecology TOPNOTCH MD
D. Endometrial ablation FROM LA SALLE)
E. Hysterectomy
18 A 60 year old postmenopausal female presents A granulose cell tumor may release estrogen KRISTEL TANHUI DIAGNOSTIC
with postmenopausal spotting. A TVS showed resulting to endometrial hyperplasia and (TOP 3 - AUG EXAM - MARCH
thickened endometrium and a right ovarian mass. sometimes even endometrial carcinoma. 2015 MED 2016
There was also elevated inhibin levels. Biopsy BOARDS;
revealed Schiller duval bodies. Which of the Source: Topnotch handout on Obstetrics and TOPNOTCH MD
following is the most likely diagnosis? Gynecology FROM LA SALLE)
A. Theca lutein cyst
B. Dysgerminoma
C. Yolk sac tumor
D. Granulosa cell tumor
E. Teratoma
19 Which of the following is not commonly associated Meig’s syndrome is commonly associated with KRISTEL TANHUI DIAGNOSTIC
with Meig’s syndrome? fibromas (benign masses), right sided pleural (TOP 3 - AUG EXAM - MARCH
A. Right sided pleural effusion effusion, and ascites. 2015 MED 2016
B. Malignant ovarian mass BOARDS;
C. Ascites Source: Topnotch handout on Obstetrics and TOPNOTCH MD
D. A and B Gynecology FROM LA SALLE)
E. None of the above

20 A 19 year old nulliparous sexually active female This is a case of trichomoniasis which is caused by a KRISTEL TANHUI DIAGNOSTIC
seeks consult for malodorous, greenish, frothy flagellated motile protozoan seen on saline wet (TOP 3 - AUG EXAM - MARCH
vaginal discharge. On obtaining specimen for TMG mount microscopy. It is associated with strawberry 2015 MED 2016
smearing, you noted petchiae all over the cervix. cervix. BOARDS;
What is the treatment for this patient? TOPNOTCH MD
A. No treatment necessary Source: Topnotch handout on Obstetrics and FROM LA SALLE)
B. Clotrimazole 2% cream 5g intravaginally for 7- Gynecology
14days
C. Metronidazole 500mg BID x 7 days
D. Metronidazole 2g single dose
E. Oral acyclovir 800mg QID x 7 days

21 Appropriate antibiotic treatment for chancroid is SIMILAR TO PREVIOUS BOARD EXAM LESTER BRYAN MIDTERM 1
A. Penicillin CONCEPT/PRINCIPLE. There are four antibiotics CO (TOP 10 - AUG EXAM - MARCH
B. TMP-SMX that can be used to treat chancroid: azithromycin, 2015 MED 2016
C. Azithromycin ceftriaxone, ciprofloxacin, and erythromycin. BOARDS;
D. Doxycycline Chancroid is a contagious sexually transmitted TOPNOTCH MD
E. Co-amoxiclav ulcerative disease of the vulva caused by FROM UST)
Haemophilus ducreyi, small gram-negative rods
that exhibit parallel alignment on Gram’s staining
(“school of fish”). After a short incubation period,
the patient usually develops multiple painful soft
ulcers on the vulva, mainly on the labia majora and,
less commonly, on the labia minora or involving the
perineal area. The chancroid ulcer has ragged,
irregular borders and a base that bleeds easily and
is covered with grayish exudates. Approximately
half the patients will develop painful inguinal
lymphadenitis within 2 weeks of an untreated
infection, which may undergo liquefaction and
presents as buboes. These may rupture and
discharge pus. Diagnosis is made by Gram’s stain
and, less commonly, by culture.
22 A 26-year-old woman underwent a Pap smear, SIMILAR TO PREVIOUS BOARD EXAM LESTER BRYAN MIDTERM 1
which revealed a low-grade squamous CONCEPT/PRINCIPLE. The correct answer is CO (TOP 10 - AUG EXAM - MARCH
intraepithelial lesion. There was no inflammation colposcopy with biopsy. Cervical cancer mortality is 2015 MED 2016
found. She was also found to be HIV negative, and a so preventable because it is easily revealed by Pap BOARDS;
Pap smear 2 years ago was negative. The most smear, but this is only a screening test. If the Pap TOPNOTCH MD
appropriate next step in her treatment is: smear returns with low- or high-grade squamous FROM UST)
A. Colposcopy and biopsy intraepithelial lesion, a punch biopsy is required for
B. Cervical conization diagnosis of CIN or invasive carcinoma. If the
C. Cryotherapy or laser therapy biopsy shows CIN I (slight dysplasia), this may
D. Return in 4–6 months for repeat Pap smear resolve and does not require treatment. CIN I is
E. Hysterectomy managed with follow-up and a repeat Pap smear in
4–6 months. CIN II and III (moderate and severe
dysplasia) are treated with ablative therapy using
either laser or cryotherapy. If the punch biopsy
returns showing invasive carcinoma, staging is
performed, and the patient is treated with
hysterectomy and/or radiation therapy. If the
TOPNOTCH MEDICAL BOARD PREP OBSTETRICS-GYNECOLOGY SUPEREXAM Page 3 of 84
For inquiries visit www.topnotchboardprep.com.ph or email us at topnotchmedicalboardprep@gmail.com
TOPNOTCH MEDICAL BOARD PREP OBSTETRICS-GYNECOLOGY SUPEREXAM
For inquiries visit www.topnotchboardprep.com.ph or email us at topnotchmedicalboardprep@gmail.com
Ite QUESTION EXPLANATION AUTHOR TOPNOTCH
m # EXAM
biopsy is inconclusive, then cervical conization is
indicated.

23 Which of the following physiologic changes occurs SIMILAR TO PREVIOUS BOARD EXAM LESTER BRYAN MIDTERM 1
during pregnancy? CONCEPT/PRINCIPLE. Gastric motility and lower CO (TOP 10 - AUG EXAM - MARCH
A. Increased systemic vascular resistance extremity venous return are decreased. Systemic 2015 MED 2016
B. Decreased minute ventilation vascular resistance, minute ventilation, and BOARDS;
C. Decreased gastric motility fibrinogen are all increased. TOPNOTCH MD
D. Decreased fibrinogen FROM UST)
E. Increased venous return from lower extremities
24 A 32-year-old G1 at 39 weeks gestation is admitted The pelvic inlet is considered contracted if the LESTER BRYAN MIDTERM 1
in labor at 4 cm dilated and completely effaced; the anteroposterior diameter is less than 10 cm. The CO (TOP 10 - AUG EXAM - MARCH
fetal head is at 0 station. You perform clinical inlet is digitally measured by the diagonal conjugate 2015 MED 2016
pelvimetry and find the following: the diagonal which is typically 1.5 cm greater than the inlet, BOARDS;
conjugate is 10 cm, the interischial spine distance therefore a pelvic inlet contraction is defined as a TOPNOTCH MD
is 11 cm with nonconvergent sidewalls, and the diagonal conjugate less than 11.5 cm. The midpelvis FROM UST)
intertuberous distance is 9 cm. Those extends from the inferior margin of the symphysis
measurements describe which of the following to the ischial spines bilaterally to the sacrum near
types of pelvis? the junction of the fourth and fifth vertebrae. The
A. Normal pelvis average mid-pelvis measurements include:
B. Contracted pelvic inlet interischial spinous 10.5 cm, anteroposterior from
C. Contracted midpelvis symphysis to sacrum 11.5 cm and posterior sagittal
D. Contracted pelvic outlet from midpoint of interspinous line to sacrum 5 cm.
E. Generally contracted pelvis But there is no precise manual measurement of the
midpelvis but contraction is suggested if the spines
are prominent, the pelvic sidewalls converge or if
the sacrosciatic notch is narrow. The contracted
outlet is defined as a intertuberous diameter of 8
cm or less. Outlet contraction without concomitant
midpelvis contraction is rare. A generally
contracted pelvis is caused by combinations of
contractions in the inlet, midpelvis , and outlet.
25 On routine examination, it is discovered that a 35- Diethylstilbestrol = DES LESTER BRYAN MIDTERM 1
year-old woman had been exposed in utero to CO (TOP 10 - AUG EXAM - MARCH
diethylstilbestrol administered to her mother, who 2015 MED 2016
had had a history of recurrent spontaneous BOARDS;
abortion. This history suggests that the patient TOPNOTCH MD
might be at increased risk of FROM UST)
(A) adenomyosis
(B) clear cell adenocarcinoma
(C) lichen sclerosus
(D) sarcoma botryoides
(E) squamous cell carcinoma
26 Which step in the steroid hormone synthetic Aromatase catalyzes the conversion of testosterone LESTER BRYAN MIDTERM 1
pathway is required for the development of female to estradiol in the ovarian granulosa cells. Estradiol CO (TOP 10 - AUG EXAM - MARCH
secondary sex characteristics, but not male is required for the development of female 2015 MED 2016
secondary sex characteristics? secondary sex characteristics. BOARDS;
A. Aldosterone synthase TOPNOTCH MD
B. Aromatase FROM UST)
C. Cholesterol desmolase
D. 17,20-Lyase
E. 5α-Reductase
27 A 35-year-old woman is seen 6 months after giving The history is strongly suggestive of LESTER BRYAN MIDTERM 1
birth to a normal infant. She suffered severe panhypopituitarism due to ischemic necrosis of the CO (TOP 10 - AUG EXAM - MARCH
cervical lacerations during delivery, resulting in pituitary, occurring as a sequela to childbirth 2015 MED 2016
hemorrhagic shock. Following blood transfusion complicated by hemorrhagic shock (Sheehan BOARDS;
and surgical repair, postpartum recovery has so far syndrome). This syndrome is clinically dominated TOPNOTCH MD
been uneventful. She now complains of continued by overt evidence of gonadotropin and FROM UST)
amenorrhea and loss of weight and muscle corticotropin deficiencies, along with laboratory
strength. Further investigation might be expected evidence of these deficiencies and thyrotropin
to demonstrate which of the following findings? deficiency. Overt secondary hypothyroidism
A. Decreased serum cortisol sometimes occurs.
B. Hyperestrinism
C. Hyperglycemia
D. Increased hair growth in a male distribution
pattern
E. Increased serum free thyroxine
28 Maneuver in breech delivery for the head: SIMILAR TO PREVIOUS BOARD EXAM LESTER BRYAN MIDTERM 1
A. Loveset CONCEPT/PRINCIPLE. Other maneuvers for breech CO (TOP 10 - AUG EXAM - MARCH
B. Mauriceau-Smellie-Viet delivery of the head: Bracht and Prague. Loveset: 2015 MED 2016
C. Zavanelli delivery of nuchal arm. Pinard: delivery of the foot. BOARDS;
D. McRobert's Zavanelli: reversal of the cardinal movements of TOPNOTCH MD
E. Pinard labor to perform CS, used as last resort FROM UST)
29 A 20-year-old G1 at 41 weeks has been pushing for A first-degree tear involves the vaginal mucosa or LESTER BRYAN MIDTERM 1
3 hours. The fetal head is at the introitus and perineal skin, but not the underlying tissue. In a CO (TOP 10 - AUG EXAM - MARCH
beginning to crown. It is necessary to cut an second-degree episiotomy, the underlying 2015 MED 2016
episiotomy. The tear extends through the sphincter subcutaneous tissue is also involved, but not the BOARDS;
of the rectum, but the rectal mucosa is intact. How rectal sphincter or rectal mucosa. In a third-degree TOPNOTCH MD
should you classify this type of laceration? tear, the rectal sphincter is affected. A fourth- FROM UST)
A. First-degree degree episiotomy involves a tear that extends into
B. Second-degree the rectal mucosa.
C. Third-degree
TOPNOTCH MEDICAL BOARD PREP OBSTETRICS-GYNECOLOGY SUPEREXAM Page 4 of 84
For inquiries visit www.topnotchboardprep.com.ph or email us at topnotchmedicalboardprep@gmail.com
TOPNOTCH MEDICAL BOARD PREP OBSTETRICS-GYNECOLOGY SUPEREXAM
For inquiries visit www.topnotchboardprep.com.ph or email us at topnotchmedicalboardprep@gmail.com
Ite QUESTION EXPLANATION AUTHOR TOPNOTCH
m # EXAM
D. Fourth-degree
E. Fifth-degree

30 A 23-year-old G1 at 38 weeks gestation presents in In the event of a face presentation, successful LESTER BRYAN MIDTERM 1
active labor at 6 cm cervical dilatation with vaginal delivery will occur; the majority of the time CO (TOP 10 - AUG EXAM - MARCH
ruptured membranes . On cervical examination the with an adequate pelvis. Spontaneous internal 2015 MED 2016
fetal nose, eyes, and lips can be palpated. The fetal rotation during labor is required to bring the chin BOARDS;
heart rate tracing is 140 beats per minute with to the anterior position, which allows the neck to TOPNOTCH MD
accelerations and no decelerations. The patient’s pass beneath the pubis . Therefore, the patient is FROM UST)
pelvis is adequate. Which of the following is the allowed to labor spontaneously; a CS is employed
most appropriate management for this patient? for failure to progress or for fetal distress. Manual
A. Perform immediate CS. conversion to vertex, forceps rotation, and internal
B. Allow spontaneous labor with vaginal delivery. version are no longer employed in obstetrics to
C. Perform forceps rotation in the second stage of deliver face presentation because of undue trauma
labor to convert mentum posterior to mentum to both the mother and the fetus .
anterior and to allow vaginal delivery.
D. Allow patient to labor spontaneously until
complete cervical dilatation is achieved and then
perform an internal podalic version with breech
extraction.
E. Attempt manual conversion of the face to vertex
in the second stage of labor.
31 Fetal death at 15 weeks gestation without A threatened abortion takes place when uterine LESTER BRYAN MIDTERM 1
expulsion of any fetal or maternal tissue for at least bleeding occurs without any cervical dilatation or CO (TOP 10 - AUG EXAM - MARCH
8 weeks is compatible with: effacement. In a patient bleeding during the first 2015 MED 2016
A. Complete abortion half of pregnancy, the diagnosis of inevitable BOARDS;
B. Incomplete abortion abortion is strengthened if the bleeding is profuse TOPNOTCH MD
C. Threatened abortion and associated with uterine cramping pains. If FROM UST)
D. Missed abortion cervical dilatation has occurred, with or without
E. Inevitable abortion rupture of membranes, the abortion is inevitable. If
only a portion of the products of conception has
been expelled and the cervix remains dilated, a
diagnosis of incomplete abortion is made. However,
if all fetal and placental tissue has been expelled,
the cervix is closed, bleeding from the canal is
minimal or decreasing, and uterine cramps have
ceased, a diagnosis of complete abortion can be
made. The diagnosis
of missed abortion is suspected when the uterus
fails to continue to enlarge with or without uterine
bleeding or spotting. A missed abortion is one in
which fetal death occurs before 20 weeks gestation
without expulsion of any fetal or maternal tissue for
a t least 8 weeks thereafter.
32 215. A 25-year-old G1 at 37 weeks presents with In frank breech presentation, the lower extremities LESTER BRYAN MIDTERM 1
gross rupture of membranes. On cervical are flexed at the hips and extended at the knees so CO (TOP 10 - AUG EXAM - MARCH
examination, she is noted to be 4 cm dilated, 90% that the feet lie in close proximity to the head and 2015 MED 2016
effaced with the presenting part at −3 station. The the fetal buttocks is the presenting part. With a BOARDS;
presenting part is soft and felt to be the fetal complete breech presentation, one or both knees TOPNOTCH MD
buttock. A quick bedside ultrasound reveals a are flexed. In incomplete breech presentation, FROM UST)
breech presentation with both hips flexed and single footling, one hip is not flexed and one foot or
knees extended. What type of breech presentation knee is lowermost in the birth canal.
is described?
A. Frank
B. Incomplete
C. Complete
D. Single footling
E. Double footling

33 A 26-year-old primigravida develops gestational The most common effect of maternal diabetes LESTER BRYAN MIDTERM 1
diabetes and remains hyperglycemic during the mellitus and hyperglycemia on the child is CO (TOP 10 - AUG EXAM - MARCH
remainder of her pregnancy. increased birth weight. This also increases the 2015 MED 2016
Which of the following abnormalities in the likelihood of obstetric complications, including the BOARDS;
newborn child is likely related to the maternal need for cesarean section and increased likelihood TOPNOTCH MD
hyperglycemia? of brachial plexus injuries. Another complication is FROM UST)
A. Ambiguous genitalia hyaline membrane disease. Cretinism results from
B. Cretinism deficiency of thyroid hormone during fetal
C. Increased birth weight development and during postnatal life. Ambiguous
D. Sheehan syndrome genitalia can occur in any of the adrenogenital
E. Thyroglossal duct cyst syndromes. Sheehan syndrome occurs in the
mother and has no relationship to diabetes.
Thyroglossal duct cysts do not usually result in
endocrine complications.

TOPNOTCH MEDICAL BOARD PREP OBSTETRICS-GYNECOLOGY SUPEREXAM Page 5 of 84


For inquiries visit www.topnotchboardprep.com.ph or email us at topnotchmedicalboardprep@gmail.com
TOPNOTCH MEDICAL BOARD PREP OBSTETRICS-GYNECOLOGY SUPEREXAM
For inquiries visit www.topnotchboardprep.com.ph or email us at topnotchmedicalboardprep@gmail.com
Ite QUESTION EXPLANATION AUTHOR TOPNOTCH
m # EXAM
34 A 68-year-old postmenopausal woman presents Endometrial cancer is the most common LESTER BRYAN MIDTERM 1
for evaluation of the recent onset of vaginal gynecologic malignancy, and Type I cancers CO (TOP 10 - AUG EXAM - MARCH
bleeding, and a diagnosis of type I endometrial (endometrioid) account for the majority (80%) of 2015 MED 2016
carcinoma is made on endometrial biopsy. Which these cases. A major predisposing factor to Type I BOARDS;
of the following is a risk factor for this condition? endometrial carcinoma is prolonged and TOPNOTCH MD
(A) Endometriosis unopposed exposure to estrogen. In the case of FROM UST)
(B) Multiparity obesity, adipose tissue converts androgens into
(C) Salpingitis estrogens, fueling the proliferation of endometrial
(D) Early sexual activity with multiple partners tissue. Similarly, diabetes, nulliparity (not
(E) Obesity multiparity), and hypertension are also
predisposing factors. Early sexual activity with
multiple partners is associated with cervical cancer.
Salpingitis can predispose to ectopic pregnancy.
Endometriosis is a benign condition and has no
relation to endometrial cancer.
35 Soon after an uncomplicated delivery at 38 weeks’ SIMILAR TO PREVIOUS BOARD EXAM LESTER BRYAN MIDTERM 1
gestation, a mother develops respiratory CONCEPT/PRINCIPLE. The diagnosis is amniotic CO (TOP 10 - AUG EXAM - MARCH
difficulties and uncontrolled vaginal bleeding fluid embolism, which is characterized by sudden 2015 MED 2016
progressing to shock, multiorgan failure, and peripartal respiratory difficulty, progressing to BOARDS;
death. At autopsy, masses of debris and epithelial shock and often to death. Sheehan syndrome is TOPNOTCH MD
squamous cells are apparent in the pulmonary associated with obstetric blood loss, with resultant FROM UST)
microcirculation. Which of the following was the pituitary ischemia leading to postpartal
likely cause of death? hypopituitarism. Chorioamnionitis is infection of
A. Sheehan syndrome the placenta, which can have devastating
B. Chorioamnionitis consequences for both the mother and the child.
C. Abruptio placentae Abruptio placentae is premature separation of the
D. Amniotic fluid aspiration syndrome placenta and can lead to antepartal bleeding and
E. Amniotic fluid embolism fetal death. Amniotic fluid aspiration can occur in
the child and can result in difficulties for the infant,
particularly if meconium is aspirated. Amniotic
fluid embolism and abruptio placentae are well-
known causes of DIC, as are retained dead fetus and
toxemia.
36 A 56-year-old diabetic woman has recently been C. albicans is a major cause of vulvovaginitis. Many LESTER BRYAN MIDTERM 1
treated with a 2-week course of antibiotics for a women are carriers of the yeast, and it therefore CO (TOP 10 - AUG EXAM - MARCH
skin infection. She returns to the clinic for follow- constitutes normal vaginal flora, although there are 2015 MED 2016
up with a new complaint of a “cottage cheese-like” many conditions that alter the vaginal BOARDS;
vaginal discharge with significant vaginal itching. microenvironment, including pregnancy, oral TOPNOTCH MD
The most likely cause of these symptoms is contraceptives, and systemic antibiotics, making FROM UST)
A. C. granulomatis overgrowth possible. C. granulomatis causes
B. C. albicans granuloma inguinale.
C. G. vaginalis
D. T. vaginalis
E. N. gonorrhea
37 A 23-year-old G1 at 40 weeks gestation with the This patient is most likely experiencing false labor, LESTER BRYAN MIDTERM 1
complaint of contractions. She states they are or Braxton-Hicks contractions. False labor is CO (TOP 10 - AUG EXAM - MARCH
occurring every 4 to 8 minutes and each lasts characterized by contractions that are irregular in 2015 MED 2016
approximately 1 minute. She reports good fetal timing and duration and do not result in any BOARDS;
movement and denies any leakage of fluid or cervical dilation. The intensity of false labor does TOPNOTCH MD
vaginal bleeding. The nurse places an external not change and the discomfort is mainly felt in the FROM UST)
tocometer and reports that the patient is having lower abdomen and is usually relieved by sedation.
contractions every 2 to 10 minutes. You note
contractions are mild on palpation. On
examination, the cervix is 2 cm dilated, 50%
effaced, and the vertex is at –1 station. The patient
had the same cervical examination in your office
last week. The patient is in what stage of labor?
A. Active labor
B. Latent labor
C. False labor
D. Stage 1 of labor
E. Stage 2 of labor
38 A 23-year-old woman consults an obstetrician Hydatidiform mole should be suspected when the LESTER BRYAN MIDTERM 1
because of the onset of vaginal bleeding in what uterus is enlarged beyond the expected size for the CO (TOP 10 - AUG EXAM - MARCH
she considers to be the fifth month of pregnancy; time of the pregnancy. HCG is markedly elevated in 2015 MED 2016
however, examination reveals the uterus to be this gestational trophoblastic disease. Preeclampsia BOARDS;
enlarged to the size of a 7-month pregnancy. and eclampsia are forms of toxemia of pregnancy TOPNOTCH MD
Intravaginal ultrasound fails to detect a fetal marked by severe hypertension, albuminuria, and FROM UST)
heartbeat and instead shows a “snowstorm edema, with the addition of convulsions and DIC in
pattern.” HCG is markedly elevated. These findings the latter. Placenta accreta occurs when the
are strongly suggestive of placenta adheres directly to the myometrium,
A. preeclampsia leading to severe bleeding at the time of delivery.
B. eclampsia Ectopic pregnancy is usually discovered early in a
C. placenta accreta suspected pregnancy when ultrasound examination
D. ectopic pregnancy reveals the uterus to be empty.
E. hydatidiform mole
39 A primipara is in the second stage of labor and an Midline episiotomies are easier to fix and have a LESTER BRYAN MIDTERM 1
episiotomy is about to be done. Compared with a smaller incidence of surgical breakdown, less pain, CO (TOP 10 - AUG EXAM - MARCH
midline episiotomy, which of the following is an and lower blood loss. The incidence of dyspareunia 2015 MED 2016
advantage of mediolateral episiotomy? is somewhat less. Howwever, the incidence of BOARDS;
A. Ease of repair extensions of the incision to include the rectum is TOPNOTCH MD
B. Fewer breakdowns considerably higher than with mediolateral FROM UST)
C. Less blood loss episiotomies. Regardless of technique, attention to
D. Less dyspareunia hemostasis and anatomic restoration is the key
E. Less extension of the incision element of a technically appropriate repair.

TOPNOTCH MEDICAL BOARD PREP OBSTETRICS-GYNECOLOGY SUPEREXAM Page 6 of 84


For inquiries visit www.topnotchboardprep.com.ph or email us at topnotchmedicalboardprep@gmail.com
TOPNOTCH MEDICAL BOARD PREP OBSTETRICS-GYNECOLOGY SUPEREXAM
For inquiries visit www.topnotchboardprep.com.ph or email us at topnotchmedicalboardprep@gmail.com
Ite QUESTION EXPLANATION AUTHOR TOPNOTCH
m # EXAM
40 You have just delivered an infant weighing 2.5 kg According to some studies, 25% of twins are LESTER BRYAN MIDTERM 1
(5.5 lb) at 38 weeks gestation. Because the uterus diagnosed at the time of delivery. Although CO (TOP 10 - AUG EXAM - MARCH
still feels large, you do a vaginal examination. A sonography or radiography can diagnose multiple 2015 MED 2016
second set of membranes is bulging through a fully gestations early in pregnancy, these methods are BOARDS;
dilated cervix, and you feel a small part presenting not used routinely in all medical centers. The TOPNOTCH MD
in the sac. A fetal heart is auscultated at 60 beats second twin is probably the only remaining FROM UST)
per minute. situation where internal version is permissible.
A. External version Although some obstetricians might perform a CS for
B. Internal version a second twin presenting as a footling or shoulder,
C. Midforceps rotation fetal bradycardia dictates that immediate delivery
D. Low transverse CS be done, and internal podalic version is the quickest
E. Classical CS procedure.
41 Maximum teratogenecity occurs during GEORGE MIDTERM 2
A. First two weeks after conception MICHAEL EXAM - MARCH
B. 3-8 weeks after conception SOSUAN (TOP 5 - 2016
C. >8 weeks after conception AUG 2015 MED
D. Both A and B BOARDS;
E. AOTA TOPNOTCH MD
FROM UST)
42 The smallest circumference of the fetal head BTD (8.0 cm); BPD (9.5 cm); OM (12.5 cm); OF GEORGE MIDTERM 2
A. Suboccipitobregmatic (11.5 cm); SOB (9.5 cm) --> diameter; OF (34.5 cm); MICHAEL EXAM - MARCH
B. Occipitofrontal SOB (32 cm) --> circumference SOSUAN (TOP 5 - 2016
C. Occipitomental AUG 2015 MED
D. Bitemporal BOARDS;
E. Biparietal TOPNOTCH MD
FROM UST)
43 What is most common cause of perinatal mortality Prematurity is the most common cause of perinatal GEORGE MIDTERM 2
in twin gestation mortality in twin gestation. MICHAEL EXAM - MARCH
A. Single fetal demise SOSUAN (TOP 5 - 2016
B. Twin to twin transfusion syndrome AUG 2015 MED
C. Prematurity BOARDS;
D. IUGR TOPNOTCH MD
E. Two fetal demise FROM UST)
44 True of the ischial spine GEORGE MIDTERM 2
A. Plane of the least pelvic dimension MICHAEL EXAM - MARCH
B. Pudendal block is carried out at this level SOSUAN (TOP 5 - 2016
C. Internal rotation occurs when the occiput is at AUG 2015 MED
this level BOARDS;
D. External OS of the cervix is located normally TOPNOTCH MD
E. AOTA FROM UST)
45 Pelvibc diaphragm consists of the ff, except The pelvic disphragm is composed of the levator ani GEORGE MIDTERM 2
A. puboperinealis and coccygeus muscle. The levator ani is composed MICHAEL EXAM - MARCH
B. ileococcygeus of the pubococcygeus/puboperinealis SOSUAN (TOP 5 - 2016
C. coccygeus (pubovaginalis, puboperinealis and puboanalis), AUG 2015 MED
D. puborectalis puborectalis and ileoccygeus muscle. BOARDS;
E. NOTA TOPNOTCH MD
FROM UST)
46 Engagement occurs when the BPD of the fetal head GEORGE MIDTERM 2
descends below the level of MICHAEL EXAM - MARCH
A. Midpelvis SOSUAN (TOP 5 - 2016
B. Pelvic inlet AUG 2015 MED
C. Pelvic floor BOARDS;
D. Ischial tuberosity TOPNOTCH MD
E. NOTA FROM UST)
47 A 35 y/o G2P1 at 10 weeks' gestational age The pain and bleeding subsided after passage of GEORGE MIDTERM 2
experience severe crampy hypogastric pai and meaty tissue. On PE, the cervix was closed. These MICHAEL EXAM - MARCH
vaginal bleeding. One day PTC, she noted passage findings support the diagnosis of complete SOSUAN (TOP 5 - 2016
of meaty tissue, thereafter pain and bleeding abortion. AUG 2015 MED
subsided. On IE, cervix was smooth and closed. BOARDS;
What is the most likely diagnosis? TOPNOTCH MD
A. Imminent abortion FROM UST)
B. Inevitable abortion
C. Threatened abortion
D. Incomplete aborition
E. Complete abortion
48 On what day of the endometrial cycle is subnuclear D17- subnuclear vacuoles; D18 - vacuoles move to GEORGE MIDTERM 2
vacuoles seen? the apical portion; D19 - cells begin to secrete MICHAEL EXAM - MARCH
A. D17 glycoprotein and MPS contents in the lumen SOSUAN (TOP 5 - 2016
B. D18 AUG 2015 MED
C. D19 BOARDS;
D. Never seen TOPNOTCH MD
E. D17-19 FROM UST)
49 The average length of the funis The average length of the funis/umbilical cord is 55 GEORGE MIDTERM 2
A. 40 cm. MICHAEL EXAM - MARCH
B. 48 SOSUAN (TOP 5 - 2016
C. 55 AUG 2015 MED
D. 63 BOARDS;
E. NOTA TOPNOTCH MD
FROM UST)
50 The source of HCG is the GEORGE MIDTERM 2
A. Syncytiotrophoblast MICHAEL EXAM - MARCH
B. Cytotrophoblast SOSUAN (TOP 5 - 2016
C. Decidua AUG 2015 MED
D. Fetus BOARDS;
E. Nitabuch's layer TOPNOTCH MD
FROM UST)
TOPNOTCH MEDICAL BOARD PREP OBSTETRICS-GYNECOLOGY SUPEREXAM Page 7 of 84
For inquiries visit www.topnotchboardprep.com.ph or email us at topnotchmedicalboardprep@gmail.com
TOPNOTCH MEDICAL BOARD PREP OBSTETRICS-GYNECOLOGY SUPEREXAM
For inquiries visit www.topnotchboardprep.com.ph or email us at topnotchmedicalboardprep@gmail.com
Ite QUESTION EXPLANATION AUTHOR TOPNOTCH
m # EXAM
51 What is the treatment of choice for a symptomatic Marsupialization is preferred due to the high GEORGE MIDTERM 2
Bartholin's duct cyst ot abscess? recurrence rate of incision and drainage. MICHAEL EXAM - MARCH
A. Incision and drainage SOSUAN (TOP 5 - 2016
B. Enucleation AUG 2015 MED
C. Marsupialization BOARDS;
D. Excision TOPNOTCH MD
E. Observation FROM UST)
52 In a patient with a thin, gray-white frothy GEORGE MIDTERM 2
discharge, what is the most likely diagnosis? MICHAEL EXAM - MARCH
A. Trichominiasis SOSUAN (TOP 5 - 2016
B. Atrophic vaginitis AUG 2015 MED
C. Bacterial vaginosis BOARDS;
D. Candidiasis TOPNOTCH MD
E. Cervicitis FROM UST)
53 What is the least number of mitosis per hpf with GEORGE MIDTERM 2
cytologic atypia that leads to a diagnosis of MICHAEL EXAM - MARCH
leiomyosarcoma? SOSUAN (TOP 5 - 2016
A. 1 AUG 2015 MED
B. 3 BOARDS;
C. 5 TOPNOTCH MD
D. 7 FROM UST)
E. 9
54 In radical hysterectomy, the main blood supply of The uterine atery (inferior vesical artery in males) GEORGE MIDTERM 2
the uterus is ligated close to its origin. The uterine is anterior division branch of internal iliac artery. MICHAEL EXAM - MARCH
artery is a branch of the following structure? SOSUAN (TOP 5 - 2016
A. Anterior division of internal iliac artery AUG 2015 MED
B. Posterior division of internal iliac artery BOARDS;
C. External iliac artery TOPNOTCH MD
D. Common iliac artery FROM UST)
E. Posterior iliac artery
55 The following is a risk factor for endometrial Other Risk factors include nulliparity, obesisty, GEORGE MIDTERM 2
cancer hypertension MICHAEL EXAM - MARCH
A. Multiparity SOSUAN (TOP 5 - 2016
B. HSV infection AUG 2015 MED
C. Diabetes BOARDS;
D. OCP use TOPNOTCH MD
E. NOTA FROM UST)
56 Which of the following should be investigated in Mullerian agenesis is associated with urologic GEORGE MIDTERM 2
patients with Mullerian agenesis? anomalies such unlateral renal agenesis. MICHAEL EXAM - MARCH
A. Cardiac SOSUAN (TOP 5 - 2016
B. Urologic AUG 2015 MED
C. Skeletal BOARDS;
D. CNS TOPNOTCH MD
E. GIT FROM UST)
57 What is the treatment for a 25 y/o with primary GEORGE MIDTERM 2
amenorrhea due to 45, XO? MICHAEL EXAM - MARCH
A. HRT SOSUAN (TOP 5 - 2016
B. Estrogen AUG 2015 MED
C. Progestins BOARDS;
D. OCPs TOPNOTCH MD
E. AOTA FROM UST)
58 Which factor primarily dictates the age of Genetics primarily dictates the age of menopause. GEORGE MIDTERM 2
menopause? MICHAEL EXAM - MARCH
A. Genetics SOSUAN (TOP 5 - 2016
B. Race AUG 2015 MED
C. Number of pregnancies BOARDS;
D. Age of menarche TOPNOTCH MD
E. OCP use FROM UST)
59 A 70 y/o multipara consulted because of chronic The description stated is characteristic of Paget's GEORGE MIDTERM 2
vulvar pruritus. On pelvic exam, there is a diffuse Disease of the vulva. MICHAEL EXAM - MARCH
erythematous eczematoid lesion on the right labia, SOSUAN (TOP 5 - 2016
about 3 x 4 cm? What is the probable clinical AUG 2015 MED
diagnosis? BOARDS;
A. Paget's Disease TOPNOTCH MD
B. Melanoma FROM UST)
C. HSV infection
D. Leukoplakia
E. Lichen sclerosus
60 To rule out microinvasion, the mininum procedure The minimum procedure should be cone biopsy to GEORGE MIDTERM 2
to establish diagnosis should be document microinvasion of the basement MICHAEL EXAM - MARCH
A. Hysterectomy membrane. SOSUAN (TOP 5 - 2016
B. Cone biopsy AUG 2015 MED
C. Paps smear BOARDS;
D. Colposcopy with biopsy TOPNOTCH MD
E. Incision biopsy FROM UST)
61 The ff scenario pertains to questions 181-183. A 28 Naegele's rule: Month +9; Day +7; Year +1 JAN CHRISTIAN MIDTERM 3
yr old nullipara came to your birthing clinic for the FELICIANO (TOP EXAM - MARCH
first time for a regular checkup? Her last menstrual 2 - AUG 2015 2016
period was on March 27, 2015. The mother was MED BOARDS;
asking when she expects to deliver. What is the TOPNOTCH MD
expected date of delivery based on Naegele's rule? FROM UST)
A. December 31, 2015
B. January 1, 2016
C. January 2, 2016
D. January 3, 2016
E. February 4, 2016
TOPNOTCH MEDICAL BOARD PREP OBSTETRICS-GYNECOLOGY SUPEREXAM Page 8 of 84
For inquiries visit www.topnotchboardprep.com.ph or email us at topnotchmedicalboardprep@gmail.com
TOPNOTCH MEDICAL BOARD PREP OBSTETRICS-GYNECOLOGY SUPEREXAM
For inquiries visit www.topnotchboardprep.com.ph or email us at topnotchmedicalboardprep@gmail.com
Ite QUESTION EXPLANATION AUTHOR TOPNOTCH
m # EXAM
62 The mother was asking for an exact date of The conception age/fertilization age is usally JAN CHRISTIAN MIDTERM 3
fertlization/conception. Although you cannot approximated at 2 weeks after the menstrual age. FELICIANO (TOP EXAM - MARCH
identify the exact date you can predict that it most 2 - AUG 2015 2016
likely occurred during what date? MED BOARDS;
A. March 13, 2015 TOPNOTCH MD
B. March 20, 2015 FROM UST)
C. April 3, 2015
D. April 10, 2015
E. Same date as the LMP
63 A CBC was performed as well, you will classify the Normal hgb levels depends on the AOG of the px. JAN CHRISTIAN MIDTERM 3
pregnant patient as being anemic if the Hgb level is 1st trimester- 11g; 2nd trimster- 10.5 g (because of FELICIANO (TOP EXAM - MARCH
below this value? hemodilution of the RBC's); 3rd tri- 11g 2 - AUG 2015 2016
A. 10.0 g/dL MED BOARDS;
B. 10.5 g/dL TOPNOTCH MD
C. 11.0 g/dL FROM UST)
D. 11.5 g/dL
E. 12.0 g/dL
64 In doing a clinical pelvimetry, which of the A clinically adequate pelvis: Inlet- Diagonal JAN CHRISTIAN MIDTERM 3
following findings will indicate a clincially conjugate of atleast 11cm, Engagement of the head, FELICIANO (TOP EXAM - MARCH
adequate pelvis? Positive Mueller Hillis manuever; Midpelvis- 2 - AUG 2015 2016
A. Diagonal conjugate is 10 cm Sidewall are non convergent, Sacrum is curved, MED BOARDS;
B. Side walls are convergent Bispinious diameter is atleast 10 cm; Outlet- Pubic TOPNOTCH MD
C. Bispinous diameter is 8 cm arch is wide (>110 deg), Bituberous diameter is FROM UST)
D. Sacrum is curved atleast 8cm
E. Subpubic arch is 90 degress
65 A mother came to your clinic for a regular prenatal In terms of gravidity and parity, mutifetal JAN CHRISTIAN MIDTERM 3
checkup currently pregnant with twins. Her first pregnancy are counted as onebut are counted FELICIANO (TOP EXAM - MARCH
pregnancy was an ecotpic pregnancy that was sepaerately in the TPAL score. H-mole and ectopic 2 - AUG 2015 2016
terminated at 8 weeks, her second pregnancy was pregnancies are counted as gravidities and included MED BOARDS;
an H-mole that was terminated by suction as abortions. Expect question regarding OB scoring TOPNOTCH MD
curretage at 10 weeks. The third resulted in the but not as difficult and colorful as this scenario. FROM UST)
live birth of triplet boys at 38 weeks although one
died at birth and the other triplet is dying with
terminal leukemia. The fourth resulted in the live
birth of a daughter at 34 weeks? What is the full OB
score?
A. G5P4 (3123)
B. G5P6 (3122)
C G5P4 (3113)
D. G5P6 (3123)
E. G5P3 (3113)
66 This type of maternal pelvis is narrow Gynecoid- round; Android- Triangular/heart JAN CHRISTIAN MIDTERM 3
anteroposteriorly but wide transversely? shaped; Anthropoid- wide A-P and narrow FELICIANO (TOP EXAM - MARCH
A. Gynecoid transversely 2 - AUG 2015 2016
B. Anthropoid MED BOARDS;
C. Android TOPNOTCH MD
D. Crystalloid FROM UST)
E. Platypelloid
67 At 20 weeks age of gestation, all of the ff Lung surfactant production begins at the terminal JAN CHRISTIAN MIDTERM 3
developments in the fetus has already occurred stage of the lung development at 24-25 weeks AOG FELICIANO (TOP EXAM - MARCH
EXCEPT? and beyond. 2 - AUG 2015 2016
A. Start of production of lung surfactants MED BOARDS;
B. Downy lanugo hair surrounds the skin TOPNOTCH MD
C. Gender can be determined bu UTZ FROM UST)
D. Physiologic herniation of the gut has occured
E. Urine poroduction has began
68 While performing a non stress test, you told the SIMILAR TO PREVIOUS BOARD EXAM JAN CHRISTIAN MIDTERM 3
mother the test is reactive and reassuring. The CONCEPT/PRINCIPLE FELICIANO (TOP EXAM - MARCH
mother asked what this means. What is the 2 - AUG 2015 2016
defintion of a reactive NST? MED BOARDS;
A. 3 or more FHR acelerations that peak atleast TOPNOTCH MD
10bpm for 10 secs in a 20 min period FROM UST)
B. 3 or more FHR acelerations that peak atleast
10bpm for 10 secs in a 20 min period
C. 2 or more FHR acelerations that peak atleast
15bpm for 15 secs in a 20 min period
D. 2 or more FHR acelerations that peak atleast
10bpm for 10 secs in a 10 min period
E. 2 or more FHR acelerations that peak atleast
15bpm for 15 secs in a 10 min period
69 This scenario pertains to question 189-192. A 25 All of the choices except threatened miscarriage can JAN CHRISTIAN MIDTERM 3
year old G6P4 patient with an LMP of February 2, explain bleeding in the second half of pregnancy. FELICIANO (TOP EXAM - MARCH
2015. Her prior pregancies were unremarkable 2 - AUG 2015 2016
and she delivered via NSD. She is a non smoker but MED BOARDS;
a heavy alcoholic drinker. Her BP is 140/90 and HR TOPNOTCH MD
is 78. She came to your clinic because of vaginal FROM UST)
bleeding. All of the ff are likely diferentials for the
patient EXCEPT?
A. Placenta previa
B. Threatened Miscarriage
C. Abruptio placentae
D. Imminent labor
E. Abruptio placentae

TOPNOTCH MEDICAL BOARD PREP OBSTETRICS-GYNECOLOGY SUPEREXAM Page 9 of 84


For inquiries visit www.topnotchboardprep.com.ph or email us at topnotchmedicalboardprep@gmail.com
TOPNOTCH MEDICAL BOARD PREP OBSTETRICS-GYNECOLOGY SUPEREXAM
For inquiries visit www.topnotchboardprep.com.ph or email us at topnotchmedicalboardprep@gmail.com
Ite QUESTION EXPLANATION AUTHOR TOPNOTCH
m # EXAM
70 On further examination,you noted a hard tender Sudden abdominal pain, vaginal bleeding and a JAN CHRISTIAN MIDTERM 3
globular abdomen with the fetus in a cephalic hard tender abdomen is indicative of placental FELICIANO (TOP EXAM - MARCH
presentation? What is the primary differential at abruption 2 - AUG 2015 2016
this time. MED BOARDS;
A. Placenta previa TOPNOTCH MD
B. Threatened Miscarriage FROM UST)
C. Abruptio placentae
D. Imminent labor
E. Abruptio placentae
71 Accdg to current studies, what is the greatest risk All of the choices are risk factors for the JAN CHRISTIAN MIDTERM 3
factor for the above condition? development of abruptio but a prior abruptio has a FELICIANO (TOP EXAM - MARCH
A. Prior abruptio relative risk of 10-50 to develop a subsequent one. 2 - AUG 2015 2016
B. Preeclampsia/Hypertension Preeclampsia has a relative risk of only 2.1-4.0 MED BOARDS;
C. Short umbilical cord TOPNOTCH MD
D. Preterm rupture of membranes FROM UST)
E. Chorioamnionitis
72 All of the ff are complications attributed to the The ff are complications of abruptio placentae: JAN CHRISTIAN MIDTERM 3
above condition, whoich of the ff is a least likely Hypovolemic shock, couvelaire uterus, Acute FELICIANO (TOP EXAM - MARCH
complication? kidney injury, Sheehan's syndrome and 2 - AUG 2015 2016
A. Couvelaire uterus Consumptive coagulopathy. Uterine inversion is MED BOARDS;
B. Uterine inversion assoc more with placenta accreta. TOPNOTCH MD
C. Acute Kidney Injury FROM UST)
D. Sheehan's syndrome
E. Consumptive coagulopathy
73 During an internal examination of a 37 weeks AOG Pls review the the table for the Bishop scoring. JAN CHRISTIAN MIDTERM 3
primigravid you noted the ff findings: Cervix 4 cm FELICIANO (TOP EXAM - MARCH
dilated; 60% effaced, medium, midposition, station 2 - AUG 2015 2016
-2. What is the Bishop's score of the patient? MED BOARDS;
A. 4 TOPNOTCH MD
B 5 FROM UST)
C. 6
D. 7
E. 8
74 A 30 year old women comes to your clinic Uterine fibroids are ususaly associated with a heavy JAN CHRISTIAN MIDTERM 3
complaining of increasing pelvic pressure, or prolonged menstrual flow and on physical exam, FELICIANO (TOP EXAM - MARCH
constipation and menorrhagia of about 10 large an irregularly enalrged uterus is palpated versus 2 - AUG 2015 2016
pads per day. On PE, you palpated a non tender, adenomyosis which has an symmetrical enlarged MED BOARDS;
irregularly enlarged uterus with a lumpy, bumpy uterus. TOPNOTCH MD
firm contour. Her cervix appears normal and she FROM UST)
has no evidence of ascites? The patient most likely
has what condition?
A. Uterine fibroids
B. Adenomyosis
C. Endometriosis
D. Endometrial Malignancy
E. Ovarian Malignancy
75 All of the medical therapies can be used to treat Opiod agonists are narcotic medications used to JAN CHRISTIAN MIDTERM 3
menorrhagia in women with the above conditon treat pain. They have no role in the treatment of FELICIANO (TOP EXAM - MARCH
EXCEPT? heavy bleeding in women with fibroids. 2 - AUG 2015 2016
A. Combined oral contraceptive pills MED BOARDS;
B. Antifibrinolytic agent TOPNOTCH MD
C. NSAIDS FROM UST)
D. Opiod agonists
E. Progestin only pills
76 A 26 year old G0 patient comes in complaining of This describes the classic location of thr Bartholin's JAN CHRISTIAN MIDTERM 3
intermittent painless mass on her vulva near the glands usually at the 4 oclock and 8 o clock position FELICIANO (TOP EXAM - MARCH
introitus. It seems to be aggravated by her near the introitus providing lubrication of the 2 - AUG 2015 2016
intercourse but usually goes away on its own. She's vagina. The ducts of Bartholin can become blocked MED BOARDS;
had two lifetime sexual partners and has been with resultign in a cyst formation. TOPNOTCH MD
her last partner for 5 years. She never had an STD. FROM UST)
You examine her and find a 3 cm non tender mass
in the 4 o clock position. What abnormality is
present?
A. Skene's gland cyst
B. Gartner's duct cyst
C. Bartholin's duct cyst
D. Cystocele
E. Epidermal inclusion cyst
77 In relation to the rpeviosu question, what In this case wherein the cyst is asymptomatic and JAN CHRISTIAN MIDTERM 3
treatment would you recommend for this patient? there is no sign of infection or abscess (being FELICIANO (TOP EXAM - MARCH
A. Expectant management nontender), expectant management is advised. 2 - AUG 2015 2016
B. Word cathetherization However for a large painful Bartholin's, a word MED BOARDS;
C. I and D catheter is palced to releive the obstruction. TOPNOTCH MD
D. Marsupialization Marsupialziation is typically reserved for patients FROM UST)
E. Excision in whom the catheter has failed
78 An 20 year old woman presents to your office A pelvic UTZ should be the first step aimed to JAN CHRISTIAN MIDTERM 3
complaining of amenorhhea. She never had a determine the presence or absence of the uterus, FELICIANO (TOP EXAM - MARCH
mesntrual period before but she complains of mild tubes and ovaries. Given the presence of normal 2 - AUG 2015 2016
cyclic abdominal bloating. She reports normal breasts and pubic hair, an FSH would not be helpful. MED BOARDS;
breast dev't at age 11. She reports she has become TOPNOTCH MD
sexually active bbut finds her intercourse painful. FROM UST)
Her past medical history is unremarkable. On PE
you noted normal appearing axillary and pubic
hair. Her breast dev't is normal. A pelvic exam
shows a normal appearing external genitalia and a
shortened vagina ending in a blind pouch. WHich
TOPNOTCH MEDICAL BOARD PREP OBSTETRICS-GYNECOLOGY SUPEREXAM Page 10 of 84
For inquiries visit www.topnotchboardprep.com.ph or email us at topnotchmedicalboardprep@gmail.com
TOPNOTCH MEDICAL BOARD PREP OBSTETRICS-GYNECOLOGY SUPEREXAM
For inquiries visit www.topnotchboardprep.com.ph or email us at topnotchmedicalboardprep@gmail.com
Ite QUESTION EXPLANATION AUTHOR TOPNOTCH
m # EXAM
of the ff tests would be your first step in diagnosing
the patient?
A. Karyotyping
B. Pelvic UTZ
C. Serum FSH
D. Hysterosalpingogram
E. DIagnostic laparoscopy
79 On further examination you noted normal Mullerian agenesis known as Mayer-Rokitansky- JAN CHRISTIAN MIDTERM 3
appearing bilateral ovaries as well as an absent Kuster-Hauser syndrome refers to congential FELICIANO (TOP EXAM - MARCH
utrerus and fallopian tubes. What is the most likely absence of the uterus, oviducts and upper vagina. 2 - AUG 2015 2016
diagnosis? MED BOARDS;
A. Imperforate hymen TOPNOTCH MD
B. Transverese vaginal septum FROM UST)
C. Mullerian agenesis
D. Androgen insensitivity syndrome
E. Asherman's syndrome
80 An 21 year old nulligravid woman presents to the The findings and symptoms are consistent with JAN CHRISTIAN MIDTERM 3
health clinic with a 4 week history of yellow Trichomonas vaginalis infection. The classic FELICIANO (TOP EXAM - MARCH
vaginal discharge. She also reports vulvar itching cervical finding is a strawberry looking appearance. 2 - AUG 2015 2016
and irritation. She is sexually active but Motile flagellated protozopans are visible on wet MED BOARDS;
monogamous.On pelvic exam, there was noted prep microscopic exam. TOPNOTCH MD
vulvar and vaginal erythema along wit ha yellow FROM UST)
frothy malodorous discharge with a pH of 6.5. THe
cervix appears to have erythematous punctations?
Wet smear was done. What would you expect to
see under microscopy?
A. Branching hyphae
B. Multinucleated giant cells
C.Scant WBC
D. Flagellated motile organisms
E. EPithelial cells covered with bacteria
81 A 28 year old female presents to the clinic with Williams Obstetrics 23rd edition p. 1246 ANDREW TIU FINAL EXAM -
vaginal pruritus with frothy discharge. Which of (TOP 1 - AUG MARCH 2016
the following is the most likely diagnosis? 2015 MED
a. bacterial vaginosis BOARDS;
b. trichomoniasis TOPNOTCH MD
c. candidiasis FROM CIM)
d. herpes simplex
e. gonorrhea
82 2. In the postpartum ward, PMG 28 year old VZIG is given to newborns 5 days before and 2 days ANDREW TIU FINAL EXAM -
G2P2002 mother developed pruritic vesicular after maternal clinical evidence of varicella. Give (TOP 1 - AUG MARCH 2016
lesions that crust over in 3 to 7 days spreading VariZIG to pregnant women exposed to varicella 2015 MED
from trunk to extremities. Which of the following is within 96 hours of exposure. Varicella vaccine is not BOARDS;
true? recommended for pregnant women and should not TOPNOTCH MD
a. give to newborn VZIG be given to women who may become pregnant FROM CIM)
b. give varicella vaccine to pregnant mothers who during the month following each vaccine dose. The
were exposed at the ward attenuated vaccine is not secreted in milk and
c. vaccine is secreted in breast milk when given postpartum vaccination should not be delayed
postpartum because of breastfeeding. Vaccine immunity
d. vaccine induced immunity is absolute diminishes over time and breakthrough infection
e. Varivax is given 6 months apart to adults with no occurs 5% in 10 years. Varivax is given 4 - 8 weeks
history of varicella apart.
Williams Obstetrics 23rd edition p. 1212
83 3. FU, 32 year old G3 P1011, comes to the clinic Isoniazid is considered safe in pregnancy and is ANDREW TIU FINAL EXAM -
and asks what is she going to do after being given for tuberculin positive patients younger than (TOP 1 - AUG MARCH 2016
exposed to an employee at her office who became 35 years old who have no evidence of active 2015 MED
AFB sputum positive? disease. INH is given 300mg daily for 1 year. BOARDS;
a. start with HRZE Williams Obstetrics 23rd edition p. 1006 TOPNOTCH MD
b. start with isoniazid FROM CIM)
c. start with HR
d. start with HRZES
e. start with HRZ
84 4. Ate Marites, 35 years old G3P1011 24 weeks GFR and renal plasma flow increases 50% thus ANDREW TIU FINAL EXAM -
AOG, diagnosed with medullary sponge kidney decreasing creatinine. There is dilatation of ureters (TOP 1 - AUG MARCH 2016
comes to the prenatal clinic with complaints of which resembles hydronephrosis on sonogram. 2015 MED
nocturia. Which of the following is true of renal Kidney size enlarges 1cm and returns to normal BOARDS;
changes in normal pregnancy? postpartum. Progesterone stimulates respiratory TOPNOTCH MD
a. creatinine increases during normal gestation due center and decreases bicarbonate threshold. FROM CIM)
to decrease in GFR from compression of uterus Williams Obstetrics 23rd edition p. 123
b. constriction of ureters which leads to nocturia
c. retains same size of kidneys
d. increased placental metabolism of AVP which
leads to diabetes insipidus
e. increased bicarbonate threshold
85 5. A 19 year old G0P0 came in for dysfunctional proliferative - straight to slightly coiled tubular ANDREW TIU FINAL EXAM -
uterine bleeding. Endometrial sampling was done glands lined by pseudostratified columnar (TOP 1 - AUG MARCH 2016
which revealed serrated, dilated glands with epithelium 2015 MED
intraluminal secretion lined by short columnar early secretory: coiled glands lined by simple BOARDS;
cells. Which of the following changes of menstrual columnar epithelium with subnuclear vacuoles TOPNOTCH MD
cycle is the patient most likely in? menstrual: fragmented endometrium with FROM CIM)
a. proliferative condensed stroma and glands
b. early secretory Williams Obstetrics 23rd edition p. 38
c. late secretory
d. menstrual
e. follicular

TOPNOTCH MEDICAL BOARD PREP OBSTETRICS-GYNECOLOGY SUPEREXAM Page 11 of 84


For inquiries visit www.topnotchboardprep.com.ph or email us at topnotchmedicalboardprep@gmail.com
TOPNOTCH MEDICAL BOARD PREP OBSTETRICS-GYNECOLOGY SUPEREXAM
For inquiries visit www.topnotchboardprep.com.ph or email us at topnotchmedicalboardprep@gmail.com
Ite QUESTION EXPLANATION AUTHOR TOPNOTCH
m # EXAM
86 6. A 26 year old female G2 P1001 8 weeks AOG Williams Obstetrics 23rd edition p. 239 ANDREW TIU FINAL EXAM -
came in for hypogastric pain and vaginal bleeding. (TOP 1 - AUG MARCH 2016
TVS showed an adnexal mass. Which of the 2015 MED
following is the most likely risk factor for this BOARDS;
patient? TOPNOTCH MD
a. previous PID FROM CIM)
b. smoking
c. prior CS
d. documented tubal problem
e. previous ectopic pregnancy
87 7. In the emergency room of VSMMC, the intern on trophoblastic proliferation is only focal as well as ANDREW TIU FINAL EXAM -
duty was able to witness a patient from Oslob villous edema. Theca lutein cysts are just observed (TOP 1 - AUG MARCH 2016
bringing a plastic bag containing the pieces which and will regress. The first step in management is 2015 MED
she evacuated in the toilet. Upon inspection of the evacuation of mole and the second is regular follow BOARDS;
contents, the intern noticed multiple vesicles with up to detect persistent trophoblastic disease. TOPNOTCH MD
fetal parts. Which of the following is not true in this Karyotype usually 69XXX or 69XXY for partial FROM CIM)
condition? moles.
a. trophoblastic proliferation is widespread Williams Obstetrics 23rd edition p. 258
b. theca lutein cysts must be removed
c. consistent follow up is imperative
d. first step in management is chemotherapy
e. karyotyping of the contents would reveal 46 XX
88 8. After 3 hours on 8cm cervical dilatation, Mrs MJP absolute contraindications to regional anesthesia: ANDREW TIU FINAL EXAM -
23 years old on 39 weeks AOG, was brought to the refractory hypotension, coagulopathy, use of LMWH (TOP 1 - AUG MARCH 2016
operating room for cesarean section. Which of the within 12hours, untreated maternal bacteremia, 2015 MED
following will make the anesthesiologist think skin infection at needle placement, and increased BOARDS;
otherwise in using the epidural route for ICP from mass lesion. TOPNOTCH MD
analgesia? Williams Obstetrics 23rd edition p. 453 FROM CIM)
a. use of heparin due to history of APAS
b. BP of 80/50 responsive to IV fluid
administration
c. fetal tachycardia
d. preeclampsia
e. meconium stained amniotic fluid
89 9. A 19 year old G1P0 mother came in for watery Williams Obstetrics 23rd edition p. 471 ANDREW TIU FINAL EXAM -
vaginal discharges. Patient is 38 weeks AOG with (TOP 1 - AUG MARCH 2016
regular prenatal check ups at local health center. 2015 MED
Upon speculum exam, pooling of fluid is noted and BOARDS;
pelvimetry done showed diagonal conjugate of TOPNOTCH MD
10cm. WHich of the following is not true in this FROM CIM)
condition?
a. face and shoulder presentations are encountered
more frequently
b. on internal exam, fetus is at station +3
c. absent pressure of the head on lower uterine
segment predisposes to less effective contractions
d. cord prolapse are encountered more frequently
e. early spontaneous rupture of membranes is
more likely
90 10. A 21 year old G2P1001 mother came in for The entire uterus or curettings with myometrium is ANDREW TIU FINAL EXAM -
regular uterine contractions every 2 minutes. necessary for histopathologic confirmation. (TOP 1 - AUG MARCH 2016
Patient had a history of placenta previa during the Placenta previa is the most important risk factor 2015 MED
previous pregnancy in which she underwent next is prior cesarean delivery. BOARDS;
cesearean section and delivered a live baby boy 34 Williams Obstetrics 23rd edition p. 777 TOPNOTCH MD
weeks. Currently, patient delivered another live FROM CIM)
baby boy 38 weeks by BS however the intern in
charge had difficulty in delivery of placenta. Which
of the following is true?
a. leave the placenta and give chemotherapy
b. histologic diagnosis can be made from the
placenta alone
c. safest treatment option is still hysterectomy
despite young age
d. multigravida is the most important risk factor
for placenta accreta
e. none of the above
91 11. Which of the following is not an etiology of Berek and Novak’s Gynecology 15th edition p.1051 ANDREW TIU FINAL EXAM -
secondary amenorrhea? (TOP 1 - AUG MARCH 2016
a. premature ovarian failure 2015 MED
b. Turner’s syndrome BOARDS;
c. asherman syndrome TOPNOTCH MD
d. stein - leventhal syndrome FROM CIM)
e. sheehan’s syndrome
92 12. A 12 year old female came in for consult due to it is difficult to differentiate a transverse septum or ANDREW TIU FINAL EXAM -
amenorrhea. Upon physical examination, pubic and complete absence of the cervix and uterus in a (TOP 1 - AUG MARCH 2016
axillary hair was absent and a blind vaginal pouch female from a blind vaginal pouch in a male 2015 MED
was seen. Which of the following tests would pseudohermaphrodite by examination alone. BOARDS;
confirm your impression? Androgen insensitivity is likely when pubic and TOPNOTCH MD
a. ultrasound axillary hair is absent. FROM CIM)
b. hysterosalpingogram Berek and Novak’s Gynecology 15th edition p.1047
c. estrogen and progesterone levels
d. karyotyping
e. gonadotropin levels

TOPNOTCH MEDICAL BOARD PREP OBSTETRICS-GYNECOLOGY SUPEREXAM Page 12 of 84


For inquiries visit www.topnotchboardprep.com.ph or email us at topnotchmedicalboardprep@gmail.com
TOPNOTCH MEDICAL BOARD PREP OBSTETRICS-GYNECOLOGY SUPEREXAM
For inquiries visit www.topnotchboardprep.com.ph or email us at topnotchmedicalboardprep@gmail.com
Ite QUESTION EXPLANATION AUTHOR TOPNOTCH
m # EXAM
93 13. Which of the following is the most common and Berek and Novak’s Gynecology 15th edition p.1367 ANDREW TIU FINAL EXAM -
earliest mode of dissemination of ovarian epithelial (TOP 1 - AUG MARCH 2016
cancer? 2015 MED
a. lymphatic BOARDS;
b. hematogeneous TOPNOTCH MD
c. locally invasive FROM CIM)
d. transcoelomic
e. none of the above
94 14. Upon regular PAP smear screening for a 25 any woman with cytologic specimen with HSIL ANDREW TIU FINAL EXAM -
year old female, results showed HSIL. Which of the should undergo colposcopy and directed biopsy (TOP 1 - AUG MARCH 2016
following would be the most appropriate next step becuase ⅔ of patients will have CIN2 or greater. 2015 MED
in management? After colposcopically directed biopsy is done, BOARDS;
a. conization excisional or ablative therapy should be performed. TOPNOTCH MD
b. repeat PAP smear after 6 months Berek and Novak’s Gynecology 15th edition p.588 FROM CIM)
c. HPV DNA testing
d. colposcopy
e. revert back to routine screening
95 15. A 34 year old G0P0 came in for complaints of it is the standard technique for visual inspection of ANDREW TIU FINAL EXAM -
dysmenorrhea as she did not report to work today pelvis and establishment of definitive diagnosis. (TOP 1 - AUG MARCH 2016
in Convergys. She states that she has been having Berek and Novak’s Gynecology 15th edition p.516 2015 MED
this pain for three years already usually before, BOARDS;
during, and after menses usually associated with TOPNOTCH MD
diarrhea. Which of the following is the FROM CIM)
confirmatory test for your primary working
impression?
a. TVS
b. CT scan
c. laparoscopy
d. MRI
e. CA 125
96 16. What is the most common cause of anovulation Berek and Novak’s Gynecology 15th edition p.1134 ANDREW TIU FINAL EXAM -
in women? (TOP 1 - AUG MARCH 2016
a. Kallman syndrome 2015 MED
b. Turner’s syndrome BOARDS;
c. asherman syndrome TOPNOTCH MD
d. stein - leventhal syndrome FROM CIM)
e. sheehan’s syndrome
97 17. A 34 year old female G5P0040 came in for Berek and Novak’s Gynecology 15th edition p.1213 ANDREW TIU FINAL EXAM -
consult for recurrent pregnancy losses. Which of (TOP 1 - AUG MARCH 2016
the following workup would you not order? 2015 MED
a. karyotyping BOARDS;
b. sonohysterogram TOPNOTCH MD
c. TSH, FT4 FROM CIM)
d. anticardiolipin antibodies
e. none of the above
98 18. A 52 year old female came in for complaints of overweight women and those who smoke have ANDREW TIU FINAL EXAM -
hot flashes as it disrupts her sleep most days of the more severe symptoms. SSRI/SNRI are not FDA (TOP 1 - AUG MARCH 2016
week. She asks for your advise on management of approved. Being in a cool environment is associated 2015 MED
her hot flashes. Which of the following is true? with fewer subjective and objective hot flashes. Hot BOARDS;
a. Underweight women have more severe flashes is the primary reason women seek care TOPNOTCH MD
symptoms during menopause. FROM CIM)
b. SSRI/SNRI are the mainstay nonhormonal Berek and Novak’s Gynecology 15th edition p.1235
treatment of hot flashes
c. SSRI are FDA approved
d. lifestyle interventions do not decrease
symptoms
e. vaginal dryness is the primary reason women
seek care during menopause
99 19. A 45 year old female came in for complaints of granulosa cell tumor ANDREW TIU FINAL EXAM -
heavy vaginal bleeding. On ultrasound, an adnexal Berek and Novak’s Gynecology 15th edition p.1407 (TOP 1 - AUG MARCH 2016
mass 4cm in size was found and subsequently 2015 MED
removed. Endometrium was also noted to be BOARDS;
thickened. Which of the following would be most TOPNOTCH MD
likely seen on histopathologic exam of the ovarian FROM CIM)
mass?
a. schiller duval bodies
b. flexner rosettes
c. call exner bodies
d. curschmann spirals
e. none of the above
100 20. A 11 year old came in for complaints of a rash molluscum contagiosum ANDREW TIU FINAL EXAM -
on the perineum. On close examination, (TOP 1 - AUG MARCH 2016
umbilicated pinkish pruritic papules was noted. 2015 MED
Which of the following is the most likely etiology of BOARDS;
the rash? TOPNOTCH MD
a. poxvirus FROM CIM)
b. papiilloma virus
c. herpes virus
d. spirochete
e. cocci
101 A 38 y/o G3P2 (2002) at 32 weeks AOG, Page 9 of OB2 Topnotch Handout. Placenta previa ANGELA DIAGNOSTIC
complained of minimal vaginal bleeding. She had 2 partialis the placenta is partially covering the PAULINE P. EXAM - AUG
previous LTCS, the first was due to placenta previa. internal os. CALIMAG- 2015
BP 110/70, PR 76/min, RR 18/min, no uterine LOYOLA (TOP 8 -
contractions noted, FH 32 cm, FHR 138/min, LM3 FEB 2015 MED
cephalic. Ultrasound was done and showed the BOARDS;
TOPNOTCH MEDICAL BOARD PREP OBSTETRICS-GYNECOLOGY SUPEREXAM Page 13 of 84
For inquiries visit www.topnotchboardprep.com.ph or email us at topnotchmedicalboardprep@gmail.com
TOPNOTCH MEDICAL BOARD PREP OBSTETRICS-GYNECOLOGY SUPEREXAM
For inquiries visit www.topnotchboardprep.com.ph or email us at topnotchmedicalboardprep@gmail.com
Ite QUESTION EXPLANATION AUTHOR TOPNOTCH
m # EXAM
placenta is grade II, posteriorly implanted, edge is TOPNOTCH MD
covering part of the internal os. What type of FROM UST)
previa is this? :
A. Total
B. Partialis
C. Marginalis
D. Low lying
E. Vasa previa
102 A 40 y/o G6P5 (5005), was brought to the ER due Page 10 of OB2 Topnotch Handout. The diagnosis is ANGELA DIAGNOSTIC
to epistaxis, abdominal pain with minimal vaginal abruptio placenta. Risk factors: Hypertension, PAULINE P. EXAM - AUG
bleeding. Her BP was 200/110, PR 120/min, RR advanced maternal age, increased parity. Signs and CALIMAG- 2015
20/min. LMP: July 31, 2014. She had irregular symptoms: Significant bleeding after 20 weeks LOYOLA (TOP 8 -
prenatal care at a lying in clinic. On PE her gestation. Uterine hypertonus manifested by rigid FEB 2015 MED
abdomen was rigid, tender uterus, FH 34cm, LM3 abdomen and tender uterus. FHR is 100 (fetal BOARDS;
cephalic, FHR 100/min by doppler. Speculum: distress). TOPNOTCH MD
cervix: minimal bleeding; IE: cervix long, 1 cm, FROM UST)
cephalic, floating. Leg edema ++, DTRs ++.
Ultrasound showed etroplacental blood clots. What
is the most likely diagnosis?
A. Patient is in active labor
B. Uterine rupture
C. Placenta previa
D. Abruptio placenta
E. Vasa previa
103 Which of the following are possible complications Page 10 of OB2 Topnotch Handout. All are possible ANGELA DIAGNOSTIC
of the above case?
complications of abruptio placenta. PAULINE P. EXAM - AUG
A. DIC CALIMAG- 2015
B. Couveaire uterus LOYOLA (TOP 8 -
C. Acute renal failure FEB 2015 MED
D. A and B are correct BOARDS;
E. All of the above TOPNOTCH MD
FROM UST)
104 An 18 y/o G1P1 who delivered via LTCS 7 days ago Due to the prolonged rupture of membranes (>18 ANGELA DIAGNOSTIC
is now having fever. She was in labor for 16 hours hours) the patientis now having endometritis as PAULINE P. EXAM - AUG
and ROM for 20 hours. VS BP 120/70 PR 110/min evidenced by the foul smelling vaginal discharge CALIMAG- 2015
RR 23/min T 39.90 C. On PE her abdomen is soft, and uterine tenderness. LOYOLA (TOP 8 -
uterine fundus is 2 cm below the umbilicus, (+) FEB 2015 MED
direct tenderness on the hypogastric area. Her BOARDS;
vaginal discharge is serosanguinous, foul smelling. TOPNOTCH MD
Hgb 8 gm/dl WBC 23,000 cells/ul. What is the FROM UST)
most likely diagnosis?
A. UTI
B. Pyelonephritis
C. Endometritis
D. Surgical site infection
E. None of the above
105 A 45 y/o G10P8 (8028) delivered via NSD. After 20 Page 11 and 12 of OB2 Topnotch Handout. This ANGELA DIAGNOSTIC
minutes, there was still profuse vaginal bleeding. patient is in uterine atony unresponsive to PAULINE P. EXAM - AUG
BP 100/70 PR 105/min RR 21/min T 36.80 C. If medictaions. But since she is already of advanced CALIMAG- 2015
uterine atony in this patient fails to respond to age with increased parity hence the best LOYOLA (TOP 8 -
medications in this patient what is the best management in this case is a hysterectomy. FEB 2015 MED
management? A. Uterine compression BOARDS;
B. Uterine packing TOPNOTCH MD
C. B Lynch suturing FROM UST)
D. Hysterectomy
E. None of the above
106 A 29 y/o G1P0 37 weeks AOG went to your clinic Page 3 of OB Pearls. Pawlick's grip is the third ANGELA DIAGNOSTIC
for prenatal check-up. On PE what maneuver will maneuver done in Leopolds and is used to PAULINE P. EXAM - AUG
you do to determine the engagement of the determine the engagement of the presenting part. A CALIMAG- 2015
presenting part?
is the First maneuver in Leopold's and is used to LOYOLA (TOP 8 -
A. Fundal grip determine the fetal part lying in the fundus. Pinards FEB 2015 MED
B. Pawlick's grip is a breech decomposition maneuver, wherehe BOARDS;
C. Zavanelli maneuver fingers are pressed in the baby's popliteal fossa TOPNOTCH MD
D. Rubin's maneuver causing flexion of the knee, foot is grasped and FROM UST)
E. Pinards maneuver delivered as footling. Rubins maneuver- fetal
shoulders are rocked from side to side by applying
force on the mothers abdomen. Zavanelli is cephalic
replacement into the pelvis followed by CS.
107 In BPP, which of the following parameter is most Page 3-4 of OB Pearls. The most important ANGELA DIAGNOSTIC
important and reflects chronic hypoxia?
parameter in BPP is AFV. Abnormal AFV usually PAULINE P. EXAM - AUG
A. AFV connotes chronic hypoxia. CALIMAG- 2015
B. Fetal breathing LOYOLA (TOP 8 -
C. Fetal movement FEB 2015 MED
D. Fetal tone BOARDS;
E. NST TOPNOTCH MD
FROM UST)
108 The following would warrant in-patient treatment Page 9 of Topnotch Handout. The following are ANGELA DIAGNOSTIC
for PID except?
conditions for in-patient treatment: 1) Surgical PAULINE P. EXAM - AUG
A. TVS showing thickened, fluid filled tubes with emergencies cannot be excluded; 2) Pregnant; CALIMAG- 2015
tubo-ovarian complex 3)Does not respond clinically to oral antimicrobial LOYOLA (TOP 8 -
B. Appendicitis is ruled out therapy; 4)Unable to follow or tolerate an FEB 2015 MED
C. Patient does not respond clinically to outpatient oral regimen; 5)Severe illness, nausea BOARDS;
Ambulatory Regimen A and vomiting, high fever; 6) Tuboovarian abcess TOPNOTCH MD
D. Patient is concurrently pregnant FROM UST)
E. High fever with nausea and vomiting

TOPNOTCH MEDICAL BOARD PREP OBSTETRICS-GYNECOLOGY SUPEREXAM Page 14 of 84


For inquiries visit www.topnotchboardprep.com.ph or email us at topnotchmedicalboardprep@gmail.com
TOPNOTCH MEDICAL BOARD PREP OBSTETRICS-GYNECOLOGY SUPEREXAM
For inquiries visit www.topnotchboardprep.com.ph or email us at topnotchmedicalboardprep@gmail.com
Ite QUESTION EXPLANATION AUTHOR TOPNOTCH
m # EXAM
109 A 23 y/o HIV positive on HAART with a CD4+ count Page 25 of Topnotch Handout. Efavirenz should be ANGELA DIAGNOSTIC
of <250 cells/mm3 comes for or prenatal checkup. discontinued in the first trimester. PAULINE P. EXAM - AUG
Which of the following drugs should be CALIMAG- 2015
discontinued in the first ttrimester? LOYOLA (TOP 8 -
A. Lamivudine FEB 2015 MED
B. Zidovudine BOARDS;
C. Nevirapine TOPNOTCH MD
D. Efavirenz FROM UST)
E. None of the above
110 This bipotential structure develops into the Page 4 of Gyne Topnotch Handout. The urogenital ANGELA DIAGNOSTIC
urethra, lower 3rd of the vagina, Bartholin's and sinus develops into the male urethra, Cowper's PAULINE P. EXAM - AUG
Skene's glands in the female: A. Genital gland and prostate gland in males while in females CALIMAG- 2015
folds it develops into the urethra, lower 3rd of the vagina, LOYOLA (TOP 8 -
B. Mesonephric duct Bartholin's and Skene's glands FEB 2015 MED
C. Mullerian duct BOARDS;
D. Genital tubercle TOPNOTCH MD
E. Urogenital sinus FROM UST)
111 4 y/o child with recurrent urinary tract infection Page 5 of gyne Topnotch Handout. This is a case of ANGELA DIAGNOSTIC
and vulvar itching was referred by her pedia for an Adhesive vulvitis whih is a self limiting PAULINE P. EXAM - AUG
apparent "imperforate hymen". On PE there was consequence of chronic vulvitis. Pathognomonic is CALIMAG- 2015
noted labial fusion with a translucent vertical line the translucent vertical line at the midline. LOYOLA (TOP 8 -
at the midline. What is the treatment of choice? Treatment for this is Topical estrogen cream BID FEB 2015 MED
A. Surgical correction for 3 weeks. BOARDS;
B. Clobetasol ointment TOPNOTCH MD
C. Topical estrogen cream FROM UST)
D. OCPs
E. Antibiotics
112 A 25 y/o G1P1 female with previously regular Page 12 of gyne Topnotch Handout. If patient is ANGELA DIAGNOSTIC
menses consulted due to amenorrhea for 6 months. positive in the estrogen-progeterone challenge the PAULINE P. EXAM - AUG
PE is normal with no signs of virilization. next step would be to determine the FSH level. If CALIMAG- 2015
Pregnancy test is negative. You started her on MPA low fsh it is an HP failure, if FSh is high it is LOYOLA (TOP 8 -
for one week after which she has no withdrawal probably due to ovarian failure. FEB 2015 MED
bleeding. You decided to give CEE for 21 days BOARDS;
followed by MPA for 7 days, after which she had TOPNOTCH MD
withdrawal bleeding. What will you determine FROM UST)
next to find out the cause of the amenorrhea?
A. Prolactin level
B. TSH level
C. LH level
D. FSH level
E. No need for another test
113 If the patient in the above case had no withdrawal Page 12 of Gyne Topnotch Handout. If patient is ANGELA DIAGNOSTIC
bleeding after the estrogen-progesterone challenge negative in the estrogen-progeterone challenge the PAULINE P. EXAM - AUG
test, which of the following could have caused her next step would be to determine if there is outflow CALIMAG- 2015
ammenorrhea?
tract obstuction LOYOLA (TOP 8 -
A. Hypothyroidism FEB 2015 MED
B. Prolactinoma BOARDS;
C. Premature ovarian failure TOPNOTCH MD
D. PCOS FROM UST)
E. Asherman's syndrome
114 Decreased libido in menopause is improved by:
Page 13 of gyne Topnotch Handout. Decreased ANGELA DIAGNOSTIC
A. Testosterone 200mg IM/month libido is improved by IM testosterone. PAULINE P. EXAM - AUG
B. CEE 0.625 mg daily
CALIMAG- 2015
C. MPA 2.5 mg daily LOYOLA (TOP 8 -
D. Ethinyl estradiol 0.002 mg FEB 2015 MED
E. None of the above BOARDS;
TOPNOTCH MD
FROM UST)
115 A 33 y/o female G2P2 presented at the ER with a SIMILAR TO PREVIOUS BOARD EXAM ANGELA DIAGNOSTIC
2-month history of progressive dyspnea, CONCEPT/PRINCIPLE. Page 26 of Gyne Topnotch PAULINE P. EXAM - AUG
abdominal distension, and vague pelvic pressure. Handout. This is a case of Meig's syndrome typically CALIMAG- 2015
Physical examination showed dullness in thoracic associated with a fibroma.Meigs syndrome is LOYOLA (TOP 8 -
percussion on the right, (+) fluid wave on defined as the triad of benign ovarian tumor with FEB 2015 MED
abdominal percussion and a unilateral, movable ascites and pleural effusion that resolves after BOARDS;
mass on bimanual pelvic exam. Chest radiography resection of the tumor. Ovarian fibromas constitute TOPNOTCH MD
showed pleural effusion on the right, and the majority of the benign tumors seen in Meigs FROM UST)
abdominal ultrasonography showed ascites, and a syndrome. Meigs syndrome, however, is a diagnosis
unilateral 5 cm solid mass on the left ovary.The of exclusion, only after ovarian carcinoma is ruled
symptoms resolved after removal of the tumor. out
What ovarian tumor would be the most likely
cause of the patient's symptoms?
A. Choriocarcinoma
B. Sertoli-Leydig cell tumor
C. Fibroma
D. Mucinous tumor
E. Clear cell tumor
116 Main objective in hormonal treatment in Page 23 of Topnotch Handout. The primary goal of ANGELA DIAGNOSTIC
endometriosis: medical management of endometriosis is induction PAULINE P. EXAM - AUG
A. Ovulation induction of amenorrhea. CALIMAG- 2015
B. Induce hypomenorrhea LOYOLA (TOP 8 -
C. Induce amenorrhea FEB 2015 MED
D. Regulate menstruation BOARDS;
E. Fertility TOPNOTCH MD
FROM UST)

TOPNOTCH MEDICAL BOARD PREP OBSTETRICS-GYNECOLOGY SUPEREXAM Page 15 of 84


For inquiries visit www.topnotchboardprep.com.ph or email us at topnotchmedicalboardprep@gmail.com
TOPNOTCH MEDICAL BOARD PREP OBSTETRICS-GYNECOLOGY SUPEREXAM
For inquiries visit www.topnotchboardprep.com.ph or email us at topnotchmedicalboardprep@gmail.com
Ite QUESTION EXPLANATION AUTHOR TOPNOTCH
m # EXAM
117 When is it recommended to begin screening with Page 19 of Topnotch Handout. Screening ANGELA DIAGNOSTIC
PAP smear? recommendation for PAP smear is 3 years after PAULINE P. EXAM - AUG
a. 21 years old onset of sexual activity or 21 years old which ever CALIMAG- 2015
b. 1 year after the initiation of vaginal occurs first. LOYOLA (TOP 8 -
intercourse FEB 2015 MED
c. Once sexually active BOARDS;
d. A and B are correct TOPNOTCH MD
E. All of the above FROM UST)
118 During which phase in a woman's lifetime do Page 15 of OB1 Topnotch Handout. At birth, what ANGELA DIAGNOSTIC
oocytes complete Meiosis II:
we have are now actually primary oocytes, because PAULINE P. EXAM - AUG
A. At birth they have already entered Meiosis I, which is CALIMAG- 2015
B. Puberty specifically arrested at Prophase I. So, at birth, it LOYOLA (TOP 8 -
C. Ovulation only becomes 2 M. Why? What happened to the FEB 2015 MED
D. Fertilization other 4 M? They aged (underwent BOARDS;
E. Menopause apoptosis/atresia). So in time, after birth, they TOPNOTCH MD
decrease in population. When they reach puberty, FROM UST)
when they start to have menstruation, with
influence of LH surge that promotes ovulation,
these primary oocytes are now called secondary
oocytes because they already completed Meiosis I.
They are being prepared for ovulation. At
ovulation, when these oocytes are then released
from the follicle, the secondary oocytes undergo
Meiosis II, and then they are specifically arrested at
Metaphase 2. This is important because they
became diploid (2N) to haploid (1N). This is again
important in meeting another haploid (1N), which
is the sperm, so that during fertilization the
secondary oocyte completes meiosis II forming a
mature ovum, the secondary oocyte and sperm will
combine and become a diploid (2N), which is now
called a zygote.
119 The following vaccines may be given to pregnant SIMILAR TO PREVIOUS BOARD EXAM ANGELA DIAGNOSTIC
women except:
CONCEPT/PRINCIPLE. Page 30 of OB1 Topnotch PAULINE P. EXAM - AUG
A. Rabies vaccine Handout. Live virus vaccines eg. Varicella-zoster, CALIMAG- 2015
B. MMR MMR, polio, and yellow fever are not recommended LOYOLA (TOP 8 -
C. Meningococcal vaccine during pregnancy. Toxoids or killed bacteria or FEB 2015 MED
D. Hepatitis B vaccine viruses eg rabies, influenza, pneumococcal, BOARDS;
E. Pneumococcal vaccine meningococcal and hepatitis B has not been TOPNOTCH MD
associated with adverse fetal outcomes. FROM UST)
120 A monochorionic, diamniotic set of monozygotic Page 2 of OB pearls Topnotch Handout. ANGELA DIAGNOSTIC
twins can be expected if division occurs___: Dichorionic/Diamniotic-Days 1-3, PAULINE P. EXAM - AUG
A. <4 days Monochorionic/Diamniotic-Days 4-8, CALIMAG- 2015
B. 4-8 days Monochorionic/Monoamniotic-Days 8-13, LOYOLA (TOP 8 -
C. 8-13 days Conjoined twins-Days 13-15. FEB 2015 MED
D. 13-15 days BOARDS;
E. >15 days TOPNOTCH MD
FROM UST)
121 A 26-year old female came to your clinic This is characteristic of Trichomoniasis, caused by LYNN DARYL MIDTERM 1
complaining of vaginal itching, dysuria, and frothy Trichomonas vaginalis. Other clinical manifestation FELICIANO EXAM - AUG
greenish vaginal discharge. The most likely include strawberry cervix and foul smelling VILLAMATER, 2015
diagnosis is: dicharge. Gonococcal infection among women MD (TOP 5 - FEB
A. Bacterial vaginosis might not produce recognizable symptoms until 2015 MED
B. Candidiasis complications (e.g. PID) have occured. SIMILAR TO BOARDS;
C. Trichomoniasis PREVIOUS BOARD EXAM CONCEPT. TOPNOTCH MD
D. Gonococcal infection FROM EAC)
E. Chlamydia infection
122 According ACOG, what criteria should be met According to ACOG, 2 criteria should be met before LYNN DARYL MIDTERM 1
before arriving at a diagnosis of Dystocia? diagnosis: 1. The latent phase has been completed FELICIANO EXAM - AUG
A. Uterine contraction pattern of 200 with cervix dilated 4 cm or more. 2. Uterine VILLAMATER, 2015
Montevideo units or more in a 10 minute period contraction pattern of 200 MV units or more in 1 10 MD (TOP 5 - FEB
has been present for 2 hours without cervical minute period has been present for 2 hours witut 2015 MED
change. cervical change. (SIMILAR TO PREVIOUS BOARD BOARDS;
B. The latent phase has been completed with EXAM CONCEPT) TOPNOTCH MD
cervix dilated at 6 cms or more. FROM EAC)
C. The first stage of labor has lasted for more than
20 hours in nullipara.
D. There is a complete cessation of cervical
dilation.
E. The uterotonic has already been given but to
no avail.
123 A 23-year-old woman, G2P1 (1001) whose LMP You will suspect molar gestation for symptoms of LYNN DARYL MIDTERM 1
was 8 weeks ago presented to the emergency room pregnancy (amenorrhea, nausea/vomiting), FELICIANO EXAM - AUG
with history of nausea and vaginal bleeding for the presence of bleeding prior to 16 weeks, and fundus VILLAMATER, 2015
last 3 days. On examination, her fundus is palpated larger than dates (most common finding). MD (TOP 5 - FEB
midway between the umbilicus and symphysis 2015 MED
pubis. No adnexal tenderness elicited. What is the BOARDS;
most likely diagnosis? TOPNOTCH MD
A. Ectopic pregnancy FROM EAC)
B. Missed abortion
C. Cervical cancer
D. Dysfunctional uterine bleeding
E. Hydatidiform mole

TOPNOTCH MEDICAL BOARD PREP OBSTETRICS-GYNECOLOGY SUPEREXAM Page 16 of 84


For inquiries visit www.topnotchboardprep.com.ph or email us at topnotchmedicalboardprep@gmail.com
TOPNOTCH MEDICAL BOARD PREP OBSTETRICS-GYNECOLOGY SUPEREXAM
For inquiries visit www.topnotchboardprep.com.ph or email us at topnotchmedicalboardprep@gmail.com
Ite QUESTION EXPLANATION AUTHOR TOPNOTCH
m # EXAM
124 Milk ejection is the result of the action of: SIMILAR TO PREVIOUS BOARD EXAM CONCEPT LYNN DARYL MIDTERM 1
A. Oxytocin FELICIANO EXAM - AUG
B. Prolactin VILLAMATER, 2015
C. Estrogen MD (TOP 5 - FEB
D. Human placental lactogen 2015 MED
E. Progesterone BOARDS;
TOPNOTCH MD
FROM EAC)
125 A 24-year-old patient presented with nausea, Decision for in-patient treatment: surgical LYNN DARYL MIDTERM 1
vomiting, lower abdominal pain, fever, and emergencies cannot be excluded, pregnant, does FELICIANO EXAM - AUG
mucopurulent vaginal discharge. Pregnancy test not respond clinically to oral antimicrobial, unable VILLAMATER, 2015
was negative. On pelvic examination, there is to follow or tolerate outpatient oral regimen, severe MD (TOP 5 - FEB
presence of left adnexal tenderness. Transvaginal illness, tubo-ovarian abscess. 2015 MED
ultrasound was requested. Which of the following BOARDS;
situation warrants admission? TOPNOTCH MD
A. Presence of tubo-ovarian abscess FROM EAC)
B. Elevated ESR and CRP
C. Presence of abundant numbers of WBC on
vaginal fluid
D. Abnormal mucopurulent discharge
E. Temperature of more than 38.3 C
126 The in-patient antibiotic regimen for pelvic Option C - Ambulatory regimen LYNN DARYL MIDTERM 1
inflammatory disease include: FELICIANO EXAM - AUG
A. Ceftriaxone 250 mg IM single dose + VILLAMATER, 2015
Metronidazole 500 mg oral BID for 14 days MD (TOP 5 - FEB
B. Cefoxitin 2 g IM and Probenecid + Doxycycline 2015 MED
100 mg oral BID for 14 days. BOARDS;
C. Ceftriaxone 250 mg IM single dose + TOPNOTCH MD
Doxycycline 100 mg oral BID for 14 days with or FROM EAC)
without Metronidazole
D. Cefoxitin 2 g IV q6 + Doxycycline 100 mg oral
BID for 14 days
E. Cefotetan 2 g IM single dose + Gentamicin
3mg/kg/dose
127 A 30-year-old female comes to your clinic with 6- Progestins is the treatment of choice for LYNN DARYL MIDTERM 1
month history of irregular and unpredictable anovulatory DB. For those who desires FELICIANO EXAM - AUG
menses. Her pregnancy test is negative. She contraception: OCP. Otherwise, give cyclic progestin VILLAMATER, 2015
weighs 128 lbs, and she is 5'4". Your main therapy from Day 14-25 of each cycle. MD (TOP 5 - FEB
impression is anovulatory bleeding. How will you 2015 MED
manage this patient if she still desires fertility? BOARDS;
A. Conjugated equine estrogen TOPNOTCH MD
B. Medroxyprogesterone for 7-10 days FROM EAC)
C. Leuprolide
D. Cyclic progestin
E. Danazol
128 An 18-year-old female was brought by her mother MRKH or mullerian agenesis is a congenital LYNN DARYL MIDTERM 1
to your office. Her daughter has never had a malformation characterized by a failure of the FELICIANO EXAM - AUG
menstrual period ever. On examination, patient Mullerian duct to develop resulting in complete or VILLAMATER, 2015
had breast but with short vagina. However on partial absence of uterus and cervix and variable MD (TOP 5 - FEB
ultrasound, she had absent uterus. Karyotyping degrees of vaginal hypoplasia of its upper portion. I 2015 MED
done showed 46 XX. This is a case of: BOARDS;
A. Androgen insensitivity TOPNOTCH MD
B. Turner's syndrome FROM EAC)
C. Premature ovarian failure
D. Outflow obstruction
E. Mayer-Rokitansky-Kuster-Hauser syndrome
129 The definitive sign of pregnancy seen with An anechoic sac (intradecidual sign) without a LYNN DARYL MIDTERM 1
transvaginal ultrasound is: distinct chorionic ring is the earliest sign of FELICIANO EXAM - AUG
A. An anechoic sac pregnancy. A thin-walled sac within the uterus is VILLAMATER, 2015
B. A thin-walled sace within the uterus not definitive evidence of pregnancy. A lemon sign MD (TOP 5 - FEB
C. A gestation sac containing a yolk sac is a cranial sign of a neural tube defect. 2015 MED
D. A lemon sign BOARDS;
E. An intradecidual sign TOPNOTCH MD
FROM EAC)
130 Which of the following is a probable sign of Option A. Definitive sign. Option B - Presumptive. LYNN DARYL MIDTERM 1
pregnancy? Option D - Presumptive (according to William's), FELICIANO EXAM - AUG
A. Perception of fetal movement by the examiner Option D - Presumptive. VILLAMATER, 2015
B. Changes in cervical mucus MD (TOP 5 - FEB
C. Softening of the cervix 2015 MED
D. Chadwick's sign BOARDS;
E. Cessation of menses TOPNOTCH MD
FROM EAC)
131 Variable decelerations in fetal heart monitoring acceleration- a response to movement; Early LYNN DARYL MIDTERM 1
may indicate? deceleration - Head compression; late deceleration: FELICIANO EXAM - AUG
A. A response to movement uteroplacental insufficiency; Variable deceleration- VILLAMATER, 2015
B. Head compression umbilical cord compression. SIMILAR TO MD (TOP 5 - FEB
C. Uteroplacental insufficiency PREVIOUS BOARD EXAM CONCEPT. 2015 MED
D. Umbilical cord compression BOARDS;
E. No prognostic significance TOPNOTCH MD
FROM EAC)
132 The single most important risk factor in the LYNN DARYL MIDTERM 1
development of post partum pelvic infection is: FELICIANO EXAM - AUG
A. Prolonged labor VILLAMATER, 2015
B. Multiparity MD (TOP 5 - FEB
C. Early rupture of membranes 2015 MED
D. Route of delivery BOARDS;
TOPNOTCH MEDICAL BOARD PREP OBSTETRICS-GYNECOLOGY SUPEREXAM Page 17 of 84
For inquiries visit www.topnotchboardprep.com.ph or email us at topnotchmedicalboardprep@gmail.com
TOPNOTCH MEDICAL BOARD PREP OBSTETRICS-GYNECOLOGY SUPEREXAM
For inquiries visit www.topnotchboardprep.com.ph or email us at topnotchmedicalboardprep@gmail.com
Ite QUESTION EXPLANATION AUTHOR TOPNOTCH
m # EXAM
E. Presence of urinary tract infection TOPNOTCH MD
FROM EAC)

133 A 35-year old G2P1 at 18 weeks AOG comes for a BP > 140/90 before pregnancy or before 20 weeks : LYNN DARYL MIDTERM 1
prenatal visit. On examination, BP 160/90, 2+ chronic hypertension. The presence of new onset FELICIANO EXAM - AUG
bipedal edema, FHT 144 bpm. Spot urine dipstick proteinuria in hypertensive women : superimposed VILLAMATER, 2015
showed 2+ protein. What is the most likely preeclampsia MD (TOP 5 - FEB
diagnosis? 2015 MED
A. Preeclampsia, severe BOARDS;
B. Mild pre-eclampsia TOPNOTCH MD
C. Chronic hypertension FROM EAC)
D. Chronic hypertension with superimposed pre-
eclampsia
E. Gestational hypertension
134 A 28-year old female had her pap smear which PAP smear LSIL - observe and repeat pap smear LYNN DARYL MIDTERM 1
revealed HGSIL. Biopsies shows carcinoma in situ. after 3 mos. HGSIL with moderate dysplasia - FELICIANO EXAM - AUG
Management for this patient is: ablative or excisional; HGSIL, carcinoma in situ, VILLAMATER, 2015
A. Hysterectomy child brearing/desires fertility- therapeutic MD (TOP 5 - FEB
B. Cold knife conization conization. SIMILAR QUESTION ASKED IN THE 2015 MED
C. Chemoradiation BOARDS BOARDS;
D. Colposcopy and repeat biopsy after 3 months TOPNOTCH MD
E. Repeat pap smear after 6 months FROM EAC)
135 A 58-year old G4P4 women comes to your clinic Prolonged estrogen exposure (Tamoxifen use) is a LYNN DARYL MIDTERM 1
with a 3-week history of painless vaginal bleeding. risk factor for endometrial cancer. The most FELICIANO EXAM - AUG
Her blood pressure is 130/90. She has a history of common presentation is post menopausal bleeding. VILLAMATER, 2015
breast cancer and have taken Tamoxifen for 3 Diagnosis is done through endometrial biopsy to MD (TOP 5 - FEB
years. What will be your next step? determine presence of atypia, which warrants 2015 MED
A. Do an endometrial biopsy hysterectomy. BOARDS;
B. Advise hysterectomy TOPNOTCH MD
C. Prescribe with cyclic progestins FROM EAC)
D. Do a pap smear and colposcopy
E. Refer for TAHBSO
136 A 24-year-old woman presented with absence of If there is no withdrawal bleeding after a LYNN DARYL MIDTERM 1
menses for 6 months now. Urine B-HcG is Progesterone Challenge test and Estrogen- FELICIANO EXAM - AUG
negative. She does not display any signs of progesterone challenge test, you consider outflow VILLAMATER, 2015
virilization. Conjugated Equine Estrogen was given obstruction. Request for HSG. MD (TOP 5 - FEB
for 21 days followed by Medroxyprogesterone. No 2015 MED
withdrawal bleeding was noted. You will suspect: BOARDS;
A. Premature ovarian failure TOPNOTCH MD
B. Outflow obstruction FROM EAC)
C. MRKH
D. Hypopituitarism
E. CAH
137 A 33-year old patient, nulligravid, comes to your Classic pelvic findings in endometriosis include a LYNN DARYL MIDTERM 1
clinic due to a 3 year hisotry of dull, aching pain fixed, tender uterus with scarring and tenderness FELICIANO EXAM - AUG
during menstruation. On pelvic examination, you posterior to the uterus on IE, and characteristic VILLAMATER, 2015
note a tender, retroverted uterus, with uterosacral nodularities in the uterosacral and cul-de-sac.It MD (TOP 5 - FEB
nodularity. The most likely diagnosis is: responds to cyclic changes in ovarian hormone 2015 MED
A. Adenomyosis production. Adenomyosis usually presents with BOARDS;
B. Endometriosis diffusely enlarged uterus. Both, however, presents TOPNOTCH MD
C. Endometrial polyp with dysmenorrhea or painful menstruation. FROM EAC)
D. Leiomyoma SIMILAR TO PREVIOUS BOARD EXAM CONCEPT.
E. Cervical polyp
138 A 30-year old, non-pregnant female presented with Indications for Pelvic LYNN DARYL MIDTERM 1
a 5 cm solid adnexal mass on ultrasound. She is laparoscopy/Surgery(Cystectomy): solid adnexal FELICIANO EXAM - AUG
not on OCP. What is the next step in the mass (regardless of age), adnexal mass after VILLAMATER, 2015
management of her condition? menopause or before puberty, cystic mass >8 cm, MD (TOP 5 - FEB
A. Observe for 8-12 weeks, then repeat cystic mass of 5-8 cm persisting more than 8 weeks 2015 MED
ultrasound. in a menstruating woman, cystic mass in patient on BOARDS;
B. Pelvic laparoscopy OCP TOPNOTCH MD
C. TAHBSO FROM EAC)
D. HSG
E. OCP
139 You will advise a woman using the oral SIMILAR TO PREVIOUS BOARD EXAM CONCEPT LYNN DARYL MIDTERM 1
contraceptive for the first time to start taking it: FELICIANO EXAM - AUG
A. On Day 1 of menses VILLAMATER, 2015
B. On Day 5 of menses MD (TOP 5 - FEB
C. On Day 7 of menses 2015 MED
D. On days 14 of menses BOARDS;
E. Anytime as long as she is not pregnant TOPNOTCH MD
FROM EAC)
140 A 40-year-old non-obese female presented with a The hysterosalpingography procedure is best LYNN DARYL MIDTERM 1
5-month history of bleeding in between periods. performed on days 5-12 of menstrual cycle to make FELICIANO EXAM - AUG
Her BP is 120/70. Hysterosalphingograpy was certain that patient is not pregnant during the VILLAMATER, 2015
requested. You will advise the patient to have it exam. SIMILAR TO PREVIOUS BOARD EXAM MD (TOP 5 - FEB
done CONCEPT. 2015 MED
A. During periovulatory period BOARDS;
B. Before menses TOPNOTCH MD
C. After menses FROM EAC)
D. Anytime during the cycle
E. On day 21 of cycle
141 MI, a 28 year old G1 was admitted for NSD, while A and B are symptoms noted in a uterine rupture. EDWARD HARRY MIDTERM 2
attending to the patient, you noted uterine rupture VALLAJERA, MD EXAM - AUG
when which of the following were noted (TOP 8 - FEB 2015
A. Sudden retraction of the presenting part 2015 MED

TOPNOTCH MEDICAL BOARD PREP OBSTETRICS-GYNECOLOGY SUPEREXAM Page 18 of 84


For inquiries visit www.topnotchboardprep.com.ph or email us at topnotchmedicalboardprep@gmail.com
TOPNOTCH MEDICAL BOARD PREP OBSTETRICS-GYNECOLOGY SUPEREXAM
For inquiries visit www.topnotchboardprep.com.ph or email us at topnotchmedicalboardprep@gmail.com
Ite QUESTION EXPLANATION AUTHOR TOPNOTCH
m # EXAM
B. Sudden drop in maternal BP BOARDS;
C. Sudden onset of painful contractions TOPNOTCH MD
D. A and B only FROM
E. All of the above PERPETUAL
BINAN)
142 MJR, a 40 year old G8P7 was admitted due to labor A neglected transverse lie is one of the indications EDWARD HARRY MIDTERM 2
pains, upon PE you noted a transverse lying fetus to perform a classical Cesarean Section. VALLAJERA, MD EXAM - AUG
with the shoulder as the presenting part, what is (TOP 8 - FEB 2015
your plan of management? 2015 MED
A. Classical CS BOARDS;
B. Low transverse CS TOPNOTCH MD
C. Manipulate the fetus to convert into cephalic FROM
D. Deliver the baby vaginally PERPETUAL
E. None of the above BINAN)
143 PES, a 30 year old G2P1 at 30 weeks AOG was The loading dose for magnesium sulfate is 5 gm IM EDWARD HARRY MIDTERM 2
admitted because of severe pre-eclampsia, the on each butt cheek, 4 gm slow IV bolus for 2 hours VALLAJERA, MD EXAM - AUG
loading dose for the anti-convulsant of choice for and the maintenance dose is 1-2gm/hr IV drip or (TOP 8 - FEB 2015
seizures due severe pre-eclampsia is 5gm IM every 6 hours 2015 MED
A. 1 - 2 gm/hr IV slow bolus BOARDS;
B. 4 gm slow IV bolus to run for 2 hours TOPNOTCH MD
C. 10 g IM on each butt cheek FROM
D. All of the above PERPETUAL
E. B and C BINAN)
144 KL, a 30 year old G2P1 at 24 weeks AOG has Chloroquine is safe for pregnant women regardless EDWARD HARRY MIDTERM 2
malaria, as her OB you know that aside from of the trimester of pregnancy. VALLAJERA, MD EXAM - AUG
quinine, another alternative drug for malaria safe (TOP 8 - FEB 2015
to use during pregnancy is? 2015 MED
A. Quinine BOARDS;
B. Chloroquine TOPNOTCH MD
C. Mefloquine FROM
D. Artemether-lumefantrine PERPETUAL
E. None of the above BINAN)
145 TC, a 30 year old female came in at the ER due to Both B and C are part of the management of a EDWARD HARRY MIDTERM 2
vaginal bleeding, upon further examination, you cervical pregnancy, curettage is the procedure VALLAJERA, MD EXAM - AUG
were able to conclude cervical pregnancy, which needed to completely evacuate the products of (TOP 8 - FEB 2015
among the following is the treatment of choice conception 2015 MED
A. Exploratory laparotomy BOARDS;
B. Systemic methotrexate TOPNOTCH MD
C. Intra-amniotic injection of methotrexate, KCl and FROM
hyperosmolar glucose PERPETUAL
D. B and C BINAN)
E. All of the above
146 MSD, a 30 year old female came at the OPD clinic This is a case of trichomoniasis and the treatment EDWARD HARRY MIDTERM 2
because of vaginal discharge, on PE you noted a of choice is metronidazole VALLAJERA, MD EXAM - AUG
green-yellow, forthy vaginal discharge with a (TOP 8 - FEB 2015
strawberry cervix, what is the treatment of choice? 2015 MED
A. Clindamycin BOARDS;
B. Itraconazole TOPNOTCH MD
C. Fluconazoe FROM
D. Metronidazole PERPETUAL
E. Ciprofloxacin BINAN)
147 PM, a 28 year old G1P0 at 18 weeks AOG is being PTU does not readily cross the placenta as much as EDWARD HARRY MIDTERM 2
treated for hyperthyroidism, she asks you why you methimazole does. PTU also blocks the conversion VALLAJERA, MD EXAM - AUG
are changing her hyperthyroid medication of T4 to T3 in the peripheral tissues. Both PTU and (TOP 8 - FEB 2015
methimazole to PTU, you explain that PTU is methimazole have the same site of action. 2015 MED
preferred for pregnant patients because: BOARDS;
A. It does not cross the placenta as readily as TOPNOTCH MD
methimazole FROM
B. It does not block the peripheral conversion of T4 PERPETUAL
to T3 BINAN)
C. It acts on a different pathway of thyroid
hormone synthesis
D. All of the above
E. None of the above
148 KS, a 17 year old female was brought by her Kallman's syndrome is the congenital absence of EDWARD HARRY MIDTERM 2
mother due to amenorrhea, upon PE, you noted GnRH producing cells causing primary amenorrhea VALLAJERA, MD EXAM - AUG
anosmia, what is the most likely diagnosis? associated with anosmia. Sheehan's syndrome is (TOP 8 - FEB 2015
A. Fitz-Hugh-Curtis syndrome hypopituitarism caused by ischemic necrosis of the 2015 MED
B. Mayer-Rokitansky-Kuster-Hauser syndrome pituitary gland. Ashermann's syndrome is BOARDS;
C. Ashermann syndrome intrauterine adhesions usually from repeated TOPNOTCH MD
D. Sheehan's syndrome curettage. FROM
E. Kallman's syndrome PERPETUAL
BINAN)
149 A post-menopausal patient came in following an In a post-menopausal patient wuth complex EDWARD HARRY MIDTERM 2
endometrial biopsy, results showed complex hyperplasia with atypia, the risk of endometrial CA VALLAJERA, MD EXAM - AUG
hyperplasia with atypia, as her attending OB-Gyne, is 50%, the treatment therefore is to remove the (TOP 8 - FEB 2015
what will you do? uterus by performing hysterectomy 2015 MED
A. Observe the patient and repeat biopsy after 6 BOARDS;
months TOPNOTCH MD
B. Start chemotherapy FROM
C. Give progestin tablets PERPETUAL
D. Schedule the patient for hysterectomy BINAN)
E. Perform dilation and curettage

TOPNOTCH MEDICAL BOARD PREP OBSTETRICS-GYNECOLOGY SUPEREXAM Page 19 of 84


For inquiries visit www.topnotchboardprep.com.ph or email us at topnotchmedicalboardprep@gmail.com
TOPNOTCH MEDICAL BOARD PREP OBSTETRICS-GYNECOLOGY SUPEREXAM
For inquiries visit www.topnotchboardprep.com.ph or email us at topnotchmedicalboardprep@gmail.com
Ite QUESTION EXPLANATION AUTHOR TOPNOTCH
m # EXAM
150 The only cure for severe pre-eclampsia Delivery of the fetus is the only cure for severe pre- EDWARD HARRY MIDTERM 2
A. Control the BP eclampsia, especially if the fetus is mature enough VALLAJERA, MD EXAM - AUG
B. Delivery of the fetus to survive outside the uterus, then delivery of the (TOP 8 - FEB 2015
C. Giving tocolytics fetus will improve the mother's pre-eclampsia 2015 MED
D. Giving anti-epileptics BOARDS;
E. None of the above TOPNOTCH MD
FROM
PERPETUAL
BINAN)
151 A 28 year old G2P1 patient came in at the ER due to McRobert's maneuver - sharply flexing the thighs EDWARD HARRY MIDTERM 2
labor pains, on history, you noted she has GDM upon the patient's own abdomen which straightens VALLAJERA, MD EXAM - AUG
currently treated with insulin and the latest the sacrum relative to the lumbar vertebrae. Chavis (TOP 8 - FEB 2015
ultrasound showed that baby weighs maneuver - shoulder horn instrument consisting of 2015 MED
approximately 5kg, on PE, you noted that the a concave blade is slipped between symphysis BOARDS;
cervix is fully dilated and the head is at station 0, pubis and impacted shoulder. Hibbard's maneuver - TOPNOTCH MD
what is the maneuver that you will perform in pressure is applied to the infant's jaw and neck in FROM
order to push the head and the baby back into the the direction of the mother's rectum with strong PERPETUAL
uterus followed by a Cesarean Section fundal pressure applied by an assistant as anterior BINAN)
A. Zavanelli Maneuver shoulder is freed.
B. McRobert's maneuver
C. Chavis maneuver
D. Hibbard's maneuver
E. None of the above
152 You are a resident attending to a patient in labor, The presence of both signs warrant stoppage of the EDWARD HARRY MIDTERM 2
you incorporated 10 units of oxytocin on the oxytocin infusion and re assessment of the mother VALLAJERA, MD EXAM - AUG
present IVF, you know that you have to stop the for any problems, option C only tells that the heart (TOP 8 - FEB 2015
oxytocin if: rate of the fetus decreases just before each 2015 MED
A. Contractions persist at a frequency more than 5 contraction and is a sign of head compression BOARDS;
in a 10 minute period or 7 in a 15 minute period against the birth canal TOPNOTCH MD
B. There is a persistent non-reassuring fetal heart FROM
rate pattern PERPETUAL
C. There are early decelerations associated with BINAN)
uterine contractions
D. A and B only
E. All of the above
153 Which among the following is a virilizing tumor Sertoli Leydig cell tumor resembles fetal testes and EDWARD HARRY MIDTERM 2
A. Granulosa - Theca cell tumor secrete testosterone causing virilization VALLAJERA, MD EXAM - AUG
B. Sertoli - Leydig cell tumor (TOP 8 - FEB 2015
C. Endodermal sinus tumor 2015 MED
D. Choriocarcinoma BOARDS;
E. Clear cell adenocarcinoma TOPNOTCH MD
FROM
PERPETUAL
BINAN)
154 MGB, a 16 year old was brought by her mother due Pregnancy is still the most common cause of EDWARD HARRY MIDTERM 2
to cessation of menses, you asked her menstrual secondary amenorrhea and should be ruled out VALLAJERA, MD EXAM - AUG
history and noted that she started her menses at first in any woman of the reproductive age (TOP 8 - FEB 2015
12 years old and was subsequently regular presenting with amenorrhea 2015 MED
occuring every 28-30 days but suddenly ceased for BOARDS;
3 months. All other physical examination findings TOPNOTCH MD
were normal, you noted that the most common FROM
cause of secondary amenorrhea is PERPETUAL
A. Mayer-Rokitansky-Kuster-Hauser syndrome BINAN)
B. Ashermann's syndrome
C. Sheehan's syndrome
D. Pregnancy
E. Turner's syndrome
155 KCL, a 27 year old G2P2 post NSD for 2 days came Gestational hypertension is BP elevation from a EDWARD HARRY MIDTERM 2
in to your clinic due to dizziness, all other physical previously normotensive patient detected AFTER VALLAJERA, MD EXAM - AUG
exam parameters were normal except for a BP of 20 weeks AOG and should resolve by 12 weeks post (TOP 8 - FEB 2015
160/100, you asked if she had prior BP partum (Williams Obstetrics, 24th ed.) 2015 MED
measurements like these and the patient said she BOARDS;
had elevated BP measurements before but she was TOPNOTCH MD
asymptomatic, you know that to fit the diagnosis of FROM
gestational hypertension, her elevated BP must PERPETUAL
have been elevated after 20 wks AOG and up to? BINAN)
A. 12 weeks post partum
B. 6 weeks post partum
C. 8 weeks post partum
D. 4 weeks post partum
E. None of the above
156 MR, a 32 year old G2P1 at 10 weeks AOG came in This is a case of incomplete H-mole as the EDWARD HARRY MIDTERM 2
due to vaginal bleeding, upon PE, her BP was ultrasound pattern is Swiss cheese, snow storm VALLAJERA, MD EXAM - AUG
140/90, you noted on ultrasound a Swiss cheese pattern is seen in complete H-mole. Suction D & C is (TOP 8 - FEB 2015
pattern, what is the treatment of choice the treatment of choice 2015 MED
A. Metronidazole BOARDS;
B. Suction dilation and curettage TOPNOTCH MD
C. None, observe and come back after 2 weeks FROM
D. Tocolysis PERPETUAL
E. None of the above BINAN)
157 MR, same patient as above asked if she could get The patient diagnosed with molar pregnancy EDWARD HARRY MIDTERM 2
pregnant, you said that while under treatment, she undergoing treatment and observation should not VALLAJERA, MD EXAM - AUG
could not get pregnant for how long? get pregnant for 1 year if she underwent (TOP 8 - FEB 2015
A. 6 months chemotherapy for GTD to prevent teratogenic 2015 MED
B. 9 months effects to the fetus and 6 months if she was only put BOARDS;
C. 1 year on hormonal contraception to prevent pregnancy. TOPNOTCH MD
TOPNOTCH MEDICAL BOARD PREP OBSTETRICS-GYNECOLOGY SUPEREXAM Page 20 of 84
For inquiries visit www.topnotchboardprep.com.ph or email us at topnotchmedicalboardprep@gmail.com
TOPNOTCH MEDICAL BOARD PREP OBSTETRICS-GYNECOLOGY SUPEREXAM
For inquiries visit www.topnotchboardprep.com.ph or email us at topnotchmedicalboardprep@gmail.com
Ite QUESTION EXPLANATION AUTHOR TOPNOTCH
m # EXAM
D. 1 1/2 year FROM
E. None of the above PERPETUAL
BINAN)

158 MS, a 31 year old G1P1 came in to your clinic due This is a case of bacterial vaginosis and the EDWARD HARRY MIDTERM 2
to malodorous vaginal discharge, with a microscopic finding is the description of clue cells VALLAJERA, MD EXAM - AUG
characteristic fishy odor, squamous cells seen in bacterial vaginosis (TOP 8 - FEB 2015
surrounded by lots of bacteria without any 2015 MED
inflammatory cells and vaginal pH >4.5, what is the BOARDS;
treatment of choice? TOPNOTCH MD
A. Metronidazole FROM
B. Clindamycin PERPETUAL
C. Itraconazole BINAN)
D. Ciprofloxacin
E. Cefuroxime
159 MR, a 32 year old G2P1 at 10 weeks AOG came in This is a case of incomplete H-mole as the EDWARD HARRY MIDTERM 2
due to vaginal bleeding, upon PE, her BP was ultrasound pattern is Swiss cheese, snow storm VALLAJERA, MD EXAM - AUG
140/90, you noted on ultrasound a Swiss cheese pattern is seen in complete H-mole. Suction D & C is (TOP 8 - FEB 2015
pattern, what is the treatment of choice the treatment of choice 2015 MED
A. Metronidazole BOARDS;
B. Suction dilation and curettage TOPNOTCH MD
C. None, observe and come back after 2 weeks FROM
D. Tocolysis PERPETUAL
E. None of the above BINAN)
160 Hysteroscopy is best performed on what day of the The best answer is a week after menstruation. In EDWARD HARRY MIDTERM 2
menstrual cycle? this case it is in the 8th day of the menstrual cycle. VALLAJERA, MD EXAM - AUG
A. 4th day (TOP 8 - FEB 2015
B. 8th day 2015 MED
C. 15th day BOARDS;
D. 18th day TOPNOTCH MD
E. 20th day FROM
PERPETUAL
BINAN)
161 Hydatidiform mole is a delicate, friable mass of other choices are characteristic of partial moles. HAROLD JAY S. MIDTERM 3
thin-walled translucent, cystic grapelike structures. Other characteristics of complete mole are: 46 XX BAYTEC, MD EXAM - AUG
Which of the following is a characteristic of a or 46 XY; embryo fetus is absent; usually presents (TOP 10 - FEB 2015
complete mole? as molar gestation.. Partial moles are usually small 2015 MED
A. Can have 69 XXX or XXY for dates. BOARDS;
B. Embryo-fetal parts are often present TOPNOTCH MD
C. often present as missed abortion FROM FEU)
D. 50% of cases have large uterine size
E. none of the above
162 The following teratogens are correctly matched to Fluoroquinolone can cause cartilage damage HAROLD JAY S. MIDTERM 3
their effect to the fetusEXCEPT: BAYTEC, MD EXAM - AUG
A. ACE inhibitor: fetal renal damage (TOP 10 - FEB 2015
B. Lithium: ebstein anomaly 2015 MED
C. Fluoroquinolone: anemia BOARDS;
D. Isotretinoin: craniofacial malformation TOPNOTCH MD
E. Streptomycin:ototoxicity FROM FEU)
163 A post partum patient suddenly had cardiac arrest dilated cardiomyopathy is associated with post HAROLD JAY S. MIDTERM 3
which eventually lead to her demise. Biopsy to her partum cardiac arrest BAYTEC, MD EXAM - AUG
heart showed cardiomyopathy. Which among the (TOP 10 - FEB 2015
following types of cardiomyopathy is most likely 2015 MED
the cause? BOARDS;
A. dilated TOPNOTCH MD
B. restrictive FROM FEU)
C. hypertrophic
D. anaplastic
E. metaplastic
164 Where can you find the widest portion of the HAROLD JAY S. MIDTERM 3
fallopian tube? BAYTEC, MD EXAM - AUG
A. infundibulum (TOP 10 - FEB 2015
B. ampulla 2015 MED
C. isthmus BOARDS;
D. interstitial TOPNOTCH MD
E. Intramuscular FROM FEU)
165 In a menstrual cycle of a normal woman, the post HAROLD JAY S. MIDTERM 3
ovulatory phase is _________. BAYTEC, MD EXAM - AUG
A. Constantly 12 days (TOP 10 - FEB 2015
B. Usually 12 days but variable 2015 MED
C. Constantly 14 days BOARDS;
D. Usually 14 days but variably TOPNOTCH MD
E. Constantly at 16 days FROM FEU)
166 Which among the following is considered a HAROLD JAY S. MIDTERM 3
definitive sign of pregnancy? BAYTEC, MD EXAM - AUG
A. Positive pregnancy test (TOP 10 - FEB 2015
B. Breast changes 2015 MED
C. Cessation of menses BOARDS;
D. Braxton hicks contraction TOPNOTCH MD
E. Perception of fetal movement by a midwife FROM FEU)
167 This is the bluish discoloration of the cervix, hegar is softening of the uterus. Goodels is softening HAROLD JAY S. MIDTERM 3
vagina, and labia that is caused by estrogen which of vaginal portion of the cervix. Spalding and BAYTEC, MD EXAM - AUG
results in venous congestion. robert's sign are radiographic evidences of fetal (TOP 10 - FEB 2015
A. Chadwick Sign death 2015 MED
B. Hegar's Sign BOARDS;
TOPNOTCH MEDICAL BOARD PREP OBSTETRICS-GYNECOLOGY SUPEREXAM Page 21 of 84
For inquiries visit www.topnotchboardprep.com.ph or email us at topnotchmedicalboardprep@gmail.com
TOPNOTCH MEDICAL BOARD PREP OBSTETRICS-GYNECOLOGY SUPEREXAM
For inquiries visit www.topnotchboardprep.com.ph or email us at topnotchmedicalboardprep@gmail.com
Ite QUESTION EXPLANATION AUTHOR TOPNOTCH
m # EXAM
C. Goodel's sign TOPNOTCH MD
D. Spalding sign FROM FEU)
E. Robert's sign

168 In a multiparous mother, she may first perceive for nulliparous it is 18-20 weeks HAROLD JAY S. MIDTERM 3
fetal movements at ______ weeeks AOG. BAYTEC, MD EXAM - AUG
A. 14-16 weeks (TOP 10 - FEB 2015
B. 16-18 weeks 2015 MED
C. 18-20 weeks BOARDS;
D. 20-22 weeks TOPNOTCH MD
E. 22-24 weeks FROM FEU)
169 Which of the following vaccines can be given to a SIMILAR TO PREVIOUS BOARD EXAM HAROLD JAY S. MIDTERM 3
pregnant patient? CONCEPT/PRINCIPLE BAYTEC, MD EXAM - AUG
A. measles (TOP 10 - FEB 2015
B. MMR 2015 MED
C. Varicella BOARDS;
D. Vaccinia TOPNOTCH MD
E. Meningococcus FROM FEU)
170 The following are all absolute contraindications for A is only a relative contraindication HAROLD JAY S. MIDTERM 3
Aerobic exercise during pregnancy EXCEPT: BAYTEC, MD EXAM - AUG
A. Poorly controlled seizure disorder (TOP 10 - FEB 2015
B. Incompetent cervix 2015 MED
C. Restrictive lung disease BOARDS;
D. Placenta previa after 26 weeks TOPNOTCH MD
E. Preterm labor during the current pregnancy FROM FEU)
171 Which among the following Anti TB drugs is this is contraindicated because of ototoxicity to the HAROLD JAY S. MIDTERM 3
contraindicated in pregnant women? fetus. SIMILAR TO PREVIOUS BOARD EXAM BAYTEC, MD EXAM - AUG
A. ethambutol CONCEPT/PRINCIPLE (TOP 10 - FEB 2015
B. rifampicin 2015 MED
C. ethambutol BOARDS;
D. streptomycin TOPNOTCH MD
E. All of the above FROM FEU)
172 Which of the following will constitute the A and B are posterior boundaries. C is the lateral HAROLD JAY S. MIDTERM 3
boundaries of the pelvic inlet? boundary. D is the anterior boundary BAYTEC, MD EXAM - AUG
A. Sacral promontory (TOP 10 - FEB 2015
B. Ala of the sacrum 2015 MED
C. Linea terminalis BOARDS;
D. Symphysis pubis TOPNOTCH MD
E. All of the above FROM FEU)
173 The inter-spinous diameter of the midpelvis should Williams 23rd edition chapter of Labor and HAROLD JAY S. MIDTERM 3
be at least how many centimeters for it to be Delivery page 472, “there is a reason to suspect BAYTEC, MD EXAM - AUG
adequate for vaginal birth? midpelvic contraction whenever the interspinous (TOP 10 - FEB 2015
A. 9 diameter is less than 10cm.” I can also consider B 2015 MED
B. 9.5 9.5 since it appeared it anatomy topnotch day 3 BOARDS;
C. 10 handout page 16. TOPNOTCH MD
D. 11 FROM FEU)
E. 12
174 Implantation usually takes place how many days usually takes place on the 6th or 7th day after HAROLD JAY S. MIDTERM 3
after fertilization? fertilization. Williams 23rd edition page 48 BAYTEC, MD EXAM - AUG
A. 5 (TOP 10 - FEB 2015
B. 7 2015 MED
C. 9 BOARDS;
D. 10 TOPNOTCH MD
E. 12 FROM FEU)
175 During prenatal check up, you palpated the fundus OB Williams 23rd edition page 79 and OB topnotch HAROLD JAY S. MIDTERM 3
of the mother to be just above the symphysis pubis. handout page 19. at 12 weeks uterus is just BAYTEC, MD EXAM - AUG
Approximately how many weeks is the AOG? palpable above the symphysis pubis (TOP 10 - FEB 2015
A. 12 2015 MED
B. 14 BOARDS;
C. 16 TOPNOTCH MD
D. 18 FROM FEU)
E. 20
176 This plane corresponds to the smallest HAROLD JAY S. MIDTERM 3
circumference of the fetal head BAYTEC, MD EXAM - AUG
A. Biparietal diameter (TOP 10 - FEB 2015
B. occipitomental 2015 MED
C. occipitofrontal BOARDS;
D. suboccipitobregmatic TOPNOTCH MD
E. Bitemporal FROM FEU)
177 A patient came in for consult at your clinic due to HAROLD JAY S. MIDTERM 3
primary amenorrhea associated with anosmia. BAYTEC, MD EXAM - AUG
Work up showed hypogonadotropic hypogonadism (TOP 10 - FEB 2015
due to deficiency in GnRH. What do you call this 2015 MED
disease? BOARDS;
A. Asherman's syndrome TOPNOTCH MD
B. Sheehan's syndrome FROM FEU)
C. Kallman syndrome
D. Turner syndrome
E. MRKH syndrome
178 Uterine synechiae is more commonly known as: SIMILAR TO PREVIOUS BOARD EXAM HAROLD JAY S. MIDTERM 3
A. Asherman's syndrome CONCEPT/PRINCIPLE BAYTEC, MD EXAM - AUG
B. Sheehan's syndrome (TOP 10 - FEB 2015
C. Kallman syndrome 2015 MED
D. Turner syndrome BOARDS;
E. MRKH syndrome TOPNOTCH MD
FROM FEU)
TOPNOTCH MEDICAL BOARD PREP OBSTETRICS-GYNECOLOGY SUPEREXAM Page 22 of 84
For inquiries visit www.topnotchboardprep.com.ph or email us at topnotchmedicalboardprep@gmail.com
TOPNOTCH MEDICAL BOARD PREP OBSTETRICS-GYNECOLOGY SUPEREXAM
For inquiries visit www.topnotchboardprep.com.ph or email us at topnotchmedicalboardprep@gmail.com
Ite QUESTION EXPLANATION AUTHOR TOPNOTCH
m # EXAM
179 A G1P1 (1001) came in for consult due to The question is asking for the MOST likely diagnosis HAROLD JAY S. MIDTERM 3
secondary amenorrhea. Her last delivery was or the primary diagnosis. Sheehan's syndrome can BAYTEC, MD EXAM - AUG
complicated by severe post partum bleeding due to be one of your differentials. However, this is less (TOP 10 - FEB 2015
retained placental fragments which prompted the likely the cause of the secondary ammenorhea as 2015 MED
attending obstetrician to do endometrial curettage. compared to Asherman's syndrome since BOARDS;
After that, the patient had no longer had any amenorrhea is the only symptom present in the TOPNOTCH MD
mentruation. Which among the following is most case. A patient with Sheehan's syndrome should FROM FEU)
likely the cause of the secondary amenorrhea? also have other signs and symptoms pertaining to
A. Asherman's syndrome hormonal imbalance aside from amenorrhea.
B. Sheehan's syndrome SIMILAR TO PREVIOUS BOARD EXAM
C. Kallman syndrome CONCEPT/PRINCIPLE
D. Turner syndrome
E. MRKH syndrome
180 To confirm the diagnosis of the answer in number SIMILAR TO PREVIOUS BOARD EXAM HAROLD JAY S. MIDTERM 3
199, this procedure should be done. CONCEPT/PRINCIPLE BAYTEC, MD EXAM - AUG
A. Transvaginal ultrasound (TOP 10 - FEB 2015
B. Pelvic MRI 2015 MED
C. Pelvic CT scan with contrast BOARDS;
D. hysteroscopy TOPNOTCH MD
E. Progesterone-estrogen challenge test FROM FEU)
181 A G1P0 patient at 36 weeks AOG had a biophysical A biophysical score of 8-10 is normal. JEAN PAOLO M. FINAL EXAM -
score of 8 (NST, Fetal movement, Fetal muscle DELFINO, MD AUG 2015
tone, AFV have 2 points each while Fetal breathing (TOP 10 - FEB
had 0 point). What should you tell the patient 2015 MED
regarding the result of the test? BOARDS;
A. Repeat BPP should be done within 24 hours TOPNOTCH MD
B. Labor should be induced FROM FATIMA)
C. The results are normal and the patient can go
home
D. Patient should undergo emergent cesarean
section
E. The results are abnormal, admit the patient for
tocolysis
182 A 22 year old nulligravid, amenorrheic for 8 weeks, Ectopic pregnancy is treated medically if the JEAN PAOLO M. FINAL EXAM -
came in due to right lower quadrant abdominal following criteria is present DELFINO, MD AUG 2015
pain with vaginal spotting. Pelvic exam shows right <6 weeks AOG (TOP 10 - FEB
adnexal tenderness.Pregnancy test is positive. hCG tubal mass <3.5cm 2015 MED
level is 3500. Ultrasound revealed no gestational absence of fetal heart motion BOARDS;
sac. This condition is treated medically if the hCG <15000 TOPNOTCH MD
following criteria is present except unruptured FROM FATIMA)
A. >6 weeks AOG
B. tubal mass <3.5cm
C. absence of fetal heart motion
D. hCG <15000
E. unruptured
183 What condition most likely predispose a patient to Any factor delaying transit of the ovum through the JEAN PAOLO M. FINAL EXAM -
develop ectopic pregnancy? fallopian tube may predispose a patient to ectopic DELFINO, MD AUG 2015
A. Previous cervical conization pregnancy. The major predisposing factor is PID. (TOP 10 - FEB
B. Pelvic inflammatory disease 2015 MED
C. Use of IUD BOARDS;
D. Induction of ovulation TOPNOTCH MD
E. Exposure in utero to DES FROM FATIMA)
184 24 year old G1P0 at 35 weeks AOG is diagnosed JEAN PAOLO M. FINAL EXAM -
with urinary tract infection. Which of the following DELFINO, MD AUG 2015
is the best choice for treatment? (TOP 10 - FEB
A. Cephalosporin 2015 MED
B. Tetracycline BOARDS;
C. Sulfonamide TOPNOTCH MD
D. Chloramphenicol FROM FATIMA)
E. Erythromycin
185 A patient with cervical cancer underwent Cervical cancer with hydronephrosis is Stage III. JEAN PAOLO M. FINAL EXAM -
intravenous pyelogram study which showed Management for cervical cancer: Extrafascial DELFINO, MD AUG 2015
hydronephrosis. How do you manage the patient at hysterectomy for Stage IA to Stage IIA; (TOP 10 - FEB
this stage? Chemoradiation for highly advance carcinoma 2015 MED
A. Extrafascial hysterectomy BOARDS;
B. Chemoradiation TOPNOTCH MD
C. TAHBSO FROM FATIMA)
D. USO
E. TAHBSO + chemoradiation
186 Diagnostic work up for left lower quadrant pain in Germ cell tumors are the most common ovarian JEAN PAOLO M. FINAL EXAM -
an 8 year old girl revealed an ovarian neoplasm. neoplasm in young age group DELFINO, MD AUG 2015
What is the most common ovarian tumor seen in (TOP 10 - FEB
this type of patient? 2015 MED
A. Papillary serous epithelial BOARDS;
B. Fibrosarcoma TOPNOTCH MD
C. Germ cell tumor FROM FATIMA)
D. Brenner tumor
E. Sarcoma botryoides
187 Mother brought her daughter for consult because Diagnosis is precocious puberty. The most common JEAN PAOLO M. FINAL EXAM -
of early onset menstruation. The girl was noted to cause is idiopathic. DELFINO, MD AUG 2015
have thelarche at 7 years old, adrenarche at 8 years (TOP 10 - FEB
old and menarche at 9 years old. What is the most 2015 MED
common cause of this condition in girls? BOARDS;
A. CNS tumor TOPNOTCH MD
B. Hypothyroidism FROM FATIMA)
C. McCune-Albright syndrome
TOPNOTCH MEDICAL BOARD PREP OBSTETRICS-GYNECOLOGY SUPEREXAM Page 23 of 84
For inquiries visit www.topnotchboardprep.com.ph or email us at topnotchmedicalboardprep@gmail.com
TOPNOTCH MEDICAL BOARD PREP OBSTETRICS-GYNECOLOGY SUPEREXAM
For inquiries visit www.topnotchboardprep.com.ph or email us at topnotchmedicalboardprep@gmail.com
Ite QUESTION EXPLANATION AUTHOR TOPNOTCH
m # EXAM
D. Gonadal tumors
E. Idiopathic

188 A G3P2 (2012), underwent completion curettage A negative estrogen-progesterone challenge test is JEAN PAOLO M. FINAL EXAM -
for incomplete abortion 8 months ago, is being most likely caused by an outflow tract obstruction. DELFINO, MD AUG 2015
investigated for secondary amenorrhea. Pregnancy In this case, because of history of completion (TOP 10 - FEB
test is negative. There is no withdrawal bleeding in curettage for incomplete abortion, the main 2015 MED
progesterone challenge test and in estrogen- consideration in the diagnosis is Asherman BOARDS;
progesterone challenge test. What should you syndrome/endometrial adhesions.Diagnosis is TOPNOTCH MD
request next? done through Hysterosalphingogram FROM FATIMA)
A. FSH level
B. Endometrial biopsy
C. Hysterosalphingogram
D. Prolactin level
E. TSH determination
189 A Pap-smear result of a 28 year old patient Inflammation affects the cytologic study so treat the JEAN PAOLO M. FINAL EXAM -
revealed inflammation with atypia. What would inflammation first then repeat pap smear after DELFINO, MD AUG 2015
you advise the patient? treatment. (TOP 10 - FEB
A. Colposcopy and biopsy 2015 MED
B. Repeat pap-smear after 3 months BOARDS;
C. Treat inflammation then repeat pap-smear TOPNOTCH MD
D. observation FROM FATIMA)
E. None of the above
190 WHO recommends low risk pregnant patients to WHO recommends at least 4 prenatal visits for JEAN PAOLO M. FINAL EXAM -
have at least how many pre-natal check ups? uncomplicated low risk pregnant patients. 1st visit: DELFINO, MD AUG 2015
A. 7 8-12 weeks, 2nd visit: 24-26 weeks, 3rd visit: 32 (TOP 10 - FEB
B. 6 weeks, 4th visit:36-38 weeks 2015 MED
C. 5 BOARDS;
D. 4 TOPNOTCH MD
E. 8 FROM FATIMA)
191 28 year old sexually active female, nulligravid, The clinical symptoms and signs of pelvic JEAN PAOLO M. FINAL EXAM -
presented with fever, nausea, vomiting and severe tuberculosis are similar to the chronic sequelae of DELFINO, MD AUG 2015
pelvic pain. Pelvic exam is positive for adnexal nontuberculous acute PID. The predominant (TOP 10 - FEB
tenderness. Diagnosis was PID and she was presentations of this chronic infection are infertility 2015 MED
managed accordingly. After several treatment and abnormal uterine bleeding. Pelvic tuberculosis BOARDS;
regimens, symptoms still recur and there were may not be diagnosed until laparotomy or TOPNOTCH MD
noted infertility and abnormal uterine bleeding. celiotomy, when the characteristic changes may be FROM FATIMA)
Laparoscopy was done and revealed a "tobacco visualized. The distal ends of the oviduct remain
pouch appearance" of the oviducts. What is the everted, producing a “tobacco pouch” appearance.
diagnosis?
A. Gonococcal PID
B. Pelvic TB
C. Non-gonococcal PID
D. Actinomyces infection
E. Endometriosis
192 Patient is G1P0 at 36 weeks AOG, had sudden gush Steroid are given if AOG is <34 weeks. JEAN PAOLO M. FINAL EXAM -
of fluid in the vagina 2 hours prior to consult. DELFINO, MD AUG 2015
Patient is now in labor. Which of the following is (TOP 10 - FEB
not included in the management of this patient? 2015 MED
A. GBS prophylaxis BOARDS;
B. Steroids to hasten lung maturity TOPNOTCH MD
C. Deliver the baby FROM FATIMA)
D. All of the above
E. None of the above
193 What type of breech has the highest incidence of Incomplete or footling breech has the highest JEAN PAOLO M. FINAL EXAM -
cord prolapse? incidence of cord prolapse. Lowest incidence in DELFINO, MD AUG 2015
A. Complete frank breech (TOP 10 - FEB
B. Footling 2015 MED
C. Frank BOARDS;
D. B and C TOPNOTCH MD
E. A and B FROM FATIMA)
194 What is the recommended regimen for Penicillin is the recommended drug. The other JEAN PAOLO M. FINAL EXAM -
intrapartum antimicrobial prophylaxis for drugs listed in the choices are only alternative to DELFINO, MD AUG 2015
perinatal Group B Streptococcal disease? penicillin. (TOP 10 - FEB
A. Ampicillin, 2 g IV initial dose, then 1 g IV every 2015 MED
4 hours or 2 g every 6 hours until delivery BOARDS;
B. Cefazolin, 2 g IV initial dose, then 1 g IV every 8 TOPNOTCH MD
hours until delivery FROM FATIMA)
C. Clindamycin, 900 mg IV every 8 hours until
delivery
D. Vancomycin, 1 g IV every 12 hours until
delivery
E. Penicillin G, 5 million units IV initial dose, then
2.5 million units IV every 4 hours until delivery
195 In chloroquine-susceptible areas, what is the Chloroquine prophylaxis is safe and well tolerated JEAN PAOLO M. FINAL EXAM -
prophylactic drug of choice for pregnant women? in pregnancy. It has been shown to decrease DELFINO, MD AUG 2015
A. Chloroquine placental infection from 20 down to 4 percent in (TOP 10 - FEB
B. Mefloquine asymptomatic infected women in areas without 2015 MED
C. Doxycycline chloroquine resistance. For travelers to areas with BOARDS;
D. Primaquine chloroquine-resistant P. falciparum, mefloquine is TOPNOTCH MD
E. Co-Artem currently the only chemoprophylaxis FROM FATIMA)
recommended. Primaquine and doxycycline are
contraindicated in pregnancy

TOPNOTCH MEDICAL BOARD PREP OBSTETRICS-GYNECOLOGY SUPEREXAM Page 24 of 84


For inquiries visit www.topnotchboardprep.com.ph or email us at topnotchmedicalboardprep@gmail.com
TOPNOTCH MEDICAL BOARD PREP OBSTETRICS-GYNECOLOGY SUPEREXAM
For inquiries visit www.topnotchboardprep.com.ph or email us at topnotchmedicalboardprep@gmail.com
Ite QUESTION EXPLANATION AUTHOR TOPNOTCH
m # EXAM
196 Propylthiouracil is the drug of choice for PTU is the drug of choice for thyrotoxicosis in JEAN PAOLO M. FINAL EXAM -
thyrotoxicosis in pregnancy because pregnancy because it is highly protein bound, thus DELFINO, MD AUG 2015
A. It has a short half-life preventing from crossing the placental barrier. (TOP 10 - FEB
B. It is highly protein bound 2015 MED
C. It blocks peripheral conversion of T4 to T3 BOARDS;
D. It has less adverse effects TOPNOTCH MD
E. All of the above FROM FATIMA)
197 At what AOG does hCG levels start to decline? hCG secretion starts at 8 days, peaks at 8 weeks, JEAN PAOLO M. FINAL EXAM -
A. 8-10 weeks and starts to decline at 18-20 weeks AOG. DELFINO, MD AUG 2015
B. 12-14 weeks Mnemonics: HCG = 8CG (TOP 10 - FEB
C. 22-24 weeks 2015 MED
D. 18-20 weeks BOARDS;
E. 14-16 weeks TOPNOTCH MD
FROM FATIMA)
198 8 days post partum, the mother developed fever. post-partum fever most likely causes: within 24 JEAN PAOLO M. FINAL EXAM -
What is the most common cause of post-partum hours- atelectasis; 1 day- UTI; 2-3 days- DELFINO, MD AUG 2015
fever at this time? endometritis; 4-5 days- surgical site infection; 5-6 (TOP 10 - FEB
A. endometritis days- DVT; 7-21 days- mastitis. 2015 MED
B. Surgical site infection BOARDS;
C. Mastitis TOPNOTCH MD
D. DVT FROM FATIMA)
E. UTI
199 Implantation happens 1 week post-conception. MC site of implantation- posterosuperior wall JEAN PAOLO M. FINAL EXAM -
What is the most common location of implantation DELFINO, MD AUG 2015
in the uterine wall? (TOP 10 - FEB
A. posterosuperior 2015 MED
B. anterosuperior BOARDS;
C. posterolateral TOPNOTCH MD
D. anteroinferior FROM FATIMA)
E. Inferolateral
200 Pseudomenopause state is achieved by giving OCP is used to achieve pseudopregnancy state, JEAN PAOLO M. FINAL EXAM -
A. OCP single daily monophasic for 6-9 months Danazol for pseudomenopause and GnRH agonist DELFINO, MD AUG 2015
B. Danazol 400-800mg/day for 6 months (Leuprolide) for medical oophorectomy. (TOP 10 - FEB
C. Leuprolide 3.75mg IM/month Clomiphene is used for ovulation induction. 2015 MED
D. Clomiphene citrate 250mg/day BOARDS;
E. All of the above TOPNOTCH MD
FROM FATIMA)
201 The Arias-Stella reaction refers to: Pregnancy is associated with both endocarvical GRACE ARVIOLA, DIAGNOSTIC
A. Whitening of the vulva after application of gland hyperplasia and and hypersecretory MD (TOP 3 - AUG EXAM - FEB
acetoacetate appearance which makes the identification of of 2014 MED 2015
B. Poor Pap smear quality during pregnancy atypical glandular cells on Pap smear particularly BOARDS;
C. Ferning of cervical mucus observed in preterm difficult. TOPNOTCH MD)
labor
D. Violin string-like adhesions in perihepatitis
E. Increased intensity of uterine contraction with
cervical motion
202 Amnioinfusion finds clinical use in the setting of: Transvaginal amnioinfusion has been extended into GRACE ARVIOLA, DIAGNOSTIC
A. Fetal head compression three clinical areas: Treatment of variable or MD (TOP 3 - AUG EXAM - FEB
B. Uteroplacental insufficiency prolonged decelerations, prophylaxis for women 2014 MED 2015
C. Cord compression with oligohydramnios, as with prolonged rupture of BOARDS;
D. Pre-eclampsia membranes, and attempts to dilute or wash out TOPNOTCH MD)
E. Chorioamnionitis thick meconium.
203 Which hormone is reponsible for uterine Estrogen for uterine activation. GRACE ARVIOLA, DIAGNOSTIC
quiescence? MD (TOP 3 - AUG EXAM - FEB
A. Estrogen 2014 MED 2015
B. Progesterone BOARDS;
C. Prolactin TOPNOTCH MD)
D. Oxytocin
E. Cortisol
204 A 23 year-old nullipara is admitted at the labor Arrest of descent is defined as no change in fetal GRACE ARVIOLA, DIAGNOSTIC
room at 10:00 o'clock A.M. IE findings are: 4 cm descent for 1 hour. MD (TOP 3 - AUG EXAM - FEB
dilated, 70% effaced, cephalic, station -2, intact bag 2014 MED 2015
of water. Oxytocin was given. Uterine contractions BOARDS;
are strong and regular. At 12:00 o'clock P.M., IE TOPNOTCH MD)
findings are: 7 cm dilated, 80% effaced, cephalic,
station -2, ruptured bag of water. Is this patient
experiencing normal labor? Why or why not?
A. Yes, this is normal labor. Uterine contractions
are regular and strong.
B. Yes, this is normal labor. Cervix is 7 cm dilated,
80% effaced.
C. Yes, this is normal labor. Bag of water has
ruptured.
D. No, this is abnormal labor. There is no change
in descent.
E. No, this is abnormal labor. Bag of water has
already ruptured.
205 A 27 year-old G3P2(2002) at 34 weeks and 5 days For patients with preterm ruptured membranes at GRACE ARVIOLA, DIAGNOSTIC
age of gestation presents to the emergency >34 weeks AOG, proceed to delivery usually by MD (TOP 3 - AUG EXAM - FEB
department due to labor pains. IE findings are: 4 induction of labor. GBS prophylaxis is 2014 MED 2015
cm dilated, 70% effaced, cephalic, station -2, recommended. BOARDS;
ruptured bag of water. What is the best course of TOPNOTCH MD)
action?
A. Admit and administer tocolytics.
B. Induction of labor.
C. Cesarean section.
TOPNOTCH MEDICAL BOARD PREP OBSTETRICS-GYNECOLOGY SUPEREXAM Page 25 of 84
For inquiries visit www.topnotchboardprep.com.ph or email us at topnotchmedicalboardprep@gmail.com
TOPNOTCH MEDICAL BOARD PREP OBSTETRICS-GYNECOLOGY SUPEREXAM
For inquiries visit www.topnotchboardprep.com.ph or email us at topnotchmedicalboardprep@gmail.com
Ite QUESTION EXPLANATION AUTHOR TOPNOTCH
m # EXAM
D. Perform transvaginal ultrasound.
E. Send the patient home.

206 A 34 year old multigravid at 35 weeks and 3 days Some of the more common causes of transverse lie GRACE ARVIOLA, DIAGNOSTIC
AOG presents for a prenatal check up. After include: high parity, preterm fetus, placenta previa, MD (TOP 3 - AUG EXAM - FEB
performing the Leopold's maneuver, you abnormal uterine anatomy, hydramnios, and 2014 MED 2015
discovered that the fetus is in a transverse lie. If contracted pelvis. BOARDS;
this abnormal lie is due to a placental abnormality, TOPNOTCH MD)
which condition is most likely present?
A. Abruptio placenta
B. Placenta accreta
C. Placenta previa
D. Battledore placenta
E. Circumvallate placenta
207 The results of a biophysical profile done to a 40 BPS score is 6 with abnormal AFV. This translates GRACE ARVIOLA, DIAGNOSTIC
year old G1P0 at 32 weeks and 4 days AOG with to possible fetal asphyxia. MD (TOP 3 - AUG EXAM - FEB
suspected intrauterine growth restriction are as 2014 MED 2015
follows: Reactive NST, AFV 1 cm, 2 discrete body BOARDS;
movements, 1 episode of breathing lasting >30 TOPNOTCH MD)
seconds, 2 episodes of extension with return to
flexion. What should you do next?
A. No fetal indication for intervention.
B. Repeat test after 1 week.
C. Repeat test twice weekly.
D. Repeat test within 24 hours.
E. Deliver.
208 A 25 year old G1P0 at 37 weeks and 6 days of AOG The dosages for severe preeclampsia are the same GRACE ARVIOLA, DIAGNOSTIC
presents to the emergency department due to as for eclampsia. Because labor and delivery is a MD (TOP 3 - AUG EXAM - FEB
labor pains. BP was 200/120. Dipstick analysis of more likely time for convulsions to develop, women 2014 MED 2015
urine revelaed 3+ proteinuria. Patient complains of with pre-eclampsia-eclampsia usually are given BOARDS;
blurred vision and severe epigastric pain. How magnesium sulfate during labor and for 24 hours TOPNOTCH MD)
should magnesium sulfate be given for this postpartum.
patient?
A. 4 grams IV bolus followed by 5 gm IM on each
buttock
B. 5 grams IV bolus followed by 4 gm IM on each
buttock
C. 4 grams IM on each buttock followed by 5 gm
IV bolus
D. 5 grams IM on each buttock followed by 4 gm
IV bolus
E. None of the above
209 The crown-rump length is compatible with the age Up to 12 weeks, the crown-rump length is GRACE ARVIOLA, DIAGNOSTIC
of gestation up until how many weeks? predictive of gestational age within 4 days. MD (TOP 3 - AUG EXAM - FEB
A. 4 weeks 2014 MED 2015
B. 8 weeks BOARDS;
C. 12 weeks TOPNOTCH MD)
D. 28 weeks
E. 32 weeks
210 A 32 year-old G3P3 (3003) delivered via normal Clindamycin 900 mg + gentamicin 1.5 mg/kg q8h IV GRACE ARVIOLA, DIAGNOSTIC
spontaneous delivery a healthy baby boy. On her MD (TOP 3 - AUG EXAM - FEB
third postpartum day, the patient had febrile 2014 MED 2015
episodes at 39 degree Celsius accompanied by BOARDS;
chills. She complains of abdominal pain. On TOPNOTCH MD)
bimanual examination, parametrial tenderness is
elicited. Which among the following antimicrobial
regimens is considered the gold standard for this
patient's condition?
A. Clindamycin + aztreonam
B. Clindamycin + gentamicin
C. Imipenem + cilastatin
D. Extended spectrum penicillins
E. Extended spectrum cephalosporins
211 A 28 year-old G3P2 (2002) at 38 weeks and 4 days Any previous vaginal delivery, either before or GRACE ARVIOLA, DIAGNOSTIC
AOG came in due to vaginal bleeding. Her first following a cesarean birth, significantly improves MD (TOP 3 - AUG EXAM - FEB
pregnancy was delivered via low-transverse the prognosis for a subsequent vaginal delivery 2014 MED 2015
cesarean section due to malpresentation. For the with either spontaneous or induced labor. BOARDS;
second pregnancy, she delivered vaginally a TOPNOTCH MD)
healthy baby girl. At present, IE findings are: 5 cm
dilated, 80% effaced, cephalic, station -2, intact bag
of water. Which factor makes this patient a
candidate for normal spontaenous delivery?
A. Young age of patient
B. Term pregnancy
C. The patient is in active phase of labor.
D. Prior vaginal delivery
E. Low parity
212 A 16 year-old presents with vaginal bleeding for This is a case of ectopic pregnancy. Patient is GRACE ARVIOLA, DIAGNOSTIC
two days. She estimates her last menstrual period hemodynamically unstable. Proceed to surgery. MD (TOP 3 - AUG EXAM - FEB
to be 3 months ago. Pregnany test is positive. 2014 MED 2015
There is crampy right lower quadrant abdominal BOARDS;
pain. On IE, the cervical os is closed but with TOPNOTCH MD)
adnexal tenderness on the right. The abdomen is
positive for direct and rebound tenderness. Vital

TOPNOTCH MEDICAL BOARD PREP OBSTETRICS-GYNECOLOGY SUPEREXAM Page 26 of 84


For inquiries visit www.topnotchboardprep.com.ph or email us at topnotchmedicalboardprep@gmail.com
TOPNOTCH MEDICAL BOARD PREP OBSTETRICS-GYNECOLOGY SUPEREXAM
For inquiries visit www.topnotchboardprep.com.ph or email us at topnotchmedicalboardprep@gmail.com
Ite QUESTION EXPLANATION AUTHOR TOPNOTCH
m # EXAM
signs are BP 80/50, HR 105, RR 22, afebrile. What
is the proper management?
A. Laparotomy
B. Perform TVS
C. D&C
D. Do serial b-HCG
E. Discharge patient
213 Women with the Rokitansky-Kuster-Hauser RKH syndome is congenital uterine agenesis. GRACE ARVIOLA, DIAGNOSTIC
syndrome have an increased risk of developing Congenital renal abnormalities occur in about one MD (TOP 3 - AUG EXAM - FEB
abnormalities of what other organ system? third of these individuals and skeletal 2014 MED 2015
A. Cardiovascular abnormalities in about 12%. BOARDS;
B. Gastrointestinal TOPNOTCH MD)
C. Renal
D. Endocrine
E. Nervous system
214 This drug achieves “pseudomenopause” among Danazol produces a hypoestrogenic and GRACE ARVIOLA, DIAGNOSTIC
patients with endometriosis. hyperandrogenic effect on steroid-sensitive end MD (TOP 3 - AUG EXAM - FEB
A. Estrogen organs. 2014 MED 2015
B. Clomiphene BOARDS;
C. Leuprolide TOPNOTCH MD)
D. Danazol
E. Goserelin
215 In cervical carcinoma, involvement of the kidneys GRACE ARVIOLA, DIAGNOSTIC
automatically places the condition at stage ______. MD (TOP 3 - AUG EXAM - FEB
A. IIA 2014 MED 2015
B. IIB BOARDS;
C. IIIA TOPNOTCH MD)
D. IIIB
E. IVA
216 A four-year old female patient came in due to This is adhesive vulvitis. Estrogen reaches a nadir GRACE ARVIOLA, DIAGNOSTIC
vulvar irching. On physical examination, you noted during this time, predisposing the nonestrogenized MD (TOP 3 - AUG EXAM - FEB
the labia minora to be fused with a translucent labia to denudation. 2014 MED 2015
vertical midline line. This condition usually affects BOARDS;
young girls between 2 and 6 years of age because TOPNOTCH MD)
of:
A. Poor hygiene
B. Hormonal factors
C. Bacterial infection
D. Genital trauma
E. Congenital defect
217 Which of the following is a hormonal correlate of Hormonal correlates of flush activity: Increased LH, GRACE ARVIOLA, DIAGNOSTIC
the hot flush of menopause? ACTH, B-endorphin. However, these occurences are MD (TOP 3 - AUG EXAM - FEB
A. Low estrogen thought to be epiphenomena that result as a 2014 MED 2015
B. Low progesterone consequence of the flush and not related to its BOARDS;
C. High FSH cause. TOPNOTCH MD)
D. High LH
E. Low ACTH
218 A 23 year-old G1P0 at 12 weeks AOG came in due This is a possible case of congenital rubella GRACE ARVIOLA, DIAGNOSTIC
to low-grade fever for three days accompanied by a syndrome. MD (TOP 3 - AUG EXAM - FEB
diffuse maculopapular rash that began on the face 2014 MED 2015
and has now spread to the trunk and extremities. BOARDS;
On physical examination, she has prominent TOPNOTCH MD)
occipital and post-auricular lymphadenopathy.
What is the single most common defect that her
fetus might develop?
A. Cataract
B. Patent ductus arteriosus
C. Sensorineural deafness
D. Microcephaly
E. Mental retardation
219 A 24 year-old G1P0 at 15 weeks AOG came in for High risk patients should be screened as soon as GRACE ARVIOLA, DIAGNOSTIC
her first prenatal check up. She has no subjective feasbile if one or more of the following are present: MD (TOP 3 - AUG EXAM - FEB
complaints. Her BMI is 30. She has no family severe obesity, strong family history of type 2 2014 MED 2015
history of diabetes. When should you screen for diabetes, previous history of GDM, impaired glucose BOARDS;
diabetes in this patient? metabolism or glucosuria. TOPNOTCH MD)
A. No need to screen. She is at low risk of
developing diabetes.
B. Screen her at 20-24 weeks AOG.
C. Screen her at 24-28 weeks AOG.
D. Screen her at 28-32 weeks AOG.
E. Perform screening now.
220 You are treating a 35 year-old female with syphilis. GRACE ARVIOLA, DIAGNOSTIC
She is concerned whether or not her partner MD (TOP 3 - AUG EXAM - FEB
should be treated. You tell her that in syphilis, 2014 MED 2015
individuals who are exposed within the ____ days BOARDS;
preceding the diagnosis in their sexual partners TOPNOTCH MD)
should be treated presumptively because they may
be infected even when seronegative.
A. 30 days
B. 60 days
C. 90 days
D. 120 days
E. 180 days

TOPNOTCH MEDICAL BOARD PREP OBSTETRICS-GYNECOLOGY SUPEREXAM Page 27 of 84


For inquiries visit www.topnotchboardprep.com.ph or email us at topnotchmedicalboardprep@gmail.com
TOPNOTCH MEDICAL BOARD PREP OBSTETRICS-GYNECOLOGY SUPEREXAM
For inquiries visit www.topnotchboardprep.com.ph or email us at topnotchmedicalboardprep@gmail.com
Ite QUESTION EXPLANATION AUTHOR TOPNOTCH
m # EXAM
221 A 30 y/o G2P1 on her 18th week AOG came to you Chronic htn is BP greater than or equal to 140/90 LEAN ANGELO MIDTERM EXAM
for her prenatal check up. She is asymptomatic, mm Hg before pregnancy or diagnosed before 20 SILVERIO, MD 1 - FEB 2015
and PE revealed BP of 150/90, HR 80, RR 19. The weeks’ gestation not attributable to gestational (TOP 4 - AUG
rest of her PE was otherwise unremarkable. She trophoblastic disease or Hypertension first 2014 MED
claims that this was the first time that she had an diagnosed after 20 weeks’ gestation and persistent BOARDS;
elevated BP. What is your initial diagnosis?? after 12 weeks postpartum. TOPNOTCH MD),
A. Gestational htn MD
B. Chronic htn
C. Preeclampsia
D. Superimposed preeclampsia on chronic htn
E. none of the above
222 Which of the following can be observed at 12 . The CRL of 12 cm is seen at 16 weeks AOG, while LEAN ANGELO MIDTERM EXAM
weeks AOG?? gender can be determined by experienced SILVERIO, MD 1 - FEB 2015
A. CRL of 12 cm sonologist at 14 weeks AOG (TOP 4 - AUG
B. Gender can be determined 2014 MED
C. Center of ossification have appeared in most of BOARDS;
the fetal bones TOPNOTCH MD),
D. All of the above MD
E. None of the above
223 40 y/o G6P5 32-33 weeks AOG complains of abruptio placenta is associated with DIC, renal LEAN ANGELO MIDTERM EXAM
vaginal bleeding and severe abdominal pain. VS BP failure (ATN), hemorrhagic shock, couvelaire SILVERIO, MD 1 - FEB 2015
160/100 PR 123 RR 24 T 37.2C. PE of the uteurs, and fetal maternal hemorrhage. (TOP 4 - AUG
abdomen:FH 36cm FH 97bpm, UC: every minute, 2014 MED
50 sec duration, strong. what complication is BOARDS;
associated with the above condition? TOPNOTCH MD),
A. placenta accreta MD
B. uterine atony
C. DIC
D. puerperial infection
E. all of the above
224 What is the type of placental expulsion, wherein Duncan mechanism – placenta separates from the LEAN ANGELO MIDTERM EXAM
the placenta separates first from the periphery? periphery first. As a result, blood collects between SILVERIO, MD 1 - FEB 2015
A. Schultze mechanism the membranes and the uterine wall and escapes (TOP 4 - AUG
B. Duncan mechanism from the vagina. In this circumstance, the placenta 2014 MED
C. McRobert mechanism descends sideways, and the maternal surface BOARDS;
D. Simpson mechanism appears first. TOPNOTCH MD),
E. Rubins mechanism Schultze mechanism – placenta separates from the MD
center first. Blood from the placental site pours into
the membrane sac and does not escape externally
until after extrusion of the placenta
225 Which of the following is an absolute Absolute contraindication LEAN ANGELO MIDTERM EXAM
contraindication for aerobic exercise during include:Hemodynamically significant heart disease, SILVERIO, MD 1 - FEB 2015
pregnancy? Restrictive lung disease, Incompetent (TOP 4 - AUG
A. Restrictive lung disease cervix/cerclage, Multifetal gestation at risk for 2014 MED
B. Severe anemia preterm labor, Persistent second- or third- BOARDS;
C. Chronic bronchitis trimester bleeding, Placenta previa after 26 weeks, TOPNOTCH MD),
D. Poorly controlled hypertension Preterm labor during the current pregnancy, MD
E. All of the above Ruptured membranes, Preeclampsia/pregnancy-
induced hypertension. Relative Contraindications
include: Severe anemia, Unevaluated maternal
cardiac arrhythmia, Chronic bronchitis, Poorly
controlled type 1 diabetes, Extreme morbid obesity,
Extreme underweight (BMI _12), History of
extremely sedentary lifestyle, Fetal-growth
restriction in current pregnancy, Poorly controlled
hypertension, Orthopedic limitations, Poorly
controlled seizure disorder, Poorly controlled
hyperthyroidism, Heavy smoker
226 A 20 y/o G1P0 at 40 weeks AOG came to your This is the characteristic of a baby with fetal LEAN ANGELO MIDTERM EXAM
hospital. She denied of having any prenatal check hydantoin syndrome. SILVERIO, MD 1 - FEB 2015
up. She then subsequently gave birth to a baby boy. (TOP 4 - AUG
Upon delivery, you noted that the baby had 2014 MED
upturned nose, mild midfacial hypoplasia, and long BOARDS;
upper lip with thin vermilion border, distal digital TOPNOTCH MD),
hypoplasia. What medication could the mother MD
have been taking during the course of her
pregnancy??
A. ACE inhibitor
B. Phenytoin
C. Warfarin
D. Alcohol
E. Valproic acid
227 A 28 y/o G2P1 mother at 35 week AOG came to BPP=4 (NST = 2, fetal breathing = 0, fetal LEAN ANGELO MIDTERM EXAM
your clinic with the following biophysical profile movements = 0, fetal tone = 2, AFI =0). For a BPP of SILVERIO, MD 1 - FEB 2015
scoring: NST has 3 accelerations greater 15 bpm 4, there is probable fetal asphyxia. The intervention (TOP 4 - AUG
each for more than 15 seconds, 20 seconds of fetal is to repeat BPP on the same day. If the result if less 2014 MED
breathing for 30 minutes, 2 discrete body than or equal to 6, deliver the baby. BOARDS;
movements, 1 episode of extremity extension then TOPNOTCH MD),
subsequent flexion. AFI 1.2cm. What is your next MD
plan of treatment?
A. No fetal indication for intervention. Repeat test
weekly
B. Deliver
C. Repeat BPP
D. Observe
E. none of the above
TOPNOTCH MEDICAL BOARD PREP OBSTETRICS-GYNECOLOGY SUPEREXAM Page 28 of 84
For inquiries visit www.topnotchboardprep.com.ph or email us at topnotchmedicalboardprep@gmail.com
TOPNOTCH MEDICAL BOARD PREP OBSTETRICS-GYNECOLOGY SUPEREXAM
For inquiries visit www.topnotchboardprep.com.ph or email us at topnotchmedicalboardprep@gmail.com
Ite QUESTION EXPLANATION AUTHOR TOPNOTCH
m # EXAM
228 The following is a correct criteria for outlet The following are the criteria for outlet forcerps LEAN ANGELO MIDTERM EXAM
forceps extraction? extraction: 1. Scalp is visible at the introitus SILVERIO, MD 1 - FEB 2015
A. Scalp is visible at the introitus after separating without separating the labia, 2. Fetal skull has (TOP 4 - AUG
the labia reached pelvic floor,3. Sagittal suture is in 2014 MED
B. Fetal skull has reached pelvic floor anteroposterior diameter or right or left occiput BOARDS;
C. Sagittal suture is in transverse diameter or anterior or posterior position, 4. Fetal head is at or TOPNOTCH MD),
right or left occiput anterior or posterior position on perineum, and 5. Rotation does not exceed 45 MD
D. Rotation does not exceed 60 degrees degrees
E. Station +2
229 What type of placenta is characterized by fetal Placental membranacea is characterized by all or a LEAN ANGELO MIDTERM EXAM
membranes completely covered by functioning large part of the fetal membranes are covered SILVERIO, MD 1 - FEB 2015
villi?? byfunctioning villi. Placenta membranacea may (TOP 4 - AUG
A. Placenta duplex occasionally give rise to serious hemorrhage 2014 MED
B. Placenta fenestrata because of associated placenta previa or accrete. BOARDS;
C. Placenta membranacea Placenta duplex, or bilobed placenta is TOPNOTCH MD),
D. Succenturiate lobe placenta characterized by the placenta that forms as MD
E. Battledore placenta separate, near equally sized disks. The cord inserts
between the two placental lobes—either into a
connecting chorionic bridge or into intervening
membranes. Placenta fenestrate is characterized by
the missing central portion of a discoidal placenta.
In some instances, there is an actual hole in the
placenta,but more often, the defect involves only
villous tissue, and the chorionic plate remains
intact. Succenturiate lobe placenta is similar to
placenta duplex, but smaller.
One or more small accessory lobes develop in the
membranes at a distance from the main placenta, to
which they usually have vascular connections of
fetal origin.
230 How many weeks will ovulation resume after Ovulation may resume as early as 3 weeks after LEAN ANGELO MIDTERM EXAM
delivery?? delivery, even in lactating women. SILVERIO, MD 1 - FEB 2015
A. 3 weeks (TOP 4 - AUG
B. 4 weeks 2014 MED
C. 3 months BOARDS;
D. 4 months TOPNOTCH MD),
E. 2 weeks MD
231 All of the following are the correct pairs of Gubernaculum gives rise to round ligament of the LEAN ANGELO MIDTERM EXAM
embryonic structures and its derivatives, except? uterus. SILVERIO, MD 1 - FEB 2015
A. Gubernaculum: broad ligament of the uterus (TOP 4 - AUG
B. Metanephric duct: renal pelvis 2014 MED
C. Urogenital sinus : greater vestibular glands BOARDS;
D. Mesonephric duct: Gartner’s duct TOPNOTCH MD),
E. none of the above MD
232 In what cell stage does the ovum in during its The egg is released from the the ovary at its LEAN ANGELO MIDTERM EXAM
release from the ovary ? metaphase II SILVERIO, MD 1 - FEB 2015
A. Prophase I (TOP 4 - AUG
B. Metaphase I 2014 MED
C. Prophase II BOARDS;
D. Metaphase II TOPNOTCH MD),
E. Anaphase I MD
233 A 27 y/o G0 patient came to your clinic due to This patient presents with a classic case of LEAN ANGELO MIDTERM EXAM
severe dysmenorrhea. She is married for 3 years, endometriosis. The definitive diagnostic procedure SILVERIO, MD 1 - FEB 2015
and is not able to conceive. Upon IE, you noted of choice is direct visualization of the endometriotic (TOP 4 - AUG
that the uterus is fixed and retroverted. There is lesion and pathologic examination via biopsy. 2014 MED
nodularity on the uterosacral ligament. What is BOARDS;
the definitive procedure of choice for the TOPNOTCH MD),
diagnosis? MD
A. Pelvic ultrasound
B. Fractional curettage
C. Direct visualization and biopsy
D. culdocentesis
E. MRI
234 5. A 41 y/o came to your clinic due to profuse This patient presents with bacterial vaginosis. Clue LEAN ANGELO MIDTERM EXAM
whitish vaginal discharge. Upon IE, you noted that cells are pathognomonic for bacterial vaginosis. The SILVERIO, MD 1 - FEB 2015
the discharge was whitish frothy appearance with drug of choice for this condition is metronidazole, (TOP 4 - AUG
fishy amine odor. On wet mount, there are 500mg, BID for 7 days. B - is for trichomonas 2014 MED
numerous clue cells. What is your next plan of infection BOARDS;
management? TOPNOTCH MD),
A. 1% Clotrimazole cream, 5g for 7-14 days MD
B. Metronidazole, 2 g, one dose
C. Metronidazole, 500mg, BID for 7 days
D. Miconazole, 100mg per vagina for 7 days
E. Clindamycin 500mg/tab BID
235 Which of the following is true for screening for The following are ACOG guidelines for screening for LEAN ANGELO MIDTERM EXAM
cervical cancer? cervical cancer: Initial screening should begin SILVERIO, MD 1 - FEB 2015
A. Initial screening should begin within 1 year within 3 years after first sexual intercourse or by (TOP 4 - AUG
after first sexual intercourse or by age 18 age 21; annual screening until 30 y/o; after 30 y/o, 2014 MED
B. Annual screening until 30 y/o if patient has 3 consecutive negative results, repeat BOARDS;
C. After 30 y/o, if patient has 3 consecutive every 2-3 years; women with their cervix removed TOPNOTCH MD),
negative results, can discontinue screening for benign reasons do not need annual screening MD
D. Women with their cervix removed for benign
reasons need annual screening
E. all of the above

TOPNOTCH MEDICAL BOARD PREP OBSTETRICS-GYNECOLOGY SUPEREXAM Page 29 of 84


For inquiries visit www.topnotchboardprep.com.ph or email us at topnotchmedicalboardprep@gmail.com
TOPNOTCH MEDICAL BOARD PREP OBSTETRICS-GYNECOLOGY SUPEREXAM
For inquiries visit www.topnotchboardprep.com.ph or email us at topnotchmedicalboardprep@gmail.com
Ite QUESTION EXPLANATION AUTHOR TOPNOTCH
m # EXAM
236 Which of the following characterizes type 2 Type 1 endometrial carcinoma - Grade 1–2, LEAN ANGELO MIDTERM EXAM
endometrial carcinoma? Superficial or no myometrial invasion, Coexisting SILVERIO, MD 1 - FEB 2015
A. Induced by estrogen endometrial hyperplasia, Perimenopausal, Induced (TOP 4 - AUG
B. Post menopausal by estrogen, Responds to progestins, Good 2014 MED
C. Responds to progestins prognosis; Type 2 - High grade (grade 3, papillary BOARDS;
D. Good prognosis serous, clear cell), Deep myometrial invasion, TOPNOTCH MD),
E. All of the above Postmenopausal women, Not induced by estrogen, MD
Does not respond to progestins, Poor prognosis
237 What type of stress incontinence is characterized Stress urinary incontinence is involuntary leakage LEAN ANGELO MIDTERM EXAM
by involuntary leakage of urine during increased of urine on effort or exertion or on coughing or SILVERIO, MD 1 - FEB 2015
abdominal pressure in the absence of detrusor sneezing. The sign of stress urinary incontinence (TOP 4 - AUG
contraction during urodynamics?? must occur at the start of coughing and end with 2014 MED
A. Genuine stress incontinence the coughing. Urge incontinence is the strong desire BOARDS;
B. Urge incontinence to urinate. The patient may sense a degree of TOPNOTCH MD),
C. Functional incontinence urgency if voiding occurs despite the desire not to MD
D. Total urinary incontinence void. Functional incontinence is involuntary
E. leakage despite normal bladder and urethral
function in patients with dementia or
disorientation. Total urinary incontinence occurs
when the bladder is unable to store urine because
the resting urethral pressure is so low that no
resistance is present for the passage of urine.
238 A 68 y/o patient came in your clinic due to intense This is a case of lichen sclerosus. Lichen simplex LEAN ANGELO MIDTERM EXAM
vulvar pruritus. Upon physical examination, you chronicus is characterized by thickened white SILVERIO, MD 1 - FEB 2015
noted that the vulva is thin, white, wrinkled tissue epithelium on vulva. It is generally localized and (TOP 4 - AUG
with a cigarette paper appearance. There was unilateral. Biopsy reveals hyperkeratosis and 2014 MED
agglutination of the labia minora and prepuce. acanthosis producing thickening of the epithelium BOARDS;
Biopsy revealed hyperkeratotic epidermis with and elongation of the rete pegs. Dermis reveals TOPNOTCH MD),
epithelial atrophy, and flattening of the rete pegs. chronic inflammatory cells, fibrosis, and MD
There is cytoplasmic vacuolization of the basal collagenization. Lichen planus is characterized by
layer of cells. What is your initial diagnosis? erosive lesions at the vestibule with or without
A. Lichen simplex chronicus vaginal synechiae resulting in stenosis. Biopsy
B. Lichen sclerosus reveals mils, localized, lichenoid, chronic
C. Lichen planus inflammatory process at the epidermal-dermis
D. Squamous cell hyperplasia junction to ulcerative process with fibrosis.
E. Squamous cell hyperplasia is circumscribed, single
or multifocal raised white lesions on the vulva or
adjacent tissue. Biopsy reveals hyperkeratosis and
acanthosis, producing thickening of the epithelium
and elongation of the rete pegs. Dermis has no
inflammatory infiltrate.
239 A 37 y/o G1P1 (1001) consulted because of severe patient is still in a reproductive age. The goal is to LEAN ANGELO MIDTERM EXAM
dysmenorrhea for 3 consecutive years. Initial remove the mass and to preserve as much as SILVERIO, MD 1 - FEB 2015
consult revealed bilateral adnexal masses about functional ovarian tissue. (TOP 4 - AUG
7cm each with thickened uterosacral ligament. If 2014 MED
surgery is an option, what is the most appropriate BOARDS;
procedure for this patient? TOPNOTCH MD),
A. TAHBSO MD
B. bilateral oophorocystectomy
C. fluid aspiration of the bilateral mass
D. no surgical procedure is warranted.
E.
240 A 34 y/o G1P1 suddenly developed lack of sertoli leydig cell tumors exhibit production of LEAN ANGELO MIDTERM EXAM
menstruation for the past 6 months. There were testosterone which is responsible for the SILVERIO, MD 1 - FEB 2015
also growing of facial hairs and clitoral amenorhea, hirsutism and clitoral enlargement of (TOP 4 - AUG
enlargement. What type of ovaian tumor should be the patient 2014 MED
considered? BOARDS;
A. Granulosa theca cell tumor TOPNOTCH MD),
B. thecoma MD
C. dysgerminoma
D. sertoli leydig tumor
E. none of the above
241 A 17 year old female was diagnosed with primary SIMILAR TO PREVIOUS BOARD EXAM KEVIN BRYAN MIDTERM 2
amenorrhea, further work up revealed absent CONCEPT/PRINCIPLE., meyer rokitansky has LO, MD (TOP 7 - EXAM - FEB
uterus but with normally developed breasts, normal ovaries, normal karyotype, normal AUG 2014 MED 2015
normal looking external genitalia and pubic hair, secodnary sexual char no uterus and sometimes no BOARDS;
which of the following conditions is implicated? vagina or atrophic TOPNOTCH MD)
A. turner's syndrome
B. meyer rokitasky kuster hauser
C. androgen insensitivity
D. congenital adrenal hyperplasia
E. none of the above
242 A 24 y/o G2P1 (1001) was admitted due to passage SIMILAR TO PREVIOUS BOARD EXAM KEVIN BRYAN MIDTERM 2
of watery discharge and onset of regular CONCEPT/PRINCIPLE, monitor futher progress of LO, MD (TOP 7 - EXAM - FEB
abdominal cramps. Initial IE was 4cm already labor first before intervention AUG 2014 MED 2015
station -2, 4 hours later patient was 7cm dilated BOARDS;
station -2, contractions came in regularly every 3- TOPNOTCH MD)
5minutes lasting 50-60 seconds, which of the
following actions are appropriate?
A. augment labor with oxytocin
B. observe progress of labor
C. prepare patient for emergency CS
D. give patient epidural anesthesia
E. hydrate and place patient on left lateral
decubitus

TOPNOTCH MEDICAL BOARD PREP OBSTETRICS-GYNECOLOGY SUPEREXAM Page 30 of 84


For inquiries visit www.topnotchboardprep.com.ph or email us at topnotchmedicalboardprep@gmail.com
TOPNOTCH MEDICAL BOARD PREP OBSTETRICS-GYNECOLOGY SUPEREXAM
For inquiries visit www.topnotchboardprep.com.ph or email us at topnotchmedicalboardprep@gmail.com
Ite QUESTION EXPLANATION AUTHOR TOPNOTCH
m # EXAM
243 Mother with preeclampsia was placed on a SIMILAR TO PREVIOUS BOARD EXAM KEVIN BRYAN MIDTERM 2
tocodynamometer and fetal heart monitor during CONCEPT/PRINCIPLE, head compression early, LO, MD (TOP 7 - EXAM - FEB
labor, the resident suddenly remarked that there cord compression variable AUG 2014 MED 2015
were late decelerations noted on the tracings, what BOARDS;
does it connote? TOPNOTCH MD)
A. fetal head compression
B. uteroplacental insufficiency
C. umbilical cord compression
D. placental abruptio
E. impending eclampsia
244 A 60 year old female complains of scanty vaginal SIMILAR TO PREVIOUS BOARD EXAM KEVIN BRYAN MIDTERM 2
spotting for the past 2 months, physical CONCEPT/PRINCIPLE, atrophic vaginitis, LO, MD (TOP 7 - EXAM - FEB
examination revealed thinned out vaginal mucosa, postmenopausal AUG 2014 MED 2015
smooth cervix, uterus small with no palpable BOARDS;
adnexal masses, which of the following most likely TOPNOTCH MD)
is the cause of
A. endometrial polyp
B. cervical polyp
C. endometrial cancer
D. atrophic vaginitis
E. submucous myoma
245 A 30 year old female G5P1 with history of repeated SIMILAR TO PREVIOUS BOARD EXAM KEVIN BRYAN MIDTERM 2
abortions and preterm labor is pregnant, her CONCEPT/PRINCIPLE, usually done at the age of LO, MD (TOP 7 - EXAM - FEB
doctor tells her to undergo a cerclage procedure to viability at around 20 weeks and above AUG 2014 MED 2015
help prevent preterm labor, this procedure is BOARDS;
generally done at what age of the pregnancy? TOPNOTCH MD)
A. 8 weeks
B. 12 weeks
C. 22 weeks
D. 24 weeks
E. 36 weeks
246 A 40 year old female patient has depressed TSH SIMILAR TO PREVIOUS BOARD EXAM KEVIN BRYAN MIDTERM 2
levels with Increased T4 levels, she experienced CONCEPT/PRINCIPLE, hyperthyoridism = hypo or LO, MD (TOP 7 - EXAM - FEB
palpitations, weight loss, heat intolerance and and oligomenorrhea AUG 2014 MED 2015
bouts of diarrhea, which of the following is BOARDS;
expected regarding her menses? TOPNOTCH MD)
A. hypomenorrhea
B. menorrhagia
C. menometrorrhagia
D. primary amenorrhea
E. any of the above
247 A 16 year old patient comes in with grossly female SIMILAR TO PREVIOUS BOARD EXAM KEVIN BRYAN MIDTERM 2
genitalia, no menses ever since but with positive CONCEPT/PRINCIPLE, they mixed up the meyer LO, MD (TOP 7 - EXAM - FEB
breast development, further work up revealed rokitansky, androgen insensitivity, turners and AUG 2014 MED 2015
absence of ovaries, uterus, karyotyping revealed 46 CAH, androgen insensitivity = grossly female or BOARDS;
XY, which of the following is the possible diagnoses phenotypically female genitalia and secondary TOPNOTCH MD)
A. meyer rokitansky kuster hauser sexual char but XY karyotype. Turners XO, meyer
B. androgen insensitivity syndrome rokitansky XX, CAH = XX with virilization or XY with
C. turners syndrome normal phenotype
D. congenital adrenal hyperplasia
E. none of the above
248 A 36 year old female patient with previously SIMILAR TO PREVIOUS BOARD EXAM KEVIN BRYAN MIDTERM 2
regular menses consulted due to amenorrhea for 3 CONCEPT/PRINCIPLE, progesterone induced LO, MD (TOP 7 - EXAM - FEB
months with episodes of spotting occasionally, withdrawal bleed indicates endometrium is AUG 2014 MED 2015
pregnancy test was negative, she was given estrogen primed, lack of ovulation causes it to BOARDS;
progesterone for 7 days and after 3-5 days of slough off improperly due to lack of progesterone TOPNOTCH MD)
withdrawal of the drug, she had heavy menses.
Which of the following is true regarding her
condition?
A. Her irregular menses are due to a submucous
myoma
B. she has ectopic pregnancy
C. she has endometrial hyperplasia
D. she has anovulatory vaginal spotting
E. she has an endometrial polyp
249 Which of the following ultrasound findings are SIMILAR TO PREVIOUS BOARD EXAM KEVIN BRYAN MIDTERM 2
indicative of down's syndrome in the fetus ? CONCEPT/PRINCIPLE, nuchal thickening is LO, MD (TOP 7 - EXAM - FEB
A. increased nuchal thickening indicative of down's syndrome AUG 2014 MED 2015
B. increase brain ventricle size BOARDS;
C. ambiguous genitalia TOPNOTCH MD)
D. small biparietal diameter for AOG
E. all of the above
250 Which of the following trisomies are associated SIMILAR TO PREVIOUS BOARD EXAM KEVIN BRYAN MIDTERM 2
with preeclampsia? CONCEPT/PRINCIPLE, trisomy 13 preeclampsia LO, MD (TOP 7 - EXAM - FEB
A. trisomy 13 AUG 2014 MED 2015
B. trisomy 15 BOARDS;
C. trisomy 18 TOPNOTCH MD)
D. trisomy 21
E. none of the above
251 An 8 year old female was brought into consult due SIMILAR TO PREVIOUS BOARD EXAM KEVIN BRYAN MIDTERM 2
to difficulty in perineal hygiene, her mother noted CONCEPT/PRINCIPLE a case of adhesive vulvitis, LO, MD (TOP 7 - EXAM - FEB
that her labia was adherent to one another and was estrogen topical AUG 2014 MED 2015
concerned about it hence the consult, which of the BOARDS;
following treatments is advisable for this case? TOPNOTCH MD)
A. oral estrogen pills
B. topical estrogen cream
TOPNOTCH MEDICAL BOARD PREP OBSTETRICS-GYNECOLOGY SUPEREXAM Page 31 of 84
For inquiries visit www.topnotchboardprep.com.ph or email us at topnotchmedicalboardprep@gmail.com
TOPNOTCH MEDICAL BOARD PREP OBSTETRICS-GYNECOLOGY SUPEREXAM
For inquiries visit www.topnotchboardprep.com.ph or email us at topnotchmedicalboardprep@gmail.com
Ite QUESTION EXPLANATION AUTHOR TOPNOTCH
m # EXAM
C. topical 1% hydrocortisone cream
D. surgical adhesiolysis
E. all of the above

252 Which of the following is considered the most SIMILAR TO PREVIOUS BOARD EXAM KEVIN BRYAN MIDTERM 2
important indicator of progress of labor? CONCEPT/PRINCIPLE, descent according to APMC LO, MD (TOP 7 - EXAM - FEB
A. engagement book, descent of the baby infers progress of labor AUG 2014 MED 2015
B. descent and interaction between the passenger and BOARDS;
C. cervical dilatation passageway TOPNOTCH MD)
D. internal rotation
E. all of the above
253 Which of the following tests can be used to infer SIMILAR TO PREVIOUS BOARD EXAM KEVIN BRYAN MIDTERM 2
regarding the risk of preterm labor? CONCEPT/PRINCIPLE, fetal fibronectin helps infer LO, MD (TOP 7 - EXAM - FEB
A. fetal HCG regarding risk of preterm labor AUG 2014 MED 2015
B. D Dimer assay BOARDS;
C. fetal fibronectin TOPNOTCH MD)
D. maternal CRP
E. amniotic fluid AFP
254 On instances of complete breach extraction, which SIMILAR TO PREVIOUS BOARD EXAM KEVIN BRYAN MIDTERM 2
of the following forceps is used to deliver the CONCEPT/PRINCIPLE - pipers aftercoming head LO, MD (TOP 7 - EXAM - FEB
aftercoming head? AUG 2014 MED 2015
A. simpsons BOARDS;
B. tucker mcclane TOPNOTCH MD)
C. kielland
D. pipers
E. none of the above
255 A 24 year old G3P3 desires contraception. In giving SIMILAR TO PREVIOUS BOARD EXAM KEVIN BRYAN MIDTERM 2
your advice regarding the various options of birth CONCEPT/PRINCIPLE - husband consent not LO, MD (TOP 7 - EXAM - FEB
control, which of the following is a needed, even in menstruation no contraindication AUG 2014 MED 2015
contraindication to placement of an IUD for family BOARDS;
planning? TOPNOTCH MD)
A. presence of menstruation
B. presence of previous ectopic pregnancy
C. no consent from the husband
D. presence of ongoing pelvic infection
E. all of the above
256 48 year old G3P3 presents to your clinic for annual SIMILAR TO PREVIOUS BOARD EXAM KEVIN BRYAN MIDTERM 2
check up, she has had no remarkable symptoms CONCEPT/PRINCIPLE, colposcopy with LO, MD (TOP 7 - EXAM - FEB
since her last check up, you decided to do a pap acetowhitening or biopsy of suspicious lesion AUG 2014 MED 2015
smear, a week after the results came out, it turned BOARDS;
out to be high grade squamous intraepithelial TOPNOTCH MD)
neoplasm, you remember that physical
examination including pelvic exam at that time was
unremarkable what is the next step in evaluation?
A. request a transvaginal ultrasound
B. place her on progesterone and wait for
withdrawal bleed
C. do a repeat pap smear using cytobrush and
liquid based cytology for confirmation
D. do a colposcopy
E. all of the above
257 A 30 year old female G1P1 presented to the clinic SIMILAR TO PREVIOUS BOARD EXAM KEVIN BRYAN MIDTERM 2
with a 2 year history of progressive virilization, CONCEPT/PRINCIPLE, adnexal mass, virilization LO, MD (TOP 7 - EXAM - FEB
facial and bodily hair growth, oligomenorrhea and and masculinization features leydig cell tumor AUG 2014 MED 2015
irregular menses, there was also acne, deepening BOARDS;
of voice, examination revealed grossly female TOPNOTCH MD)
genitalia, clitoromegaly, and a palpable right
adnexal mass. Ultrasound revealed right complex
adnexal mass, which of the following is the
possible condition?
A. leydig cell tumor
B. granulosa cell tumor
C. yolk sac tumor
D. choriocarcinoma
E. dermoid cyst of the ovary
258 Which of the following risk factors is considered SIMILAR TO PREVIOUS BOARD EXAM KEVIN BRYAN MIDTERM 2
the greatest risk factor for ectopic pregnancy? CONCEPT/PRINCIPLE, tubal instrumentation is the LO, MD (TOP 7 - EXAM - FEB
A. repeated pelvic infections or PID strongest risk factor for having ectopic pregnancy, AUG 2014 MED 2015
B. history of tubal instrumentation infection comes next BOARDS;
C. infertility TOPNOTCH MD)
D. congenital uterine abnormalities
E. familial chromosomal disorders
259 A 26 year old G1P0 was admitted to the labor room SIMILAR TO PREVIOUS BOARD EXAM KEVIN BRYAN MIDTERM 2
due to rupture of bag of waters without CONCEPT/PRINCIPLE, fetal tachycardia = LO, MD (TOP 7 - EXAM - FEB
accompanying uterine contractions, after 18 hours, chorioamnionitis AUG 2014 MED 2015
on monitoring, mother was slightly febrile at 37.8 BOARDS;
degrees celsius and was continuously hydrated TOPNOTCH MD)
with IV fluids, electronic fetal monitoring was done
which revealed fetal tachycardia, which of the
following is the suspected condition?
A. chorioamnionitis
B. umblical cord prolapse
C. maternal dehydration
D. maternal sepsis
E. endometritis

TOPNOTCH MEDICAL BOARD PREP OBSTETRICS-GYNECOLOGY SUPEREXAM Page 32 of 84


For inquiries visit www.topnotchboardprep.com.ph or email us at topnotchmedicalboardprep@gmail.com
TOPNOTCH MEDICAL BOARD PREP OBSTETRICS-GYNECOLOGY SUPEREXAM
For inquiries visit www.topnotchboardprep.com.ph or email us at topnotchmedicalboardprep@gmail.com
Ite QUESTION EXPLANATION AUTHOR TOPNOTCH
m # EXAM
260 A 35 year old female patient G1P1 came in due to SIMILAR TO PREVIOUS BOARD EXAM KEVIN BRYAN MIDTERM 2
dysmenorrhea and vaginal spotting for the past 3 CONCEPT/PRINCIPLE - adenomyosis symmetrically LO, MD (TOP 7 - EXAM - FEB
weeks, PE revealed grossly female external enlarged uterus dysfunctional bleeding, AUG 2014 MED 2015
genitalia, no lesions noted, cervix pink smooth, no dysmenorrhea BOARDS;
discharge, cervix closed, uterus symmetrically TOPNOTCH MD)
enlarged to 2 to3 months size, no adnexal masses
noted, which of the following conditions is
suspected?
A. endometriosis
B. endometrial hyperplasia
C. multiple submucous myomas
D. adenomyosis
E. endometrial polyp
261 A 35 year old woman is complaining of Characteristic case of adenomyosis RAYMUND MIDTERM 3
increasingly severe mentrual pain with associated MARTIN LI, MD EXAM - FEB
heavy and prolonged bleeding. Pelvic examination (TOP 1 - AUG 2015
reveals a diffusely enlarged uterus that feels soft 2014 MED
and boggy. What is the pathogenesis underlying BOARDS;
the most likely condition that is causing this TOPNOTCH MD)
patient's symptoms?
A. Presence of hormone-responsive endometrial
tissue in the pelvic peritoneum
B. Benign submucous myomatous proliferation
C. Atypical endometrial hyperplasia
D. Presence of endometrial glands within the
myometrium
E. None of the above
262 A sexually active 28 year old female had her first ASC-H, LSIL, HSIL - Perform colposcopy with biosy; RAYMUND MIDTERM 3
Pap smear revealing low-grade squamous ASC-US - perform HPV testing or repeat Pap smear MARTIN LI, MD EXAM - FEB
intraepithelial lesion (LSIL). What will be the next after 6 months (TOP 1 - AUG 2015
step in your management? 2014 MED
A. Hysterectomy BOARDS;
B. LEEP procedure TOPNOTCH MD)
C. HPV testing
D. Repeat Pap smear after 6 months
E. Colposcopy-directed biopsy
263 A G1P0 preganant comes for prenatal consult in Vitamin A megadoses is teratogenic. Beta carotene- RAYMUND MIDTERM 3
your clinic. History reveals that she takes vitamin rich foods has lesser vitamin A content and is less MARTIN LI, MD EXAM - FEB
supplements because she is afraid of getting sick. likely to cause teratogenesis. (TOP 1 - AUG 2015
You counsel her to avoid taking which of the 2014 MED
following? BOARDS;
A. Vitamin C TOPNOTCH MD)
B. Vitamin A
C. Beta carotene
D. Vitamin E
E. Folic acid
264 Which of the following findings is expected in the Menopausal - parabasal; Reproductive nonpregnant RAYMUND MIDTERM 3
cervical cytology of a menopausal woman? - superficial and intermediate; Pregnant - MARTIN LI, MD EXAM - FEB
A. Parabasal cells intermediate (TOP 1 - AUG 2015
B. Superficial cells 2014 MED
C. Intermediate cells BOARDS;
D. Superficial and intermediate cells TOPNOTCH MD)
E. All of the above
265 A G2P2 patient is in labor undergoes pelvic Pergnant patients with herpes simplex genital RAYMUND MIDTERM 3
examination revealing vesicular lesions in the lesions should have cesarean delivery to lessen risk MARTIN LI, MD EXAM - FEB
genital area. You are highly suspecting a herpes of transmission to newborn (TOP 1 - AUG 2015
simplex infection. How will you manage this 2014 MED
patient? BOARDS;
A. Treat the mother with IV acyclovir and deliever TOPNOTCH MD)
via SVD
B. Administer oxytocin to accelerate labor
C. Perform cesarean delivery
D. Perform forceps extraction when fully dilated
E. Continue labor watch.
266 A 30 year G3P3 patient gave birth to a live baby A case of postpartum endometritis. Most are caused RAYMUND MIDTERM 3
girl 2 days prior via NSVD. Currently, she claims by mixed aerobic and anaerobic microorganisms MARTIN LI, MD EXAM - FEB
that she feels febrile. Temperature taken was 37.9 but Bacteroides is frequently isolated in culture (TOP 1 - AUG 2015
degrees Celsius. Bimanual pelvic examination 2014 MED
reveals a tender uterus and foul-smelling vaginal BOARDS;
discharge. You will most likely isolate which of the TOPNOTCH MD)
following?
A. Bacteroides fragilis
B. Chlamydia trachomatis
C. Neisseria gonorrheae
D. Pseudomonas aeruginosa
E. Gardnerella vaginalis
267 What will be the expected results of a Quad screen Elevated AFP - neural tube defects; Decreased AFP, RAYMUND MIDTERM 3
in a mother carrying a fetus with trisomy 21? HCG, estriol - trisomy 18 MARTIN LI, MD EXAM - FEB
A. Decreased B-HCG and inhibin, Elevated estriol (TOP 1 - AUG 2015
and AFP 2014 MED
B. Elevated B-HCG and inhibin, Decreased estriol BOARDS;
and AFP TOPNOTCH MD)
C. Elevated B-HCG and inhibin, Elevated estriol and
AFP
D. Decreased B-HCG and inhibin, Decreased estriol
and AFP
TOPNOTCH MEDICAL BOARD PREP OBSTETRICS-GYNECOLOGY SUPEREXAM Page 33 of 84
For inquiries visit www.topnotchboardprep.com.ph or email us at topnotchmedicalboardprep@gmail.com
TOPNOTCH MEDICAL BOARD PREP OBSTETRICS-GYNECOLOGY SUPEREXAM
For inquiries visit www.topnotchboardprep.com.ph or email us at topnotchmedicalboardprep@gmail.com
Ite QUESTION EXPLANATION AUTHOR TOPNOTCH
m # EXAM
E. None of these

268 A 34 year old patient comes to your clinic Characteristic picture of a case of leiomyoma which RAYMUND MIDTERM 3
complaining of heavy menses. She claims that she is a common cause of heavy menstrual bleeding esp MARTIN LI, MD EXAM - FEB
occasionally pass clots and she feels a heavy in premenopausal women (TOP 1 - AUG 2015
sensation and a palpable mass on the lower 2014 MED
abdominal area. Which of the following is the most BOARDS;
likely diagnosis? TOPNOTCH MD)
A. Ovarian cyst
B. Endometrial carcinoma
C. Endometriosis
D. Endometrioma
E. Leiomyoma
269 An incidental finding of a 6cm multilocular ovarian Simple ovarian cysts may be managed with RAYMUND MIDTERM 3
cyst with solid components was seen during observation except if size is >8cm, if 6-8 cm but MARTIN LI, MD EXAM - FEB
ultrasound in a 30 year old woman. History is doesn’t resolve after 2-3 months, if there is rapid (TOP 1 - AUG 2015
unremarkable. What is the next best step in the increase in size, or if multiloculated or with solid 2014 MED
management? components. These are suspicous for malignancy BOARDS;
A. Observe for 2-3 months and repeat ultrasound and have to be treated with surgery TOPNOTCH MD)
B. Perform percutaneous biopsy
C. OCP therapy
D. Exploratory laparotomy
E. Do nothing.
270 A 38 weeks AOG G3P2 mother who has chronic A case of abruptio placenta. May be managed RAYMUND MIDTERM 3
hypertension presents to the ER due to vaginal conservatively if vital signs are stable or fetal status MARTIN LI, MD EXAM - FEB
bleeding associated abdominal pain and uterine is reassuring. If not, immediate delivery is (TOP 1 - AUG 2015
contractions. Physical examination reveals a firm, necessary 2014 MED
tender uterus with noted regular contractions BOARDS;
every 4-5 minutes and cervical dilatation of 4-5cm. TOPNOTCH MD)
Bleeding has subsided and vital signs of the mother
is stable. Fetal monitoring is reassuring.
Ultrasound reveals a high-lying placenta. How
should this patient be managed?
A. Stat cesarean delivery
B. Give tocolytics
C. Monitor progress of labor and do fetal status
monitoring
D. Send the patient home
E. None of the above
271 A G2P0 patient has a previous history of RAYMUND MIDTERM 3
incompetent cervix. If she is to undergo an elective MARTIN LI, MD EXAM - FEB
cervical cerclage, at what age of gestation is the (TOP 1 - AUG 2015
ideal time to perform the procedure? 2014 MED
A. 8-10 weeks BOARDS;
B. 10-12 weeks TOPNOTCH MD)
C. 12-14 weeks
D. 18-20 weeks
E. 24-28 weeks
272 Examination of a woman reveals presence of fleshy Descriptive of condyloma acuminata/genital wart RAYMUND MIDTERM 3
growths covered with small, papillary surface MARTIN LI, MD EXAM - FEB
prijections on her external genitalia. What is the (TOP 1 - AUG 2015
most likely diagnosis? 2014 MED
A. Condyloma acuminata BOARDS;
B. Condyloma lata TOPNOTCH MD)
C. Genital herpes
D. Phthirus pubis
E. Molluscum contagiosum
273 A 16 year old female is brought to you by her Anorexia nervosa may cause hypothalamic RAYMUND MIDTERM 3
mother because of amenorrhea. Upon physical dysfunction leading to imapired release of MARTIN LI, MD EXAM - FEB
examination, patient was noted to have a BMI of Gonadotropin releasing hormone (TOP 1 - AUG 2015
16.1. Dry skin, thinning hair, parotid glnad 2014 MED
swelling, and fine body hair were also noted. She BOARDS;
tells you that she has not attained her ideal body TOPNOTCH MD)
weight and that she thinks she is fat. Based from
these findings, you expect that the amenorrhea is
caused by?
A. Hyperprolactinemia
B. Hypothalamic dysfunction
C. Anterior pituitary hormone deficiency
D. Increased androgen production
E. Anovulatory disorder
274 Which of the following is most predictive of RAYMUND MIDTERM 3
recurrent preterm labor? MARTIN LI, MD EXAM - FEB
A. Group B streptococcus colonization (TOP 1 - AUG 2015
B. Prior preterm delivery 2014 MED
C. History of cigarette smoking BOARDS;
D. History of early trimester abortion TOPNOTCH MD)
E. None of the above
275 A patient presents to the ER complaining of Equivocal findings necessitate repeat TVS. Absence RAYMUND MIDTERM 3
amenorrhea, lower abdominal pain, and vaginal of Intrauterine pregnancy at B-HCG levels of >1500 MARTIN LI, MD EXAM - FEB
bleeding. Pregnancy test is positive. Vital signs are or failure of BHCG to double after 48 hours rules (TOP 1 - AUG 2015
stable. Physical examination reveals a uterus that out uterine pregnancy 2014 MED
is small for gestational age, closed cervix and BOARDS;
bleeding. No adnexal mass is palpated. TVS fails to TOPNOTCH MD)

TOPNOTCH MEDICAL BOARD PREP OBSTETRICS-GYNECOLOGY SUPEREXAM Page 34 of 84


For inquiries visit www.topnotchboardprep.com.ph or email us at topnotchmedicalboardprep@gmail.com
TOPNOTCH MEDICAL BOARD PREP OBSTETRICS-GYNECOLOGY SUPEREXAM
For inquiries visit www.topnotchboardprep.com.ph or email us at topnotchmedicalboardprep@gmail.com
Ite QUESTION EXPLANATION AUTHOR TOPNOTCH
m # EXAM
reveal an intrauterine pregnancy. Which of the
following is the most appropriate management?
A. Send the patien home.
B. Admit the patient and prepare for dilatation and
curettage.
C. Repeat TVS when B-HCG level is >1500
D. Perform emergency exploratory laparotomy
E. Admit the patient for therapeutic abortion
276 A 23 year old obese patient has been diagnosed Increased LH to FSH ratio is characteristic of PCOS. RAYMUND MIDTERM 3
with Polycystic ovarian syndrome. You, therefore, FSH is decreased while estrogen, androgen, and LH MARTIN LI, MD EXAM - FEB
expect decreased levels of which of the following? are increased (TOP 1 - AUG 2015
A. FSH 2014 MED
B. LH BOARDS;
C. Estrogen TOPNOTCH MD)
D. Androstenedione
E. Testosterone
277 Physiologic changes during pregnancy will include There is an increase in tidal volume, cardiac output, RAYMUND MIDTERM 3
which of the following? GFR, and plasma volume. There is a decrease in SVR MARTIN LI, MD EXAM - FEB
A. Increase in systemic vascular resistance and hamatocrit (TOP 1 - AUG 2015
B. Increase in tidal volume 2014 MED
C. Decrease in hematocrit BOARDS;
D. A and B TOPNOTCH MD)
E. B and C
278 Which of the following cardinal movements of Flexion allows the narrowest diameter RAYMUND MIDTERM 3
labor allows the narrowest fetal head diameter to (suboccipitobregmatic diameter) to present. MARTIN LI, MD EXAM - FEB
present in the birth canal? Review cardinal movements of labor (TOP 1 - AUG 2015
A. Flexion 2014 MED
B. Extension BOARDS;
C. Internal Rotation TOPNOTCH MD)
D. Engagement
E. Descent
279 A 30 year old patient who is desirious of pregnancy Endometriosis may be managed medically but if RAYMUND MIDTERM 3
presented to your infertility clinic with associated desire for fertility is an issue then treatment should MARTIN LI, MD EXAM - FEB
symptoms of dysmenorrhea and dyspareunia be surgery (TOP 1 - AUG 2015
undergoes fertility evaluation. You are highly 2014 MED
suspecting endometriosis as the cause. How will BOARDS;
you manage this patient? TOPNOTCH MD)
A. NSAIDS
B. Continuous OCP's
C. Watchful waiting
D. Laparoscopic surgery and fulguration of
endometriotic implants
E. Danazol therapy
280 What is the characteristic liver pathologic finding Periportal necrosis and sinusoidal thrombin RAYMUND MIDTERM 3
in preeclampsia? deposition is characteristic of MARTIN LI, MD EXAM - FEB
A. Centrilobular necrosis preeclampsia/eclampsia (TOP 1 - AUG 2015
B. Macrovesicular steatosis 2014 MED
C. Bridging portal fibrosis BOARDS;
D. Periportal necrosis TOPNOTCH MD)
E. None of the above
281 A 19 year old G1P0, 14-15 weeks AOG, consulted Dx: H. mole. Treatment of choice females <35 years ERIC ROYD FINAL EXAM -
due to vaginal bleeding. Vital signs were stable. old and still desires pregnancy is suction curettage. TALAVERA, MD FEB 2015
Fundic height was noted at 23 cm. Internal exam: (TOP 1 - AUG
cervix is soft, long and closed with uterus enlarged 2014 MED
to 5 months size. UTZ showed a complex, BOARDS;
echogenic mass with numerous cystic spaces with TOPNOTCH MD)
no fetus and aminotic sac seen. Serum Beta HCG is
elevated at 170,000 miU/mL. What is the most
appropriate management for this case?
A. TAH
B. TAHBSO
C. Radiotherapy
D. Suction Curettage
E. Any of the above
282 What is the maximum blood loss (in mL) to be Menorrhagia: Prolonged (more than 7 days) or ERIC ROYD FINAL EXAM -
considered in a normal menstrual cycle? excessive (greater than 80 mL) uterine bleeding TALAVERA, MD FEB 2015
A. 30 occurring at regular intervals. (TOP 1 - AUG
B. 40 2014 MED
C. 60 BOARDS;
D. 70 TOPNOTCH MD)
E. 80
283 A 27 year old mulitpara who delivered at home DX: Uterine inversion which is most commonly ERIC ROYD FINAL EXAM -
was brough to the ER due to profuse vaginal brough about by excessive traction of the umbilical TALAVERA, MD FEB 2015
bleeding. At the ER, BP was 80/50 and HR was 126 cord upon delivery of the placenta (TOP 1 - AUG
bpm. She was visibly pale. Abdomen was soft and 2014 MED
the fundus of the uterus was not appreciated. On BOARDS;
inspection of the vulva, a pear shaped reddish pink TOPNOTCH MD)
mass was see protruding from the introitus. What
is the most common cause of the above condition?
A. Abruptio placenta
B. Placenta previa
C. Strong traction of the umbilical cord
D. Short umbilical cord
E. None of the above

TOPNOTCH MEDICAL BOARD PREP OBSTETRICS-GYNECOLOGY SUPEREXAM Page 35 of 84


For inquiries visit www.topnotchboardprep.com.ph or email us at topnotchmedicalboardprep@gmail.com
TOPNOTCH MEDICAL BOARD PREP OBSTETRICS-GYNECOLOGY SUPEREXAM
For inquiries visit www.topnotchboardprep.com.ph or email us at topnotchmedicalboardprep@gmail.com
Ite QUESTION EXPLANATION AUTHOR TOPNOTCH
m # EXAM
284 Which of the following is not an indication for in- Other indications: severe illness, nausea or ERIC ROYD FINAL EXAM -
patient treatment of patients with Pelvic vomiting. Response to treatment is usually assessed TALAVERA, MD FEB 2015
Inflammatory Disease? after 3 days (72 hours) (TOP 1 - AUG
A. Acute appendicitis cannot be ruled out 2014 MED
B. No response after 24 hours of treatment BOARDS;
C. Cannot tolerate oral regimen TOPNOTCH MD)
D. Presence of tubo-ovarian abscess
E. Currently pregnant
285 A 38 year old G1P0, 30-31 weeks AOG with no In severe pre eclampsia: SBP >160, DBP >110. With ERIC ROYD FINAL EXAM -
known co-morbids, sought consult due to headache proteinuria >5 g/24 hours, (+) headache, visual TALAVERA, MD FEB 2015
and blurring of vision. Patient had 8 prenatal disturbance, upper abdominal pain, oliguria. (TOP 1 - AUG
checkups since 6 weeks AOG. BP monitoring was Elevated crea. Thrombocytopenia 2014 MED
advised which ranges from 100/70 to 110/80 BOARDS;
mmHg. She has a (+) family history for both HTN TOPNOTCH MD)
and DM. VS: BP 180/110 mmHg, PR 87/min, RR
21/min. FH 30 cm, FHT 120 bpm, LM1-cephalic.
What is the most probable diagnosis for this case?
A. Mild Preeclampsia
B. Severe Preeclampsia
C. Chronic hypertesion
D. Preeclampsia superimposed on Chronic
hypertension
E. Eclampsia
286 What laboratory examination can be done to proteinuria would differentiate pre eclampsia with ERIC ROYD FINAL EXAM -
confirm the diagnosis in #185? gestatinal hypertension and chronic hypertension TALAVERA, MD FEB 2015
A. CXR (TOP 1 - AUG
B. Liver Enzymes 2014 MED
C. CBC BOARDS;
D. 24 hour urine protein TOPNOTCH MD)
E. Ultrasound
287 A 35 year old G4P4 (4004) underwent TAH-BSO Hyaline droplets represent the Schiller Duval ERIC ROYD FINAL EXAM -
with omentectomy, peritoneal washing and pelvic bodies which are seen in Yolk sac tumor TALAVERA, MD FEB 2015
lymph node dissection for a right ovarian new (TOP 1 - AUG
growth. Histopathologic report stated that 2014 MED
numerous hyaline droplets were seen. What BOARDS;
specific type of ovarian tumor was removed from TOPNOTCH MD)
this patient?
A. Yolk sac tumor
B. Dysgerminoma
C. Granulosa cell tumor
D. Mucinous adenocarcinoma
E. Sertoli Leydig cell tumor
288 What tumor marker would be characteristically The tumor marker associated with yolk sac tumor ERIC ROYD FINAL EXAM -
elevated in association with the diagnosis in #187 is Alpha feto protein (AFP) TALAVERA, MD FEB 2015
? (TOP 1 - AUG
A. HCG 2014 MED
B. AFP BOARDS;
C. LDH TOPNOTCH MD)
D. CA-125
E. CEA
289 A 40 year old G4P3 (3003), 31-32 weeks AOG, Dx: Abruptio placenta. Immediate delivery via CS is ERIC ROYD FINAL EXAM -
known hypertensive and a heavy smoker was warranted due to evidence of fetal distress as noted TALAVERA, MD FEB 2015
brought to the ER due to severe hypogastric pain by a decreased fetal heart rate ranging from 90-100 (TOP 1 - AUG
and vaginal bleeding. BP 140/90, HR 100/min, RR bom 2014 MED
21/min. FH 32 cm, FHT 90-100/min, LM1-cephalic, BOARDS;
uterine contractions - every 2 minutes with a TOPNOTCH MD)
strong intensity, (+) uterine tenderness. What is
the best management for this case?
A. Corticosteroids alone
B. Tocolysis and corticosteroids
C. Labor induction
D. Cesarean section
E. None of the above
290 Which of the following is the expected hormonal There is ovarian atresia resulting in depletion of ERIC ROYD FINAL EXAM -
profile in individuals with Turner's syndrome? oocytes in Turner's syndrome hence there is TALAVERA, MD FEB 2015
A. Hypogonadotropic hypogonadism decreased estrogen secretion. However since the (TOP 1 - AUG
B. Hypergonadotropic hypogonadism pituitary function is intact, the decreased estrogen 2014 MED
C. Hypogonadotropic normogonadism will send feedback impulses to the pituitray BOARDS;
D. Hypergonadotropic normogonadism increasing its output of FSH and LH TOPNOTCH MD)
E. Normogonadotropic normogonadism
291 When is the usual onset of quickening among Quickening among multi: 16-18 weeks, for primi: ERIC ROYD FINAL EXAM -
primigravids? 18-20 weeks TALAVERA, MD FEB 2015
A. 12-14 weeks AOG (TOP 1 - AUG
B. 13-15 weeks AOG 2014 MED
C. 16-18 weeks AOG BOARDS;
D. 18-20 weeks AOG TOPNOTCH MD)
E. 22-24 weeks AOG
292 Fetal attitude is primarily determined by what LM1 presentation LM2 position LM3 engagement ERIC ROYD FINAL EXAM -
Leopold's maneuver (LM)? TALAVERA, MD FEB 2015
A. LM1 (TOP 1 - AUG
B. LM2 2014 MED
C. LM3 BOARDS;
D. LM4 TOPNOTCH MD)
E. Both B and C

TOPNOTCH MEDICAL BOARD PREP OBSTETRICS-GYNECOLOGY SUPEREXAM Page 36 of 84


For inquiries visit www.topnotchboardprep.com.ph or email us at topnotchmedicalboardprep@gmail.com
TOPNOTCH MEDICAL BOARD PREP OBSTETRICS-GYNECOLOGY SUPEREXAM
For inquiries visit www.topnotchboardprep.com.ph or email us at topnotchmedicalboardprep@gmail.com
Ite QUESTION EXPLANATION AUTHOR TOPNOTCH
m # EXAM
293 A 34 year old nulliparous sought consult due to Leuprolide acts as an agonist ERIC ROYD FINAL EXAM -
progressive dysmenorrhea. Internal examination at pituitary GnRH receptors. By interrupting the TALAVERA, MD FEB 2015
showed a fixed, retroverted uterus with normal pulsatile stimulation of, and thus (TOP 1 - AUG
characteristic nodularities palpated in the desensitizing, the GnRH receptors, it indirectly 2014 MED
uterosacral ligaments and cul-de-sac. A diagnosis downregulates the secretion of BOARDS;
of endometriosis was made for which the patient gonadotropins luteinizing hormone (LH) TOPNOTCH MD)
opted to have medical treatment for the moment. and follicle-stimulating hormone(FSH), leading
Which of the following medications will induce a to hypogonadism and thus a dramatic reduction
"medical oophorectomy"? in estradiol and testosterone levels in both sexes.
A. Danazol
B. Progestins
C. OCP
D. Leuprolide
E. Clomiphene
294 Which of the following is not a sign of midpelvis A narrow subpubic arch is a sign of an inadequate ERIC ROYD FINAL EXAM -
contraction? pelvic outlet TALAVERA, MD FEB 2015
A. Prominent ischial spines (TOP 1 - AUG
B. Narrow subpubic arch 2014 MED
C. Convergent sidewalls BOARDS;
D. Shallow sacral concavity TOPNOTCH MD)
E. Narrow sacroiliac notch
295 A 38 year old G4P4 (4004) who is s/p LTCS due to Dx: Endometritis: Route of delivery is the single ERIC ROYD FINAL EXAM -
PROM of 24 hours about 2 days ago now presents most important risk factor for this condition TALAVERA, MD FEB 2015
with fever and chills. BP 100/60, PR 110/min, (TOP 1 - AUG
RR23/min T 39.5 C. There is noted abdominal 2014 MED
tenderness over the hypogastric area. Speculum: BOARDS;
(+) foul smelling lochia rubra. IE: cervix soft, long TOPNOTCH MD)
and closed with (+) uterine tenderness. What is
the single most important risk factor for this
condition?
A. Anemia
B. Socioeconomic status
C. Prolonged labor
D. Route of delivery
E. Malnutrition
296 What is the earliest stage of cervical cancer where Please see FIGO staging system for cervical CA. Of ERIC ROYD FINAL EXAM -
in hydronephrosis, renal failure and uremia would important note, uremia is the most common cause TALAVERA, MD FEB 2015
be present? of death in cervical cancer patients. (TOP 1 - AUG
A. Stage IIIA 2014 MED
B. Stage IIIB BOARDS;
C. Stage IIA1 TOPNOTCH MD)
D. Stage IIA2
E. Stage IIB
297 A 37 year old G3P1 (1011), 39 weeks AOG, was Once in the active phase: rate of cervical dilatation ERIC ROYD FINAL EXAM -
admitted due to labor pains. VS are normal. FH 35 should be 1.5 cm/hour for multiparas. TALAVERA, MD FEB 2015
cm, FHT 145 bpm. IE: Cervix 3-4 cm dilated 70% (TOP 1 - AUG
effaced, ROT, st -2, (+) BOW. Labor admission test 2014 MED
showed reassuring fetal heart rate pattern. 2 hours BOARDS;
after a repeat IE was done: cervix 5 cm dilated 80% TOPNOTCH MD)
effaced, ROT, st -1, (+) BOW, FHT 140 bpm. After
another 2 hours, a repeat IE was done: cervix 6 cm
dilated, 90% effaced, ROA, st-1, (+) BOW, FHT 143
bpm. EFM tracing showed a reassuring fetal heart
rate pattern with 190 Montevideo Units. How
would you diagnose the labor pattern of the
patient?
A. Normal labor pattern
B. Arrest in cervical dilatation
C. Protracted cervical dilatation
D. Prolonged latent phase
E. Arrest in descent
298 What would be the best approach for the problem The patient is in the active phase and in cephalic ERIC ROYD FINAL EXAM -
in the previous case? presentation. Adequate Montevideo Units should be TALAVERA, MD FEB 2015
A. None, it is a normal labor pattern >200, hence we can augment labor with oxytocin (TOP 1 - AUG
B. Emergency CS 2014 MED
C. Labor Induction BOARDS;
D. Oxytocin Augmentation TOPNOTCH MD)
E. Early amniotomy
299 After another 2 hours, the repeat IE showed the Dx: Arrest in cervical dilatation. Perform CS ERIC ROYD FINAL EXAM -
following findings: cervix 6 cm dilated, 90% TALAVERA, MD FEB 2015
effaced, ROA, st-1, (+) BOW, FHT 135 bpm. How (TOP 1 - AUG
would you know manage this case ? 2014 MED
A. Expectant management BOARDS;
B. Labor Induction TOPNOTCH MD)
C. Cesarean section
D. Cervical ripening
E. None of the above
300 Which of the following is NOT an indication for Other indications: thinned out LUS, severely obese ERIC ROYD FINAL EXAM -
classical cesarean section ? mothers w/c precludes safe access to LUS TALAVERA, MD FEB 2015
A. Anteriorly implanted placenta previa (TOP 1 - AUG
B. Multiple gestation 2014 MED
C. Neglected transverse lie BOARDS;
D. A large myoma in the lower uterine segment TOPNOTCH MD)
E. Invasive cervical carcinoma

TOPNOTCH MEDICAL BOARD PREP OBSTETRICS-GYNECOLOGY SUPEREXAM Page 37 of 84


For inquiries visit www.topnotchboardprep.com.ph or email us at topnotchmedicalboardprep@gmail.com
TOPNOTCH MEDICAL BOARD PREP OBSTETRICS-GYNECOLOGY SUPEREXAM
For inquiries visit www.topnotchboardprep.com.ph or email us at topnotchmedicalboardprep@gmail.com
Ite QUESTION EXPLANATION AUTHOR TOPNOTCH
m # EXAM
301 what is the minimum age of gestation wherein fetal the external genitale are beginning to show LEAN ANGELO BACK-UP
gender can be determined? definitve signs of male and female gender by the SILVERIO, MD MIDTERM EXAM
A. 8 weeks end of 12 weeks of gestation. Williams 22nd ed p93. (TOP 4 - AUG - FEB 2015
B. 10 weeks 2014 MED
C. 12 weeks BOARDS;
D. 14 weeks TOPNOTCH MD),
E. 16 weeks MD
302 which of the following is/are a function of human williams 22nd ed - pp72-73 LEAN ANGELO BACK-UP
Chorionic Gonadotropin? SILVERIO, MD MIDTERM EXAM
A. Rescue and maintenance of the corpus luteum (TOP 4 - AUG - FEB 2015
B. To promote male sexual differentiation 2014 MED
C. Promote uterine vascular dilatation and BOARDS;
myometrial smooth muscle relaxation TOPNOTCH MD),
D. stimulation of maternal thyroid hormone MD
production
E. all of the above
303 which of the following tocolytics can induce a prostaglandin inhibitors are associated with LEAN ANGELO BACK-UP
reversible type of oligohydramnios? oligohydramnios. Williams 22nd ed p 871 SILVERIO, MD MIDTERM EXAM
A. Magnesium sulfate (TOP 4 - AUG - FEB 2015
B. ritodrine 2014 MED
C. atosiban BOARDS;
D. indomethacin TOPNOTCH MD),
E. Terbutaline MD
304 which of the following fetal malformation is Williams 22nd ed p1177 LEAN ANGELO BACK-UP
strongly correlated with poorly controlled diabetes SILVERIO, MD MIDTERM EXAM
mellitus both preconceptionally and during (TOP 4 - AUG - FEB 2015
pregnancy? 2014 MED
A. Situs inversus BOARDS;
B. Spina bifida TOPNOTCH MD),
C. Caudal regression MD
D. Renal agenesis
E. Anencephaly
305 use of atenolol as an antihypertensive medication Williams 22nd ed p 1051 LEAN ANGELO BACK-UP
in pregnnacy is associated with which of the SILVERIO, MD MIDTERM EXAM
following perinatal moribidity? (TOP 4 - AUG - FEB 2015
A. Fetal growth restriction 2014 MED
B. Cardiac defects BOARDS;
C. Hyperglycemia TOPNOTCH MD),
D. Preterm birth MD
E. Renal defects
306 what is the type of placenta is assoicated with there is an increased risk with circumvallate LEAN ANGELO BACK-UP
placental abruptio and fetal hemorrhage? placentas of antepartum hemmorrhage both from SILVERIO, MD MIDTERM EXAM
A. Bipartite placenta placental abruption and from fetal hemorrhage. (TOP 4 - AUG - FEB 2015
B. Circumvallate placenta Circummarginate placenta risk are less well 2014 MED
C. Succenturiate placenta defined. Placenta membranacea can give rise to BOARDS;
D. Circummaginate placenta serious hemorrhage because of its associated TOPNOTCH MD),
E. Placenta membranacea placenta previa or accreta. MD
307 What type of forceps should be use in delivery of In general, Simpsons forceps are used to deliver the LEAN ANGELO BACK-UP
fetus with molded head ? fetus with a molded head, as is common in SILVERIO, MD MIDTERM EXAM
A. Pipers nulliparous women. The tucker-Mclane instrument (TOP 4 - AUG - FEB 2015
B. Kielland is often ued for the fetus with rounded head seen in 2014 MED
C. Chamberlain most multiparas. Wlliams 22nd ed p549. BOARDS;
D. Simpsons TOPNOTCH MD),
E. Tucker-Mclane MD
308 40 y/o G6P5 32-33 weeks AOG complains of abruptio placenta is associated with DIC, renal LEAN ANGELO BACK-UP
vaginal bleeding and severe abdominal pain. VS BP failure (ATN), hemorrhagic shock, couvelaire SILVERIO, MD MIDTERM EXAM
160/100 PR 123 RR 24 T 37.2C. PE of the uteurs, and fetal maternal hemorrhage. (TOP 4 - AUG - FEB 2015
abdomen:FH 36cm FH 97bpm, UC: every minute, 2014 MED
50 sec duration, strong. what complication is BOARDS;
associated with the above condition?? TOPNOTCH MD),
A placenta accreta MD
B. uterine atony
C. DIC
D. puerperial infection
E. all of the above
309 A 28 y/o G2P1 mother at 35 week AOG came to BPP=4 (NST = 2, fetal breathing = 0, fetal LEAN ANGELO BACK-UP
your clinic with the following biophysical profile movements = 0, fetal tone = 2, AFI =0). For a BPP of SILVERIO, MD MIDTERM EXAM
scoring: NST has 3 accelerations greater 15 bpm 4, there is probable fetal asphyxia. The intervention (TOP 4 - AUG - FEB 2015
each for more than 15 seconds, 20 seconds of fetal is to repeat BPP on the same day. If the result if less 2014 MED
breathing for 30 minutes, 2 discrete body than or equal to 6, deliver the baby. BOARDS;
movements, 1 episode of extremity extension then TOPNOTCH MD),
subsequent flexion. AFI 1.2cm. What is your next MD
plan of treatment?
A. No fetal indication for intervention. Repeat test
weekly
B. Deliver
C. Repeat BPP
D. Observe
E. none of the above

TOPNOTCH MEDICAL BOARD PREP OBSTETRICS-GYNECOLOGY SUPEREXAM Page 38 of 84


For inquiries visit www.topnotchboardprep.com.ph or email us at topnotchmedicalboardprep@gmail.com
TOPNOTCH MEDICAL BOARD PREP OBSTETRICS-GYNECOLOGY SUPEREXAM
For inquiries visit www.topnotchboardprep.com.ph or email us at topnotchmedicalboardprep@gmail.com
Ite QUESTION EXPLANATION AUTHOR TOPNOTCH
m # EXAM
310 A 32 y/o G3P2 went for her 1st prenatal check up. it seems reasonable to remove all ovarian masses LEAN ANGELO BACK-UP
Currently, she is at 18 weeks AOG. Upon doing over 10cm because of the substantive risk of SILVERIO, MD MIDTERM EXAM
routine pelvic ultrasound, there was an incidental malignancy. Tumors from 6-10 cm should be (TOP 4 - AUG - FEB 2015
finding of adnexal mass with solid and cystic carefully evaluated. For the possibility of neoplastic 2014 MED
structures measuring 13cm. What is the next best disease by ultrasound, MRI or both. If evaluation BOARDS;
management for this case? suggests a neoplasm, then resection is indicatied. if TOPNOTCH MD),
A. observation with serial sonography the corpus luteum is removed before 10 weeks, MD
B. immediate laparotomy then 17OH progesterone IM is given weekly until
C. laparotomy at 24th week AOG 10 weeks. cystic mass that are benign with less than
D. sonographic aspiration of the cyst 6cm should undergo serial sonography. Williams
E. laparotomy after delivery 22nd ed pp 965-966.
311 A 68 y/o patient came in your clinic due to intense This is a case of lichen sclerosus. Lichen simplex LEAN ANGELO BACK-UP
vulvar pruritus. Upon physical examination, you chronicus is characterized by thickened white SILVERIO, MD MIDTERM EXAM
noted that the vulva is thin, white, wrinkled tissue epithelium on vulva. It is generally localized and (TOP 4 - AUG - FEB 2015
with a cigarette paper appearance. There was unilateral. Biopsy reveals hyperkeratosis and 2014 MED
agglutination of the labia minora and prepuce. acanthosis producing thickening of the epithelium BOARDS;
Biopsy revealed hyperkeratotic epidermis with and elongation of the rete pegs. Dermis reveals TOPNOTCH MD),
epithelial atrophy, and flattening of the rete pegs. chronic inflammatory cells, fibrosis, and MD
There is cytoplasmic vacuolization of the basal collagenization. Lichen planus is characterized by
layer of cells. What is your initial diagnosis? erosive lesions at the vestibule with or without
A. Lichen simplex chronicus vaginal synechiae resulting in stenosis. Biopsy
B. Lichen sclerosus reveals mils, localized, lichenoid, chronic
C. Lichen planus inflammatory process at the epidermal-dermis
D. Squamous cell hyperplasia? junction to ulcerative process with fibrosis.
E. none of the above Squamous cell hyperplasia is circumscribed, single
or multifocal raised white lesions on the vulva or
adjacent tissue. Biopsy reveals hyperkeratosis and
acanthosis, producing thickening of the epithelium
and elongation of the rete pegs. Dermis has no
inflammatory infiltrate.
312 A 37 y/o G1P1 (1001) consulted because of severe patient is still in a reproductive age. The goal is to LEAN ANGELO BACK-UP
dysmenorrhea for 3 consecutive years. Initial remove the mass and to preserve as much as SILVERIO, MD MIDTERM EXAM
consult revealed bilateral adnexal masses about functional ovarian tissue (TOP 4 - AUG - FEB 2015
7cm each with thickened uterosacral ligament. If 2014 MED
surgery is an option, what is the most appropriate BOARDS;
procedure for this patient? TOPNOTCH MD),
A. TAHBSO MD
B. bilateral oophorocystectomy
C. fluid aspiration of the bilateral mass
D. no surgical procedure is warranted.
E.
313 A 25 y/o nullipara went for consult secondary to 7 exclusion of other causes of hyperandrogenism is LEAN ANGELO BACK-UP
years of irregular menses occuring every 2 to 3 needed before arriving at a diagnosis of PCOS. TSH, SILVERIO, MD MIDTERM EXAM
months with menorrhaggia lasting for 8 days. Her prolactin should also be checked. Testosterone (TOP 4 - AUG - FEB 2015
BMI is 32kg/m2. BP 130/90. PE shows darkened levels >200ng/dl may suggest adrenal tumor. FSH 2014 MED
skin flexures particularly in the dorsal neck. hairs and estradiol is requested for work up of POF. BOARDS;
are also noted in the upper lip, chin and anterior Although approximately 40-50% of PCOS patient TOPNOTCH MD),
chest. which of the following is not needed as part have elevated LH/FSH ratio, it is not required for MD
of her work up? the diagnosis of PCOS.
A. LH/FSH ratio
B. TSH, T4
C. Testosterone
D. 17-OH progesterone
E. estradiol
314 A 24 y/o sexually active nullipara went for consult patient is suffering from severe PID and at the same LEAN ANGELO BACK-UP
secondary to 4 days of fever and continuous lower time, early pregnancy ( as noted by lack of UTZ SILVERIO, MD MIDTERM EXAM
abdominal pain. PE noted direct tenderness on (B) findings with (+) hcg). The most appropriate (TOP 4 - AUG - FEB 2015
adnexal region without palpable mass. IE: (+) treatment for this case is IV clindamycin and 2014 MED
cervical motion tenderness. Speculum ex: gentamicin. B, C- is ineffective since resistance is BOARDS;
hyperemic cervix with purulent discharge. common in N gonnorrhea infection in the setting of TOPNOTCH MD),
pregnancy test is positive, urinalysis shows are PID. D- fluoroquinolone should never be given in MD
within normal limits, CBC: neutrophilic pregnant patient. furthermore, this combination
leukocytosis. pelvic ultrasound is unremarkable. therapy is prescribed in milder cases of PID.
what is the next best step in terms of management
?
A . IV clindamycin and gentamicin
B. IV ampicillin and gentamicin
C. IM spectinomycin and ceftriaxone
D. levofloxacin and metronidazole
E. Laparoscopy
315 True about cervical cancer screening except? A- recommendation indicates that continued Pap LEAN ANGELO BACK-UP
A. Pap testing should be discontinued in patients testing should be done on hysterectomized patients SILVERIO, MD MIDTERM EXAM
who underwent hysterectomy regardless of if she has a history of HSIL, immunocompromised (TOP 4 - AUG - FEB 2015
immune status or DES exposure in utero. D- this is the basis of 2014 MED
B. Invasive cervical cancer is never been found in screening at 21 years old even in the absence of BOARDS;
women less than 21 years old sexual activity. TOPNOTCH MD),
C. HPV DNA testing and Pap smear should be MD
tested in women >30years old
D. Fomite transmission can cause HPV infection.
E. none of the above
316 A 29 y/o G4P4 asked you about the best method of the patient who is currently on breastfeeding and LEAN ANGELO BACK-UP
contraception. She is currently breastfeeding her 4 has questionable compliance should be advised to SILVERIO, MD MIDTERM EXAM
month old child and admitted to have a very tight have progesterone IUD as a form of contraception. (TOP 4 - AUG - FEB 2015
schedule since she is a single parent. Patient is Progesterone will not affect the quantity and 2014 MED
working both as a gym instructor and a fashion quality of the milk in contrast to estrogen BOARDS;
model. containing contraceptive methods. TOPNOTCH MD),
TOPNOTCH MEDICAL BOARD PREP OBSTETRICS-GYNECOLOGY SUPEREXAM Page 39 of 84
For inquiries visit www.topnotchboardprep.com.ph or email us at topnotchmedicalboardprep@gmail.com
TOPNOTCH MEDICAL BOARD PREP OBSTETRICS-GYNECOLOGY SUPEREXAM
For inquiries visit www.topnotchboardprep.com.ph or email us at topnotchmedicalboardprep@gmail.com
Ite QUESTION EXPLANATION AUTHOR TOPNOTCH
m # EXAM
A. progesterone impregnated IUD MD
B. progestin only pill
C. Depot medroxyprogesterone acetate
D. combined OCP
E. any of the above
317 Which of the following is true about dysgerminoma Dysgerminoma is the most common malignant LEAN ANGELO BACK-UP
? germ cell tumors primarily occuring in women SILVERIO, MD MIDTERM EXAM
A. The median age of presentation is 45 years old younger than 30 years old. Unlike other ovarian (TOP 4 - AUG - FEB 2015
B. Bilaterality is seen as high as 80% of the cases malignancy, bilaterality is more commonly seen in 2014 MED
C. Analogous to seminoma in the male dysgerminoma with an estimate of around 10% of BOARDS;
counterpart the cases. Only 10% of the tumor express HcG. ( TOPNOTCH MD),
D. Approximately 90% of the tumors are highly SIMILAR TO PREVIOUS BOARD EXAM MD
differentiated and HcG producing. CONCEPT/PRINCIPLE). Compre Gyne 5th ed pa 867
E. all of the above.
318 Which of the following characterizes type 2 Type 1 endometrial carcinoma - Grade 1–2, LEAN ANGELO BACK-UP
endometrial carcinoma? Superficial or no myometrial invasion, Coexisting SILVERIO, MD MIDTERM EXAM
A. Induced by estrogen endometrial hyperplasia, Perimenopausal, Induced (TOP 4 - AUG - FEB 2015
B. Post menopausal by estrogen, Responds to progestins, Good 2014 MED
C. Responds to progestins prognosis; Type 2 - High grade (grade 3, papillary BOARDS;
D. Good prognosis serous, clear cell), Deep myometrial invasion, TOPNOTCH MD),
E. all of the above Postmenopausal women, Not induced by estrogen, MD
Does not respond to progestins, Poor prognosis
319 what chromosomal abnormality accounts for the ( SIMILAR TO PREVIOUS BOARD EXAM LEAN ANGELO BACK-UP
majority of spontaneous abortion? CONCEPT/PRINCIPLE). The most common SILVERIO, MD MIDTERM EXAM
A. triploidy abormality seen in spontaneous abortion is (TOP 4 - AUG - FEB 2015
B. trisomy trisomy. While the most common single anomaly 2014 MED
C. 45 XO seen in abortion is 45XO. BOARDS;
D. translocation TOPNOTCH MD),
E. None of the above MD
320 which of the following BPP parameter is the last to During Hypoxia, the first parameter to be affected is LEAN ANGELO BACK-UP
be affected by hypoxia ? NST while the last is the fetal tone. SILVERIO, MD MIDTERM EXAM
A. Fetal breathing (TOP 4 - AUG - FEB 2015
B. Fetal movement 2014 MED
C. Fetal tone BOARDS;
D. Amniotic fluid volume TOPNOTCH MD),
E. NST MD
321 Component/s of active management of labor SIMILAR TO PREVIOUS BOARD EXAM DEBBIE ROSE BACK-UP
is/are: CONCEPT/PRINCIPLE. Williams Obstetrics 23rd TANENGSY, MD MIDTERM EXAM
A. oxytocin and cesarean delivery edition p.405 (TOP 5 - AUG - FEB 2015
B. cesarean delivery 2014 MED
C. oxytocin and amniotomy BOARDS;
D. all of the above TOPNOTCH MD)
E. none of the above
322 The most common explanation for fetal Williams Obstetrics 23rd edition p.413 DEBBIE ROSE BACK-UP
tachycardia: TANENGSY, MD MIDTERM EXAM
A. fetal compromise (TOP 5 - AUG - FEB 2015
B. fetal cardiac dysrhythmias 2014 MED
C. maternal administration of atropine BOARDS;
D. maternal fever from infection TOPNOTCH MD)
323 Which is a true statement with regards to The baselines HR becomes more physiologically DEBBIE ROSE BACK-UP
variability? fixed as rate increases. Good variability is not TANENGSY, MD MIDTERM EXAM
A. The baseline HR becomes more variable as rate necessarily reassuring. Normal variability (TOP 5 - AUG - FEB 2015
increases. (moderate): 6-25 bpm. Baseline variability is an 2014 MED
B. Good variability is an assurance of fetal well- important index of cardiovascular function. BOARDS;
being. Williams Obstetrics 23rd edition pp.415-417 TOPNOTCH MD)
C. Normal variability: 10-30 bpm
D. Baseline variability is an important index of
respiratory function.
E. A reduced baseline HR variability is the single
most reliable sign of fetal compromise.
324 What is the most important factor when Williams Obstetrics 23rd edition p.569 DEBBIE ROSE BACK-UP
considering a trial of labor after cesarean delivery? TANENGSY, MD MIDTERM EXAM
A. type of prior incision (TOP 5 - AUG - FEB 2015
B. history of preterm cesarean delivery 2014 MED
C. interdelivery interval BOARDS;
D. maternal age TOPNOTCH MD)
E. maternal parity
325 True regarding trial of labor after cesarean: Williams Obstetrics 23rd edition p.568 DEBBIE ROSE BACK-UP
A. Twin pregnancy may increase the risk of uterine TANENGSY, MD MIDTERM EXAM
rupture in VBAC. (TOP 5 - AUG - FEB 2015
B. Obesity has no effect in the success of VBAC. 2014 MED
C. The most favorable prognositc factor is prior BOARDS;
vaginal delivery. TOPNOTCH MD)
D. A woman with a previous low segment rupture
has a 16% recurrence risk.
E. Women with prior vertical incision in the lower
uterine segment without fundal extension are not
candidates for VBAC.
326 What is the most common manifestation of Williams Obstetrics 23rd edition p.222 DEBBIE ROSE BACK-UP
postabortal infection? TANENGSY, MD MIDTERM EXAM
A. septicemia (TOP 5 - AUG - FEB 2015
B. endocarditis 2014 MED
C. parametritis BOARDS;
D. peritonitis TOPNOTCH MD)
E. endomyometritis

TOPNOTCH MEDICAL BOARD PREP OBSTETRICS-GYNECOLOGY SUPEREXAM Page 40 of 84


For inquiries visit www.topnotchboardprep.com.ph or email us at topnotchmedicalboardprep@gmail.com
TOPNOTCH MEDICAL BOARD PREP OBSTETRICS-GYNECOLOGY SUPEREXAM
For inquiries visit www.topnotchboardprep.com.ph or email us at topnotchmedicalboardprep@gmail.com
Ite QUESTION EXPLANATION AUTHOR TOPNOTCH
m # EXAM
327 Which confers the highest risk for ectopic Williams Obstetrics 23rd edition p.239 DEBBIE ROSE BACK-UP
pregnancy? TANENGSY, MD MIDTERM EXAM
A. infertility (TOP 5 - AUG - FEB 2015
B. prior tubal damage 2014 MED
C. multiple sexual partners BOARDS;
D. previous genital infection TOPNOTCH MD)
E. intrauterine device use
328 A physiologic change in pregnancy: Respiratory rate is unchanged, creatinine is DEBBIE ROSE BACK-UP
A. transient loss of accommodation decreased, tidal volume is increased, and creatinine TANENGSY, MD MIDTERM EXAM
B. increase in serum creatinine clearance is increased in pregnancy. Williams (TOP 5 - AUG - FEB 2015
C. decreased creatinine clearance Obstetrics 23rd edition p.121-130 2014 MED
D. tachypnea BOARDS;
E. unchanged tidal volume TOPNOTCH MD)
329 True regarding gestational diabetes: Fetal anomalies are not increased in GD. Elective DEBBIE ROSE BACK-UP
A. GD causes an increase in fetal anomalies. cesarean delivery has no effect on the incidence of TANENGSY, MD MIDTERM EXAM
B. Fetus' brain is unaffected by macrosomia. brachial plexus injury. There is a 50% likelihood (TOP 5 - AUG - FEB 2015
C. Elective cesarean delivery was found to that women with GD will develop overt DM within 2014 MED
significantly decrease the incidence of brachial 20 years. Williams Obstetrics 23rd edition BOARDS;
plexus injury. pp.1111-1112 TOPNOTCH MD)
D. There is a 15% likelihood that women with
gestational diabetes to develop overt diabetes
mellitus within 20 years.
E. There is a high risk of fetal death.
330 Which satisfies one of the two clinical criteria for Williams Obstetrics 23rd edition p.1153 DEBBIE ROSE BACK-UP
the diagnosis of antiphospholid antibody TANENGSY, MD MIDTERM EXAM
syndrome? (TOP 5 - AUG - FEB 2015
A. >/= 1 unexplained fetal losses after 10 weeks 2014 MED
B. >/= 2 consecutive miscarriages < 10 weeks BOARDS;
C. preterm delivery for severe preeclampsia at 35 TOPNOTCH MD)
completed weeks
D. preterm delivery for placental insufficiency at
35 completed weeks
E. none of the above
331 ACOG recommends single dose treatment of There is no concensus regarding giving antenatal DEBBIE ROSE BACK-UP
antenatal steroids for women with preterm steroids at 32-24 weeks. Steroids are not TANENGSY, MD MIDTERM EXAM
membrane rupture at an age of gestation of: recommended at <24 weeks AOG. Williams (TOP 5 - AUG - FEB 2015
A. 32-34 weeks Obstetrics 23rd edition p.820 2014 MED
B. <24 weeks BOARDS;
C. 20-30 weeks TOPNOTCH MD)
D. 24-32 weeks
332 Most frequently occurring congenital malformation Williams Obstetrics 23rd edition p.1114 table 52-8 DEBBIE ROSE BACK-UP
in infants of women with overt diabetes mellitus: TANENGSY, MD MIDTERM EXAM
A. situs inversus (TOP 5 - AUG - FEB 2015
B. anencephaly 2014 MED
C. cardiac abnormalities BOARDS;
D. rectal atresia TOPNOTCH MD)
E. caudal regression
333 True regarding should dystocia: There is no evidence that any one maneuver is DEBBIE ROSE BACK-UP
A. McRoberts is a maneuver most likely to release superior to another in releasing an impacted TANENGSY, MD MIDTERM EXAM
an impacted shoulder. shoulder. McRoberts is a reasonable INITIAL (TOP 5 - AUG - FEB 2015
B. McRoberts is a reasonable approach when all approach in shoulder dystocia. Williams Obstetrics 2014 MED
other maneuvers fail. 23rd edition p.485 BOARDS;
C. McRoberts requires 2 assistants to perform. TOPNOTCH MD)
D. all of the above
E. A & B
334 A 40-year-old woman reports heavy and prolonged DEBBIE ROSE BACK-UP
menstrual bleeding, associated with TANENGSY, MD MIDTERM EXAM
dysmenorrhea. On examination, the uterus is noted (TOP 5 - AUG - FEB 2015
to be diffusely enlarged and soft, with its mobility 2014 MED
not limited. There is no associated adnexal BOARDS;
pathology. What is the most likely diagnosis? TOPNOTCH MD)
A. endometriosis
B. ectopic pregnancy
C. leiomyoma
D. adenomyosis
335 What is considered abnormal after the first two Abnormal: >42 days cycle, <21 days cycle, bleeding DEBBIE ROSE BACK-UP
years from onset of menarche? >7 days. Average blood loss: 35 cc. Berek & Novaks TANENGSY, MD MIDTERM EXAM
A. cycle lasting 22 days Gynecology 14th edition pp.446-447 (TOP 5 - AUG - FEB 2015
B. menstrual bleeding lasting 10 days 2014 MED
C. blood loss amounting 40 cc per cycle BOARDS;
D. cycles lasting 30 days TOPNOTCH MD)
E. none of the above
336 The most common symptom in a patient with Berek & Novaks Gynecology 14th edition p.1404 DEBBIE ROSE BACK-UP
cervical cancer: TANENGSY, MD MIDTERM EXAM
A. dyspareunia (TOP 5 - AUG - FEB 2015
B. vaginal bleeding 2014 MED
C. foul vaginal discharge BOARDS;
D. weight loss TOPNOTCH MD)
E. pruritus
337 True of breast masses: Treatment for phyllodes tumor is wide local DEBBIE ROSE BACK-UP
A. It is easier to distinguish between benign excision. Fibroadenomas don’t cause skin dimpling TANENGSY, MD MIDTERM EXAM
phyllodes from fibroadenomas than benign from or nipple retraction. Phyllodes tumors are rarely (TOP 5 - AUG - FEB 2015
malignant phyllodes. bilateral. Berek & Novaks Gynecology 14th edition 2014 MED
B. Treatment for phyllodes tumor is radiation pp.657-659 BOARDS;
therapy. TOPNOTCH MD)
C. Fibroadenomas are not associated with an
TOPNOTCH MEDICAL BOARD PREP OBSTETRICS-GYNECOLOGY SUPEREXAM Page 41 of 84
For inquiries visit www.topnotchboardprep.com.ph or email us at topnotchmedicalboardprep@gmail.com
TOPNOTCH MEDICAL BOARD PREP OBSTETRICS-GYNECOLOGY SUPEREXAM
For inquiries visit www.topnotchboardprep.com.ph or email us at topnotchmedicalboardprep@gmail.com
Ite QUESTION EXPLANATION AUTHOR TOPNOTCH
m # EXAM
increased risk of breast cancer.
D. Fibroadenomas are benign despite causing some
skin dimpling and nipple retraction
E. Phyllodes tumors are difficult to distinguish
clinically from fibroadenomas but the former are
usually bilateral.
338 An involuntary reflex involving contraction of Novaks p.333 DEBBIE ROSE BACK-UP
pelvic muscles precipitated by attempts at vaginal TANENGSY, MD MIDTERM EXAM
entry is known as: (TOP 5 - AUG - FEB 2015
A. dyspareunia 2014 MED
B. genital arousal disorder BOARDS;
C. orgasmic dysfunction TOPNOTCH MD)
D. vaginismus
E. sexual assault
339 The probability of achieving a live birth within a Fecundability - the probability of achieving a DEBBIE ROSE BACK-UP
single cycle is termed: pregnancy within a single cycle. Berek & Novaks TANENGSY, MD MIDTERM EXAM
A. fecundability Gynecology 14th edition p.1186 (TOP 5 - AUG - FEB 2015
B. fertility 2014 MED
C. fecundity BOARDS;
D. gestation TOPNOTCH MD)
340 The fecundability of a normal couple is estimated Berek & Novaks Gynecology 14th edition p.1186 DEBBIE ROSE BACK-UP
at: TANENGSY, MD MIDTERM EXAM
A. 10-15% (TOP 5 - AUG - FEB 2015
B. 15-20% 2014 MED
C. 20-25% BOARDS;
D. 25-30% TOPNOTCH MD)
E. 30-35%
341 A 33 year old G6P5 (5005) consulted due to This is a possible case of placenta previa (patient is JESSICA MAE BACK-UP
vaginal bleeding. She is currently on her 30th week G5P5, breech presentation), therefore internal SANCHEZ, MD MIDTERM EXAM
AOG. Past OB History were all NSD. On PE, BP - exam is contraindicated. (TOP 4 - AUG - FEB 2015
110/70; PR - 78/min; RR - 20/min; T - 36.7C; FH - 2014 MED
31cm; FHT - 132/min; LM3 - breech, no uterine BOARDS;
contractions. Speculum exam showed brownish TOPNOTCH MD)
discharge per cervical os. Which of the following is
contraindicated in this patient?
A. Digital exam
B. Speculum exam
C. Transvaginal ultrasound
D. All of the above
342 What is the single most important risk factor for JESSICA MAE BACK-UP
intrauterine infection? SANCHEZ, MD MIDTERM EXAM
A. Age (TOP 4 - AUG - FEB 2015
B. Parity 2014 MED
C. PROM BOARDS;
D. Route of delivery TOPNOTCH MD)
343 What is the most important criterion for the JESSICA MAE BACK-UP
diagnosis of postpartum metritis? SANCHEZ, MD MIDTERM EXAM
A. Lochia (TOP 4 - AUG - FEB 2015
B. Dysuria 2014 MED
C. Fever BOARDS;
D. Tenderness over the wound TOPNOTCH MD)
344 A 25 year old G2P1 (1001) just delivered by low Since the patient underwent forceps extraction, the JESSICA MAE BACK-UP
forceps extraction to a live baby boy with BW 6 lbs. most likely cause of her bleeding would be due to SANCHEZ, MD MIDTERM EXAM
Continued oozing of bright red blood was noted lacerations. Uterine atony is unlikely since she is (TOP 4 - AUG - FEB 2015
with BP dropping to 80/50, CR 120/min, RR just only gravida 2, the birth weight is 6 lbs, and 2014 MED
20/min, T 37C. In 2009, she had a low transverse there are no other clues in the history that would BOARDS;
cesarean section for placenta previa. What is the make the patient at risk for this condition. Rupture TOPNOTCH MD)
most probable cause of the bleeding? of CS scar is also unlikely because her previous CS
A. Abruptio placenta was a low transverse type.
B. Vaginal lacerations
C. Rupture of CS scar
D. Uterine atony
345 Which of the following is the most common cause JESSICA MAE BACK-UP
of postpartum hemorrhage? SANCHEZ, MD MIDTERM EXAM
A. Retained placental cotyledons (TOP 4 - AUG - FEB 2015
B. Coagulopathies 2014 MED
C. Uterine atony BOARDS;
D. Lacerations TOPNOTCH MD)
346 What is the most common cause of spontaneous JESSICA MAE BACK-UP
abortions? SANCHEZ, MD MIDTERM EXAM
A. Maternal systemic disease (TOP 4 - AUG - FEB 2015
B. Chromosomal anomalies 2014 MED
C. Uterine defects BOARDS;
D. Immunologic factors TOPNOTCH MD)
347 What is the best approach in the management of Laporotomy is usually indicated for patient who are JESSICA MAE BACK-UP
unruptured tubal pregnancy 5-6 weeks with a live hemodynamically unstable or for those with SANCHEZ, MD MIDTERM EXAM
fetus? cornual pregnancies. Methotrexate therapy is only (TOP 4 - AUG - FEB 2015
A. Laparoscopy done when the ff. criteria are met: 2014 MED
B. Laparotomy patient should be hemodynamically stable with no BOARDS;
C. Methotrexate signs of hemoperitoneum TOPNOTCH MD)
D. Watchful observation gestational sac should be <=4cm
no fetal cardiac activity
no tubal rupture
beta HCG less than 5000 mIU/mL

TOPNOTCH MEDICAL BOARD PREP OBSTETRICS-GYNECOLOGY SUPEREXAM Page 42 of 84


For inquiries visit www.topnotchboardprep.com.ph or email us at topnotchmedicalboardprep@gmail.com
TOPNOTCH MEDICAL BOARD PREP OBSTETRICS-GYNECOLOGY SUPEREXAM
For inquiries visit www.topnotchboardprep.com.ph or email us at topnotchmedicalboardprep@gmail.com
Ite QUESTION EXPLANATION AUTHOR TOPNOTCH
m # EXAM
348 Which of the following is a marker of chronic JESSICA MAE BACK-UP
hypoxia? SANCHEZ, MD MIDTERM EXAM
A. Non stress test (TOP 4 - AUG - FEB 2015
B. Contraction stress test 2014 MED
C. Amniotic fluid index BOARDS;
D. Fetal tone TOPNOTCH MD)
349 A 23 year old G1P0 on her 15th week of pregnancy JESSICA MAE BACK-UP
consulted because of fever and right flank pains. T - SANCHEZ, MD MIDTERM EXAM
38.9C; BP - 110/70; PR - 85/min; RR - 20/min; (TOP 4 - AUG - FEB 2015
FHT 137/min by Doppler. The rest of the PE was 2014 MED
essentially normal. What is the probably diagnosis? BOARDS;
A. Asymptomatic bacteriuria TOPNOTCH MD)
B. Cystitis
C. Acute pyelonephritis
D. Acute glomerulonephritis
350 Which of the following is not an imminent sign of JESSICA MAE BACK-UP
eclampsia? SANCHEZ, MD MIDTERM EXAM
A. Severe headache (TOP 4 - AUG - FEB 2015
B. Visual disturbances 2014 MED
C. Epigastric pain BOARDS;
D. Facial edema TOPNOTCH MD)
351 What is the anaesthesia of choice for pregnant JESSICA MAE BACK-UP
cardiac patients? SANCHEZ, MD MIDTERM EXAM
A. General anesthesia (TOP 4 - AUG - FEB 2015
B. Epidural anesthesia 2014 MED
C. Subarachnoid block BOARDS;
D. Spinal anesthesia TOPNOTCH MD)
352 Conjoined twinning occurs when fertilisation JESSICA MAE BACK-UP
occurs after the first: A. 48 hours SANCHEZ, MD MIDTERM EXAM
B. 5 days (TOP 4 - AUG - FEB 2015
C. 8 days 2014 MED
D. 12 days BOARDS;
TOPNOTCH MD)
353 In evaluating a 28 year old patient complaining of Always rule out pregnancy first in patients who are JESSICA MAE BACK-UP
vaginal bleeding, what information is most of reproductive age SANCHEZ, MD MIDTERM EXAM
important? (TOP 4 - AUG - FEB 2015
A. Age at menarche 2014 MED
B. History of dysmenorrhea BOARDS;
C. Last menstrual period TOPNOTCH MD)
D. OCP use
354 What is the usual cause of dysfunctional uterine JESSICA MAE BACK-UP
bleeding in the postmenarcheal and SANCHEZ, MD MIDTERM EXAM
premenopausal age groups? (TOP 4 - AUG - FEB 2015
A. Hematologic disorder 2014 MED
B. ncreased prostaglandin BOARDS;
C. Anovulation TOPNOTCH MD)
D. Hyperprolactinemia
355 A 16 year old patient consulted because of primary (+) breast (-) uterus: Mullerian agenesis or JESSICA MAE BACK-UP
amenorrhea. On PE, her breasts are fully Testicular feminization. SANCHEZ, MD MIDTERM EXAM
developed. Pelvic exam: normal external genitalia; (TOP 4 - AUG - FEB 2015
a cotton tip applicator could be inserted 3 cm from 2014 MED
the vulvar introitus. On rectal exam, there was BOARDS;
nothing palpated. What is the most likely cause of TOPNOTCH MD)
her problem?
A. Ovarian agenesis
B. Transverse vaginal septum
C. Mullerian agenesis
D. Imperforate hymen
356 What procedure should be done next when a Pap JESSICA MAE BACK-UP
smear result shows “high-grade squamous SANCHEZ, MD MIDTERM EXAM
intraepithelial lesion”? (TOP 4 - AUG - FEB 2015
A. Repeat Pap smear 2014 MED
B. Cone biopsy BOARDS;
C. Colposcopy TOPNOTCH MD)
D. Random punch biopsies
357 Which histologic type of cervical cancer is JESSICA MAE BACK-UP
associated with a tendency for late diagnosis and a SANCHEZ, MD MIDTERM EXAM
“barrel-shaped” morphology? (TOP 4 - AUG - FEB 2015
A. Squamous carcinoma 2014 MED
B. Sarcoma BOARDS;
C. Adenocarcinoma TOPNOTCH MD)
D. Lymphoma
358 A 16 year old, sexually active nulligravid, Vesicular lesions, shallow ulcers, and most notably JESSICA MAE BACK-UP
complains of vulvar pain. She noted yellowish pain, are all characteristic of a herpes infection. SANCHEZ, MD MIDTERM EXAM
green foul-smelling vaginal discharge for the past 3 (TOP 4 - AUG - FEB 2015
days, and since 1 day ago has been experiencing 2014 MED
pain at the start of urination. On PE: (+) firm, BOARDS;
tender, bilateral inguinal lymph nodes, (+) multiple TOPNOTCH MD)
vesicular lesions and shallow superficial ulcers.
What is the most likely diagnosis for this case?
A. Chancroid
B. Herpes
C. Syphilis
D. Lymphogranuloma venereum

TOPNOTCH MEDICAL BOARD PREP OBSTETRICS-GYNECOLOGY SUPEREXAM Page 43 of 84


For inquiries visit www.topnotchboardprep.com.ph or email us at topnotchmedicalboardprep@gmail.com
TOPNOTCH MEDICAL BOARD PREP OBSTETRICS-GYNECOLOGY SUPEREXAM
For inquiries visit www.topnotchboardprep.com.ph or email us at topnotchmedicalboardprep@gmail.com
Ite QUESTION EXPLANATION AUTHOR TOPNOTCH
m # EXAM
359 A 23 year old, nulligravid, recently married, This is most likely a case of bacterial vaginosis, the JESSICA MAE BACK-UP
consulted because of foul smelling, copious grayish treatment for which is metronidazole. SANCHEZ, MD MIDTERM EXAM
vaginal discharge for the past week. On PE: (TOP 4 - AUG - FEB 2015
speculum exam showed copious creamy, grayish, 2014 MED
foul-smelling vaginal discharge; cervix is smooth BOARDS;
and erythematous. What is the appropriate TOPNOTCH MD)
treatment for this patient?
A. None
B. Metronidazole
C. Doxycycline
D. Fluconazole
360 What is the possible aetiology of the PID of a JESSICA MAE BACK-UP
woman who is not responding to the usual SANCHEZ, MD MIDTERM EXAM
antibiotic regimen for acute bacterial PID? (TOP 4 - AUG - FEB 2015
A. Gonococcal 2014 MED
B. Chlamydia BOARDS;
C. Mycoplasma TOPNOTCH MD)
D. Tuberculous
361 A patient is diagnosed to have Mayer-Rokitansky This syndrome is characterized by congenital MAIRRE JAMES BACK-UP
syndrome. This patient will have: absence of the vagina and uterus. These individuals GADDI, MD (TOP MIDTERM EXAM
A. 46XY karyotype have a 46,XX karyotype. Up to 50% of women with 4 - AUG 2013 - FEB 2015
B. Normal ovaries Mullerian agenesis have concur- rent urinary tract MED BOARDS;
C. Infrequent association with skeletal anomalies. One study described a 12% incidence of TOPNOTCH MD)
abnormalities skeletal anomalies, usually involving congenital
D. Normal uterus fusion or absence of vertebrae in these patients.
E. No involvement of the urinary tract Katz 6th pg 191-192
362 A 22 year-old primigravid at 35 weeks age of MAIRRE JAMES BACK-UP
gestation was brought to the emergency room due GADDI, MD (TOP MIDTERM EXAM
to severe abdominal pain. Vital signs were BP = 4 - AUG 2013 - FEB 2015
160/100, HR 104, RR 22, T 37C. This was allegedly MED BOARDS;
her first episode of elevated BP. Physical TOPNOTCH MD)
examination revealed a fundic height of 29cm. Stat
albumin was 1+ while laboratory tests revealed
elevated liver enzymes. What is your diagnosis?
A. Gestational Hypertension
B. Mild preeclampsia
C. Severe preeclampsia
D. Chronic hypertension
E. HELLP syndrome
363 A 25 year-old, G1P0 is on the 12th hour of labor, This is a case of failure of descent. Preferred MAIRRE JAMES BACK-UP
cervix 6cm dilated, at station -2 with contracted treatment in this case would be cesarean delivery if GADDI, MD (TOP MIDTERM EXAM
pelvis. Amniotomy was done. After 2 hours, cervix with CPD and oxytocin if without CPD. Williams 4 - AUG 2013 - FEB 2015
is now fully dilated, still at station -2. What is the 23rd pg 465 MED BOARDS;
preferred management for this case? TOPNOTCH MD)
A. Sedation and observation
B. Augmentation with oxytocin
C. Forceps delivery under anesthesia
D. Abdominal delivery
E. None of the above
364 A 1 year old was brought to your clinic due to Congenital adrenal hyperplasia is the most common MAIRRE JAMES BACK-UP
ambiquous genitalia. The patient has normal cause of androgenic excess in fetuses with female GADDI, MD (TOP MIDTERM EXAM
ovaries with normal development of the uterus. pseudohermaphroditism. In this condition, 4 - AUG 2013 - FEB 2015
There was also note of virilization with scrotal müllerian-inhibiting substance is not produced. MED BOARDS;
formation and penile urethra. Karyotyping done Androgen exposure is excessive, but variable, for a TOPNOTCH MD)
revealed a 46XX karyotype. The patient most fetus genetically predestined to be female. The
probably has: karyotype is 46,XX and ovaries are present. Because
A. Congential adrenal hyperplasia müllerian-inhibiting substance is not produced, the
B. Mullerian agenesis uterus, fallopian tubes, and upper vagina develop. If
C. Androgen insensitivity syndrome androgen levels increase earlier in embryonic
D. Testicular feminization syndrome develop- ment, then more severe virilization can be
E. None of the above seen. This includes formation of labioscrotal folds;
development of a urogenital sinus, in which the
vagina empties into the posterior urethra; and
development of a penile urethra with scrotal
formation.
365 A 36 year old comes to your clinic because of Premature ovarian failure (POI) or premature MAIRRE JAMES BACK-UP
amenorrhea. Her last normal menstrual period was ovarian insufficiency (POI), which is the more GADDI, MD (TOP MIDTERM EXAM
10 months ago. Physical and internal examination recently used term, is defined as 4 - AUG 2013 - FEB 2015
were essentially normal. Endocrine assays reveal hypergonadotropic ovarian failure occurring prior MED BOARDS;
estradiol levels as <40pg/ml and FSH was to age 40. POI has occurred in 5% to 10% of women TOPNOTCH MD)
>30mIU/ml. What is your likely diagnosis? who are evaluated for amenorrhea. Patients will
A. Hypothalamic-pituitary failure have abnormally low levels of estrogen with high
B. Polycystic ovarian syndrome levels of FSH.
C. Premature ovarian failure
D. Hypothalamic-pituitary dysfunction
E.
366 TRUE of gestational diabetes: Screening should be done between 24-28 weeks in MAIRRE JAMES BACK-UP
A. Screening should be done between 28-32 weeks those women not known to have glucose GADDI, MD (TOP MIDTERM EXAM
in those women not known to have glucose intolerance earlier in pregnancy. Maternal 4 - AUG 2013 - FEB 2015
intolerance earlier in pregnancy. hyperglycemia prompts fetal hyperinsulinemia MED BOARDS;
B. Maternal hyperglycemia prompts fetal particularly during the second half of gestation, TOPNOTCH MD)
hyperinsulinemia particularly during the first half which in turn stimulates excessive somatic growth.
of gestation, which in turn stimulates excessive Women diagnosed with GDM have a 50-percent
somatic growth. likelihood of developing overt diabetes within 20
C. Women diagnosed with GDM have a 25-percent years. Williams 23rd pg 1106-1112
likelihood of developing overt diabetes within 20
TOPNOTCH MEDICAL BOARD PREP OBSTETRICS-GYNECOLOGY SUPEREXAM Page 44 of 84
For inquiries visit www.topnotchboardprep.com.ph or email us at topnotchmedicalboardprep@gmail.com
TOPNOTCH MEDICAL BOARD PREP OBSTETRICS-GYNECOLOGY SUPEREXAM
For inquiries visit www.topnotchboardprep.com.ph or email us at topnotchmedicalboardprep@gmail.com
Ite QUESTION EXPLANATION AUTHOR TOPNOTCH
m # EXAM
years.
D. Women with gestational diabetes who do not
require insulin seldom require early delivery or
other interventions.
E. All of the above

367 A 27 year old G3P2 on her 36th week of gestation, Placental abruption presents with vaginal bleeding MAIRRE JAMES BACK-UP
with a history of chronic hypertension, had severe in 78 percent, uterine tenderness or back pain in 66 GADDI, MD (TOP MIDTERM EXAM
abdominal pains of several hours with percent, and fetal distress in 60 percent of patients. 4 - AUG 2013 - FEB 2015
accompanying vaginal bleeding. Vital signs are BP Other findings included frequent uterine MED BOARDS;
= 140/90, PR = 108bpm, RR = 22/min and T = 37.2 contractions and persistent uterine hypertonus. TOPNOTCH MD)
C. Abdomen is tender and cervix is 3cm dilated, Williams 23rd pg 765-766
membranes are bulging. CTG is non-reactive with
hypertonic uterine contractions. The most likely
diagnosis is:
A. Normal early labor
B. Uterine rupture
C. Placental abruption
D. Placenta previa
E. None of the above
368 A 24 year-old primigravid complains of an ulcer in Lymphogranuloma venereum (LGV) is a chronic MAIRRE JAMES BACK-UP
the genital area. According to her, it started as a infection of lymphatic tissue produced by GADDI, MD (TOP MIDTERM EXAM
shallow painless ulcer three weeks prior to consult. Chlamydia trachomatis. There are three distinct 4 - AUG 2013 - FEB 2015
On physical examination, you noted tender phases of vulvar and perirectal LGV. The primary MED BOARDS;
inguinal lymph nodes. What is the most likely infection is a shallow painless ulcer that heals rap- TOPNOTCH MD)
etiology? idly without therapy. One to 4 weeks after the
A. Chlamydia trachomatis primary infection, a secondary phase marked by
B. Herpes simplex virus painful adenopathy develops in the inguinal and
C. Hemophilus ducreyi peri- rectal areas. When the disease is untreated,
D. Klebsiella granulomatosis the infected nodes become increasingly tender,
E. Treponema pallidum enlarged, matted together, and ad- herent to
overlying skin, forming a bubo (tender lymph
nodes). Katz 6th pg 526
369 A 34 year old G2P2 presents at the OPD with white Bacterial vaginosis and Trichomoniasis will have a MAIRRE JAMES BACK-UP
thick vaginal discharge associated with vaginal vaginal pH greater than 4.5. GADDI, MD (TOP MIDTERM EXAM
erythema and pruritus. Measurement of vaginal 4 - AUG 2013 - FEB 2015
acidity reveals a pH of 4. What is your diagnosis? MED BOARDS;
A. Bacterial Vaginosis TOPNOTCH MD)
B. Trichomoniasis
C. Vulvovaginal candiasis
D. Atrophic Vaginitis
E. Gonorrhea
370 A 26 year-old G2P1 consulted at the Emergency MAIRRE JAMES BACK-UP
room. On internal examination, you noted that the GADDI, MD (TOP MIDTERM EXAM
cervix is 3 cm dilated, at midposition, with 40% 4 - AUG 2013 - FEB 2015
effacement, with a soft consistency, and at station - MED BOARDS;
2. What is the Bishop score? TOPNOTCH MD)
A. 6
B. 7
C. 8 3cm - 2, midposition - 1, 40% - 1, soft - 2, station -2
D. 9 - 1
E. 10
371 What is the mechanism of action of combined oral Progestins prevent ovulation by suppressing LH MAIRRE JAMES BACK-UP
contraceptives? and also thicken cervical mucus, thereby retarding GADDI, MD (TOP MIDTERM EXAM
A. Progesterone suppresses LH action while sperm passage. In addition, they render the 4 - AUG 2013 - FEB 2015
estrogen suppresses FSH action. endometrium unfavorable for implantation. MED BOARDS;
B. Progesterone suppresses FSH action while Estrogen prevents ovulation by suppressing FSH TOPNOTCH MD)
estrogen suppresses LH action. release. Williams 23rd pg 673
C. Progesterone suppresses GnRH action while
estrogen suppresses FSH action.
D. Progesterone suppresses FSH action while
estrogen suppresses GnRH action.
372 The most common adnexal mass in the adolescent Most ovarian masses in the pediatric and MAIRRE JAMES BACK-UP
age group. adolescent age group are functional ovarian cysts, GADDI, MD (TOP MIDTERM EXAM
A. Germ cell tumor and if a tumor is present it most often is a benign 4 - AUG 2013 - FEB 2015
B. Dermoid cysts teratoma (dermoid). Malignancies can, however, MED BOARDS;
C. Infectious cyst occur and are most often of germ cell origin, but TOPNOTCH MD)
D. Functional ovarian cyst they can also be sex cord tumors such as a
granulosa cell malignancy. Katz 6th pg 211
373 A 56 year-old G0 had abnormal postmenopausal This is a case of Stage IA endometrial MAIRRE JAMES BACK-UP
bleeding and biopsy done revealed endometrial adenocarcinoma since tumor is limited to the GADDI, MD (TOP MIDTERM EXAM
adenocarcinoma. Patient underwent EHBSO, PFC, endometrium. Positive cytology has to be reported 4 - AUG 2013 - FEB 2015
BLND and final histopath showed that tumor is separately without changing the stage. Katz 6th pg MED BOARDS;
confined to the endometrium. All other pelvic 720 TOPNOTCH MD)
structures and lymph nodes are negative.
However, peritoneal fluid cytology was positive.
What is the stage?
A. IA
B. IB
C. II
D. IIIA
E. IIIC

TOPNOTCH MEDICAL BOARD PREP OBSTETRICS-GYNECOLOGY SUPEREXAM Page 45 of 84


For inquiries visit www.topnotchboardprep.com.ph or email us at topnotchmedicalboardprep@gmail.com
TOPNOTCH MEDICAL BOARD PREP OBSTETRICS-GYNECOLOGY SUPEREXAM
For inquiries visit www.topnotchboardprep.com.ph or email us at topnotchmedicalboardprep@gmail.com
Ite QUESTION EXPLANATION AUTHOR TOPNOTCH
m # EXAM
374 The following are techniques employed to deliver Methods used in the delivery of the aftercoming MAIRRE JAMES BACK-UP
the aftercoming head EXCEPT: head include: Piper's Forceps, Mauriceau GADDI, MD (TOP MIDTERM EXAM
A. Mauriceau maneuver maneuver, Prague maneuver, Bracht maneuver and 4 - AUG 2013 - FEB 2015
B. Lovesets maneuver Pinard's maneuver. Lovesets maneuver is used to MED BOARDS;
C. Modified Prage maneuver deliver the shoulder. Topnotch TOPNOTCH MD)
D. Pinard's maneuver
375 The following can be observed in the puerperium Women approach their self-reported pre- MAIRRE JAMES BACK-UP
EXCEPT: pregnancy weight 6 months after delivery but still GADDI, MD (TOP MIDTERM EXAM
A. Women approach their self-reported pre- retain an average surplus of 3 lbs. Williams 23rd pg 4 - AUG 2013 - FEB 2015
pregnancy weight 3 months after delivery. 649 MED BOARDS;
B. By 2 weeks postpartum, the uterus has TOPNOTCH MD)
descended into the true pelvis.
C. Ovulation may resume as early as 3 weeks after
delivery, even in lactating women.
D. The bladder has increased capacity and relative
insensitivity to intravesical pressure.
376 A 27 year-old G2P1 36 weeks of age of gestation MAIRRE JAMES BACK-UP
came in the emergency room for uterine GADDI, MD (TOP MIDTERM EXAM
contractions. Internal examination revealed 2cm 4 - AUG 2013 - FEB 2015
dilatation, station -2, with adequate pelvimetry. MED BOARDS;
After 14 hours, IE revealed 3cm dilatation still at TOPNOTCH MD)
station -2. What is the preferred management in
this case?
A. Cesarean delivery
B. Augment with oxytocin This is case of prolonged latent phase.
C. Advise bed rest Preferred management would be bed rest.
D. Administer tocolytics Administration of oxytocin or abdominal delivery is
done if with urgent problem Williams 23rd pg 465
377 In placenta accreta, what layer is defective or MAIRRE JAMES BACK-UP
absent? GADDI, MD (TOP MIDTERM EXAM
A. decidua basalis 4 - AUG 2013 - FEB 2015
B. decidua functionalis MED BOARDS;
C. decidua stroma TOPNOTCH MD)
D. Nitabuch's layer
378 The most common cause of obstetrical hemorrhage Failure of the uterus to contract properly following MAIRRE JAMES BACK-UP
is delivery is the most common cause of obstetrical GADDI, MD (TOP MIDTERM EXAM
A. Coagulopathy hemorrhage. 4 - AUG 2013 - FEB 2015
B. Placental abruption MED BOARDS;
C. Placental Accreta TOPNOTCH MD)
D. Retained Placental fragments
E. Uterine atony
379 Triad of ectopic pregnancy: The most common symptoms of ectopic pregnancy MAIRRE JAMES BACK-UP
A. Amenorrhea, vaginal bleeding and adnexal mass are abdominal pain, absence of menses, and GADDI, MD (TOP MIDTERM EXAM
B. Amenorrhea, vaginal bleeding and abdominal irregular vaginal bleeding Katz 6th pg 367 4 - AUG 2013 - FEB 2015
pain MED BOARDS;
C. Positive pregnancy test, vaginal bleeding, and an TOPNOTCH MD)
adnexal mass
D. Positive pregnancy test, vaginal bleeding and
abdominal pain
380 The single most important risk factor for the The route of delivery is the single most significant MAIRRE JAMES BACK-UP
development of uterine infecion after childbirth. risk factor for the development of uterine infection. GADDI, MD (TOP MIDTERM EXAM
A. Prolonged labor induction Williams 23rd pg 661 4 - AUG 2013 - FEB 2015
B. Route of delivery MED BOARDS;
C. Bacterial colonization of the lower genital tract TOPNOTCH MD)
D. Meconium-stained amniotic fluid
E. None of the above
381 Which of the following represents the shortest AP Diagonal conjugate: measured clinically and is SCOTT RILEY BACK-UP
diameter of the pelvic inlet? roughly 1.5 cm longer than the obstetric conjugate. ONG, MD (TOP 5 - MIDTERM EXAM
A. Diagonal conjugate AUG 2014 MED - FEB 2015
B. Obstetric conjugate BOARDS;
C. True conjugate TOPNOTCH MD)
D. Interspinous diameter
E. Intertuberous diameter
382 Which of the following is the most common SIMILAR TO PREVIOUS BOARD EXAM SCOTT RILEY BACK-UP
menstrual abnormality associated with CONCEPT/PRINCIPLE. Oligomenorrhea and ONG, MD (TOP 5 - MIDTERM EXAM
hyperthyroidism? hypomenorrhea are the most common menstrual AUG 2014 MED - FEB 2015
A. Oligomenorrhea abnormalities in hyperthyroidism. BOARDS;
B. Menorrhagia TOPNOTCH MD)
C. Metrorrhagia
D. Polymenorrhea
E. Intermenstrual bleeding
383 Which uterine ligament provides the major SCOTT RILEY BACK-UP
support to the cervix and vagina? ONG, MD (TOP 5 - MIDTERM EXAM
A. Broad ligament AUG 2014 MED - FEB 2015
B. Cardinal ligament BOARDS;
C. Uterosacral ligament TOPNOTCH MD)
D. Round ligament
E. Infundibulopelvic ligament
384 The Arias-Stella reaction is seen in which of the Arias-Stella reaction refers to benign endocervical SCOTT RILEY BACK-UP
following conditions/periods? gland hyperplasia of pregnancy. It can cause ONG, MD (TOP 5 - MIDTERM EXAM
A. Pregnancy state misdiagnosis of Pap smear specimens from AUG 2014 MED - FEB 2015
B. Luteal phase pregnant patients as its appearance mimics BOARDS;
C. Proliferative phase endometrial CA. TOPNOTCH MD)
D. Cervical cancer
E. Menopause

TOPNOTCH MEDICAL BOARD PREP OBSTETRICS-GYNECOLOGY SUPEREXAM Page 46 of 84


For inquiries visit www.topnotchboardprep.com.ph or email us at topnotchmedicalboardprep@gmail.com
TOPNOTCH MEDICAL BOARD PREP OBSTETRICS-GYNECOLOGY SUPEREXAM
For inquiries visit www.topnotchboardprep.com.ph or email us at topnotchmedicalboardprep@gmail.com
Ite QUESTION EXPLANATION AUTHOR TOPNOTCH
m # EXAM
385 Which of the following antihypertensive ACEIs and ARBs are contraindicated in pregnancy. SCOTT RILEY BACK-UP
medications are contraindicated in pregnancy? ONG, MD (TOP 5 - MIDTERM EXAM
A. Labetalol AUG 2014 MED - FEB 2015
B. Methyldopa BOARDS;
C. Hydralazine TOPNOTCH MD)
D. Captopril
E. None of the above
386 A 16-year old female presented to your clinic with SIMILAR TO PREVIOUS BOARD EXAM SCOTT RILEY BACK-UP
primary amenorrhea. On physical examination, she CONCEPT/PRINCIPLE. ONG, MD (TOP 5 - MIDTERM EXAM
has signs of normal breast development. AUG 2014 MED - FEB 2015
Ultrasound evaluation failed to demonstrate the BOARDS;
presence of any uterus and cervix. Karyotype TOPNOTCH MD)
analysis revealed that she is a 46 XX. What is your
most likely diagnosis at this point?
A. Androgen insensitivity
B. Mayer-Rokitansky-Kuster-Hauser syndrome
C. Imperforate hymen
D. Premature ovarian failure
E. Hypogonadotropic hypogonadism
387 During pregnancy, the uterus normally undergoes The gravid uterus normally rotates to the right due SCOTT RILEY BACK-UP
which of the following positional change? to the presence of the sigmoid colon in the left ONG, MD (TOP 5 - MIDTERM EXAM
A. Rotates to the left aspect of the pelvic cavity. AUG 2014 MED - FEB 2015
B. Rotates to the right BOARDS;
C. Flexes anteriorly TOPNOTCH MD)
D. Flexes posteriorly
E. None of the above
388 Mittelschmerz pain occurs during which of the Mittelschmerz refers to the lower abdominal and SCOTT RILEY BACK-UP
following periods? pelvic pain experienced by some women midway in ONG, MD (TOP 5 - MIDTERM EXAM
A. Early trimester pregnancy their menstrual cycle, corresponding to the period AUG 2014 MED - FEB 2015
B. Late trimester pregnancy of ovulation. BOARDS;
C. Ovulation TOPNOTCH MD)
D. Menstruation
E. Perimenopause
389 Among pregnant patients, when is the best time to SCOTT RILEY BACK-UP
perform screening for Group B streptococcal ONG, MD (TOP 5 - MIDTERM EXAM
infection? AUG 2014 MED - FEB 2015
A. on initial visit BOARDS;
B. 24-28 weeks AOG TOPNOTCH MD)
C. 33-35 weeks AOG
D. 35-37 weeks AOG
E. once labor has begun
390 In a pregnant patient without known risk factors, For high-risk patients, perform OGTT on initial SCOTT RILEY BACK-UP
when is the optimal time to perform OGTT to prenatal visit. ONG, MD (TOP 5 - MIDTERM EXAM
screen for gestational diabetes? AUG 2014 MED - FEB 2015
A. on initial visit BOARDS;
B. 16-20 weeks AOG TOPNOTCH MD)
C. 24-28 weeks AOG
D. 32-36 weeks AOG
E. 20-24 weeks AOG
391 A 33-year old pregnant patient on her 36-week SIMILAR TO PREVIOUS BOARD EXAM SCOTT RILEY BACK-UP
AOG was admitted for sudden onset of painful CONCEPT/PRINCIPLE. General anesthesia is ONG, MD (TOP 5 - MIDTERM EXAM
vaginal bleeding. You diagnosed her to have recommended in the following cases: nternal AUG 2014 MED - FEB 2015
abruptio placenta and decided to perform podalic version, breech decomposition, inverted BOARDS;
emergency cesarean delivery. Given the nature of uterus, severe hemorrhage and emergent C/S. TOPNOTCH MD)
this operation and anticipating severe blood loss, Epidural anesthesia is preferred for severe
which is of the following types of anesthesia would preeclampsia, eclampsia and CHF.
you recommend for the patient?
A. Regional anesthesia
B. Epidural anesthesia
C. Combined regional and epidural anesthesia
D. General anesthesia
E. Local anesthesia
392 A G4P4 mother presented with complaints of SIMILAR TO PREVIOUS BOARD EXAM SCOTT RILEY BACK-UP
dysmenorrhea and menorrhagia. Pelvic exam CONCEPT/PRINCIPLE. Diagnosis: adenomyosis. ONG, MD (TOP 5 - MIDTERM EXAM
revealed a diffusely and symmterically enlarged AUG 2014 MED - FEB 2015
uterus. Which of the following would you expect on BOARDS;
histopathologic investigation? TOPNOTCH MD)
A. Presence of ectopic endometrial glands and
stroma on the peritoneal lining of the uterus
B. Presence of endometrial glands and stroma in
the myometrium
C. Increased endometrial thickness without
invasion of the myometrium
D. Proliferation of smooth muscle cells in the
myometrium with areas of hyaline and cystic
degeneration
E. Atrophic endometrial mucosa with friable
myometrial blood vessels
393 A 23-year old female came into your clinic due to SIMILAR TO PREVIOUS BOARD EXAM SCOTT RILEY BACK-UP
5-day history of vaginal discharge. On pelvic CONCEPT/PRINCIPLE. Diagnosis: Trichomoniasis ONG, MD (TOP 5 - MIDTERM EXAM
examination, you noted green-yellow frothy AUG 2014 MED - FEB 2015
discharge and multiple erythematous punctate BOARDS;
lesions in her vagina. With your diagnosis in mind, TOPNOTCH MD)
which of the following regimens would you
prescribe to your patient?
A. Metronidazole 2 g PO single dose
TOPNOTCH MEDICAL BOARD PREP OBSTETRICS-GYNECOLOGY SUPEREXAM Page 47 of 84
For inquiries visit www.topnotchboardprep.com.ph or email us at topnotchmedicalboardprep@gmail.com
TOPNOTCH MEDICAL BOARD PREP OBSTETRICS-GYNECOLOGY SUPEREXAM
For inquiries visit www.topnotchboardprep.com.ph or email us at topnotchmedicalboardprep@gmail.com
Ite QUESTION EXPLANATION AUTHOR TOPNOTCH
m # EXAM
B. Metronidazole 500 mg BID x 7 days
C. Fluconazole 150 mg PO single dose
D. Nystatin vaginal wash
E. Either A or B is acceptable

394 Which of the following forceps is used to assist in SIMILAR TO PREVIOUS BOARD EXAM SCOTT RILEY BACK-UP
the delivery of the aftercoming head during breech CONCEPT/PRINCIPLE. ONG, MD (TOP 5 - MIDTERM EXAM
extraction? AUG 2014 MED - FEB 2015
A. Simpson forceps BOARDS;
B. Tucker-McLane forceps TOPNOTCH MD)
C. Kielland forceps
D. Piper forceps
E. Elliot forceps
395 Which of the following substances is responsible SCOTT RILEY BACK-UP
for myometrial contractility and is implicated in ONG, MD (TOP 5 - MIDTERM EXAM
dysmenorrhea? AUG 2014 MED - FEB 2015
A. Oxytocin BOARDS;
B. Prostaglandin F2-alpha TOPNOTCH MD)
C. Enkephalinase
D. Endothelin
E. BMP-15
396 A 4-year old female presented at your clinic with SIMILAR TO PREVIOUS BOARD EXAM SCOTT RILEY BACK-UP
symptoms of UTI. On physical examination, you CONCEPT/PRINCIPLE. Diagnosis: adhesive vulvitis ONG, MD (TOP 5 - MIDTERM EXAM
noted labial adhesion with translucent vertical line AUG 2014 MED - FEB 2015
at midline. Which of the following would you BOARDS;
prescribe in relation to your finding? TOPNOTCH MD)
A. Topical clobetasol cream
B. Topical estrogen cream
C. Topical testosterone cream
D. Low-dose oral estrogen therapy
E. Surgical incision of the adhesion
397 A 35-year old female came into your clinic with a Pap smear showing ASC-US, LSIL, HSIL or SCC SCOTT RILEY BACK-UP
Pap smear result indicating "low-grade squamous should always be followed up with colposcopy and ONG, MD (TOP 5 - MIDTERM EXAM
intraepithelial lesion". What is the next biopsy. Therapeutic management would then AUG 2014 MED - FEB 2015
appropriate step in your management? depend on the result of the biopsy. BOARDS;
A. Request for transvaginal ultrasound to assess TOPNOTCH MD)
the depth of the lesion
B. Request for CT scan to assess regional node
status
C. Request for colposcopy and biopsy
D. Perform definitive management with cold-knife
conization since this is a clinical diagnosis
E. Performed loop electrosurgical excision
procedure.
398 A 43-year old multigravid presented with 1-year Diagnosis: cervical CA, stage IIIB. The presence of SCOTT RILEY BACK-UP
history of gradually enlarging, friable mass arising dilated ureters and renal pelves is an indicator of ONG, MD (TOP 5 - MIDTERM EXAM
from her cervix. On pelvic examination, you noted pelvic side wall involvement. AUG 2014 MED - FEB 2015
nodularities in the bilateral parametria. Abdominal BOARDS;
ultrasound showed no liver metastasis; however, TOPNOTCH MD)
bilateral ureteropelvocaliectasia was noted. What
would be the best management for this patient?
A. Radical hysterectomy, followed by radiotherapy
and adjuvant chemotherapy.
B. Radical hysterectomy, followed by radiotherapy
only
C. Cisplatin-based chemotherapy with concurrent
external beam radiotherapy, followed by
brachytherapy
D. Cisplatin-based chemotherapy, followed by
EBRT, then brachytherapy
E. Cisplatin-based chemotherapy only due to
presence of ureteropelvocaliectasia
399 Which of the following regimens would you Ethinyl estradiol + levonorgestrel combination, SCOTT RILEY BACK-UP
recommend for a 18-year female seeking more popularly known as Yuzpe, is an effective ONG, MD (TOP 5 - MIDTERM EXAM
emergency contraception? emergency contraception when taken up to 2 days AUG 2014 MED - FEB 2015
A. Progestin only pills for 7 days following coitus. Levonorgestrel x 2 doses is also an BOARDS;
B. Mifepristone x 5 doses acceptable alternative. TOPNOTCH MD)
C. Copper IUD
D. Ethinyl estradiol + levonorgestrel x 2 doses
E. Misoprostol x 5 doses
400 At what gestational age does surfactant production SCOTT RILEY BACK-UP
begin in the developing fetal lungs? ONG, MD (TOP 5 - MIDTERM EXAM
A. 16 weeks AOG AUG 2014 MED - FEB 2015
B. 18 weeks AOG BOARDS;
C. 20 weeks AOG TOPNOTCH MD)
D. 24 weeks AOG
E. 28 weeks AOG
401 A 35 year old patient was diagnosed with an This type of Branner's tumor is benign due to the JOSE CARLO DIAGNOSTIC
Ovarian tumor, a biopsy was done revealing presence of an outer fibrous stroma. All benign MASANGKAY III, EXAM - AUG
Transitional cells, similar to a urinary bladder-type tumors require only a unilateral salphingo- MD (TOP 8 - FEB 2014
of histology surrounded by a massive growth of a oophorectomy. (SIMILAR TO PREVIOUS BOARD 2014 MED
fibrous stroma. What would be the most EXAM CONCEPT/PRINCIPLE) BOARDS;
appropriate management? TOPNOTCH MD)
A. Unilateral Oophorectomy
B. Bilateral Oophorectomy
C. Unilateral Salphingo-oophorectomy

TOPNOTCH MEDICAL BOARD PREP OBSTETRICS-GYNECOLOGY SUPEREXAM Page 48 of 84


For inquiries visit www.topnotchboardprep.com.ph or email us at topnotchmedicalboardprep@gmail.com
TOPNOTCH MEDICAL BOARD PREP OBSTETRICS-GYNECOLOGY SUPEREXAM
For inquiries visit www.topnotchboardprep.com.ph or email us at topnotchmedicalboardprep@gmail.com
Ite QUESTION EXPLANATION AUTHOR TOPNOTCH
m # EXAM
D. TAH-BSO
E. TAH-BSO with lymph node dissection and PF
analysis

402 Which of the following cardinal movements is the SIMILAR TO PREVIOUS BOARD EXAM JOSE CARLO DIAGNOSTIC
1st prerequisite for birth? CONCEPT/PRINCIPLE MASANGKAY III, EXAM - AUG
A. Engagement MD (TOP 8 - FEB 2014
B. Descent 2014 MED
C. Flexion BOARDS;
D. Internal Rotation TOPNOTCH MD)
E. Extension
403 Bartholin's Gland is a derivative of which of the Derivatives of the Urogenital Sinus include the JOSE CARLO DIAGNOSTIC
following anlagen? female urethra, lower vagina (2/3), Bartholin's MASANGKAY III, EXAM - AUG
A. Wolffian Duct Gland and Skene's Gland. MD (TOP 8 - FEB 2014
B. Mullerian Duct 2014 MED
C. Genital Tubercle BOARDS;
D. Genital Swelling TOPNOTCH MD)
E. Urogenital Sinus
404 A 15-year old patient was brought to you by her Primary amenorrhea in a patient with the presence JOSE CARLO DIAGNOSTIC
mother apparently because she still has not of secondary sexual characteristics is considered if MASANGKAY III, EXAM - AUG
menstruated, secondary sexual characteristics are the patient is already 16 years old. Our patient is MD (TOP 8 - FEB 2014
present in this patient and there is no just 15, a more appropriate step is to wait for her 2014 MED
developmental delay. What would be your next 16th birthday, if still with no menses then do BOARDS;
step? diagnostics. TOPNOTCH MD)
A. Measure FSH and LH
B. Do a transrectal UTZ
C. Do CNS imaging
D. Do Karyotyping
E. Observe
405 The most common symptom of Vulvar Pruritus is the most common symptom of VIN JOSE CARLO DIAGNOSTIC
Intraepithelial neoplasia is: hence in all pruritic valvualr lesions, biopsy is MASANGKAY III, EXAM - AUG
A. Pruritus warranted. MD (TOP 8 - FEB 2014
B. Pain 2014 MED
C. Palpable nontender mass BOARDS;
D. Bleeding TOPNOTCH MD)
E. Acanthosis
406 True of Lichen sclerosus, EXCEPT: Lichen sclerosus does not involve the vagina. JOSE CARLO DIAGNOSTIC
A. Treated with Testosterone cream MASANGKAY III, EXAM - AUG
B. Has a 5-15% risk for CA in premenopausal MD (TOP 8 - FEB 2014
women 2014 MED
C. Pruritus is the most common presentation BOARDS;
D. May involve the vaginal wall TOPNOTCH MD)
E. Creates an hourglass appearance
407 A 20-year old Nulligravid has just underwent a Pap For patients with High grade SIL (CIN II and III) and JOSE CARLO DIAGNOSTIC
Smear revealing a CIN II result, which of the still desires infertility, ablative therapy or excisional MASANGKAY III, EXAM - AUG
following is the best option for this patient? procedures are the options, hysterectomy is only MD (TOP 8 - FEB 2014
A. Observe and repeat Pap smear after 3 months warranted for patients who has no desire for 2014 MED
B. Ablative therapy fertility/ or has completed child-bearing years. BOARDS;
C. Perform Hysterectomy only TOPNOTCH MD)
D. Do a TAHBSO
E. Do chemoradiation only
408 Which of the following statements is NOT true HPV strains 16 and 18 poses a high risk for JOSE CARLO DIAGNOSTIC
regarding Cervical Cancer? carcinoma, not 6 and 11 which causes only genital MASANGKAY III, EXAM - AUG
A. HPV 6, 11 infections poses a high risk for warts. MD (TOP 8 - FEB 2014
developing Carcinoma 2014 MED
B. Squamous Cell Carcinoma is the most common BOARDS;
type TOPNOTCH MD)
C. Vaginal Bleeding is the most common symptom
D. Uremia is the most common cause of death
E. HPV vaccine can be administered starting 9
years old
409 A 50-year old asymptomatic patient with a Myoma Asymptomatic patients with myoma are managed JOSE CARLO DIAGNOSTIC
as large as a baseball, sought consult from an OB- conservatively. MASANGKAY III, EXAM - AUG
GYN, what would be the most appropriate MD (TOP 8 - FEB 2014
management for this patient? 2014 MED
A. Observe BOARDS;
B. Medical Therapy with a GnRH Analogue TOPNOTCH MD)
C. Do Myomectomy
D. Do embolization
E. Do a hysterectomy
410 A patient with a recently discovered endometriosis JOSE CARLO DIAGNOSTIC
found on her lungs asks for an explanation from MASANGKAY III, EXAM - AUG
you why this occurred, you would explain which of MD (TOP 8 - FEB 2014
the following theories of endometriosis? 2014 MED
A. Retrograde Menstruation BOARDS;
B. Metaplasia of Coelomic Epithelium TOPNOTCH MD)
C. Lymphatic and Vascular Metastasis
D. Iatrogenic Dissemination
E. Malignant Degeneration
411 A patient with an ovarian fibroma, suddenly Meig's Syndrome: Ovarian Fibroma, Ascites, JOSE CARLO DIAGNOSTIC
developed Meig's Syndrome, among which of the Hydrothorax MASANGKAY III, EXAM - AUG
following would be seen in this patient? MD (TOP 8 - FEB 2014
A. Pneumothorax 2014 MED
B. Ascites BOARDS;
C. Amenorrhea TOPNOTCH MD)

TOPNOTCH MEDICAL BOARD PREP OBSTETRICS-GYNECOLOGY SUPEREXAM Page 49 of 84


For inquiries visit www.topnotchboardprep.com.ph or email us at topnotchmedicalboardprep@gmail.com
TOPNOTCH MEDICAL BOARD PREP OBSTETRICS-GYNECOLOGY SUPEREXAM
For inquiries visit www.topnotchboardprep.com.ph or email us at topnotchmedicalboardprep@gmail.com
Ite QUESTION EXPLANATION AUTHOR TOPNOTCH
m # EXAM
D. Menorrhagia
E. Palpable abdominal mass

412 A patient presenting with a pelvic mass underwent all others are probable signs that the mass is JOSE CARLO DIAGNOSTIC
a Pelvic ultrasound, which of the following benign. MASANGKAY III, EXAM - AUG
characteristics will make you suspect that the mass MD (TOP 8 - FEB 2014
is probably malignant? 2014 MED
A. Unilateral Mass BOARDS;
B. Absence of septations TOPNOTCH MD)
C. Noted Calcifications
D. Multicystic or multilocular mass
E. size of less than 8 cm
413 Drugs are categorized according to their safety JOSE CARLO DIAGNOSTIC
profile for consumption of pregnant patients, MASANGKAY III, EXAM - AUG
Phenytoin is an example. Phenytoin is found to MD (TOP 8 - FEB 2014
have positive evidence of human fetal risk based 2014 MED
on studies in humans, but potential benefits may BOARDS;
warrant use of the drug despite potential risks. TOPNOTCH MD)
What category is Phenytoin classified?
A. A
B. B
C. C
D. D
E. X
414 Your sister is an excited new mom-to-be, being The phenotypic sex is well formed at 17 weeks but JOSE CARLO DIAGNOSTIC
enthusiastic as she is, she then asks you when can be identified by expert sonologists at 14 weeks. MASANGKAY III, EXAM - AUG
would they know the sex of their baby on UTZ? You MD (TOP 8 - FEB 2014
said that it is as early as: 2014 MED
A. 7 weeks AOG BOARDS;
B. 9 weeks AOG TOPNOTCH MD)
C. 11 weeks AOG
D. 14 weeks AOG
E. 20 weeks AOG
415 Which of the following skin changes in pregnancy All hyperpigmented lesions are due to MSH, Striae JOSE CARLO DIAGNOSTIC
is an effect of Hyperestrogenemia? gravidarum is due to the effects of corticosteroids. MASANGKAY III, EXAM - AUG
A. Linea Nigra MD (TOP 8 - FEB 2014
B. Chloasma 2014 MED
C. Darkening of Areola BOARDS;
D. Striae Gravidarum TOPNOTCH MD)
E. Spider Nevi
416 A G1P0 on her 34th week AOG underwent a BPS is 4 with an abnormal AFV hence protocol JOSE CARLO DIAGNOSTIC
Biophysical Assesment which revealed the states that delivery should be prompted. MASANGKAY III, EXAM - AUG
following: 1.)Reactive NST, 2.)One breathing MD (TOP 8 - FEB 2014
period lasting 10 seconds, 3.) 2 discrete 2014 MED
movements of arms, 4.) One definite extension and BOARDS;
return to flexion, 5.) AF volume with the largest TOPNOTCH MD)
pocket at 1 cm. What would be the nest step of the
OB-GYN?
A. Observe and repeat per protocol
B. Repeat BPF in 24 hours
C. Repeat test weekly
D. Do a CST
E. Deliver
417 The earliest prenatal screening that can be done as Chorionic villus sampling can be done as early as 9 JOSE CARLO DIAGNOSTIC
early as 9 weeks AOG is among which of the weeks to 12 weeks AOG. MASANGKAY III, EXAM - AUG
following? MD (TOP 8 - FEB 2014
A. Cordocentesis 2014 MED
B. Chorionic villus sampling BOARDS;
C. Early amniocentesis TOPNOTCH MD)
D. Percutaneous Umbilical cord blood sampling
E. None of the above
418 The highest risk for the development of a future SIMILAR TO PREVIOUS BOARD EXAM JOSE CARLO DIAGNOSTIC
Ectopic Pregnancy is which among the following? CONCEPT/PRINCIPLE MASANGKAY III, EXAM - AUG
A. Previous PID MD (TOP 8 - FEB 2014
B. Tubal Corrective Surgery 2014 MED
C. Previous Ectopic Pregnancy BOARDS;
D. IUD placement TOPNOTCH MD)
E. Previous abdominal surgery
419 A G2P1 patient on her 16th week AOG was rushed Ectopic pregnancy of the interstitial part of the FT JOSE CARLO DIAGNOSTIC
in the ED due to severe abdominal pain, will rupture only at around 16 weeks, rupture of MASANGKAY III, EXAM - AUG
hypotension, tachycardia and signs of peritonitis, the ampulla occurs at around 8-12 weeks, while the MD (TOP 8 - FEB 2014
which among the following would be your isthmus which is the narrowest will most likely 2014 MED
consideration? rupture in the first 6-8 weeks of pregnancy. D and E BOARDS;
A. Rupture Ectopic Pregnancy, Ampulla are seen in the 3rd trimester of pregnancy. TOPNOTCH MD)
B. Ruptured Ectopic Pregnancy, Isthmus
C. Ruptured Ectopic Pregnancy, Interstitial
D. Abruptio Placenta
E. Placenta Previa
420 CPD can be totally ruled out in a nulligravid patient CPD can not be totally ruled out in a nulligravid JOSE CARLO DIAGNOSTIC
by which of the following procedure? patient. A history of CPD in a multipara will give a MASANGKAY III, EXAM - AUG
A. Xray Pelvimetry great idea to a clinician that the patient may again MD (TOP 8 - FEB 2014
B. Clinical Pelvimetry come out with the same problem. 2014 MED
C. History taking BOARDS;
D. Pelvic CT-Scan TOPNOTCH MD)
E. None of the above
TOPNOTCH MEDICAL BOARD PREP OBSTETRICS-GYNECOLOGY SUPEREXAM Page 50 of 84
For inquiries visit www.topnotchboardprep.com.ph or email us at topnotchmedicalboardprep@gmail.com
TOPNOTCH MEDICAL BOARD PREP OBSTETRICS-GYNECOLOGY SUPEREXAM
For inquiries visit www.topnotchboardprep.com.ph or email us at topnotchmedicalboardprep@gmail.com
Ite QUESTION EXPLANATION AUTHOR TOPNOTCH
m # EXAM
421 A female patient was noted to be of short stature, Noonan syndrome shares clinical features with WEBSTER MIDTERM 1
with webbed neck, shield chest, and a congenital Turner syndrome, including the signs mentioned in ALINDOG, MD EXAM - AUG
heart disease. She has normal mentation. Genetic the item. However, the observation that patients (TOP 3 - FEB 2014
analysis, however, revealed a normal karyotype. with Noonan syndrome have normal karyotypes is 2014 MED
This patient has: important in allowing the distinction to be made BOARDS;
A. Superfemale syndrome between the two (Turner has the karyotype 45 XO). TOPNOTCH MD)
B. Turner syndrome Significantly, Noonan syndrome patients are able to
C. Rokitansky Kuster Hauser Mayer syndrome bear offsprings, however management is
D. Noonan syndrome complicated with uterine anatomic anomalies
which are also common in these patients.
422 A 35-year old patient was rushed to the OB ER All OB-GYN emergencies, especially if the patient is WEBSTER MIDTERM 1
after experiencing excessive pelvic pain and hemodynamically stable, warrants a pregnancy test ALINDOG, MD EXAM - AUG
cramping with notable vaginal bleeding. She is prior to any intervention. This is a case of (TOP 3 - FEB 2014
febrile and tachycardic but with normal BP and secondary dysmenorrhea with several possible 2014 MED
good peripheral pulses and perfusion. During etiologies including ectopic pregnancy, septic BOARDS;
history taking, she admits that she has been abortion, pelvic infection, etc. TOPNOTCH MD)
working as a prostitute and does not use condoms
or OCPs. The first test that you will order for this
patient is:
A. Pelvic sonography
B. Blood culture
C. Diagnostic laparoscopy
D. β-hCG
E. None of the above
423 Which of the following is not true about Menopause is defined as permanent cessation of WEBSTER MIDTERM 1
menopause: menstruation, featured with 3 consecutive months ALINDOG, MD EXAM - AUG
A. The age of onset is genetically determined. of amenorrhea with elevations of FSH and LH. The (TOP 3 - FEB 2014
B. There is a significant decrease in the number of mean age of onset among Filipinos is said to be at 2014 MED
ovarian follicles accompanied by degeneration of 51. The increase in the gonadotropins as well as the BOARDS;
granulosa and theca cells. cessation of mestrual cycle is brought about by the TOPNOTCH MD)
C. Stromal cells of the ovaries sustain their loss of ovarian follicles secreting inhibin and
capacity to produce androgenic hormones. estrogen. Estrone (E1) is the predominant estrogen
D. The remaining estrogen levels in the in menopause. It is converted from
circulation during menopause is dictated by the androstenedione which comes from body fat.
proportion of body fat.
E. None of the above.
424 Ferning or arborization of the cervical mucus is On the other hand, BEADING is seen during the WEBSTER MIDTERM 1
brought about by the crystallization of sodium secretory phase of the menstrual cycle and is due to ALINDOG, MD EXAM - AUG
chloride on mucus fibers in the presence of high levels of progesterone. When ferning is seen in (TOP 3 - FEB 2014
elevated levels of estrogen. This pattern can be all phases of the cycle, it suggests infertility, 2014 MED
normally seen in which phase of the menstrual autonomic ovarian failure or menopause. BOARDS;
cycle? TOPNOTCH MD)
A. Proliferative phase
B. Secretory phase
C. Luteal phase
D. Any phase
425 A 24-year old G2P1 at 38 weeks AOG is admitted at Prolonged latent phase is considered if the patient WEBSTER MIDTERM 1
the OB ward. She reports that she has been has been experiencing regular uterine contractions ALINDOG, MD EXAM - AUG
experiencing regular uterine contractions for 19 for >20 hours if nulliparous, or >14 hours if with (TOP 3 - FEB 2014
hours now. IE reveals cervical dilation remaining at previous delivery, with cervical dilation remaining 2014 MED
2 cms. She is afebrile with normal heart rate and at 2 cms. Primary manegement is still expectant BOARDS;
BP. Fetal monitoring is also reassuring. You and supportive, although one should consider that TOPNOTCH MD)
consider prolonged latent phase of labor. What will the mother may suffer from exhaustion or uterine
be your next plan? infection.
A. Expectant management
B. Induction of labor
C. Augmentation of labor
D. Refer to a perinatologist
426 Cesarean section is an operative procedure Dystocia, on the other hand, is the most common WEBSTER MIDTERM 1
requiring a uterine incision which aims to deliver indication for a primary cesarean section. ALINDOG, MD EXAM - AUG
the fetus abdominally . Which of the following is an (TOP 3 - FEB 2014
absolute indication for this procedure? 2014 MED
A. Massive maternal obesity BOARDS;
B. Transverse lie TOPNOTCH MD)
C. Contracted pelvis
D. Placenta previa
427 A 23-year old G2P2 has just given birth to a live Peurperium usually lasts up to 6 weeks. Decidua WEBSTER MIDTERM 1
healthy term 2.8 kg baby girl via spontaneous differentiates in 2-3 days after delivery. At 2 weeks, ALINDOG, MD EXAM - AUG
vaginal delivery. She is stable with adequate uterus returns to the true pelvis; at 3 weeks, the (TOP 3 - FEB 2014
response to post-delivery care and heading to an entire endometrium becomes restored; and at 4 2014 MED
unremarkable peuperal stage. At what time is her weeks, the uterus reaches its non-gestational size. BOARDS;
uterus expected to descend into the true pelvis? TOPNOTCH MD)
A. At 1 week post-partum
B. At 2 weeks post-partum
C. At 3 weeks post-partum
D. At 4 weeks post-partum
E. Soon after delivery of the placenta
428 The type of anesthesia ideal for severe pre- Pudendal block is used in manual exploration of WEBSTER MIDTERM 1
eclampsia and eclampsia and is also considered as uterine cavity, in outlet forceps delivery and in ALINDOG, MD EXAM - AUG
the gold standard in obstetrical anesthesia is: repair of vagina and cervix. Spinal anesthesia is (TOP 3 - FEB 2014
A. Spinal anesthesia more commonly used in elective CS (the duration of 2014 MED
B. Pudendal anesthesia procedure is more time-limited, since the BOARDS;
C. Epidural anesthesia anesthesia is given as a single shot). While general TOPNOTCH MD)
D. General anesthesia anesthesia is indicated in breech decomposition,
replacement of inverted uterus, and internal
podalic version of second twin.
TOPNOTCH MEDICAL BOARD PREP OBSTETRICS-GYNECOLOGY SUPEREXAM Page 51 of 84
For inquiries visit www.topnotchboardprep.com.ph or email us at topnotchmedicalboardprep@gmail.com
TOPNOTCH MEDICAL BOARD PREP OBSTETRICS-GYNECOLOGY SUPEREXAM
For inquiries visit www.topnotchboardprep.com.ph or email us at topnotchmedicalboardprep@gmail.com
Ite QUESTION EXPLANATION AUTHOR TOPNOTCH
m # EXAM
429 A 33-year old G3P1 at 36 weeks AOG was referred This is a case of placenta accreta where chorionic WEBSTER MIDTERM 1
to your hospital due to abnormal placentation villi penetrate the decidua but not the myometrium ALINDOG, MD EXAM - AUG
detected sonographically at 29 weeks AOG. She had (accreta vera, 75%); or deeper into the (TOP 3 - FEB 2014
CS for placenta previa 3 years ago. At the admitting myometrium but not the seros (increta, 15%); or 2014 MED
section, the patient reports vaginal spotting but no may reach and even perforate the serosa and BOARDS;
severe abdominal complaints. Repeat ultrasound invade adjacent structures (percreta, 5%). Option A TOPNOTCH MD)
reveals a viable baby and CS was immediately explains vasa previa while option C pertains to
performed. The surgeon however noted profuse placenta previa.
bleeding that was not ameliorated by any means.
Hysterectomy was done. This abnormal condition
can be best explained by:
A. Velamentous insertion of the umbilical cord
B. Absence of the decidua basalis with imperfect
development of the Nitabuch layer
C. Avulsion of anchoring villi of low implanted
placenta
D. Uterine ischemia secondary to prolonged
myometrial contractions
430 Which of the following conditions will least likely WEBSTER MIDTERM 1
cause uterine inversion? ALINDOG, MD EXAM - AUG
A. Strong umbilical cord traction (TOP 3 - FEB 2014
B. Myometrial fatigue 2014 MED
C. Placenta accreta BOARDS;
D. Abruptio placenta TOPNOTCH MD)
431 A 17-year old primi at 37 weeks AOG, who is VZV Ig should be given to neonates born to mothers WEBSTER MIDTERM 1
expected to deliver in a week, returned to your who have clinical evidence of the infection 5 days ALINDOG, MD EXAM - AUG
clinic after observing multiple pruritic vesicular before or up to 2 days after delivery. Varicella (TOP 3 - FEB 2014
rashes on her torso and head. You know that this is vaccine is not recommended to pregnant women, or 2014 MED
infectious and requires intervention, therefore you to those who expect to conceive in the month BOARDS;
will consider: following vaccination. TOPNOTCH MD)
A. Giving pregnancy-graded oral anti-viral
medications to prevent spread in the circulation
saving the feto-maternal blood circuit.
B. Giving vaccine to the mother to prevent
transplacental transfer.
C. Administering immune globulins to the
newborn.
D. An emergency cesarean section.
E. All can be considered.
432 Which Leopold's maneuver will determine the A FAQ as well. Leopold I is the fundal grip and is WEBSTER MIDTERM 1
degree of flexion (attitude) of fetal head? used to determine the fetal part that lies in the ALINDOG, MD EXAM - AUG
A. Leopold's I fundus (presentation). Leopold II identifies the (TOP 3 - FEB 2014
B. Leopold's II location of fetal back and small parts (fetal lie). 2014 MED
C. Leopold's III While Leopold's III determines engagement and is BOARDS;
D. Leopold's IV also known as the Pawlick's sign. TOPNOTCH MD)
433 A pre-eclamptic primigravid is admitted at the There are 3 parameters that we monitor in patients WEBSTER MIDTERM 1
maternal ICU. Her condition is adequately maintained on MgSO4: the deep tendon reflex, RR, ALINDOG, MD EXAM - AUG
controlled by MgSO4 drip. You ask the clerk-on- and urine output. The therapeutic level of MgSO4 is (TOP 3 - FEB 2014
duty to monitor the possible adverse effects of at 4-7 mEq/L; disappearance of the patellar reflex is 2014 MED
such medication. In your mind, you expect that this set at 10 mEq/L. On the onter hand, respiratory BOARDS;
patient will start having respiratory depression if depression sets in if value is >12 mEq/L, and it may TOPNOTCH MD)
her MgSO4 blood level reaches: require mechanical ventilation if >15 mEq/L.
A. 7 mEq/L Cardiac arrest is expected to happen at levels >30-
B. 10 mEq/L 35 mEq/L. Calcium gluconate (1 gm IV) is the
C. 14 mEq/L antidote.
D. 18 mEq/L
E. 30 mEq/L
434 Which of the following is not found in HELLP HELLP means hemolysis (high LDH - enzyme WEBSTER MIDTERM 1
syndrome? released from RBCs), elevated liver enzymes,and ALINDOG, MD EXAM - AUG
A. Elevated LDH low platelet. (TOP 3 - FEB 2014
B. Thrombocytopenia 2014 MED
C. Elevated liver enzymes BOARDS;
D. Hemorrhage with prolonged PT/PTT TOPNOTCH MD)
E. None of the above
435 What CNS anomaly in infants is considered to be However, other causes are presumably involved, as WEBSTER MIDTERM 1
most specific to maternal DM? demonstrated by the rare incidence of sacral ALINDOG, MD EXAM - AUG
A. Down syndrome agenesis compared to diabetes and certainly, not all (TOP 3 - FEB 2014
B. Sacral agenesis children born with the condition have diabetic 2014 MED
C. Cretinism mothers. BOARDS;
D. Lissencephaly TOPNOTCH MD)
436 Which will you give/recommend to a lactating Please study the methods of contraception, WEBSTER MIDTERM 1
woman who desires contraception? especially the hormonal pills. Minipill is also known ALINDOG, MD EXAM - AUG
A. Combined oral pill as progestin only pill. It has 0.5 mg of progesterone (TOP 3 - FEB 2014
B. Levonorgestrel and is considered safe and appropriate for 2014 MED
C. Minipill breastfeeding women. BOARDS;
D. Copper T IUD TOPNOTCH MD)
437 A 27-year old G3P2 mother who is now at her 14 The manifestations are most consistent with a case WEBSTER MIDTERM 1
weeks AOG, is rushed to the OB ER after of missed abortion. Expected findings in the UTZ ALINDOG, MD EXAM - AUG
complaining of minimal vaginal spotting. She is include an empty gestational sac in blighted ovum (TOP 3 - FEB 2014
afebrile with normal BP and heart rate. On IE, her or a fetus without cardiac activity. Elective D&C is 2014 MED
cervix is closed with uterine size estimating 10 Complete abortion will have no bleeding and will BOARDS;
weeks AOG. Which of the following is the next best show an empty cavity on UTZ. TOPNOTCH MD)
step?
A. Discharge the patient and advise her to have
complete bed rest.
B. Perform an ultrasound and perform an elective
TOPNOTCH MEDICAL BOARD PREP OBSTETRICS-GYNECOLOGY SUPEREXAM Page 52 of 84
For inquiries visit www.topnotchboardprep.com.ph or email us at topnotchmedicalboardprep@gmail.com
TOPNOTCH MEDICAL BOARD PREP OBSTETRICS-GYNECOLOGY SUPEREXAM
For inquiries visit www.topnotchboardprep.com.ph or email us at topnotchmedicalboardprep@gmail.com
Ite QUESTION EXPLANATION AUTHOR TOPNOTCH
m # EXAM
dilatation and curettage.
C. Perform a prompt dilatation and evacuation.
D. Perform an ultrasound, observe the patient for
recurrence of spotting, and advise for expectant
management.
438 Vulvar carcinoma is strongly related to HPV with Pruritus is the most common symptom of vulvar WEBSTER MIDTERM 1
an excellent orverall survival rate. It is usually of carcinoma. ALINDOG, MD EXAM - AUG
the squamous cell CA type and may present either (TOP 3 - FEB 2014
as a flat, raised, plaque-like, ulcerated or polypoid 2014 MED
masses on the vulva. If a patient with this condition BOARDS;
would come to you for consult she would most TOPNOTCH MD)
likely complain of vulvar:
A. Pain
B. Bleeding
C. Itching
D. Adhesions
439 A subseptate uterus can result in 1st trimester Septate and subseptate are associated with first WEBSTER MIDTERM 1
pregnancy loss because of: trimester miscarriage due to inadequate ALINDOG, MD EXAM - AUG
A. Poor vascularization vascularization. Pregnancy losses during 2nd (TOP 3 - FEB 2014
B. Limited uterine space hence restricting fetal trimester can be seen in unicornuate and 2014 MED
growth bicornuate uteri. With these anomalies, fetal growth BOARDS;
C. Increased risk of bacterial invasion and is more physically restricted. TOPNOTCH MD)
chorioamnionitis
D. High chance of placental fragmentation and
loss
440 What is the most common type of uterine myoma? *FAQ. The submucosal type is the one most WEBSTER MIDTERM 1
A. Subserosal commonly associated with heavy and prolonged ALINDOG, MD EXAM - AUG
B. Submucosal bleeding. (TOP 3 - FEB 2014
C. Intramural 2014 MED
D. Cervical BOARDS;
TOPNOTCH MD)
441 The presence of heavy concentration of Clue cells are epithelial cells of the vagina that get JULIET KRISTINE MIDTERM 2
coccobacilli surrounding vaginal epithelial cells their distinctive stippled appearance by being EVANGELISTA, EXAM - AUG
with loss of distinct cell margins is the appearance covered with coccobacilli. Donovan bodies are rod- MD (TOP 9 - FEB 2014
of: shaped, oval organisms that can be seen in the 2014 MED
A. Donovan bodies cytoplasm of mononuclear phagocytes or BOARDS;
B. Chancre histiocytes in tissue samples from patients with TOPNOTCH MD)
C. Clue cells granuloma inguinale. A chancre is a painless ulcer
D. Inclusion cells most commonly found in primary stage of syphilis.
E. Koilocytes Koilocytes are cells found with HPV infection.
442 Speculum exam of a 27 year-old female Trichomoniasis is caused by Trichomonas vaginalis JULIET KRISTINE MIDTERM 2
complaining of leucorrhea showed copious frothy manifested with green-yellow frothy vaginal EVANGELISTA, EXAM - AUG
greenish vaginal discharge with strawberry-like discharge associated with strawberry cervix. MD (TOP 9 - FEB 2014
mucosa. This is most likely due to: Candidiasis has cottage cheese-like discharge, 2014 MED
A. Candida albicans gonorrhea and chlamydia infection has no BOARDS;
B. Trichomonas vaginalis dischrage in infected women, Bacterial vaginosis TOPNOTCH MD)
C. Neisseria gonorrheae has grayish-white discharge.
D. Chlamydia trachomatis
E. Gardnerella vaginalis
443 A 28 year-old, G1P0 woman was diagnosed of Magnesium sulfate is the anticonvulsant of choice JULIET KRISTINE MIDTERM 2
pregnancy-induced hypertension was seen at the for preeclampsia and eclampsia. Administration is EVANGELISTA, EXAM - AUG
ER due to convulsion. The anticonvulsant of choice limited if the patient had depressed tendon reflexes, MD (TOP 9 - FEB 2014
is magnesium sulfate. Which of the following respiratory rate <12cpm, and urine output 2014 MED
findings would limit administration of the <30cc/hour hence, these should be monitored. BOARDS;
magnesium sulfate? TOPNOTCH MD)
A. PR of 70bpm
B. bradypnea
C. +2 tendon reflexes
D. BP of 140/90
E. urine output of 35cc/hour
444 It is a metastatic tumor to the ovary, usually A Krukenberg tumor refers to a malignancy in the JULIET KRISTINE MIDTERM 2
bilateral, consisting of signet ring cells, usually ovary that metastasized from a primary site, EVANGELISTA, EXAM - AUG
originating from gastrointestinal tract: classically the gastrointestinal tract, although it can MD (TOP 9 - FEB 2014
A. Mucinous tumor arise in other tissues such as the breast. 2014 MED
B. Serous tumor Krukenberg tumors are often found in both ovaries, BOARDS;
C. Sex-cord stromal tumor consistent with its metastatic nature, most TOPNOTCH MD)
D. Krukenberg tumor commonly from gastric adenocarcinoma.
E. Brenner tumor Lymphogranuloma venereum (LGV) is an
uncommon sexually transmitted disease (STD)
caused by Chlamydia trachomatis.
445 What ulcerative lesion of the genital tract is Lymphogranuloma venereum (LGV) is an JULIET KRISTINE MIDTERM 2
characterized by the presence of "groove sign"? uncommon sexually transmitted disease (STD) EVANGELISTA, EXAM - AUG
A. Granuloma inguinale caused by Chlamydia trachomatis. This condition is MD (TOP 9 - FEB 2014
B. Lymphogranuloma venereum characterized by self-limited genital papules or 2014 MED
C. Chancroid ulcers followed by painful inguinal and/or femoral BOARDS;
D. Syphilis lymphadenopathy. The ‘groove sign’ characteristic TOPNOTCH MD)
E. TB of genital tract of LGV is seen if both the inguinal and the femoral
nodes are involved.
446 A 25 year-old G1P1 was diagnosed with cervical Conization is a treatment of choice for women who JULIET KRISTINE MIDTERM 2
intraepithelial neoplasia involving the entire are still desirous of pregnancy with high grade EVANGELISTA, EXAM - AUG
thickness of the cervical epithelium. This is best cervical dysplasia. Conization removes a cone MD (TOP 9 - FEB 2014
managed by: shaped piece of tissue from the cervix. It is also 2014 MED
A. Cryosugery called a cone biopsy and can be used to help BOARDS;
B. Electrocautery diagnose cervical cancer. Cocaine, a small molecule, TOPNOTCH MD)
C. Conization is able to cross the placenta into the bloodstream of
D. Hysterectomy the fetus.
TOPNOTCH MEDICAL BOARD PREP OBSTETRICS-GYNECOLOGY SUPEREXAM Page 53 of 84
For inquiries visit www.topnotchboardprep.com.ph or email us at topnotchmedicalboardprep@gmail.com
TOPNOTCH MEDICAL BOARD PREP OBSTETRICS-GYNECOLOGY SUPEREXAM
For inquiries visit www.topnotchboardprep.com.ph or email us at topnotchmedicalboardprep@gmail.com
Ite QUESTION EXPLANATION AUTHOR TOPNOTCH
m # EXAM
E. CO2 laser ablation

447 Cocaine use in pregnancy is associated with: Cocaine is a a small molecule which is able to cross JULIET KRISTINE MIDTERM 2
A. Spontaneous abortion the placenta into the bloodstream of the fetus. EVANGELISTA, EXAM - AUG
B. Prematurity Cocaine-using pregnant women deliver MD (TOP 9 - FEB 2014
C. Abruptio placenta prematurely. There are also data showing that 2014 MED
D. A and B spontaneous abortion and low birth weight. The BOARDS;
E. All of the above increased risk of placental abruption with cocaine TOPNOTCH MD)
use has been well documented.
448 The third stage of labor commences: Delivery of the fetus commences the third stage of JULIET KRISTINE MIDTERM 2
A. After expulsion of placenta labor and is also the end of second stage of labor. EVANGELISTA, EXAM - AUG
B. After delivery of fetus First stage of the labor ends when cervix is fully MD (TOP 9 - FEB 2014
C. When cervix is fully dilated dilated and fully effaced. 2014 MED
D. After episiorrhapy BOARDS;
E. Cervix is fully effaced TOPNOTCH MD)
449 On endometrial biopsy, glycogen-rich subnuclear Basal vacuolation is the earliest histological JULIET KRISTINE MIDTERM 2
vacuoles were seen in the base of the cells lining evidence of progesterone action. EVANGELISTA, EXAM - AUG
the glands. The hormone that is predominant in MD (TOP 9 - FEB 2014
this phase is: 2014 MED
A. Estrogen BOARDS;
B. FSH TOPNOTCH MD)
C. LH
D. Progesterone
E. Gonadotropin
450 The order of the 4 divisions of the fallopian tubes The order of the 4 divisions of the fallopian tubes JULIET KRISTINE MIDTERM 2
from the ovary to the uterus are: from the ovary to the uterus are infundibulum, EVANGELISTA, EXAM - AUG
A. Infundibulum, isthmus, ampulla, interstitium ampulla, isthmus, interstitium. MD (TOP 9 - FEB 2014
B. Infundibulum, ampulla, isthmus, interstitium 2014 MED
C. Infundibulum, ampulla, interstitium, isthmus BOARDS;
D. isthmus, ampulla, interstitium, infundibulum TOPNOTCH MD)
E. interstitium, isthmus, ampulla, infundibulum
451 Abdominal enlargement and a positive hormonal Abdominal enlargement, changes in the servix, JULIET KRISTINE MIDTERM 2
test maybe considered: hegar's sign, goodell's sign, braxton hick's EVANGELISTA, EXAM - AUG
A. Positive signs of pregnancy contractions, ballottement, physical outlining of MD (TOP 9 - FEB 2014
B. Presumptive signs of pregnancy fetus in the uterus and positive pregnancy test are 2014 MED
C. Probable signs of pregnancy the probable evidence of pregnancy. BOARDS;
D. Negative signs of pregnancy TOPNOTCH MD)
E. None of the above
452 Immunization during pregnancy may be given Vaccines contraindicated for pregnant women are JULIET KRISTINE MIDTERM 2
except the one which is absolutely contraindicated MMR and varicella. EVANGELISTA, EXAM - AUG
is: MD (TOP 9 - FEB 2014
A. Tetanus 2014 MED
B. Rubella BOARDS;
C. Hepatitis B TOPNOTCH MD)
D. Poliomyelitis
E. Cholera
453 A 27 year-old, G2P1, 41 weeks AOG was admitted The parturient is in the active stage of normal labor. JULIET KRISTINE MIDTERM 2
in labor. Fundic height was measured at 30cms Monitoring, observation is the only appropriate EVANGELISTA, EXAM - AUG
with good fetal heart tone. Cervix is 4cm dilated, thing to do. MD (TOP 9 - FEB 2014
not effaced, station -1. About 1 hour after 2014 MED
admission, IE findings remained the same. What BOARDS;
would be the most appropriate thing to do? TOPNOTCH MD)
A. deliver the baby by CS
B. observe and evaluate
C. give oxytocin drip to augment labor
D. ask patient to walk around to hasten labor
E. do amniotomy
454 Seven minutes after a normal delivery under The length of the third stage itself is usually 5-15 JULIET KRISTINE MIDTERM 2
pudendal anesthesia, the patient has not minutes. Expectant, or physiologic, management EVANGELISTA, EXAM - AUG
completed the third stage of labor. The uterus is involves waiting for the typical signs of placental MD (TOP 9 - FEB 2014
discoid and firm, no bleeding is evident. You separation such as fundal rise, a gush of blood, and 2014 MED
should: lengthening of the umbilical cord, then allowing the BOARDS;
A. Manually remove the placenta placenta to deliver spontaneously. TOPNOTCH MD)
B. Pull the cord vigorously
C. Invert the uterus
D. Gently massage the uterus and wait
E. Remove placenta from inverted uterus
455 In vaginal delivery for breech presentation, the Piper's forceps is used in the delivery of the JULIET KRISTINE MIDTERM 2
forceps of choice in delivery of the aftercoming aftercoming head in a vaginal delivery for breech EVANGELISTA, EXAM - AUG
head is: presentation. Simpson forceps is the most common MD (TOP 9 - FEB 2014
A. Kielland forceps to deliver babies with molded head in 2014 MED
B. Simpson's nulliparas. Tucker-Mclane is used to deliver babies BOARDS;
C. Piper's with rounded head in multiparas. Kielland forceps TOPNOTCH MD)
D. Tucker-Mclane is ideal for rotating the head with occiput
E. Barton transverse. Barton forceps is used for rotation of
the head in transverse arrest.
456 A 30 year-old, who just delivered a healthy male Endometritis is a uterine infection with JULIET KRISTINE MIDTERM 2
neonate, had fever, hypogastric pain and odorous polymicrobial cause. Fever is the most important in EVANGELISTA, EXAM - AUG
vaginal discharge. Infection developed is most 2-3 days postpartum associated with abdominal MD (TOP 9 - FEB 2014
likely in the form of: pain and malodorous lochia. 2014 MED
A. Vaginitis BOARDS;
B. Salpingitis TOPNOTCH MD)
TOPNOTCH MEDICAL BOARD PREP OBSTETRICS-GYNECOLOGY SUPEREXAM Page 54 of 84
For inquiries visit www.topnotchboardprep.com.ph or email us at topnotchmedicalboardprep@gmail.com
TOPNOTCH MEDICAL BOARD PREP OBSTETRICS-GYNECOLOGY SUPEREXAM
For inquiries visit www.topnotchboardprep.com.ph or email us at topnotchmedicalboardprep@gmail.com
Ite QUESTION EXPLANATION AUTHOR TOPNOTCH
m # EXAM
C. Pelvic abscess
D. Endometritis
E. Peritonitis

457 A 26 year-old female with past history of Ectopic pregnancy is the implanation of trophoblast JULIET KRISTINE MIDTERM 2
gonococcal infection was seen at the Emergency other than the endometrium of the uterine cavity. EVANGELISTA, EXAM - AUG
Room due to moderate to severe lower abdominal The most frequent site is ampulla of the fallopian MD (TOP 9 - FEB 2014
pain associated with intermittent vaginal spotting tube. Clinical manifestations begin at 6-8 weeks 2014 MED
about 6 days duration. Her LMP was 6 weeks prior. with the triad of amenorrhea, vaginal bleeding and BOARDS;
Your initial diagnosis is: abdominal pain. TOPNOTCH MD)
A. ectopic pregnancy
B. ruptured ovarian cyst
C. endometriosis
D. recurrent gonorrhea
E. salpingitis
458 A 32 year-old, G1P0 at term suddenly had severe Abruptio placenta is usually a 3rd trimester painful JULIET KRISTINE MIDTERM 2
continuous low abdominal pain and tenderness bleeding with crampy abdominal pain in a patient EVANGELISTA, EXAM - AUG
associated with hypotension, tachycradia and with hypertension or a history of trauma. MD (TOP 9 - FEB 2014
nonreassuring fetal heart tone. She is most Stabilization of the mother and immediate delivery 2014 MED
probably having: of the fetus is warranted. BOARDS;
A. Start of labor TOPNOTCH MD)
B. Placenta previa
C. Abruptio placenta
D. Amniotic fluid embolism
E. Uterine rupture
459 Overriding of the fetal skull bones in x-ray at term Spalding sign is the overlapping of fetal skull bones, JULIET KRISTINE MIDTERM 2
is: a radiographic evidence to establish fetal death. EVANGELISTA, EXAM - AUG
A. Halo sign MD (TOP 9 - FEB 2014
B. Spalding sign 2014 MED
C. Indicates fetal prematurity BOARDS;
D. Possible cephalopelvic disproportion TOPNOTCH MD)
E. No significance
460 A 27 year-old woman who has been amenorrheic Complete mole is a dyspermic fertilization of an JULIET KRISTINE MIDTERM 2
for 12 weeks has an elevated serum HCG titer. D&C empty egg by one normal sperm. It is characterized EVANGELISTA, EXAM - AUG
was performed on the patient due to an incomplete by severe trophoblastic hyperplasia, hydropic or MD (TOP 9 - FEB 2014
abortion. Pathology report was available swollen chorionic villi and absent fetus and blood 2014 MED
describing a generalized trophoblastic vessels. BOARDS;
proliferation, hydropic villi without blood vessels TOPNOTCH MD)
and fetal parts. Diagnosis is most likely:
A. Choriocarcinoma
B. Partial mole
C. Complete mole
D. Incomplete abortion
E. Complete abortion
461 A 3 year old child was noticed by her mother to this is a case of adhesive vulvitis, in which the LUISA BACK-UP
frequently scratch her vulva. On examination, labia treatment is topical estrogen SARANILLO, MD MIDTERM EXAM
minora adheres in the midline with a translucent (TOP 6 - FEB AUG 2014 - FOR
vertical line. What is the treatment? 2014 MED INCLUSION IN
A. topical androgen BOARDS; THE SAMPLEX
B. topical combined estrogen and progesterone TOPNOTCH MD)
C. topical progesterone
D. Topical estrogen
E. Topical steroid
462 A 32 year old G2P2 mother is on her 2nd week The 3 types of secretions during puerperium are as LUISA BACK-UP
postpartum. She noticed vaginal secretions that are follows: lochia rubra - red color on days 1-3 SARANILLO, MD MIDTERM EXAM
pinkish in color. What do you call this secretions? postpartum; lochia serosa - more pale in color or (TOP 6 - FEB AUG 2014 - FOR
A. Lochia alba pinkish on days 4-10; and lochia alba -white to 2014 MED INCLUSION IN
B. Lochia rubra yellowish white on days 10 to 4-8 weeks BOARDS; THE SAMPLEX
C. Lochia serosa postpartum. TOPNOTCH MD)
D. Normal secretions
E. None of the choices
463 In preparation for labor, the uterus has increased In preparation for labor, estrogen is the principal LUISA BACK-UP
responsiveness to uterotonins and increased mediator, while progesterone level decreases SARANILLO, MD MIDTERM EXAM
contractility which is mediated primarily by: (TOP 6 - FEB AUG 2014 - FOR
A. estrogen 2014 MED INCLUSION IN
B. progesterone BOARDS; THE SAMPLEX
C. Beta HCG TOPNOTCH MD)
D. glycosaminoglycans
E. Prostaglandins
464 It is defined as menstrual cycle occuring every oligomenorrhea is defined as menstrual cycle LUISA BACK-UP
>35days with normal flow. occuring every >35 days with normal flow, while SARANILLO, MD MIDTERM EXAM
A. oligomenorrhea polymenorrhea is every <21 days. Hypomenorrhea (TOP 6 - FEB AUG 2014 - FOR
B. polymenorrhea is scanty menstruation, while menorrhagia is 2014 MED INCLUSION IN
C. hypomenorrhea excessive heavy menstruation. Metrorrhagia is any BOARDS; THE SAMPLEX
D. menorrhagia bleeding between normal menses. TOPNOTCH MD)
E. Metrorrhagia
465 The nerve supply of suprapubic area comes from: LUISA BACK-UP
A. Ilioinguinal nerve SARANILLO, MD MIDTERM EXAM
B. Iliolumbar nerve (TOP 6 - FEB AUG 2014 - FOR
C. Iliohypogastric nerve 2014 MED INCLUSION IN
D. Obturator nerve BOARDS; THE SAMPLEX
E. Pudendal nerve TOPNOTCH MD)
466 The following are the major criteria for polycystic the major criteria for PCOS are chronic anovulation, LUISA BACK-UP
ovarian syndrome: hyperandrogenemia and exclusion of other causes. SARANILLO, MD MIDTERM EXAM
A. Chronic anovulation Insulin resistance is one of the minor criteria. (TOP 6 - FEB AUG 2014 - FOR
B. hyperandrogenemia 2014 MED INCLUSION IN
TOPNOTCH MEDICAL BOARD PREP OBSTETRICS-GYNECOLOGY SUPEREXAM Page 55 of 84
For inquiries visit www.topnotchboardprep.com.ph or email us at topnotchmedicalboardprep@gmail.com
TOPNOTCH MEDICAL BOARD PREP OBSTETRICS-GYNECOLOGY SUPEREXAM
For inquiries visit www.topnotchboardprep.com.ph or email us at topnotchmedicalboardprep@gmail.com
Ite QUESTION EXPLANATION AUTHOR TOPNOTCH
m # EXAM
C. Insulin resistance BOARDS; THE SAMPLEX
D. All of the above TOPNOTCH MD)
E. Only A and B
467 Round ligament is continuous with the broad roud ligament arises below and anterior to the LUISA BACK-UP
ligament and extends from the lateral portion of origin of the oviducts SARANILLO, MD MIDTERM EXAM
the uterus. In relation to the oviducts, it is located: (TOP 6 - FEB AUG 2014 - FOR
A. Below and posterior to origin of the oviducts 2014 MED INCLUSION IN
B. Below and anterior to origin of the oviducts BOARDS; THE SAMPLEX
C. above and anterior to origin of the oviducts TOPNOTCH MD)
D. above and posterior to the origin of oviducts
E. lateral to the origin of the oviducts
468 A 50 year old female had amenorrhea for 12 IM testosterone will improved the libido. LUISA BACK-UP
months, hot flushes, and decreased libido. She SARANILLO, MD MIDTERM EXAM
wants to improved her libido. What will you give? (TOP 6 - FEB AUG 2014 - FOR
A. IM testosterone 2014 MED INCLUSION IN
B. IM estrogen BOARDS; THE SAMPLEX
C. IM progesterone TOPNOTCH MD)
D. Combined estrogen and progesterone IM
E. combined oral contraceptive
469 A 55 year old nulligravid presents with this is a case of endometrial hyperplasia. The LUISA BACK-UP
amenorrhea for 6 months followed by irregular management depends on the age, cytologic atypia SARANILLO, MD MIDTERM EXAM
vaginal bleeding. Endometrial sampling done and type of hyperplasia. In patients with complex (TOP 6 - FEB AUG 2014 - FOR
which shows complex hyperplasia with atypia. hyperplasia with atypia in menopausal women, the 2014 MED INCLUSION IN
What is your management? management is hysterectomy, while for BOARDS; THE SAMPLEX
A. Total abdominal hysterectomy with bilateral premenopausal women, it is high dose progestin. TOPNOTCH MD)
salpingo-oophoectomy
B. high dose progestin
C. hysterectomy
D. progestin
E. repeat endometrial sampling after 3 months
470 What is the method used in breech delivery if the When the entire body is extracted by the doctor, it LUISA BACK-UP
entire body is extracted by the doctor? is total breech delivery; it is partial if breech is SARANILLO, MD MIDTERM EXAM
A. Spontaneous breech delivery delivered spontaneously as far as umbilicus, but the (TOP 6 - FEB AUG 2014 - FOR
B. Total breech delivery remainder of the body is assisted; it is spontaneous 2014 MED INCLUSION IN
C. Partial breech delivery if the baby is expelled entirely without any traction BOARDS; THE SAMPLEX
D. Internal podalic version other than support. TOPNOTCH MD)
E. External cephalic version
471 A 35 year old G5P5 smoker, hypertensive mother painful vaginal bleeding after 20 weeks AOG with LUISA BACK-UP
at 34 weeks AOG presents with vaginal bleeding abdominal or uterine tenderness supports the SARANILLO, MD MIDTERM EXAM
associated with crampy abdominal pain. Upon diagnosis of abruptio placenta. Hypertension, (TOP 6 - FEB AUG 2014 - FOR
abdominal palpation, there's extreme tenderness. multiparity, advanced age, and smoking are risk 2014 MED INCLUSION IN
What is your primary consideration? factors of abruptio placenta. Placenta previa on the BOARDS; THE SAMPLEX
A. Placenta previa other hand, will present with painless vaginal TOPNOTCH MD)
B. Placenta accreta bleeding.
C. Placenta abruptio
D. vasa previa
E. normal labor
472 What is the most common type of cervical cancer? LUISA BACK-UP
A. Squamous type SARANILLO, MD MIDTERM EXAM
B. adenocarcinoma (TOP 6 - FEB AUG 2014 - FOR
C. squamoadenocarcinoma 2014 MED INCLUSION IN
D. Transitional type BOARDS; THE SAMPLEX
E. None of the choices TOPNOTCH MD)
473 A 38 year old female presents with dysmenorrhea adenomyosis is usually symptomatic, however it is LUISA BACK-UP
and heavy vaginal bleeding. Upon examination, you symptomatic in women >35 years old presenting SARANILLO, MD MIDTERM EXAM
noted a diffusely enlarged uterus approximately 2x with dysmenorrhea and menorrhagia with classice (TOP 6 - FEB AUG 2014 - FOR
its normal size. What is your diagnosis? pelvic exam of diffusely enlarged uterus 2-3x larger. 2014 MED INCLUSION IN
A. Endometriosis BOARDS; THE SAMPLEX
B. Pregnancy TOPNOTCH MD)
C. Myoma
D. Adenomyosis
E. none of the choices
474 One of the following is not a fetal risk factor Tobacco is a maternal risk factor, while the rest of LUISA BACK-UP
causing intrauterine growth restriction. the choices are fetal risk factors. SARANILLO, MD MIDTERM EXAM
A. Chromosomal abnormalities (TOP 6 - FEB AUG 2014 - FOR
B. Congenital infection 2014 MED INCLUSION IN
C. Tobacco BOARDS; THE SAMPLEX
D. Multiple gestation TOPNOTCH MD)
E. Congenital anomalies
475 What is the most common symptom of ectopic Abdominal pain, vaginal bleeding, and amenorrhea LUISA BACK-UP
pregnancy? are the components of ectopic triad. The most SARANILLO, MD MIDTERM EXAM
A. Abdominal pain common symptom is abdominal pain. Hypotension, (TOP 6 - FEB AUG 2014 - FOR
B. Amenorrhea tachycardia, along with peritoneal signs are signs of 2014 MED INCLUSION IN
C. Vaginal bleeding ruptured ectopic pregnancy. BOARDS; THE SAMPLEX
D. Hypotension TOPNOTCH MD)
E. Tachycardia
476 Which of the following is/are considered LUISA BACK-UP
physiologic cyst? SARANILLO, MD MIDTERM EXAM
A. Follicular cyst (TOP 6 - FEB AUG 2014 - FOR
B. Corpus luteum cyst 2014 MED INCLUSION IN
C. Theca lutein cyst BOARDS; THE SAMPLEX
D. All of the choices TOPNOTCH MD)
E. None of the choices

TOPNOTCH MEDICAL BOARD PREP OBSTETRICS-GYNECOLOGY SUPEREXAM Page 56 of 84


For inquiries visit www.topnotchboardprep.com.ph or email us at topnotchmedicalboardprep@gmail.com
TOPNOTCH MEDICAL BOARD PREP OBSTETRICS-GYNECOLOGY SUPEREXAM
For inquiries visit www.topnotchboardprep.com.ph or email us at topnotchmedicalboardprep@gmail.com
Ite QUESTION EXPLANATION AUTHOR TOPNOTCH
m # EXAM
477 A 70 year old female had uterine prolapse. Her Grading of uterine prolapse is as follows: when the LUISA BACK-UP
cervix descends halfway to the introitus. This is cervix descends half way to the introitus - I; to the SARANILLO, MD MIDTERM EXAM
grade: introitus - II; outside the introitus - III; entire uterus (TOP 6 - FEB AUG 2014 - FOR
A. I outside the introitus at all times -IV. 2014 MED INCLUSION IN
B. II BOARDS; THE SAMPLEX
C. III TOPNOTCH MD)
D. IV
E. V
478 What layer of decidua is eventually lost as decidua capsularis is the layer overlying the LUISA BACK-UP
pregnancy progresses? blastocyst.It is lost eventually due to obliteration of SARANILLO, MD MIDTERM EXAM
A. Decidua parietalis the growing fetus. (TOP 6 - FEB AUG 2014 - FOR
B. Decidua capsularis 2014 MED INCLUSION IN
C. Decidua basalis BOARDS; THE SAMPLEX
D. All of the above TOPNOTCH MD)
E. A and B only
479 What is the most common benign solid tumor of LUISA BACK-UP
the vulva? SARANILLO, MD MIDTERM EXAM
A. Lipoma (TOP 6 - FEB AUG 2014 - FOR
B. Bartholin's cyst 2014 MED INCLUSION IN
C. Urethral diverticulum BOARDS; THE SAMPLEX
D. Fibroma TOPNOTCH MD)
E. Urethral diverticulum
480 In the ovarian-endometrial cycle, what is GDF 9- regulates proliferation and differentiation of LUISA BACK-UP
responsible in the regulation of granulosa cells granulosa cells; TGF-beta - regulates extracellular SARANILLO, MD MIDTERM EXAM
proliferation and differentiation as primary matrix; epidermal growth factor - stimulates (TOP 6 - FEB AUG 2014 - FOR
follicles grow? stromal cell differentiation; vascular endothelial 2014 MED INCLUSION IN
A. Transforming Growth Factor-beta growth factor modulates angiogenesis; IGF I and II - BOARDS; THE SAMPLEX
B. Growth differentiation factor 9 differentiation of endometrium. TOPNOTCH MD)
C. Insulin-like growth factor
D. vascular endothelial growth factor
E. epidermal growth factor
481 What is the single most important risk factor for Cesarean section is the single most significant risk ANGELIS FINAL EXAM -
the development of uterine infection? factor for post-partum infections. ANDREA COCOS, AUG 2014
A. Duration of labor MD (TOP 1 - FEB
B. Route of delivery 2014 MED
C. Duration of membrane rupture BOARDS;
D. Number of internal vaginal examination TOPNOTCH MD)
482 Intake of oral contraceptive pills increases which Studies show that the use of OCPs lead to a 2x ANGELIS FINAL EXAM -
type of cancer? increase in cervical cancer even when confounding ANDREA COCOS, AUG 2014
A. cervical factors such as age at first intercourse, number of MD (TOP 1 - FEB
B. endometrial sexual partners, exposure to HPV, cytologic 2014 MED
C. breast screening, and the use of barrier methods were BOARDS;
D. Ovarian excluded. Some references say that this is due to TOPNOTCH MD)
OCPs effect in everting the transformation zone.
483 The following are known sequelae for babies of ANGELIS FINAL EXAM -
HBV-infected mothers, EXCEPT: ANDREA COCOS, AUG 2014
A. Low birth weight MD (TOP 1 - FEB
B. Prematurity 2014 MED
C. Congenital malformations BOARDS;
D. There is no exception TOPNOTCH MD)
484 A sexually active female presents with a beefy red Granuloma inguinale or donovanosis is caused by ANGELIS FINAL EXAM -
lesion with granulation tissue on her vulva. Which Klebsiella granulomatis. Chancre is a painless ANDREA COCOS, AUG 2014
is the most likely diagnosis? ulcerated lesion of primary syphilis. Chancroid is a MD (TOP 1 - FEB
A. chancre painful genital ulcer of H. ducreyi. LGV presents 2014 MED
B. chancroid with buboes or enlarged, tender lymphnodes. BOARDS;
C. granuloma inguinale TOPNOTCH MD)
D. LGV
485 A prostitute presents at the emergency room with The gold standard for the diagnosis of acute PID is ANGELIS FINAL EXAM -
recurrent fever, foul smelling vaginal discharge laparoscopy. ANDREA COCOS, AUG 2014
and hypogastric pain. The gold standard to MD (TOP 1 - FEB
diagnose the most likely disease of this patient is: 2014 MED
A. ultrasound BOARDS;
B. laparoscopy TOPNOTCH MD)
C. laparotomy
D. endometrial biopsy
486 How long does it take for the sperm to reach the SIMILAR TO PREVIOUS BOARD EXAM ANGELIS FINAL EXAM -
egg during intercourse? CONCEPT/PRINCIPLE. References say that the ANDREA COCOS, AUG 2014
A. 1 minute travel of the sperm from the vagina to reach the egg MD (TOP 1 - FEB
B. 30 minutes would take around 15-30 minutes. 2014 MED
C. 6 hours BOARDS;
D. 24 hours TOPNOTCH MD)
487 At 20 weeks AOG, the fundus of the gravid uterus 12 weeks AOG is at the level of the symphysis pubis, ANGELIS FINAL EXAM -
can be palpated: 16 weeks is midway between the symphysis pubis ANDREA COCOS, AUG 2014
A. At the level of the symphysis pubis and umbilicus. MD (TOP 1 - FEB
B. Midway between the symphysis pubis and 2014 MED
umbilicus BOARDS;
C. At the level of the umbilicus TOPNOTCH MD)
D. Within the pelvic cavity
488 A patient at the OPD consulted for post-coital Involvement of the parametria is the key phrase for ANGELIS FINAL EXAM -
bleeding associated with foul-smelling discharge. stage 2B. SIMILAR TO PREVIOUS BOARD EXAM ANDREA COCOS, AUG 2014
On internal exmaination, the cervix is converted to CONCEPT/PRINCIPLE. Study the recent FIGO MD (TOP 1 - FEB
a 5x5 cm nodular and friable mass with no staging. 2014 MED
extension to the vagina. The corpus is small. On BOARDS;
rectovaginal examination, there was involvement TOPNOTCH MD)
of the parametria. What stage is she in?
A. Cervical cancer stage 1B
TOPNOTCH MEDICAL BOARD PREP OBSTETRICS-GYNECOLOGY SUPEREXAM Page 57 of 84
For inquiries visit www.topnotchboardprep.com.ph or email us at topnotchmedicalboardprep@gmail.com
TOPNOTCH MEDICAL BOARD PREP OBSTETRICS-GYNECOLOGY SUPEREXAM
For inquiries visit www.topnotchboardprep.com.ph or email us at topnotchmedicalboardprep@gmail.com
Ite QUESTION EXPLANATION AUTHOR TOPNOTCH
m # EXAM
B. Cervical cancer stage 2B
C. Cervical cancer stage 3B
D. Cervical cancer stage 4B

489 The pathognomonic symptom of menopause is: The rest of the choices are also symptoms of ANGELIS FINAL EXAM -
A. Hot flushes menopause. ANDREA COCOS, AUG 2014
B. Cessation of menses MD (TOP 1 - FEB
C. Atrophic vaginitis 2014 MED
D. Osteoporosis BOARDS;
TOPNOTCH MD)
490 A 7-year-old girl is seen by her pediatrician for left Germ cell tumors are more likely in the pediatric ANGELIS FINAL EXAM -
lower quadrant pain. You identified an ovarian age group. Dermoid cysts or mature teratomas are ANDREA COCOS, AUG 2014
tumor by ultrasound. Of the following, the most more likely in the young adult. MD (TOP 1 - FEB
common ovarian tumor in this age group is? 2014 MED
A. Dermoid cyst BOARDS;
B. Germ cell tumor TOPNOTCH MD)
C. Fibrosarcoma
D. Papillary serous adenoma
491 The drug of choice for a patient complaining of GnRH antagonists are the drug of choice for ANGELIS FINAL EXAM -
dyspareunia, severe cyclical dysmenorrhea and endometriosis. ANDREA COCOS, AUG 2014
infertility is: MD (TOP 1 - FEB
A. estrogen 2014 MED
B. estrogen with progesterone BOARDS;
C. leuprolide TOPNOTCH MD)
D. danazol
492 A pre-eclamptic patient on prolonged labor has Preeclamptic patients are at risk for developing ANGELIS FINAL EXAM -
vaginal bleeding accompanied by non-reassuring abruptio placenta. ANDREA COCOS, AUG 2014
fetal heart rate pattern. Which is the most likely MD (TOP 1 - FEB
complication which occurred? 2014 MED
A. Placenta previa BOARDS;
B. Uterine rupture TOPNOTCH MD)
C. Vasa previa
D. Abruptio placenta
493 A syndrome of multiple congenital anomalies This is a classic case of fetal alcohol syndrome. ANGELIS FINAL EXAM -
including microcephaly, small palpebral fissues, ANDREA COCOS, AUG 2014
short nose, cleft lip and plate and CNS MD (TOP 1 - FEB
abnormalities can be see if the mother, during 2014 MED
pregnancy: BOARDS;
A. Drinks alcohol TOPNOTCH MD)
B. Smokes cigarettes
C. Takes phenytoin
D. Is diabetic
494 The following should be monitored in patients DTR and RR should be monitored to detect signs of ANGELIS FINAL EXAM -
being given magnesium sulfate, EXCEPT: toxicity. Monitoring of urine output is vital because ANDREA COCOS, AUG 2014
A. Deep tendon reflexes MgSO4 is renally excreted. MD (TOP 1 - FEB
B. Respiratory rate 2014 MED
C. Urine output BOARDS;
D. None of the above TOPNOTCH MD)
495 A 32-year-old G4P3 underwent suction curettage A similar trophoblastic disease question was asked ANGELIS FINAL EXAM -
for hydatidiform mole. A chest xray was done during our board exam. Methotrexate prophylaxis ANDREA COCOS, AUG 2014
revealing essentially normal findings. beta-HCG can be considered for high-risk patients (e.g. MD (TOP 1 - FEB
levels were high. Which is the most appropriate Metastasis to other organs). 2014 MED
next step? BOARDS;
A. Do a hysterectomy TOPNOTCH MD)
B. Monitor beta-HCG levels
C. Give methotrexate prophylaxis
D. Advice EMACO therapy
496 Depot medroxyprogesterone acetate can cause: Some disadvantages of DMPA include irregular ANGELIS FINAL EXAM -
A. Irregular menstrual bleeding menstrual bleeding, delay in fertility resumption ANDREA COCOS, AUG 2014
B. Immediate fertility resumption and weight gain. MD (TOP 1 - FEB
C. Weight loss 2014 MED
D. Infertility BOARDS;
TOPNOTCH MD)
497 The phase normally comprising 95% of pregnancy Phase 1, which composes 95% of pregnancy, is ANGELIS FINAL EXAM -
is: characterized by maintenance of cervical ANDREA COCOS, AUG 2014
A. Phase 1 anatomical and structural integrity. MD (TOP 1 - FEB
B. Phase 2 2014 MED
C. Phase 3 BOARDS;
D. Phase 4 TOPNOTCH MD)
498 A 35-year-old primigravid at 36 weeks AOG ANGELIS FINAL EXAM -
consulted for her prenatal care. She complained ANDREA COCOS, AUG 2014
that her abdomen seems to be smaller and she MD (TOP 1 - FEB
feels as if "the baby dropped". This is termed as: 2014 MED
A. descent BOARDS;
B. engagement TOPNOTCH MD)
C. lightening
D. Labor progression
499 A 45-year-old G6P6 consulted at the OPD with a Colposcopy is a diagnostic procedure to closely ANGELIS FINAL EXAM -
finding of LSIL in her Papsmear. The next step in examine the cervix. This is the next step in order to ANDREA COCOS, AUG 2014
the management of her case is: evaluate an abnormal finding in papsmear. MD (TOP 1 - FEB
A. Total hysterectomy with bilateral salpingo- 2014 MED
oophorectomy BOARDS;
B. conization TOPNOTCH MD)
C. observation
D. Colposcopy

TOPNOTCH MEDICAL BOARD PREP OBSTETRICS-GYNECOLOGY SUPEREXAM Page 58 of 84


For inquiries visit www.topnotchboardprep.com.ph or email us at topnotchmedicalboardprep@gmail.com
TOPNOTCH MEDICAL BOARD PREP OBSTETRICS-GYNECOLOGY SUPEREXAM
For inquiries visit www.topnotchboardprep.com.ph or email us at topnotchmedicalboardprep@gmail.com
Ite QUESTION EXPLANATION AUTHOR TOPNOTCH
m # EXAM
500 The presence of a uterus and fallopian tubes in an The default sex is female, hence without the ANGELIS FINAL EXAM -
otherwise phenotypically normal male is due to: mullerian-inhibiting factor, the female internal ANDREA COCOS, AUG 2014
A. Lack of mullerian-inhibiting factor organs would develop despite the predominant MD (TOP 1 - FEB
B. Lack of testosterone presence of testosterone. 2014 MED
C. Increased levels of estrogen BOARDS;
D. Presence of ovarian tissue TOPNOTCH MD)
501 This provides surgical access to the peritoneal Topnotch handouts: posterior fornix- for JAN CHARMAINE BACK-UP
cavity: culdocentesis PALOMAR, MD MIDTERM EXAM
A. Anterior fornix (TOP 9 - FEB AUG 2014
B. Right lateral fornix 2014 MED
C. Left lateral fornix BOARDS;
D. Posterior fornix TOPNOTCH MD)
E. All of the above
502 A 3 year old little girl was brought to the ER Topnotch handouts JAN CHARMAINE BACK-UP
because of perineal discomfort and yellowish PALOMAR, MD MIDTERM EXAM
vaginal discharge. You suspect rape. During the (TOP 9 - FEB AUG 2014
first coitus, which part of the hymen would most 2014 MED
likely rupture first? BOARDS;
A. 12 o' clock position TOPNOTCH MD)
B. 3 o' clock position
C. 6 o' clock position
D. 9 o' clock position
E. none of the above
503 Which of the following is not true regarding also prevents expansion of the urogenital hiatus JAN CHARMAINE BACK-UP
perineal body? PALOMAR, MD MIDTERM EXAM
A. It anchors the anorectum and the vagina. (TOP 9 - FEB AUG 2014
B. It helps maintain urinary and fecal continence. 2014 MED
C. It provides physical barrier between the vagina BOARDS;
and rectum. TOPNOTCH MD)
D. It prevents expansion of the urogenital hiatus.
E. None of the above.
504 This ligament extend from the lateral portion of the JAN CHARMAINE BACK-UP
uterus, arising below and anterior to origin of the PALOMAR, MD MIDTERM EXAM
oviducts, that is continuous with the broad (TOP 9 - FEB AUG 2014
ligament, outward and downward to the inguinal 2014 MED
canal terminating at upper portion of labium BOARDS;
majus: TOPNOTCH MD)
A. Broad ligament
B. Cardinal ligament
C. Transverse cervical ligament
D. Uterosacral ligament
E. Round ligament
505 This segment of the uterine tube is the widest and Intramural/Interstitial- within muscular wall; JAN CHARMAINE BACK-UP
most tortuous. This is the most common site of ectopic pregnancy here result in severe maternal PALOMAR, MD MIDTERM EXAM
occurrence of ectopic prenancy: morbidity Isthmus-narrowest portion; preferred (TOP 9 - FEB AUG 2014
A. Intramural portion for BTL Infundibulum - fimbriated 2014 MED
B. Interstitial extremity; funnel-shaped opening of the distal end BOARDS;
C. Isthmus of the fallopian tube TOPNOTCH MD)
D. Ampulla
E. Infundibulum
506 This is the shortest distance between the diagonal conjugate has to >11.5cm to be adequate JAN CHARMAINE BACK-UP
promontory of the sacrum to the lower margin of PALOMAR, MD MIDTERM EXAM
the symphysis pubis that can be measured (TOP 9 - FEB AUG 2014
clinically. 2014 MED
A. True conjugate BOARDS;
B. Obstetrical conjugate TOPNOTCH MD)
C. Diagonal conjugate
D. All of the above
E. None of the above
507 This is the male homologue of the vagina: A. Prostate gland - Skene's glands JAN CHARMAINE BACK-UP
A. Prostate gland B. Prostatic utricle - vagina PALOMAR, MD MIDTERM EXAM
B. Prostatic utricle C. Bulbourethral gland- greater vestibular glands (TOP 9 - FEB AUG 2014
C. Bulbourethral gland D. Seminal colliculus-hymen 2014 MED
D. Seminal colliculus E. Penis - clitoris BOARDS;
E. Penis TOPNOTCH MD)
508 A 6 year old girl was incidentally found to have a Exploratory laparotomy is advised in premenarchal JAN CHARMAINE BACK-UP
cystic right adnexal mass measuring 4 cm. What patients with adnexal mass >2cm. -topnotch PALOMAR, MD MIDTERM EXAM
course of management should you advise the handout (TOP 9 - FEB AUG 2014
mother of patient? 2014 MED
A. Observe for 8-12 weeks BOARDS;
B. Do exploratory laparotomy TOPNOTCH MD)
C. Reassure mother
D. Do serial Transabdominal ultrasound
E. Do scout film of the abomen
509 The following are definitive evidence of prenancy, JAN CHARMAINE BACK-UP
except: PALOMAR, MD MIDTERM EXAM
A. Identification of fetal heart action. (TOP 9 - FEB AUG 2014
B. Perception of fetal movement by the doctor. 2014 MED
C. Recognition fetus by sonographic exam. BOARDS;
D. Beta- HCG titer of more than 1500 IU/L TOPNOTCH MD)
E. None of the above
510 CTG tracing showed mirror images of uterine Early decelerations- head compression Variable JAN CHARMAINE BACK-UP
contractions and fetal heart rate deceleration. decelerations- umbilical cord compression Late PALOMAR, MD MIDTERM EXAM
What caused this pattern? deceleration-Uteroplacental insufficiency (TOP 9 - FEB AUG 2014
A. Fetal movement 2014 MED
B. Head compression BOARDS;
TOPNOTCH MEDICAL BOARD PREP OBSTETRICS-GYNECOLOGY SUPEREXAM Page 59 of 84
For inquiries visit www.topnotchboardprep.com.ph or email us at topnotchmedicalboardprep@gmail.com
TOPNOTCH MEDICAL BOARD PREP OBSTETRICS-GYNECOLOGY SUPEREXAM
For inquiries visit www.topnotchboardprep.com.ph or email us at topnotchmedicalboardprep@gmail.com
Ite QUESTION EXPLANATION AUTHOR TOPNOTCH
m # EXAM
C. Umbilical cord compression TOPNOTCH MD)
D. Uteroplacental insufficiency
E. Any of the above

511 Marcia 21 year old G1P0 14 weeks AOG was Inevitable or imminent abortion JAN CHARMAINE BACK-UP
rushed to the hospital because of vaginal bleeding, PALOMAR, MD MIDTERM EXAM
watery vaginal discharge and hypogastric pain. (TOP 9 - FEB AUG 2014
Examination revealed cervix to be 4 cm dilated 2014 MED
with no fetal cardiac activity on ultrasonography. BOARDS;
What is your impression? TOPNOTCH MD)
A. Threatened abortion
B. Inevitable abortion
C. Complete abortion
D. Incomplete abortion
E. Missed abortion
512 Maria, 19 year old G1P1, previously treated for PID JAN CHARMAINE BACK-UP
2 years ago was brought in the hospital because of PALOMAR, MD MIDTERM EXAM
severe hypogastric pain and vaginal spotting (TOP 9 - FEB AUG 2014
consuming 3 pads per day, lightly soaked. History 2014 MED
revealed delayed menses for 3 weeks now. What is BOARDS;
the gold standard procedure for the diagnosis of TOPNOTCH MD)
Ectopic pregnancy?
A. Quatitative serum B-HCG
B. Sonography
C. Laparotomy
D. Laparoscopy
E. Culdocentesis
513 The 3rd stage of labor includes the period from SIMILAR TO PREVIOUS BOARD EXAM JAN CHARMAINE BACK-UP
fetal delivery to placental expulsion. What does CONCEPT/PRINCIPLE:all refer to the signs of PALOMAR, MD MIDTERM EXAM
Calkin sign refer to? placental separation (TOP 9 - FEB AUG 2014
A. Uterus become globular and firmer 2014 MED
B. Sudden gush of blood BOARDS;
C. Uterus rise in the abdomen TOPNOTCH MD)
D. Lengthening of the umbilical cord
E. None of the above
514 What is the therapeutic level of Magnesium SIMILAR TO PREVIOUS BOARD EXAM JAN CHARMAINE BACK-UP
sulfate? CONCEPT/PRINCIPLE: 4-7 mEq/L-therapeutic PALOMAR, MD MIDTERM EXAM
A. 5 mEq/L level; 10 mEq/L- disappearance of patellar reflex; (TOP 9 - FEB AUG 2014
B. 10 mEq/L >12 mEq/L- respiratory depression; >15mEq/L- 2014 MED
C. 12 mEq/L respiratory depression with mechanical ventilation; BOARDS;
D. >15 mEq/L >30-35-cardiac arrest TOPNOTCH MD)
E. 30-35 mEq/L
515 This syndrome result from meiotic nondysjunction JAN CHARMAINE BACK-UP
leading to 47, XXY genotype, with associated PALOMAR, MD MIDTERM EXAM
testicular atrophy, eunuchoid body shape, tall (TOP 9 - FEB AUG 2014
stature, long extremities, gynecomastia and female 2014 MED
hair distribution: BOARDS;
A. Hermaphroditism TOPNOTCH MD)
B. Klinefelter Syndrome
C. Turner Syndrome
D. Androgen insensitivity
E. Down Syndrome
516 This is the first sign of puberty in females: Thelarche or breast bud formation is the first sign JAN CHARMAINE BACK-UP
A. Axillary hair of puberty in females PALOMAR, MD MIDTERM EXAM
B. Onset of menses (TOP 9 - FEB AUG 2014
C. Growth spurt 2014 MED
D. Breast bud formation BOARDS;
E. None of the above TOPNOTCH MD)
517 This syndrome is characterized by hypothalamic- Kallman Syndrome- Hypogonadotropic JAN CHARMAINE BACK-UP
pituitary dysfunction caused by congenital absence hypogonadism characterized by isolated PALOMAR, MD MIDTERM EXAM
of GNRH associated with anosmia: gonadotropin deficiency associated with anosmia (TOP 9 - FEB AUG 2014
A. Savage Syndrome 2014 MED
B. Kallman Syndrome BOARDS;
C. Turner Syndrome TOPNOTCH MD)
D. Premature ovarian Failure
E. None of the above
518 Shally is a 37 year old G2P2 who came in to your Asherman syndrome- intrauterine adhesion with JAN CHARMAINE BACK-UP
clinic because of absence od menses for the last 8 history of previous endometrial curettage PALOMAR, MD MIDTERM EXAM
months. Her mother had her menopause at age 39. Premature ovarian failure- cessation of ovarian (TOP 9 - FEB AUG 2014
What is the most probable diagnosis of Shally's function before age 40 2014 MED
condition? BOARDS;
A. Asherman syndrome TOPNOTCH MD)
B. Premature ovarian failure
C. Polycystic Ovarian Syndrome
D. All of the above
E. None of the above
519 What is the treatment of choice for hot flushes of JAN CHARMAINE BACK-UP
menopause? PALOMAR, MD MIDTERM EXAM
A. Progesterone supplement (TOP 9 - FEB AUG 2014
B. Leuprolide 2014 MED
C. Estrogen BOARDS;
D. Calcium + vitamin D TOPNOTCH MD)
E. DEXA

TOPNOTCH MEDICAL BOARD PREP OBSTETRICS-GYNECOLOGY SUPEREXAM Page 60 of 84


For inquiries visit www.topnotchboardprep.com.ph or email us at topnotchmedicalboardprep@gmail.com
TOPNOTCH MEDICAL BOARD PREP OBSTETRICS-GYNECOLOGY SUPEREXAM
For inquiries visit www.topnotchboardprep.com.ph or email us at topnotchmedicalboardprep@gmail.com
Ite QUESTION EXPLANATION AUTHOR TOPNOTCH
m # EXAM
520 Mila, 78 year old G5P5 was recently diagnosed to JAN CHARMAINE BACK-UP
have Endometrial Carcinoma. What is the PALOMAR, MD MIDTERM EXAM
definitive management? (TOP 9 - FEB AUG 2014
A. Progestin 2014 MED
B. Progestin, Do hysterectomy if with bleeding BOARDS;
C. Hysterectomy TOPNOTCH MD)
D. TAH-BSO
E. Explratory laparotomy
521 A 19 year old primigravid at 40 weeks AOG MIGUEL RAFAEL MIDTERM 1
undergoes fetal heart monitoring. Pregnancy has RAMOS, MD (TOP EXAM - FEB
been uncomplicated. External monitoring shows a 3 - FEB 2012 2013
baseline heart rate of 140 bpm with good MED BOARDS;
variability; over a period of 30 minutes, the rate TOPNOTCH MD)
increases twice to 160 bpm for 25 to 30 seconds.
Which of the following is the most appropriate step
in management?
A) Reassurance
B) Biophysical profile
C) Oxytocin challenge test
D) Induction of labor
522 A 32 year old G2P1 at 40 weeks AOG is brought to MIGUEL RAFAEL MIDTERM 1
the ER by her husband because she has been RAMOS, MD (TOP EXAM - FEB
confused for 45 minutes. Her husband says that 3 - FEB 2012 2013
she has been in labor for 3 days at home, and she MED BOARDS;
has received all her prenatal care from an TOPNOTCH MD)
alternative provider. On arrival, she is obtunded.
Her pulse is 140 bpm, and palpable systolic blood
pressure is 60 mmHg. Abdominal examination
shows distention and rigidity and a 25-cm
irregular, mobile mass in the upper right quadrant.
The cervix is 3 cm dilated and 50% effaced. Which
of the following is the most likely cause of these
findings?
A) Coagulopathy
B) Uterine rupture
C) Endomyometritis
D) Uterine atony
523 A previously healthy 23 year old G2P1 at 32 weeks MIGUEL RAFAEL MIDTERM 1
AOG comes to the physician because of urinary RAMOS, MD (TOP EXAM - FEB
urgency and pain with urination or 3 days. Her 3 - FEB 2012 2013
pregnancy has ben uncomplicated. Examination MED BOARDS;
shows a uterus consistent in size with a 32 week TOPNOTCH MD)
gestation. There is mild suprapubic tenderness but
no costovertebral angle tenderness on palpation.
Urinalysis reveals numerous WBCs and RBCs and
3+ bacteria. The patient is at greatest risk for
which of the following?
A) Chorioamnionitis
B) Renal failure
C) Pyelonephritis
D) Abruption placentae
524 A 32 year old woman comes for follow-up MIGUEL RAFAEL MIDTERM 1
examination 1 week after a Pap smear showed a RAMOS, MD (TOP EXAM - FEB
high-grade squamos intraepithelial lesion. 3 - FEB 2012 2013
Examination shows no abnormalities. Which of the MED BOARDS;
following is the most appropriate next step in TOPNOTCH MD)
management?
A) Trichloroacetic acid therapy
B) Colposcopic-direct biopsy
C) Cone biopsy of cervix
D) Repeat Pap smear
525 An 82 year old woman comes to the physician MIGUEL RAFAEL MIDTERM 1
because of a 9 month history of progressive RAMOS, MD (TOP EXAM - FEB
urinary incontinence. At least once daily, she has a 3 - FEB 2012 2013
strong urge to void, is unable to reach a bathroom MED BOARDS;
in time, and spontaneously passes a large amount TOPNOTCH MD)
of urine. She had a mild cerebral infarction 2 years
ago with no residual weakness. She has severe
arthritis of her knees and hips, which severely
limits her mobility. Current medications include
lovastatin and aspirin. Examination, including
pelvic examination, shows no abnormalities. Which
of the following is the most likely explanation for
this patient’s incontinence?
A) Detrusor hyperactivity
B) Intrinsic weakness of the urethral sphincter
C) Poor pelvic support
D) Outflow obstruction
526 An 18-year-old primigravid woman comes for her MIGUEL RAFAEL MIDTERM 1
initial prenatal visit at 16 weeks' gestation. She is RAMOS, MD (TOP EXAM - FEB
not sure about the date of her last menstrual 3 - FEB 2012 2013
period but says that the pregnancy probably MED BOARDS;
occurred immediately after she stopped taking oral TOPNOTCH MD)
contraceptives 5 months ago. Maternal serum α-
fetoprotein (MSAFP) level is increased to 3
multiples of the median. Which of the following is
TOPNOTCH MEDICAL BOARD PREP OBSTETRICS-GYNECOLOGY SUPEREXAM Page 61 of 84
For inquiries visit www.topnotchboardprep.com.ph or email us at topnotchmedicalboardprep@gmail.com
TOPNOTCH MEDICAL BOARD PREP OBSTETRICS-GYNECOLOGY SUPEREXAM
For inquiries visit www.topnotchboardprep.com.ph or email us at topnotchmedicalboardprep@gmail.com
Ite QUESTION EXPLANATION AUTHOR TOPNOTCH
m # EXAM
the most appropriate next step in management?
A) Repeat measurement of MSAFP level
B) Triple screening for MSAFP, serum β-hCG, and
serum estriol levels
C) Ultrasonography
D) Amniocentesis for measurement of α-
fetoprotein level
527 A previously healthy 87-year-old woman comes to MIGUEL RAFAEL MIDTERM 1
the physician because of a 4-month history of RAMOS, MD (TOP EXAM - FEB
vulvar itching. Examination shows excoriated 3 - FEB 2012 2013
areas from scratching and a white, thin vulva. The MED BOARDS;
labia minora are absent, and there are small TOPNOTCH MD)
fissures at the introitus. The remainder of the
examination shows no abnormalities. Which of the
following is the most likely diagnosis?
A) Lichen planus
B) Lichen sclerosus
C) Squamous cell carcinoma
D) Atrophic vaginitis
528 Four hours after undergoing a cesarean delivery at MIGUEL RAFAEL MIDTERM 1
term followed by tubal ligation, a 37-year-old RAMOS, MD (TOP EXAM - FEB
woman, gravida 2, para 2, has dizziness and 3 - FEB 2012 2013
confusion. The operation was uncomplicated, and MED BOARDS;
blood loss is estimated to be 800 mL. Patient- TOPNOTCH MD)
controlled epidural analgesia has been moderately
effective for pain. Her blood pressure now is 80/40
mm Hg, decreased from 120/72 mm Hg
intraoperatively, and pulse is 152/min, increased
from 96/min intraoperatively. Breath sounds are
decreased bilaterally. No murmurs are heard.
Abdominal examination shows distention and
tenderness. Bowel sounds are absent. The incision
is intact with no drainage. She is disoriented to
person, place, and time. Her hematocrit is 23%;
preoperative hematocrit was 35%. Which of the
following is the most likely cause of the
hemodynamic changes?
A) Epidural-related hypotension
B) Insufficient intraoperative fluid replacement
C) Postoperative intra-abdominal hemorrhage
D) Supine hypotensive syndrome
529 A healthy 24-year-old woman comes for a routine MIGUEL RAFAEL MIDTERM 1
health maintenance examination. Menses occur at RAMOS, MD (TOP EXAM - FEB
regular 28-day intervals and last 5 to 6 days. Her 3 - FEB 2012 2013
last menstrual period was 3 weeks ago. She takes MED BOARDS;
no medications. Bimanual examination shows a 5- TOPNOTCH MD)
cm, mildly tender left adnexa. A pregnancy test is
negative. Which of the following is the most
appropriate next step in management?
A) Repeat examination in 2 weeks
B) Measurement of serum CA 125 level
C) Measurement of serum α-fetoprotein level
D) CT scan of the pelvis
530 A 32-year-old woman at 38 weeks' gestation MIGUEL RAFAEL MIDTERM 1
comes for a routine prenatal visit. During routine RAMOS, MD (TOP EXAM - FEB
screening at 28 weeks' gestation, she tested 3 - FEB 2012 2013
positive for hepatitis B surface antigen. Her MED BOARDS;
pregnancy has been otherwise uncomplicated. TOPNOTCH MD)
Examination shows a uterus consistent in size with
a 38-week gestation. Which of the following
measures is most likely to decrease the risk for
hepatitis B infection in her newborn?
A) Recommendation of bottle-feeding rather than
breast-feeding
B) Maternal administration of hepatitis B immune
globulin (HBIG) now
C) Neonatal administration of HBIG after delivery
and hepatitis B vaccine at 3 months of age
D) Neonatal administration of HBIG and hepatitis B
vaccine immediately after delivery
531 Five weeks after vaginal delivery of a healthy full- thoughts of suicide and infanticide >> indications MIGUEL RAFAEL MIDTERM 1
term newborn, a 22-year-old woman, gravida 1, for hospitalization RAMOS, MD (TOP EXAM - FEB
para 1, is brought to the physician by her mother 3 - FEB 2012 2013
because of depressed mood for 2 weeks. Her MED BOARDS;
mother is concerned that her daughter is not able TOPNOTCH MD)
to take care of her infant. Physical examination
shows no abnormalities. She is quiet and tearful
and does not engage in conversation easily. She
states that she lives alone with her infant and has
had thoughts of suicide and infanticide. Which of
the following is the most appropriate next step in
management?
A) Reassurance
B) Long-term outpatient counseling
C) Admission to the hospital for treatment
D) Selective serotonin reuptake inhibitor therapy

TOPNOTCH MEDICAL BOARD PREP OBSTETRICS-GYNECOLOGY SUPEREXAM Page 62 of 84


For inquiries visit www.topnotchboardprep.com.ph or email us at topnotchmedicalboardprep@gmail.com
TOPNOTCH MEDICAL BOARD PREP OBSTETRICS-GYNECOLOGY SUPEREXAM
For inquiries visit www.topnotchboardprep.com.ph or email us at topnotchmedicalboardprep@gmail.com
Ite QUESTION EXPLANATION AUTHOR TOPNOTCH
m # EXAM
532 A 23-year-old woman has pain, cramping, and MIGUEL RAFAEL MIDTERM 1
swelling of the right calf 3 days after an RAMOS, MD (TOP EXAM - FEB
uncomplicated labor and delivery. The right foot is 3 - FEB 2012 2013
swollen, and there is marked tenderness with MED BOARDS;
dorsiflexion and palpation of the right calf. TOPNOTCH MD)
Examination shows no other abnormalities. A
complete blood count and serum electrolyte levels
are within normal limits. Which of the following is
the most likely cause of this condition?
A) Hypercoagulable state of pregnancy
B) Hyperuricemia
C) Peripheral artery aneurysm
D) Platelet embolus
533 A 28-year-old nulligravid woman comes for a MIGUEL RAFAEL MIDTERM 1
routine health maintenance examination. She has RAMOS, MD (TOP EXAM - FEB
had progressively severe dysmenorrhea over the 3 - FEB 2012 2013
past 6 months adequately controlled by MED BOARDS;
nonsteroidal anti-inflammatory agents. Pelvic TOPNOTCH MD)
examination shows a normal vagina and cervix.
The uterus is retroverted and fixed, and there is
nodularity of the cul-de-sac. A 6-cm left adnexal
mass is palpated. Transvaginal ultrasonography
shows a 7-cm septated adnexal mass. Four weeks
later, there is no change in the size of the adnexal
mass. Which of the following is the most
appropriate diagnostic test?
A) Measurement of serum CA 125 level
B) Laparoscopy
C) CT scan of the pelvis
D) MRI of the pelvis
534 A 32-year-old woman, gravida 2, para 2, comes to (+) Progestin challenge test >> anovulation MIGUEL RAFAEL MIDTERM 1
the physician because she has been amenorrheic RAMOS, MD (TOP EXAM - FEB
for 4 months. Examination shows a well- 3 - FEB 2012 2013
estrogenized vagina and no evidence of virilization MED BOARDS;
or other abnormalities. A serum pregnancy test is TOPNOTCH MD)
negative. She is given medroxyprogesterone and
has the onset of bleeding 3 days later. Which of the
following is the most likely cause of her condition?
A) Anovulation
B) Asherman's syndrome
C) Hypopituitarism
D) Premature ovarian failure
535 A healthy 37-year-old primigravid woman at 12 MIGUEL RAFAEL MIDTERM 1
weeks' gestation comes for a routine prenatal visit. RAMOS, MD (TOP EXAM - FEB
The pregnancy was achieved by in vitro 3 - FEB 2012 2013
fertilization. She does not use tobacco, alcohol, or MED BOARDS;
drugs. She is a hematologist and works 10 to 12 TOPNOTCH MD)
hours daily. Two previous ultrasonographies have
shown a triplet gestation. She weighs 66 kg (145
lb) and is 178 cm (70 in) tall. Her blood pressure is
116/70 mm Hg, and pulse is 72/min. Examination
shows a uterus consistent in size with a 16-week
gestation. Her pelvis is normal-sized. This patient
is at greatest risk for which of the following?
A) Abruptio placentae
B) Hepatitis B
C) HIV infection
D) Preterm labor
536 A 30-year-old woman, gravida 2, para 1, is brought MIGUEL RAFAEL MIDTERM 1
to the emergency department in labor. An RAMOS, MD (TOP EXAM - FEB
episiotomy is performed. Following delivery of the 3 - FEB 2012 2013
head, the shoulders do not follow with the usual MED BOARDS;
traction and maternal pushing. Which of the TOPNOTCH MD)
following is the most appropriate next step in
management?
A) Flexing the woman's knees toward her
shoulders
B) More forceful traction and fundal pressure
C) Delivering the posterior arm
D) Symphysiotomy
537 A 13-year-old girl is brought for a well-child MIGUEL RAFAEL MIDTERM 1
examination. Menses have occurred every other RAMOS, MD (TOP EXAM - FEB
month since menarche 10 months ago. Her last 3 - FEB 2012 2013
menstrual period was 1 week ago. She is not MED BOARDS;
sexually active. Sexual development is Tanner TOPNOTCH MD)
stage 3. Examination shows no abnormalities.
Which of the following is the most appropriate
next step in management?
A) Discussion of pregnancy prevention
B) Measurement of serum luteinizing and follicle-
stimulating hormone levels
C) Urine β-hCG test
D) Pelvic examination

TOPNOTCH MEDICAL BOARD PREP OBSTETRICS-GYNECOLOGY SUPEREXAM Page 63 of 84


For inquiries visit www.topnotchboardprep.com.ph or email us at topnotchmedicalboardprep@gmail.com
TOPNOTCH MEDICAL BOARD PREP OBSTETRICS-GYNECOLOGY SUPEREXAM
For inquiries visit www.topnotchboardprep.com.ph or email us at topnotchmedicalboardprep@gmail.com
Ite QUESTION EXPLANATION AUTHOR TOPNOTCH
m # EXAM
538 A 52-year-old woman comes to the physician MIGUEL RAFAEL MIDTERM 1
because of decreased libido; this symptom began 8 RAMOS, MD (TOP EXAM - FEB
months ago, after she underwent a total abdominal 3 - FEB 2012 2013
hysterectomy and bilateral salpingo-oophorectomy MED BOARDS;
for leiomyomata uteri and menorrhagia. She has TOPNOTCH MD)
been taking hormone replacement therapy with
conjugated estrogen since the operation.
Examination shows a moist, rugated vagina. Which
of the following is the most likely cause of these
findings?
A) Decreased androgens
B) Decreased estrogen
C) Decreased follicle-stimulating hormone (FSH)
D) Decreased luteinizing hormone (LH)
539 A 38-year-old woman, gravida 2, para 1, at 38 MIGUEL RAFAEL MIDTERM 1
weeks' gestation has had no fetal movement for 36 RAMOS, MD (TOP EXAM - FEB
hours. Her prenatal course, prenatal tests, and fetal 3 - FEB 2012 2013
growth have been normal. Fetal heart tones are MED BOARDS;
heard by Doppler. Which of the following is the TOPNOTCH MD)
most appropriate next step in management?
A) Routine prenatal visit in 1 week
B) Maternal hydration
C) Nonstress test
D) Immediate induction of labor
540 A 22-year-old primigravid woman at 16 weeks' cannon ball lesions MIGUEL RAFAEL MIDTERM 1
gestation is brought to the emergency department RAMOS, MD (TOP EXAM - FEB
because of progressive shortness of breath over 3 - FEB 2012 2013
the past 48 hours. Her temperature is 37 C (98.6 MED BOARDS;
F), blood pressure is 120/70 mm Hg, pulse is TOPNOTCH MD)
100/min, and respirations are 24/min. Scattered
wheezes are heard. Pelvic examination shows a
uterus that extends to the umbilicus. Her
hematocrit is 32%, leukocyte count is
11,000/mm3, and serum β-hCG level is 300,000
mIU/mL. Pulse oximetry on room air shows an
oxygen saturation of 92%. An x-ray film of the
chest shows multiple round densities throughout
all lung fields. Which of the following is the most
likely diagnosis?
A) Bacterial pneumonia
B) Choriocarcinoma
C) Pulmonary embolism
D) Tuberculosis
541 Which of the following bears the greatest risk for ABDELSIMAR FINAL EXAM -
ectopic pregnancy? OMAR II, MD FEB 2014
A. Previous ectopic pregnancy (TOP 2 - AUG
B. Tubal corrective surgery 2013 MED
C. Infertility BOARDS;
D. Previous genital infection TOPNOTCH MD -
E. Prior Abortion 200 QUESTIONS)
AND MARC
DENVER
TIONGSON, MD
(40 QUESTIONS)
542 The appearance of stromal nodules in the prostate ABDELSIMAR FINAL EXAM -
occurs in the: OMAR II, MD FEB 2014
A. 2nd decade of life (TOP 2 - AUG
B. 3rd decade of life 2013 MED
C. 4th decade of life BOARDS;
D. 5th decade of life TOPNOTCH MD -
E. 6th decade of life 200 QUESTIONS)
AND MARC
DENVER
TIONGSON, MD
(40 QUESTIONS)
543 A hot flush is usually followed by ABDELSIMAR FINAL EXAM -
A. increased digital perfusion OMAR II, MD FEB 2014
B. increased peripheral skin temperature (TOP 2 - AUG
C. decreased LH 2013 MED
D. decreased HR BOARDS;
E. decrease in cortisol TOPNOTCH MD -
200 QUESTIONS)
AND MARC
DENVER
TIONGSON, MD
(40 QUESTIONS)
544 A 46 year old asymptomatic patient is found to ABDELSIMAR FINAL EXAM -
have an 8 week size irregular myomatous uterus. OMAR II, MD FEB 2014
The best management would be: (TOP 2 - AUG
A. administration of GnRH analogues 2013 MED
B. reevaluation in 6 to 12 months BOARDS;
C. fractional dilatation and curettage TOPNOTCH MD -
D. myomectomy 200 QUESTIONS)
E. hysterectomy AND MARC
DENVER
TIONGSON, MD
TOPNOTCH MEDICAL BOARD PREP OBSTETRICS-GYNECOLOGY SUPEREXAM Page 64 of 84
For inquiries visit www.topnotchboardprep.com.ph or email us at topnotchmedicalboardprep@gmail.com
TOPNOTCH MEDICAL BOARD PREP OBSTETRICS-GYNECOLOGY SUPEREXAM
For inquiries visit www.topnotchboardprep.com.ph or email us at topnotchmedicalboardprep@gmail.com
Ite QUESTION EXPLANATION AUTHOR TOPNOTCH
m # EXAM
(40 QUESTIONS)

545 Correct use of lactation amenorrhea EXCEPT The use of lactation amenorrhea must first fulfill 3 ABDELSIMAR FINAL EXAM -
A. Mother must breast feed day and night criteria: exclusive or almost exclusively OMAR II, MD FEB 2014
B. Baby is more than 6 months old breastfeeding, have not had menses since giving (TOP 2 - AUG
C. Mother has not started menses birth, and mother should be less than 6 months 2013 MED
D. Infant must receive at least 85% of feeding as post-partum. BOARDS;
breastfeeding TOPNOTCH MD -
E. None of the above 200 QUESTIONS)
AND MARC
DENVER
TIONGSON, MD
(40 QUESTIONS)
546 A 28 year old patient G2P1 (1001) 39 weeks AOG ABDELSIMAR FINAL EXAM -
CIL, is undergoing contraction stress test. You note OMAR II, MD FEB 2014
that there are fetal heart rate decelerations that (TOP 2 - AUG
occur in the presence of contractions lasting longer 2013 MED
than 90 seconds. You interpret this as: BOARDS;
A. Negative CST TOPNOTCH MD -
B. Positive CST 200 QUESTIONS)
C. Equivocal-suspicious AND MARC
D. Equivocal-hyperstimulatory DENVER
E. Unsatisfactory TIONGSON, MD
(40 QUESTIONS)
547 Which of the following INCORRECTLY describes there is no posterior rectal and inferior labial ABDELSIMAR FINAL EXAM -
the perineum? arteries. Its supposed to be inferior rectal and OMAR II, MD FEB 2014
A. The pelvic diaphragm consists of the levator ani posterior labial (TOP 2 - AUG
muscles plus the coccygeus muscles posteriorly. 2013 MED
B. The levator ani muscles form a broad muscular BOARDS;
sling that originates from the posteriorsurface of TOPNOTCH MD -
the superior pubic rami, from the inner surface of 200 QUESTIONS)
the ischial spine, and between these two sites, from AND MARC
the obturator fascia. DENVER
C. The major blood supply to the perineum is via TIONGSON, MD
the internal pudendal artery and its branches. (40 QUESTIONS)
These include the posterior rectal artery and
inferior labial artery.
D. The innervation of the perineum is primarily via
the pudendal nerve and its branches. The pudendal
nerve originates from the S2, S3, and S4 level of the
spinal cord.
548 A 34 year old G2P1 (1001) patient on her 28th ABDELSIMAR FINAL EXAM -
week APG came in at the OBAS for consult. She was OMAR II, MD FEB 2014
referred from the OPD due to BP of 140/90. (TOP 2 - AUG
According to the patient, she has been diagnosed 2013 MED
with hypertension since she was 32 yo. Her stat BOARDS;
albumin is (-). the diagnosis is TOPNOTCH MD -
A. Gestational Hypertension 200 QUESTIONS)
B. Chronic Hypertension AND MARC
C. Preeclampsia, mild DENVER
D. Preeclampsia, severe TIONGSON, MD
E. Eclampsia (40 QUESTIONS)

549 L.Q. a 28 yo nulligravid, desirous of pregnancy, ABDELSIMAR FINAL EXAM -


presents with 7x4x3 cm myomatous growth by OMAR II, MD FEB 2014
UTZ. She is symptomatic with intermittent colicky (TOP 2 - AUG
hypograstric pain but with no menstrual 2013 MED
disturbance. What would be the best management BOARDS;
for her? TOPNOTCH MD -
A. hysterectomy 200 QUESTIONS)
B. Myomectomy AND MARC
C. Observation DENVER
D. Conceptual trial TIONGSON, MD
E. Pain relievers (40 QUESTIONS)
550 Aside from health benefits, the use of OCP has been ABDELSIMAR FINAL EXAM -
proven to reduce the incidence of: OMAR II, MD FEB 2014
A. thromboembolic events (TOP 2 - AUG
B. Colon CA 2013 MED
C. Benign Breast BOARDS;
D. Endometrial CA TOPNOTCH MD -
E. Cervical Ca 200 QUESTIONS)
AND MARC
DENVER
TIONGSON, MD
(40 QUESTIONS)
551 R.L. 36 year old G1P1 (1001) is 2 weeks post ABDELSIMAR FINAL EXAM -
partum. She consults her physician for tenderness OMAR II, MD FEB 2014
of the breast on the upper outer quadrant of the (TOP 2 - AUG
left breast. On Physical examination, it was noted 2013 MED
erythema and warmth on the area. Which of the BOARDS;
following conditions most likely explains her TOPNOTCH MD -
problem? 200 QUESTIONS)
A. breast abscess AND MARC
TOPNOTCH MEDICAL BOARD PREP OBSTETRICS-GYNECOLOGY SUPEREXAM Page 65 of 84
For inquiries visit www.topnotchboardprep.com.ph or email us at topnotchmedicalboardprep@gmail.com
TOPNOTCH MEDICAL BOARD PREP OBSTETRICS-GYNECOLOGY SUPEREXAM
For inquiries visit www.topnotchboardprep.com.ph or email us at topnotchmedicalboardprep@gmail.com
Ite QUESTION EXPLANATION AUTHOR TOPNOTCH
m # EXAM
B. nipple trauma DENVER
C. breast cyst TIONGSON, MD
D. acute mastitis (40 QUESTIONS)
E. inflammatory breast ca

552 A 32 yo primigravid on her 34th wk AOG came to ABDELSIMAR FINAL EXAM -


your clinic for lots of spider veins in the abdomen. OMAR II, MD FEB 2014
What would you advice her? (TOP 2 - AUG
A. she has to have liver function test done to rule 2013 MED
out liver disease BOARDS;
B. refer her to dermatology for further work-up TOPNOTCH MD -
and evealuation 200 QUESTIONS)
C. This is normal during pregnancy and resolves AND MARC
spontaneously after delivery DENVER
D. she needs abdominal support to relieve pressure TIONGSON, MD
E. Work up for possible liver disease (40 QUESTIONS)
553 Which of the following is NOT a documented ABDELSIMAR FINAL EXAM -
outcome of uncontrolled thyrotoxicosis in OMAR II, MD FEB 2014
pregnancy? (TOP 2 - AUG
A. Preeclampsia 2013 MED
B. Preterm delivery BOARDS;
C. Still Birth TOPNOTCH MD -
D. Neonatal Goiter 200 QUESTIONS)
AND MARC
DENVER
TIONGSON, MD
(40 QUESTIONS)
554 37 yo G3P3 consulted at the health center for ABDELSIMAR FINAL EXAM -
cervical CA screening. She tested (+) for aceto- OMAR II, MD FEB 2014
whitening. What should be done next? (TOP 2 - AUG
A. Colposcopy 2013 MED
B. Annual screening until 2 consecutive normal BOARDS;
tests than every 5 years thereafter TOPNOTCH MD -
C. pap smear 200 QUESTIONS)
D. do a punch biopsy AND MARC
E. Treat with antibiotics DENVER
TIONGSON, MD
(40 QUESTIONS)
555 A 34 yo G2P1 came to OBAS for labor pains. She ABDELSIMAR FINAL EXAM -
has no records of prenatal check-up and was noted OMAR II, MD FEB 2014
to have BP elevations during her 3rd preganancy. (TOP 2 - AUG
She gives you an AOG based on LNMP of 33 wks 2013 MED
and 6 days. On PE: BP 160/90 with grade I bipedal BOARDS;
edema. Urinalysis reveals albumin +1. On IE, she TOPNOTCH MD -
has a patulous cervix with hard ballotable mass. 200 QUESTIONS)
What is your admitting impression? AND MARC
A. PU 33 6/7 by LNMP, cephalic, NIL, preeclampsia, DENVER
mild TIONGSON, MD
B. PU 33 6/7 by LNMP, cephalic, NIL, chronic HPN (40 QUESTIONS)
w/ superimposed preeclampsia
C. PU 33 6/7 by LNMP, cephalic, NIL, gestational
HPN
D. PU 33 6/7 by LNMP, cephalic, NIL, gestational
HPN w/ superimposed preeclampsia
E. PU 33 6/7 by LNMP, cephalic, NIL, chronic
hypertension
556 A 29 year old female G2P1 (1001) experiences ABDELSIMAR FINAL EXAM -
biliary colic for the past 1 week. She is diagnosed to OMAR II, MD FEB 2014
have acute cholecystitis. How do you treat this (TOP 2 - AUG
case? 2013 MED
A. Treat as if the patient is not pregnant BOARDS;
B. Wait until the fetus is viable then perform TOPNOTCH MD -
surgery 200 QUESTIONS)
C. Medical management and perform intervention AND MARC
after delivery DENVER
D. Delay treatment and perform surgery at second TIONGSON, MD
trimester (40 QUESTIONS)
557 A 26 year old G1P0 PU 39 weeks AOG CIL, is noted ABDELSIMAR FINAL EXAM -
to have a BP of 160/100. You have managed the OMAR II, MD FEB 2014
hypertension by give hydralazine. The next step is (TOP 2 - AUG
toprevent eclampsia by giving magnesium sulfate. 2013 MED
How will you administer magnesium sulfate in this BOARDS;
patient? TOPNOTCH MD -
A. Loading dose of 4g IV and 5g on each buttock 200 QUESTIONS)
then 5g in alternating buttock every 4 hours to AND MARC
complete a cycle of 24 hours DENVER
B. Loading dose of 4g IV and 5g on each buttock TIONGSON, MD
then 5g in alternating buttock every 4 hours upto (40 QUESTIONS)
24 hours post delivery
C. Loading dose of 4g IV and 5g on each buttock
then 5g in alternating buttock every 4 hours until
delivery
D. Loading dose of 4g IV then 5g in alternating
buttock every 4 hours until delivery

TOPNOTCH MEDICAL BOARD PREP OBSTETRICS-GYNECOLOGY SUPEREXAM Page 66 of 84


For inquiries visit www.topnotchboardprep.com.ph or email us at topnotchmedicalboardprep@gmail.com
TOPNOTCH MEDICAL BOARD PREP OBSTETRICS-GYNECOLOGY SUPEREXAM
For inquiries visit www.topnotchboardprep.com.ph or email us at topnotchmedicalboardprep@gmail.com
Ite QUESTION EXPLANATION AUTHOR TOPNOTCH
m # EXAM
558 A 38y y.o. G7P5 came in a stretcher. She appears to ABDELSIMAR FINAL EXAM -
be pale, incoherent with cold, clammy extremities. OMAR II, MD FEB 2014
There is note of profuse vaginal bleeding. History (TOP 2 - AUG
from the husband: came from a lying-in clinic 2013 MED
where the midwife performed fundal pressure. BOARDS;
Rigid abdomen, (-) FHT. What could the patient TOPNOTCH MD -
have? 200 QUESTIONS)
A. Bleeding placenta previa AND MARC
B. Abruptio placenta with full blown DIC DENVER
C. Ruptured vasa previa TIONGSON, MD
D. Uterine rupture (40 QUESTIONS)
559 A 35 y.o. G1P0 37wks AOG with a history of ABDELSIMAR FINAL EXAM -
chronic hypertension presents with severe OMAR II, MD FEB 2014
abdominal pains for several hours. BP 140/90, PR (TOP 2 - AUG
110, RR 18, T 37C, tender abdomen, cervix 4cm 2013 MED
dilated, membranes bulging. CTG is non reactive BOARDS;
with hypertonic contraction. Diagnosis? TOPNOTCH MD -
A. Intrauterine infection 200 QUESTIONS)
B. Normal early labor AND MARC
C. Ruptured uterus DENVER
D. Abruptio placenta TIONGSON, MD
(40 QUESTIONS)
560 What is the complete obstretic score of the patient ABDELSIMAR FINAL EXAM -
given the following history? OMAR II, MD FEB 2014
1. 1970 FT via SVD male, alive (TOP 2 - AUG
2. 1971 Spontaneous abortion 15 weeks AOG 2013 MED
3. 1980 H. mole BOARDS;
4. 1985 Preterm twin gestation via SVD, both TOPNOTCH MD -
females, 1 died after 1 yr, the other, alive 200 QUESTIONS)
5. 1986 FT via SVD male, FDU AND MARC
6. 1987 Ectopic pregnancy DENVER
7. 1990 Present pregnancy TIONGSON, MD
(40 QUESTIONS)
A. G7P3 (1232)
B. G7P3 (2132)
C. G7P3 (1232)
D. G7P3 (2133)
561 What is the most common presenting feature of Most common feature of adult hypopituitarism BLAKE WARREN MIDTERM 2
adult hypopituitarism? even when other pituitary hormones are deficient ANG, MD (TOP 1 - EXAM - FEB
A. Hypogonadism Trophic hormone failure associated with pituitary AUG 2013 MED 2014
B. Hypothyroidism compression or destruction usually occurs BOARDS;
C. Hypoprolactinemia sequentially: GH>FSH>LH>TSH>ACTH TOPNOTCH MD)
D. Short stature Childhood: Growth retardation is often the
presenting feature
Adult: Hypogonadism is the earliest symptom
562 Most important step in the evaluation of male BLAKE WARREN MIDTERM 2
infertility ANG, MD (TOP 1 - EXAM - FEB
A. Serum testosterone AUG 2013 MED 2014
B. HCG stimulation test BOARDS;
C. Semen Analysis TOPNOTCH MD)
D. Testicular biopsy
563 Fetal lung maturity is expected to be completed at: BLAKE WARREN MIDTERM 2
A.) 28 wks AOG ANG, MD (TOP 1 - EXAM - FEB
B.) 30 wks AOG AUG 2013 MED 2014
C.) 34 wks AOG BOARDS;
D.) 32 wks AOG TOPNOTCH MD)
564 Around the middle of the menstrual cycle, the BLAKE WARREN MIDTERM 2
thickening of the cervical mucus is brought about ANG, MD (TOP 1 - EXAM - FEB
by this hormone: AUG 2013 MED 2014
A.) progesterone BOARDS;
B.) estrogen TOPNOTCH MD)
C.) oxytonin
D.) FSH
565 During hysterectomy, ligating this ligament would Cardinal ligament aka Mackenrodt’s ligament BLAKE WARREN MIDTERM 2
cause injury to the uterine artery, vein and ureter. ANG, MD (TOP 1 - EXAM - FEB
A.) cardinal ligament AUG 2013 MED 2014
B.) ligament of Treitz BOARDS;
C.) Mackenrodt's ligament TOPNOTCH MD)
D.) A & B
E.) A & C
566 In a fully mature fetal lung, the expected BLAKE WARREN MIDTERM 2
Lecithin/Sphingomyelin ratio should be at least: ANG, MD (TOP 1 - EXAM - FEB
A.) 1.75 AUG 2013 MED 2014
B.) 2.0 BOARDS;
C.) 1.0 TOPNOTCH MD)
D.) 1.5
567 Kassandra, a 38-year old G7P6 (6006) just gave BLAKE WARREN MIDTERM 2
birth to a healthy 2.7 kg baby boy via spontaneous ANG, MD (TOP 1 - EXAM - FEB
vaginal delivery. The Obstetrician on duty noticed AUG 2013 MED 2014
continued vaginal bleeding despite absence of any BOARDS;
vaginal or cervical lacerations. What is the most TOPNOTCH MD)
prudent thing to do to stop the bleeding?
A.) continue packing with OS
B.) nipple stimulation
C.) administer Oxytocin drip
D.) IV fluids and vasopressors
TOPNOTCH MEDICAL BOARD PREP OBSTETRICS-GYNECOLOGY SUPEREXAM Page 67 of 84
For inquiries visit www.topnotchboardprep.com.ph or email us at topnotchmedicalboardprep@gmail.com
TOPNOTCH MEDICAL BOARD PREP OBSTETRICS-GYNECOLOGY SUPEREXAM
For inquiries visit www.topnotchboardprep.com.ph or email us at topnotchmedicalboardprep@gmail.com
Ite QUESTION EXPLANATION AUTHOR TOPNOTCH
m # EXAM
568 On endometrial biopsy, you saw saw-toothed secretory phase of the menstrual cycle since the BLAKE WARREN MIDTERM 2
glands, edematous stroma, prominent arterioles presence of glycogen already marks the ovulatory ANG, MD (TOP 1 - EXAM - FEB
and glycogen vacuoles at the cell surfaces. What phase AUG 2013 MED 2014
phase of the menstrual cycle is this? BOARDS;
A.) Luteal TOPNOTCH MD)
B.) Proliferative
C.) Secretory
D.) Follicular
569 The most common cause of benign vaginal BLAKE WARREN MIDTERM 2
bleeding among post menopausal women is: ANG, MD (TOP 1 - EXAM - FEB
A.) leiomyosarcoma AUG 2013 MED 2014
B.) Atrophic vaginitis BOARDS;
C.) foreign body vaginitis TOPNOTCH MD)
D.) infectious vaginitis
570 Mary, a 32-year old non-diabetic and non- BLAKE WARREN MIDTERM 2
hypertensive, is currently pregnant with her ANG, MD (TOP 1 - EXAM - FEB
second child (at 34 wks AOG) suddenly AUG 2013 MED 2014
experienced non-painful vaginal bleeding while BOARDS;
asleep. Her husband Roel, woke her up in a pool of TOPNOTCH MD)
blood and accompanied her to the nearest private
hospital. Roel is anxious to know the cause of her
bleeding. As the physician on duty, your primary
diagnosis is:
A.) abruptio placenta
B.) placenta previa
C.) pre-eclampsia
D.) inevitable abortion
571 Which of the following is responsible for BLAKE WARREN MIDTERM 2
production of beta HCG? ANG, MD (TOP 1 - EXAM - FEB
A. Syncytiotrophoblast AUG 2013 MED 2014
B. cytotrophoblast BOARDS;
C. Decidua TOPNOTCH MD)
D. Endometrial glands
572 What is the histologic date of an endometrium BLAKE WARREN MIDTERM 2
showing subnuclear vacuoles and pseudostratified ANG, MD (TOP 1 - EXAM - FEB
nuclei? AUG 2013 MED 2014
A. Day 16 BOARDS;
B. day 18 TOPNOTCH MD)
C. Day 20
D. Day 22
573 Which of the following is an expected feature in the BLAKE WARREN MIDTERM 2
placenta of a mother with severe preeclampsia? ANG, MD (TOP 1 - EXAM - FEB
A. Neutrophilic infiltrates AUG 2013 MED 2014
B. granulation BOARDS;
C. Infarcts TOPNOTCH MD)
D. Chronic inflammatory pattern
574 The most common predisposing factor to tubal BLAKE WARREN MIDTERM 2
pregnancy? ANG, MD (TOP 1 - EXAM - FEB
A.Chronic salphingitis AUG 2013 MED 2014
B. Congenital tubal defects BOARDS;
C. Functional tubal disturbances TOPNOTCH MD)
D. Salphingitis isthmica nodosa
575 As the villi of placenta mature, they begin to form Answer: 6 weeks BLAKE WARREN MIDTERM 2
vessels. At how many weeks do the villous vessels ANG, MD (TOP 1 - EXAM - FEB
become apparent? AUG 2013 MED 2014
A. 4 BOARDS;
B. 5 TOPNOTCH MD)
C. 6
D. 7
576 How many arteries do the umbilical cord posses? 2 arteries and 1 vein BLAKE WARREN MIDTERM 2
A. O ANG, MD (TOP 1 - EXAM - FEB
B. 1 AUG 2013 MED 2014
C. 2 BOARDS;
D. 3 TOPNOTCH MD)
577 Your patient comes to you 28 weeks pregnant. She BLAKE WARREN MIDTERM 2
asked if she can go to Tacloban by plane to visit her ANG, MD (TOP 1 - EXAM - FEB
relatives who have been victims of the typhoon AUG 2013 MED 2014
Yolanda. Till how many weeks is she considered BOARDS;
safe to travel by air? TOPNOTCH MD)
A. 30
B. 32
C. 34
D. 36
578 Strawberry cervix/colpitis macularis. What is the This is a case of trichomoniasis treated with BLAKE WARREN MIDTERM 2
treatment? Metronidazole ANG, MD (TOP 1 - EXAM - FEB
A. Metronidazole AUG 2013 MED 2014
B. clotrimazole BOARDS;
C. Ciprofloxacin TOPNOTCH MD)
D. Fluconazole
579 Which of the following vaccines has established BLAKE WARREN MIDTERM 2
evidence of causing harm to the fetus? ANG, MD (TOP 1 - EXAM - FEB
A. Rubella AUG 2013 MED 2014
B. varicella BOARDS;
C. Measles TOPNOTCH MD)
D. Smallpox

TOPNOTCH MEDICAL BOARD PREP OBSTETRICS-GYNECOLOGY SUPEREXAM Page 68 of 84


For inquiries visit www.topnotchboardprep.com.ph or email us at topnotchmedicalboardprep@gmail.com
TOPNOTCH MEDICAL BOARD PREP OBSTETRICS-GYNECOLOGY SUPEREXAM
For inquiries visit www.topnotchboardprep.com.ph or email us at topnotchmedicalboardprep@gmail.com
Ite QUESTION EXPLANATION AUTHOR TOPNOTCH
m # EXAM
580 Ovarian artery/vein is found coursing in the They are all referring to infundibulopelvic ligament BLAKE WARREN MIDTERM 2
ligament of: containing the ovarian vessels ANG, MD (TOP 1 - EXAM - FEB
A. Infundibulopelvic ligament AUG 2013 MED 2014
B. Suspensory ligament BOARDS;
C. Ovarian ligament TOPNOTCH MD)
D. All of the above
581 This ligament of the uterus comes from the The uterosacrals are the landmarks used by the TIMOTHY TANG MIDTERM 1
posterolateral wall going to the supravaginal gynecologist to locate the cervix. LEE SAY, MD EXAM - FEB
portion of the cervix? (TOP 4 - AUG 2014
A. Broad ligament 2013 MED
B. Cardinal ligament BOARDS;
C. Uterosacral ligament TOPNOTCH MD)
D. Round ligament
E. Transverse cervical ligament
582 In the menstraul cycle, what is the earliest sign that Presence of basal vacuolization in the endometrium TIMOTHY TANG MIDTERM 1
the endometrium is in its secretory phase? is the earliest sign of progesterone action in the LEE SAY, MD EXAM - FEB
A. Formation and coiling of the spiral arteries endometrium. Luteinization of the corpus and (TOP 4 - AUG 2014
B. Decidualization of the endometrium ovum reaching the tubes comes before the 2013 MED
C. Luteinization of the corpus and secretion of vacoulization but are not signs that the BOARDS;
progesterone endometrium has reached the secretory phase. TOPNOTCH MD)
D. Ovum has reached the fallopian tube They are signs that the ovaries has reached the
E. Presence of subnuclear glycogen vacuoles luteal phase.
583 Which of the following is a presumptive symptom Anatomic breast changes and thermal signs are TIMOTHY TANG MIDTERM 1
of pregnancy? presumptive signs, while enlargement of the LEE SAY, MD EXAM - FEB
A. Anatomic breast changes abdomen and Braxton Hick's contraction are (TOP 4 - AUG 2014
B. Perception of fetal movement painless and irregular. 2013 MED
C. Increased in basal body temperature BOARDS;
D. Englargement of the abdomen TOPNOTCH MD)
E. Braxton-Hick's contractions
584 In assessing results of fetal heart monitoring, what The presence of moderate variability (beat to beat TIMOTHY TANG MIDTERM 1
is the single best indicator that the fetus is well? variation) will tell us that the fetus is not LEE SAY, MD EXAM - FEB
A. Reactive Non-stress test compromised more than a reactive NST or negative (TOP 4 - AUG 2014
B. Negative Contraction stress test CST. 2013 MED
C. Presence of accelaration BOARDS;
D. Absence of deceleration TOPNOTCH MD)
E. Moderate variability
585 When the fetal head is engaged, this tells us that The shortest and narrowest dimensions that the TIMOTHY TANG MIDTERM 1
the fetal head has pass through which narrowest fetal head must pass through for engagement to LEE SAY, MD EXAM - FEB
diameter? occur is the obstetric conjugate. The ischial spine is (TOP 4 - AUG 2014
A. True/Anatomic conjugate at the level of the presenting part but the fetal head 2013 MED
B. Obstetric conjugate has not pass through it yet. BOARDS;
C. Diagonal conjugate TOPNOTCH MD)
D. Interspinous diameter
E. All of the above
586 A 32 year old primigravid went to your clinic Deliver immediately since we have already TIMOTHY TANG MIDTERM 1
complaining of loss of perceived fetal movements. documented lung maturity (L/S ratio of 2) and with LEE SAY, MD EXAM - FEB
The fetus is 33 3/7 weeks AOG by early ultrasound. a BPP of 6, the fetus may be chronically (TOP 4 - AUG 2014
You did a biophysical profile (BPP) with a score of asphyxiated. 2013 MED
6 with normal AFV. Lung maturity testing revealed BOARDS;
L/S ratio of 2. What is your next step in TOPNOTCH MD)
management?
A. Observe and reassure the patient
B. Admit the patient and repeat BPP after 24
hours. If on repeat shows improvement, do
expectant management and repeat per protocol
C. Admit the patient, give Dexamethasone and
repeat BPP daily. If not improving, deliver.
D. Deliver immediately
E. Refer to a perinatologist
587 Which of the following scenarios will it be LESS The longer is the duration of labor, the more prone TIMOTHY TANG MIDTERM 1
likely to form the pathologic retraction ring of is the uterus to form the pathologic band, LEE SAY, MD EXAM - FEB
Bandl? precipituous labor would have lesser chance of (TOP 4 - AUG 2014
A. Precipituous labor having this complication. 2013 MED
B. Cephalopelvic disproportion BOARDS;
C. Prolonged 2nd stage of labor TOPNOTCH MD)
D. Labor augmentation
E. Nulliparous women
588 The average duration of the first stage of labor in Average duration of the first stage of labor is 8 TIMOTHY TANG MIDTERM 1
nulliparous women is? hours and 5 hours for nulliparous and multiparous LEE SAY, MD EXAM - FEB
A. 5 hours women respectively. (TOP 4 - AUG 2014
B. 8 hours 2013 MED
C. 10 hours BOARDS;
D. 14 hours TOPNOTCH MD)
E. 20 hours
589 Which of the following characteristics would make HCG should be below <40,000 TIMOTHY TANG MIDTERM 1
a patient NOT amenable to single agent LEE SAY, MD EXAM - FEB
chemotherpay for gestational trophoblastic (TOP 4 - AUG 2014
disease? 2013 MED
A. 2 months since pregnancy BOARDS;
B. Serum hCG 50, 000 mIU/mL TOPNOTCH MD)
C. Metastasis to the lung
D. No prior chemotherapy
E. None of the above

TOPNOTCH MEDICAL BOARD PREP OBSTETRICS-GYNECOLOGY SUPEREXAM Page 69 of 84


For inquiries visit www.topnotchboardprep.com.ph or email us at topnotchmedicalboardprep@gmail.com
TOPNOTCH MEDICAL BOARD PREP OBSTETRICS-GYNECOLOGY SUPEREXAM
For inquiries visit www.topnotchboardprep.com.ph or email us at topnotchmedicalboardprep@gmail.com
Ite QUESTION EXPLANATION AUTHOR TOPNOTCH
m # EXAM
590 22 year old female complains of sudden onset right Laparoscopy is the gold standard to diagnose TIMOTHY TANG MIDTERM 1
lower quadrant pain accompanied by vaginal ectopic pregnancy LEE SAY, MD EXAM - FEB
spotting and missed menses. Patient has pallor (TOP 4 - AUG 2014
and tachycardia. Pregnancy test was positive. 2013 MED
Pelvic exam is (+) for wiggling tenderness. What is BOARDS;
the gold-standard in diagnosing this disorder? TOPNOTCH MD)
A. Transvaginal ultrasound
B. Culdocentesis
C. Laparoscopy
D. Quantitative hCG determintation
E. Laparotomy
591 What forceps is used for deep transverse arrest of Piper - aftercoming head in breeach TIMOTHY TANG MIDTERM 1
the head? Simpson - molded head LEE SAY, MD EXAM - FEB
A. Piper Tucker McLane - round head (mulitparas) (TOP 4 - AUG 2014
B. Simpson 2013 MED
C. Tucker MacLane BOARDS;
D. Kielland TOPNOTCH MD)
E. Prague
592 Which of the following statements regarding Fraternal twins have genetic basis, identical twins TIMOTHY TANG MIDTERM 1
multifetal gestation is TRUE? is an abnormal event. Dichorionic twins have the LEE SAY, MD EXAM - FEB
A. Fertilization of two different ova forms least complications. Thoracophagus is the most (TOP 4 - AUG 2014
fraternal twins. common form of conjoined twins. Mothers have a 2013 MED
B. Identical twinning has been known to have a higher rate of preeclampsia, hypertension or BOARDS;
hereditary basis. premature birth than other pregnant population TOPNOTCH MD)
C. Dichorionic diamniotic twins have higher groups.
morbidity than those enclosed in one chorion.
D. Ischiophagus is the most common type of
conjoined twinning.
E. Pregnant mothers of twins have a higher
incidence of GDM than other pregnant women.
593 The following statements regarding pre-eclampsia Being nulliparous is a risk factor for pre-eclampsia. TIMOTHY TANG MIDTERM 1
is true EXCEPT? LEE SAY, MD EXAM - FEB
A. The visual disturbances of pre-eclampsia such (TOP 4 - AUG 2014
as serous retinal detachment improves several 2013 MED
weeks after the onset of symptoms. BOARDS;
B. The most consistent anatomic finding in pre- TOPNOTCH MD)
eclampsia is glomerular capillary endotheliosis in
the kidneys.
C. The main pathogenesesis of pre-eclampsia is
vasospasm and endothelial cell activation leading
to increase sensitivity to vasopressors.
D. Pathognomonic lesion in eclampsia is the
presence of periportal hemorrhagic necrosis in the
liver
E. Pre-eclampsia tends to be more severe as the
parity of women increases.
594 A mother brought her 15 year old daughter Reasure mother, primary amenorrhea is defined as TIMOTHY TANG MIDTERM 1
because of absence of menarche. Physical no menses by age 16 with the presence of sexual LEE SAY, MD EXAM - FEB
examination of the child is normal. Sexual maturity characteristics. The patient exhibited secondary (TOP 4 - AUG 2014
index is 3. What is your management? sexual characteristic and it will just be a matter of 2013 MED
A. Reaasure mother and observe time till menarche sets in. BOARDS;
B. Measure FSH and LH TOPNOTCH MD)
C. Request for a transrectal ultrasound
D. Karyotyping
E. Cranial MRI
595 A couple came in your office who wants to consult In any work-up of infertility, one must do a semen TIMOTHY TANG MIDTERM 1
regarding fertility issues. They have been trying to analysis first no matter what history the husband or LEE SAY, MD EXAM - FEB
conceive for over a year now, however they were wife give. Rationale: Easier to rule out a male factor (TOP 4 - AUG 2014
unsuccessful. The wife narrated that she was the than a female factor in the extensive work-up of 2013 MED
eldest of 3 siblings and her mother had the same infertility. BOARDS;
problems and was able to conceive only after 3 TOPNOTCH MD)
years of marriage with consult to a reproductive
endocrinologist. Physical examination of the
couple is unremarkable. What is your initial
procedure in this couple's work-up for infertility?
A. Do Semen Analysis
B. Request for a Hysterosalpingogram
C. Request for a Transvaginal ultrasound
D. Do Clomiphene citrate challenge test
E. Perform Laparoscopy
596 A 25 year old model with multiple sexual partners Treat with both, you assume there is concomittant TIMOTHY TANG MIDTERM 1
came to your office complaining of vaginal pruritus chlamydial infection with gonorrheal PID. LEE SAY, MD EXAM - FEB
and discharge accompanied by intermittent pelvic (TOP 4 - AUG 2014
pain. TVS showed normal anatomy of the uterus, 2013 MED
oviducts and ovaries. Speculum exam was done BOARDS;
which showed mucopurulent discharge with TOPNOTCH MD)
cervical tenderness. Gram stain revealed Gram(-)
intracellular diplococci. What is your next step?
A. Treat with 250 mg IM Ceftriaxone
B. Treat with 1g Azithromycin
C. Do Laparoscopy
D. Use both ceftriazone and azithromycin
E. Do surgical debridement

TOPNOTCH MEDICAL BOARD PREP OBSTETRICS-GYNECOLOGY SUPEREXAM Page 70 of 84


For inquiries visit www.topnotchboardprep.com.ph or email us at topnotchmedicalboardprep@gmail.com
TOPNOTCH MEDICAL BOARD PREP OBSTETRICS-GYNECOLOGY SUPEREXAM
For inquiries visit www.topnotchboardprep.com.ph or email us at topnotchmedicalboardprep@gmail.com
Ite QUESTION EXPLANATION AUTHOR TOPNOTCH
m # EXAM
597 Which will confer the highest risk factor for the It is known that HPV infection confers the highest TIMOTHY TANG MIDTERM 1
development of cervical cancer? risk but it should be the oncogenic serotypes and LEE SAY, MD EXAM - FEB
A. Multiple sexual partners the presence of a mass in the cervix will confer the (TOP 4 - AUG 2014
B. HPV 6 infection highest risk for malignancy 2013 MED
C. Intercourse at an early age BOARDS;
D. Genetic predisposition TOPNOTCH MD)
E. Exophytic mass on ectocervix
598 Which ovarian tumor has a higher chance of Granulosa theca cell tumor are funcional and TIMOTHY TANG MIDTERM 1
concommitant endometrial hyperplasia? secrete estrogen, so there is a risk for endometrial LEE SAY, MD EXAM - FEB
A. Serous epithelial tumor hyperplasia or carcinoma. (TOP 4 - AUG 2014
B. Endometrioid epithelial tumor 2013 MED
C. Granulosa-theca cell tumor BOARDS;
D. Dysgerminoma TOPNOTCH MD)
E. Meigs syndrome
599 Which parameter is not included in the Sassone Size of mass is not one of the 4 criteria of Sassone TIMOTHY TANG MIDTERM 1
criteria in diagnosis of ovarian malignancy by scoring. LEE SAY, MD EXAM - FEB
ultrasound? (TOP 4 - AUG 2014
A. Inner wall structure 2013 MED
B. Wall thickness BOARDS;
C. Size of mass TOPNOTCH MD)
D. Septa
E. Echogenicity
600 Which of the following structures in the female Ovaries, oviducts and uterus came from the TIMOTHY TANG MIDTERM 1
genital tract comes from the endoderm? mesoderm, the vagina, urethra comes from both the LEE SAY, MD EXAM - FEB
A. Ovaries endoderm and mesoderm. Endoderm from the (TOP 4 - AUG 2014
B. Oviducts cloaca which are part of the primitive hindgut. The 2013 MED
C. Uterus vulva is form from ectoderm. BOARDS;
D. Vagina TOPNOTCH MD)
E. Vulva
601 The female equivalent of the gubernaculum testis Round ligament is the female equivalent of RACHELLE FINAL EXAM -
is: gubernaculum testis. This ligament courses from MENDOZA, MD FEB 2013
A. Round ligament lateral uterus inserting to the labia majora. (TOP 9 - AUG
B. Ovarian epithelium 2012 MED
C. Suspensory ligament of the ovary BOARDS;
D. Epoophoron TOPNOTCH MD)
E. None of the above
602 Which of the following statements is/are true The average interval between cycles is 28 days. A RACHELLE FINAL EXAM -
regarding the normal menstrual cycle : normal menstrual cycle lasts from 21 to 35 days MENDOZA, MD FEB 2013
A. It has an average blood loss of 100 ml (average of 28 days, +/- 7 days) with 2 to 6 days of (TOP 9 - AUG
B. The average interval between cycles is 28 days flow (average of 4 days) and with a blood loss of 30 2012 MED
C. The average duration of flow is 7 days - 80 ml (average of 50 ml). Studies of large numbers BOARDS;
D. Luteal phase has a variable duration, while of normally cycling women have shown that only TOPNOTCH MD)
secretory phase is constant approximately 2/3 of adult women have cycles
E. All of the above lasting 21 - 35 days. Duration of luteal phase is
always constant (14 days), while that of the
secretory phase is variable.
603 A 34-year old G4P3 (3003) was sent to the OR for Theca-lutein cyst is common in pregnancy. It is a RACHELLE FINAL EXAM -
an emergency CS. Aftter repair of the uterus, the benign cyst that usually presents with honeycomb MENDOZA, MD FEB 2013
obstetrician noted that the left ovary has a cyst appearance on gross inspection. This cyst (TOP 9 - AUG
with a honeycomb appearance. The physician spontaneously regresses after completion of 2012 MED
should: pregnancy and menstruation resumes. BOARDS;
A. Do unilateral oophorectomy TOPNOTCH MD)
B. Obtain a biopsy specimen of the cyst
C. Refer intra-op to general surgery for possible
presence of gastric CA
D. Leave the cyst and do follow-up ultrasound
E. Proceed with TAHBSO
604 A 26-year old G1P1 (1001) seeks consult for Method effectiveness refers to the pregnancy rate RACHELLE FINAL EXAM -
contraception. She has no co-morbidity and had no of 100 women using a particular contraceptive MENDOZA, MD FEB 2013
complication during her previous pregnancy. method correctly for 1 year (100 women-years of (TOP 9 - AUG
Which of the following will be most effective for use). The effectiveness reflects failures due to 2012 MED
this patient? patient misuse of the contraceptive method and is BOARDS;
A. Intra-uterine device (IUD) less than method effectiveness. Oral contraceptive TOPNOTCH MD)
B. Spermicidal agent is proven by studies to be most effective among
C. Condom female patients when properly used. Coitus
D. Oral contraceptives interruptus is an unreliable contraceptive method
E. Coitus interruptus in which the penis is withdrawn from the vagina
prior to ejaculation.
605 Inlet contraction can be diagnosed with which of The obstetric conjugate is the most important RACHELLE FINAL EXAM -
the following clinical finding: anteroposterior diameter of the inlet which is the MENDOZA, MD FEB 2013
A. Obstetric conjugate of less than 11.5 cm shortest distance between the sacral promontory (TOP 9 - AUG
B. Diagonal conjugate of less than 11.5 cm and the mid posterior aspect of the symphysis 2012 MED
C. Interspinous diameter of less than 10.5 cm pubis through which the fetal head must pass. It BOARDS;
D. Sidewalls that are convergent measures 10 cm or more but may be considerably TOPNOTCH MD)
E. All of the above shortened in abnormal pelvis. The obstetric
conjugate cannot be measured directly with the
examining finger. Measurement is indirectly done
by getting the diagonal conjugate which is the
distance between the sacral promontory and
inferior margin of the symphysis pubis and subtract
1.5 to 2 cm. The diagonal conjugate length may vary
depending on the height and inclination of the
symphysis pubis. Therefore inlet contraction is
usually defined as a diagonal conjugate of less than
11.5 cm. Obstetric conjugate in inlet contraction
should be less than 10cm.
TOPNOTCH MEDICAL BOARD PREP OBSTETRICS-GYNECOLOGY SUPEREXAM Page 71 of 84
For inquiries visit www.topnotchboardprep.com.ph or email us at topnotchmedicalboardprep@gmail.com
TOPNOTCH MEDICAL BOARD PREP OBSTETRICS-GYNECOLOGY SUPEREXAM
For inquiries visit www.topnotchboardprep.com.ph or email us at topnotchmedicalboardprep@gmail.com
Ite QUESTION EXPLANATION AUTHOR TOPNOTCH
m # EXAM
606 Cytomegalovirus is the most common cause CMV is the most common cause of perinatal RACHELLE FINAL EXAM -
perinatal infection. This may be transmitted by infection. This is transmitted horizontally by MENDOZA, MD FEB 2013
which of the following routes : droplet infection and contact with saliva and urine, (TOP 9 - AUG
A. droplet infections vertically from mother to fetus, infant and as a 2012 MED
B. coitus sexually transmitted disease. Williams, 21th ed, p BOARDS;
C. uteroplacental 1468. TOPNOTCH MD)
D. all of the above
E. A and C only
607 A 33-year old G0 came to your clinic complaining Pen G is the recommended treatment for adult with RACHELLE FINAL EXAM -
of genital lesions. Suspecting syphilis, the most primary, secondary or early latent syphilis. Patients MENDOZA, MD FEB 2013
appropriate drug therapy is: with latent (patients seroactive without evidence of (TOP 9 - AUG
A. Ceftriaxone 1gm IM single dose disease) syphilis should be given 3 doses of 2.4 2012 MED
B. Erythromycin 500 mg orally 4 x 1 day for 7 days million. Benzathine Pen G IM, at 1-week intervals. BOARDS;
C. Benzathine Penicillin G, 2.4 million units IM TOPNOTCH MD)
single dose
D. Cefoxitin 2 gms IM single dose
608 A 19 year old G1P0 PU 40 weeks, not in labor, was Reactive non-stress test biophysical scoring (8-10) RACHELLE FINAL EXAM -
seen at the OPD for decreased fetal movement. She shows normal findings and indicates favorable fetal MENDOZA, MD FEB 2013
was hooked to an electronic fetal monitor and survival. A non-reactive non-stress test (< 6) refers (TOP 9 - AUG
tracing showed: Baseline FHT- 140’s, good to poor fetal condition and may be an indication for 2012 MED
variability, with more than 2 accelerations of 20 an emergent delivery. BOARDS;
bpm lasting for 20 secs. The tracing is interpreted TOPNOTCH MD)
as:
A. reactive
B. non-reactive
C. positive
D. negative
E. inconclusive
609 An obstetrician who attempts to do forceps In trial forceps, the OPB is fully aware that vaginal RACHELLE FINAL EXAM -
extraction with the knowledge that there is certain delivery may not be successful. In elective forceps, MENDOZA, MD FEB 2013
degree of disproportion at the level of the vaginal delivery is possible even without operative (TOP 9 - AUG
midpelvis is doing a: delivery. 2012 MED
A. Failed forceps BOARDS;
B. Prophylactic forceps TOPNOTCH MD)
C. Elective forceps
D. Trial forceps
E. Structural forceps
610 A patient underwent THBSO omentectomy and Stage I Growth limited to the ovaries. RACHELLE FINAL EXAM -
lymph node dissection for an ovarian cancer. IA Growth limited to one ovary; no ascites MENDOZA, MD FEB 2013
Histopathological report showed the tumor in the present containing malignant cells. No tumor on the (TOP 9 - AUG
right ovary measured 10x8x8 cm ruptured with external surface; capsule intact. 2012 MED
extension to the fallopian tube and omentum. One IB Growth limited to both ovaries; no ascites BOARDS;
para aortic node showed malignant cells. Using the present containing malignant cells. No tumor on the TOPNOTCH MD)
FIGO staging, this patient can be categorized external surfaces; capsules intact.
under: IC Tumor either stage IA or IB but with tumor on
A. Stage II-C surface of one or both ovaries; or with capsule
B. Stage III-A ruptured; or with ascites present containing
C. Stage III-B malignant cells; or with positive peritoneal
D. Stage III-C washings.
II Growth involving one or both ovaries with pelvic
extension.
IIA Extension and/or metastases to the uterus
and/or tubes.
IIB Extension to other pelvic tissues.
IIC Tumor either stage IIA or IIB, but with tumor
on surface of one or both ovaries; or with
capsule(s) ruptured; or with ascites present
containing malignant cells; or with positive
peritoneal washings.
III Tumor involving one or both ovaries with
peritoneal implants outside the pelvis and/or
positive retroperitoneal or inguinal nodes.
Superficial liver metastasis equals stage III. Tumor
is limited to the true pelvis but with histologically
proven malignant extension to small bowel or
omentum.
IIIA Tumor grossly limited to the true pelvis with
negative nodes but with histologically confirmed
microscopic seeding of abdominal peritoneal
surfaces.
IIIB Tumor of one or both ovaries with
histologically confirmed implants of abdominal
peritoneal surfaces, none exceeding 2 cm in
diameter. Nodes are negative.
IIIC Abdominal implants greater than 2 cm in
diameter and/or positive retroperitoneal or
inguinal nodes.
IV Growth involving one or both ovaries with
distant metastases. If pleural effusion is present,
there must be positive cytology to allot a case to
stage IV.
IVA Parenchymal liver metastasis equals stage IV
611 In the above case, what is the chance that the Stage RACHELLE FINAL EXAM -
patient may still live for the next 5 years? IA 86.9%, IB 71.3%, IC 79.2%, IIA 66.6%, IIB 55.1, MENDOZA, MD FEB 2013
A. 11.1% IIC 57.0, IIIA 41.1, IIIB 24.9, IIIC 23.4, IV 11.1 (TOP 9 - AUG
B. 23.4% (Comprehensive Gynecology, 4th ed) 2012 MED

TOPNOTCH MEDICAL BOARD PREP OBSTETRICS-GYNECOLOGY SUPEREXAM Page 72 of 84


For inquiries visit www.topnotchboardprep.com.ph or email us at topnotchmedicalboardprep@gmail.com
TOPNOTCH MEDICAL BOARD PREP OBSTETRICS-GYNECOLOGY SUPEREXAM
For inquiries visit www.topnotchboardprep.com.ph or email us at topnotchmedicalboardprep@gmail.com
Ite QUESTION EXPLANATION AUTHOR TOPNOTCH
m # EXAM
C. 36.7% BOARDS;
D. 41.1% TOPNOTCH MD)
E. 55.1%
612 The following statement is TRUE about Polycystic Polycystic Ovarian Syndrome (PCOS) is an RACHELLE FINAL EXAM -
Ovarian Syndrome, EXCEPT: endocrinologic disorder characterized by excessive MENDOZA, MD FEB 2013
A. This disorder begins perimenarcheally ovarian androgen production, abnormal (TOP 9 - AUG
B. Clinical manifestations include hirsutism, gonadotrophin secretion, and chronic anovulation 2012 MED
menstrual irregularity and obesity with morphologic changes in the ovary consisting BOARDS;
C. Ovaries contain multiple (>10) small (2- to 10- of multiple small subcapsular follicles, increased TOPNOTCH MD)
mm diameter) follicles in the periphery amounts of stromal tissue, and ovarian
D. Sonographic findings also include echogenicity enlargement. This disorder begins
of the stroma and ovarian enlargement perimenarcheally, and its clinical manifestations
E. None of the above include hirsutism, menstrual irregularity
(oligomenorrhea or amenorrhea), and obesity.
Findings upon sonographic visualization of the
ovaries include multiple (>10) small (2- to 10-mm
diameter) follicles in the periphery, increased
echogenicity of the stroma, and ovarian
enlargement. (Comprehensive Gynecology, 4th ed)
613 The major androgen produced by the ovaries is: The sources of androgen production in the human RACHELLE FINAL EXAM -
A. Testosterone female are the ovaries and the adrenal glands. The MENDOZA, MD FEB 2013
B. Dehydroepiandrosterone sulfate major androgen produced by the ovaries is (TOP 9 - AUG
C. Androstenedione testosterone and that of the adrenal glands is 2012 MED
D. Etiocholanolone dehydroepiandrosterone sulfate (DHEA-S). BOARDS;
E. The ovaries do not produce androgens (Comprehensive Gynecology, 4th ed) TOPNOTCH MD)
614 During active labor, the fetus was observed to have Early deceleration of the fetal heart rate consists of RACHELLE FINAL EXAM -
episodes of decreased fetal heart rate and return to a gradual decrease and return to baseline MENDOZA, MD FEB 2013
baseline in association with a contraction. The associated with a contraction. Freeman and co- (TOP 9 - AUG
degree of deceleration is generally proportional to authors (2003) defined early decelerations as those 2012 MED
the contraction strength and does not fall below generally seen in active labor between 4 and 7 cm BOARDS;
100 beats/min.Which of the following can be dilatation. In their definition, the degree of TOPNOTCH MD)
attributed to this phenomenon? deceleration is generally proportional to the
A. Fetal hypoxia contraction strength and rarely falls below 100 to
B. Utero-placental insufficiency 110 beats/min or 20 to 30 beats/min below
C. Umbilical cord occlusion baseline. Importantly, early decelerations are not
D. Head compression associated with fetal hypoxia, acidemia, or low
E. A and B Apgar scores. Ball and Parer (1992) concluded that
fetal head compression is a likely cause of early
deceleration. Head compression probably causes
vagal nerve activation as a result of dural
stimulation and that mediates the heart rate
deceleration (Paul and co-workers, 1964).
(William's obstetrics)
615 The following are associated with increased risk of According to William's obstetrics, placenta previa RACHELLE FINAL EXAM -
placenta previa, EXCEPT: may be assocaiated with 4 etiologies: advanced MENDOZA, MD FEB 2013
A. Advance maternal age maternal age, multiparity, prior CS and smoking. (TOP 9 - AUG
B. Previous caesarean section 2012 MED
C. Multiparity BOARDS;
D. Smoking TOPNOTCH MD)
E. Family history of placenta revia
616 A 26-year old G1P0 at 39 weeks AOG was noted to Types of amniotomy include: elective amniotomy, RACHELLE FINAL EXAM -
be 1 cm dilated but without regular uterine amniotomy induction and amniotomy MENDOZA, MD FEB 2013
contractions during her pre-natal check up. If her augmentation. Artificial rupture of the membranes (TOP 9 - AUG
obstetrician decides to admit her and plans to do can be used to induce labor, but it implies a 2012 MED
an early amniotomy, the type of amniotomy would commitment to delivery. The main disadvantage of BOARDS;
be a/an: amniotomy when used alone for labor induction is TOPNOTCH MD)
A. Elective amniotomy the unpredictable and occasionally long interval to
B. Amniotomy induction the onset of contractions. In a randomized trial,
C. Amniotomy augmentation Bakos and Backstrom (1987) found that
D. Therapeutic amniotomy amniotomy alone or combined with oxytocin was
E. Late amniotomy superior to oxytocin alone. Mercer and colleagues
(1995) randomized 209 women undergoing
oxytocin induction to amniotomy either at 1 to 2 cm
dilatation (early amniotomy) or at 5 cm (late
amniotomy). Early amniotomy was associated with
significantly shorter labor by approximately 4
hours. There was, however, an increased incidence
of chorioamnionitis and cord-compression patterns
with early amniotomy.
617 The most common malignancy gicing rise to a Malignant tumors rarely metastasize to the RACHELLE FINAL EXAM -
placnetal metastasis is: placenta. Of those that do, melanoma accounts for MENDOZA, MD FEB 2013
A. Gastric carcinoma nearly one third of reported cases, and leukemias (TOP 9 - AUG
B. Pilocytic astrocytoma and lymphomas comprise another third. 2012 MED
C. Rhabdomyosarcoma BOARDS;
D. Lung cancer TOPNOTCH MD)
E. Melanoma
618 A 3 year-old girl was brought to the ER because of Sarcoma Botryoides (Embryonal RACHELLE FINAL EXAM -
vaginal bleeding. On inspection of the external Rhabdomyosarcoma) is usually diagnosed in the MENDOZA, MD FEB 2013
genitalia, there was a mass protruding through the vagina of a young female. Rarely does it occur in a (TOP 9 - AUG
vaginal introitus. It measured 4x3 cm and young child over 8 years of age, although cases in 2012 MED
resembled a cluster of grapes forming multiple adolescents have been reported. The most common BOARDS;
polypoid masses. The most likely diagnosis is: symptom is abnormal vaginal bleeding, with an TOPNOTCH MD)
A. Sarcoma botyroides occasional mass at the introitus. The tumor grossly
B. Epidermoid carcinoma will resemble a cluster of grapes forming multiple
C. Vaginal adenosis polypoid masses.

TOPNOTCH MEDICAL BOARD PREP OBSTETRICS-GYNECOLOGY SUPEREXAM Page 73 of 84


For inquiries visit www.topnotchboardprep.com.ph or email us at topnotchmedicalboardprep@gmail.com
TOPNOTCH MEDICAL BOARD PREP OBSTETRICS-GYNECOLOGY SUPEREXAM
For inquiries visit www.topnotchboardprep.com.ph or email us at topnotchmedicalboardprep@gmail.com
Ite QUESTION EXPLANATION AUTHOR TOPNOTCH
m # EXAM
D. Condyloma acuminata
E. Congenital vulvar malformation

619 The first index/indices of declining ovarian Inhibin is a glycoprotein produced by the granulosa RACHELLE FINAL EXAM -
function in perimenopausal women is/are: cells of developing follicles during the follicular MENDOZA, MD FEB 2013
A. Decrease in FSH phase of the cycle. The gonadal production of (TOP 9 - AUG
B. Decrease in LH inhibin is stimulated by FSH, and inhibin 2012 MED
C. Decrease estradiol suppresses pituitary FSH secretion as part of a BOARDS;
D. Decrease inhibin closed-loop feedback system. MacNaughton et al. TOPNOTCH MD)
E. A and C have shown that circulating follicular phase inhibin
levels are significantly lower among women aged
45 to 49 than among women younger than age 45
(Table 42-1) . The fall in inhibin levels may be due
to the decreased number of ovarian follicles or to
altered granulosa cell function that accompanies
increasing age. Because estradiol levels do not
undergo a similar significant decrease between
ages 45 and 49, it is possible that synthesis of these
two hormones are a result of separate functions of
the granulosa cells. As inhibin levels fall there is a
concomitant rise in FSH, which initially results in
greater secretion of estradiol from the follicle.
620 A 33 year-old G3P2 (2002) went to OPD due to Waiting for spontaneous labor is the treatment of RACHELLE FINAL EXAM -
spotting. Her LMP was 28 weeks ago and she choice for intra-uterine fetal death (IUFD). MENDOZA, MD FEB 2013
claims that she has not felt any fetal movement for Induction of labor may be done 72 hours without (TOP 9 - AUG
the past month. On examination, fundal height is spontaneous labor. 2012 MED
18 cm, no fetal heart tone detected. Ultrasound BOARDS;
was done, revealing no cardiac acitivity. TOPNOTCH MD)
Management of this patient would be:
A. Wait for spontaneous labor
B. Induction of labor
C. Dilatation & Curettage
D. Hysterotomy
E. Prostaglandin
621 In the clinical pelvimetry of a pateint in the third Interspinous diameter and Diagonal conjugate can VON ANDRE DIAGNOSTIC
trimester of pregnancy, which of the following can be measured clinically. MEDINA, MD EXAM - AUG
be measured clinically? (TOP 4 - FEB 2012
A. Transverse diameter of the inlet 2012 MED
B. Obstetric conjugate BOARDS;
C. Interspinous diameter TOPNOTCH MD)
D. Posterior sagittal diameter of the outlet
E. None of the above
622 The iron deficiency anemia of pregnancy is due to: VON ANDRE DIAGNOSTIC
A. Expected decreased absorption of iron from MEDINA, MD EXAM - AUG
the GI tract during pregnancy (TOP 4 - FEB 2012
B. Nausea and vomiting that causes dehydration 2012 MED
and electrolyte imbalance BOARDS;
C. Expansion of plasma volume without normal TOPNOTCH MD)
expansion of maternal hemoglobin mass
D. Decreased expansion of RBC mass because of
decreased erythropoietin activity during
pregnancy
E. Any of the above choices
623 Which of the folloqwing is true regarding maternal Highest rate is seen in age group 40-44. VON ANDRE DIAGNOSTIC
mortality in the Philippines? MEDINA, MD EXAM - AUG
A. Fifty (50%) percent of the women who died (TOP 4 - FEB 2012
had prenatal care 2012 MED
B. Hypertension is the leading cause of mortality BOARDS;
C. Pulmonary disease is the leading medical TOPNOTCH MD)
cause
D. Highest rate is seen in the 40-44 years old age
group
E. Cervical cancer remain as the second leading
cause of maternal mortality
624 At what age of gestation is scanning for congenital congenital scanning is usually done at 16-18weeks VON ANDRE DIAGNOSTIC
abnormalities best done? MEDINA, MD EXAM - AUG
A. 5-6 weeks (TOP 4 - FEB 2012
B. 10-14 weeks 2012 MED
C. 16-18 weeks BOARDS;
D. 24-28 weeks TOPNOTCH MD)
E. 30-32 weeks
625 The following are components of quadruple Quadruple screen- AFP, BHCG, Estriol and Inhibin VON ANDRE DIAGNOSTIC
marker screen, EXCEPT? MEDINA, MD EXAM - AUG
A. AFP (TOP 4 - FEB 2012
B. B-HCG 2012 MED
C. Estriol BOARDS;
D. Estradiol TOPNOTCH MD)
E. Inhibin
626 Th e following structures originate from ureteric glomerulus originates from metanephric VON ANDRE DIAGNOSTIC
bud/ metanephric duct , EXCEPT? mesenchyme. MEDINA, MD EXAM - AUG
A. Ureter (TOP 4 - FEB 2012
B. Renal pelvis 2012 MED
C. Calyces BOARDS;
D. Collecting system TOPNOTCH MD)
E. Renal glomerulus
TOPNOTCH MEDICAL BOARD PREP OBSTETRICS-GYNECOLOGY SUPEREXAM Page 74 of 84
For inquiries visit www.topnotchboardprep.com.ph or email us at topnotchmedicalboardprep@gmail.com
TOPNOTCH MEDICAL BOARD PREP OBSTETRICS-GYNECOLOGY SUPEREXAM
For inquiries visit www.topnotchboardprep.com.ph or email us at topnotchmedicalboardprep@gmail.com
Ite QUESTION EXPLANATION AUTHOR TOPNOTCH
m # EXAM
627 In females, what is the remnant of mesonephric Gartners duct is a remnant of mesonephric duct VON ANDRE DIAGNOSTIC
duct/ wolffian duct ? MEDINA, MD EXAM - AUG
A. Gartner's Duct (TOP 4 - FEB 2012
B. Bartholin's Gland 2012 MED
C. Skene's gland BOARDS;
D. Uterus TOPNOTCH MD)
E. Nabothian cyst
628 The most sensitive test for the diagnosis of Laparoscopy is the gold standard in the diagnosis of VON ANDRE DIAGNOSTIC
abdominopelvic tuberculosis is? abdominopelvic tuberculosis MEDINA, MD EXAM - AUG
A. Tissue biopsy (TOP 4 - FEB 2012
B. Peritoneal fluid culture 2012 MED
C. ELISA BOARDS;
D. Laparoscopy TOPNOTCH MD)
E. Exploratory laparotomy
629 Forcep delivery is NOT recommended in which of VON ANDRE DIAGNOSTIC
the following conditions? MEDINA, MD EXAM - AUG
A. Class II-IV gravidocardiac (TOP 4 - FEB 2012
B. Vaginal delivery after Cesarian section 2012 MED
C. Presence of acute pulmonary edema BOARDS;
D. Presence of persistent late decelerations at TOPNOTCH MD)
station +4
E. None of the above
630 Of the following possible complications, which postpartum hemorrhage is a common complication VON ANDRE DIAGNOSTIC
should be anticipated in a multifetal pregnancy? following a multifetal pregnancy MEDINA, MD EXAM - AUG
A. Shoulder dystocia (TOP 4 - FEB 2012
B. Postmaturity 2012 MED
C. Multiple congenital abnormalities BOARDS;
D. Postpartum hemorrhage TOPNOTCH MD)
E. All of the above
631 Fetal flexion, rotation , extension and expulsion VON ANDRE DIAGNOSTIC
occur during which phase of labor? MEDINA, MD EXAM - AUG
A. Preparatory division (TOP 4 - FEB 2012
B. Acceleration phase 2012 MED
C. Phase of maximum slope BOARDS;
D. Deceleration phase TOPNOTCH MD)
E.Latent phase
632 A 25 year old at 32 weeks AOG is noted to have a CS is the only cure for eclampsia. VON ANDRE DIAGNOSTIC
BP of 160/100 mmHg with the presence of +2 MEDINA, MD EXAM - AUG
proteinuria. The platelet count and liver function (TOP 4 - FEB 2012
tests were abnormal. After few hours, patient went 2012 MED
into seizure. What is the best management plan in BOARDS;
this case? TOPNOTCH MD)
A. Induction of labor
B. Intelligent expectancy
C. Control the seizure then expectant
management
D. Cesarian section
E. Forcep delivery
633 A 26 year old came in to the emergency This is a case of ectopic pregnancy. PID is the most VON ANDRE DIAGNOSTIC
department due to vaginal bleeding and severe common predisposing factor. MEDINA, MD EXAM - AUG
abdominal pain. She was amenorrheic for about 2 (TOP 4 - FEB 2012
months. On physical examination, patient was pale 2012 MED
looking and hypotensive. Which of the following BOARDS;
events would be most likely predispose this patient TOPNOTCH MD)
to your diagnosis?
A. Pelvic inflammatory disease
B. Use of IUD
C. Previous tubal surgery
D. Exposure to diethylstilbestrol
E. None of the above
634 Amor is pregnant and visits your clinic for a G4P2 (1 1 1 3) VON ANDRE DIAGNOSTIC
regular pre-natal check up. She had one ectopic MEDINA, MD EXAM - AUG
pregnancy at 4 weeks. She has one baby born at 40 (TOP 4 - FEB 2012
weeks and one born at 32 weeks which is a set of 2012 MED
twins. What is her OB score? BOARDS;
A. G3P3 (1113) TOPNOTCH MD)
B. G4P2 (1113)
C. G3P3 (2113)
D. G4P2 (2113)
E. G4P2 (1213)
635 A primigravid consulted for amenorrhea of 12 This is a case of missed abortion VON ANDRE DIAGNOSTIC
weeks. Home pregnancy test was positive but you MEDINA, MD EXAM - AUG
were unable to appreciate fetal heart tones by (TOP 4 - FEB 2012
doppler. Transvaginal ultrasound showed a fetus 2012 MED
within an enlarged uterus without cardiac or BOARDS;
somatic activity. IE revealed a closed cervix. The TOPNOTCH MD)
most likely diagnosis is?
A. Blighted ovum
B. Missed abortion
C. Threatened abortion
D. Imminent abortion
E. Inevitable abortion
636 The structure that is cut during median episiotomy VON ANDRE DIAGNOSTIC
is the: MEDINA, MD EXAM - AUG
A. Ischiocavernosus (TOP 4 - FEB 2012
B. Bulbocavernosus 2012 MED
TOPNOTCH MEDICAL BOARD PREP OBSTETRICS-GYNECOLOGY SUPEREXAM Page 75 of 84
For inquiries visit www.topnotchboardprep.com.ph or email us at topnotchmedicalboardprep@gmail.com
TOPNOTCH MEDICAL BOARD PREP OBSTETRICS-GYNECOLOGY SUPEREXAM
For inquiries visit www.topnotchboardprep.com.ph or email us at topnotchmedicalboardprep@gmail.com
Ite QUESTION EXPLANATION AUTHOR TOPNOTCH
m # EXAM
C. Internal anal sphincter BOARDS;
D. Rectal mucosa TOPNOTCH MD)
E. None of the above
637 When the fetus is in "military attitude", the VON ANDRE DIAGNOSTIC
presenting diameter is? MEDINA, MD EXAM - AUG
A. Suboccipitobregmatic (TOP 4 - FEB 2012
B. occipitomental 2012 MED
C. Occipitofrontal BOARDS;
D. Submentobregmatic TOPNOTCH MD)
E. None of the above
638 The most common clinical manifestation of molar VON ANDRE DIAGNOSTIC
pregnancy is: MEDINA, MD EXAM - AUG
A. Vagial bleeding (TOP 4 - FEB 2012
B. Uterine size smaller than the age of gestation 2012 MED
C. Uterine size larger than the expected AOG BOARDS;
D. Fetal heart tones absent at 20 weeks AOG TOPNOTCH MD)
E. None of the above
639 A 41 year old had a baby with Down syndrome 5 Chorionic villous sampling is usually done in the 1st VON ANDRE DIAGNOSTIC
years ago. She is now anxious to know the trimester. MEDINA, MD EXAM - AUG
chromosome status of her fetus in her current (TOP 4 - FEB 2012
pregnancy . The test that has the fastest lab 2012 MED
processing time for karyotyping is? BOARDS;
A. Amniocentesis TOPNOTCH MD)
B. Cordocentesis
C. Chorionic villous sampling
D. Doppler flow ultrasound
E. None of the above
640 Zavanelli, as a management of shoulder dystocia Zavanelli- cephalic placement into pelvis followed VON ANDRE DIAGNOSTIC
refers to: by CS MEDINA, MD EXAM - AUG
A. Cephalic placement into pelvis followed by (TOP 4 - FEB 2012
Cesarian section 2012 MED
B. Pressure is applied to the infants jaw and neck BOARDS;
in the direction of the mother's rectum with strong TOPNOTCH MD)
fundal pressure applied by the assistant as anterior
shoulder is freed
C. Cutting of the clavicle with scissors or other
sharp instruments
D. Surgical incision into the fibrocartilage of the
symphysis pubis
E. None of the above choices
641 Maja Rotzky, a 19 y/o primigravid consulted on at 12 weeks, FH is at the level of symphysis pubis. LITO JAY DIAGNOSTIC
your clinic for her first prenatal check-up. The At 16 weeks, FH is halfway between the symphysis MACARAIG, MD EXAM - AUG
fundic height was measured to be at the level of and umbilicus. At 20 weeks at the level of (TOP 8 - FEB 2013
umbilicus. Assuming that this is NOT a mutifetal umbilicus. At 32 weeks about the level of xiphoid 2013 MED
pregnancy and there is NO structural, or medical process. BOARDS;
abnormality, approximately how many weeks AOG TOPNOTCH MD)
is this pregnancy?
A. 12 weeks
B. 16 weeks
C. 18 weeks
D. 20 weeks
E. 24 weeks
642 Cathy Poe is a 31 year old female who is Endometriosis is the presence of endometrial tissue LITO JAY DIAGNOSTIC
complaining of cyclic pelvic pains since 8 months outside the uterus. Hence accumulation of blood to MACARAIG, MD EXAM - AUG
PTC. She thought this was just an ordinary case of solid organs during menstrual periods, causes (TOP 8 - FEB 2013
dysmenorrhea hence she self-medicated with cyclic pain. 2013 MED
NSAID's. However, 2 weeks PTC, patient BOARDS;
experienced defecating with blood. You requeste TOPNOTCH MD)
dultrasound which revealed hyperechoic sacs on
both ovaries which possibly blood-containing.
What is your diagnosis?
A. Teratoma
B. Theca Lutein Cysts
C. Endometriosis
D. Adenomyosis
E. Polycystic ovarian syndrome
643 Halley is a 23 year old promiscuous lady who came Fitz Hugh Curtis Syndrome is a rare complication of LITO JAY DIAGNOSTIC
in due to right upper abdominal pain. Further PID due to ascending infection from lower genital MACARAIG, MD EXAM - AUG
history revealed previous episodes of STD but did tract towrds uterus, oviduct and upto the hepatic (TOP 8 - FEB 2013
not comply with the complete treatment course. capsule. 2013 MED
Laparoscopy was done and showed periportal BOARDS;
hepatitis with characteristic violin string TOPNOTCH MD)
adhesions. What is your impression?
A. Fitz-Hugh Curtis Syndrome
B. Weil's Syndrome
C. Waterhouse Friderichsen syndrome
D. Sheehan's Syndrome
E. Asherman Syndrome
644 You are consulted by a 28 year multigravid who is Sheehan's syndrome is the postpartum pituitary LITO JAY DIAGNOSTIC
on her 2nd week post-partum because she was necrosis. During gestation, the pituitary gland MACARAIG, MD EXAM - AUG
unable to express milk for her child. Further increases it's size to almost 130%. If there is acute (TOP 8 - FEB 2013
history revealed profuse bleeding during her last blood loss, ischemic necrosis may occur hence 2013 MED
childbirth. With that kind of history, you are able to oxytocin is not produced, leading to lack of milk BOARDS;
diagnose the patient with? ejection, and amenorrhea. TOPNOTCH MD)
A. Fitz-Hugh Curtis Syndrome

TOPNOTCH MEDICAL BOARD PREP OBSTETRICS-GYNECOLOGY SUPEREXAM Page 76 of 84


For inquiries visit www.topnotchboardprep.com.ph or email us at topnotchmedicalboardprep@gmail.com
TOPNOTCH MEDICAL BOARD PREP OBSTETRICS-GYNECOLOGY SUPEREXAM
For inquiries visit www.topnotchboardprep.com.ph or email us at topnotchmedicalboardprep@gmail.com
Ite QUESTION EXPLANATION AUTHOR TOPNOTCH
m # EXAM
B. Weil's Syndrome
C. Waterhouse Friderichsen syndrome
D. Sheehan's Syndrome
E. Asherman Syndrome

645 A 24 year-old primigravid with 9 weeks AOG this is a case of Complete abortion, hence watchful LITO JAY DIAGNOSTIC
consulted due to passing of meaty material. History observation is warranted to prevent severe MACARAIG, MD EXAM - AUG
started a day PTC, patient had crampy abdominal bleeding, sepsis and alike. D and C is not indicated, (TOP 8 - FEB 2013
pain (7-8/10 in pain scale) asscociated with just like Oxytocin, Tranex and antibiotics (as long 2013 MED
spotting. Few hours PTC, patient noted passing-out that this is NOT an instrumentally-induced) BOARDS;
of meaty material per vagina. Speculum exam TOPNOTCH MD)
showed closed cervix. To be sure, you requested
ultrasound and revealed empty uterine cavity.
What is your next step?
A. Dilatation and curettage
B. Give Tranexamic acid 500mg, 2 capsules 3x a
day for 3 days
C. Administer 10 units of Oxytocin via 30 minute
drip
D. Administer 1.2 grams of Co-amoxiclav IV every
12 hours for 3 days
E. Do watchful observation
646 The most common type of breech presentation is Frank breech is the most common type of breech LITO JAY DIAGNOSTIC
characterized by? and is characterized by lower extremities flexed at MACARAIG, MD EXAM - AUG
A. Lower extremities extended at the hips and the hips and extended at the knees (TOP 8 - FEB 2013
extended at the knees 2013 MED
B. Lower extremities flexed at the hips and flexed BOARDS;
at the knees TOPNOTCH MD)
C. Lower extremities extended at the hips and
flexed at the knees
D. Lower extremities flexed at the hips and
extended at the knees
E. None of the above
647 The most common symptom found in complete the question is asking for the MOST COMMON LITO JAY DIAGNOSTIC
mole is? SYMPTOM hence vaginal bleeding is the only and MACARAIG, MD EXAM - AUG
A. Vaginal bleeding definite answer (TOP 8 - FEB 2013
B. Abdominal pain 2013 MED
C. Uterine enlargement BOARDS;
D. Nausea and vomiting TOPNOTCH MD)
E. A and D
648 A 25 year-old female, with several previous The one being asked is the discriminatory level of LITO JAY DIAGNOSTIC
episodes of PID, came in due to severe right lower beta-hCG. Meaning, if beta-hCG is >1,500 units, a MACARAIG, MD EXAM - AUG
quadrant pain. No associated fever, diarrhea or gestational sac should be seen via ultrasound (TOP 8 - FEB 2013
vomiting noted. Further history revealed missed 2013 MED
menses for 2 months. Pregnancy test revealed BOARDS;
positive and you are entertaining ectopic TOPNOTCH MD)
pregnancy. If you are to perform ultrasound, you
should know that a gestational sac should be seen
once the level of hCG is?
A. > 1,500 units
B. > 1.50 units
C. > 2, 500 units
D. > 2.5 units
E. > 3,500 units
649 Which among the following statements is/are tru LITO JAY DIAGNOSTIC
regarding medical treatment of Ectopic pregnancy? MACARAIG, MD EXAM - AUG
A. < 6 weeks AOG (TOP 8 - FEB 2013
B. < 3.5 cms in length 2013 MED
C. Beta hCG of < 15,000 miU/mL BOARDS;
D. A and C only TOPNOTCH MD)
E. All of the above
650 What is the most common cause of DIC in pregnant premature separation of placenta causes activation LITO JAY DIAGNOSTIC
women? of coagulation cascade via exposure to tissue factor MACARAIG, MD EXAM - AUG
A. Placenta Previa which leads to a viscious cycle called DIC. (TOP 8 - FEB 2013
B. Vasa Previa 2013 MED
C. Placenta Acreta BOARDS;
D. Abruptio Placenta TOPNOTCH MD)
E. Placenta Increta
651 Vaginal delivery follows a series of fetal movement Descent occurs even before engagement and is LITO JAY DIAGNOSTIC
in relation to uterine contractions, which are called continuous until the fetus is delivered. MACARAIG, MD EXAM - AUG
Cardinal movements. Which cardinal movement is (TOP 8 - FEB 2013
the most important and is the first prerequisite for 2013 MED
vaginal birth? BOARDS;
A. Engagement TOPNOTCH MD)
B. Descent
C. Flexion
D. Extension
E. Internal rotation
652 Which of the following is/are criteria for using The judicial use of Oxytocin should start when the LITO JAY DIAGNOSTIC
Oxytocin? labor is already at the active phase (4cms cervical MACARAIG, MD EXAM - AUG
A. Cervix should atleast 4cms opened dilatation), no CPD and the child is on cephalic (TOP 8 - FEB 2013
B. CPD is ruled out presentation. 2013 MED
C. Cephalic presentation BOARDS;
D. B and C only TOPNOTCH MD)
E. All of the above

TOPNOTCH MEDICAL BOARD PREP OBSTETRICS-GYNECOLOGY SUPEREXAM Page 77 of 84


For inquiries visit www.topnotchboardprep.com.ph or email us at topnotchmedicalboardprep@gmail.com
TOPNOTCH MEDICAL BOARD PREP OBSTETRICS-GYNECOLOGY SUPEREXAM
For inquiries visit www.topnotchboardprep.com.ph or email us at topnotchmedicalboardprep@gmail.com
Ite QUESTION EXPLANATION AUTHOR TOPNOTCH
m # EXAM
653 A 27 year-old nulligravid have undergone explore Dermoid cyst is the most common ovarian tumor in LITO JAY DIAGNOSTIC
laparotomy due to ovarian tumor. Grossly, the patients less than 30 years old. Since it is derived MACARAIG, MD EXAM - AUG
tumor contains teeth, hair and bone. What is your from all germ layers, it can differentiate into any (TOP 8 - FEB 2013
impression? other tissues like bones, teeth, or cartilage. 2013 MED
A. Dermoid cyst BOARDS;
B. Seminoma TOPNOTCH MD)
C. Teratoma
D. A and C only
E. All of the above
654 Which of the following is/are probable sign/s of Probable signs are changes related to placenta or LITO JAY DIAGNOSTIC
pregnancy? uterus MACARAIG, MD EXAM - AUG
A. Positive pregnancy test (TOP 8 - FEB 2013
B. Braxton-Hicks contractions 2013 MED
C. Goodell's sign BOARDS;
D. A and B only TOPNOTCH MD)
E. All of the above
655 Human Placental Lactogen is being pointed-out as HPL is at its peak between 24-28 weeks, and this is LITO JAY DIAGNOSTIC
the cause of gestational diabetes. Its peak levels the ideal time to screen for GDM MACARAIG, MD EXAM - AUG
can be detected on what age of gestation? (TOP 8 - FEB 2013
A. 20-22weeks 2013 MED
B. 22-26weeks BOARDS;
C. 24-28 weeks TOPNOTCH MD)
D. 28-32weeks
E. 30 weeks
656 The minimum criteria for diagnosing Pelvic These are the 3 minimun criteria for diagnosing LITO JAY DIAGNOSTIC
Inflammatory Disease includes PID. MACARAIG, MD EXAM - AUG
A. Cervical tenderness (TOP 8 - FEB 2013
B. Uterine tenderness 2013 MED
C. Adnexal tenderness BOARDS;
D. A and C only TOPNOTCH MD)
E. All of the above
657 Call Exner bodies are the pathognomonic finding LITO JAY DIAGNOSTIC
in? MACARAIG, MD EXAM - AUG
A. Sertoli-Leydig cell tumor (TOP 8 - FEB 2013
B. Granulosa-theca tumor 2013 MED
C. Endodermal Sinus tumor BOARDS;
D. Struma Ovarii TOPNOTCH MD)
E. Choriocarcinoma
658 This is an abnormal uterine bleeding characterized Hypomenorrhea refers to scanty menstruation. LITO JAY DIAGNOSTIC
by scanty menstruation. Oligomenorrhea refers to prolonged intervals of MACARAIG, MD EXAM - AUG
A. Menorrhagia menstruation. (TOP 8 - FEB 2013
B. Hypomenorrhea 2013 MED
C. Oligomenorrhea BOARDS;
D. Polymenorrhea TOPNOTCH MD)
E. Metrorrhagia
659 The most common cause of secondary amenorrhea SPERM is the most common cause of secondary LITO JAY DIAGNOSTIC
is? amenorrhea ! ! ! MACARAIG, MD EXAM - AUG
A. Ovarian tumor (TOP 8 - FEB 2013
B. PCOS 2013 MED
C. Imperforated hymen BOARDS;
D. A or B TOPNOTCH MD)
E. None of the above
660 A 28 year-old primigravid consulted at ER due to Internal exam is a NO NO!! The patient is possibly LITO JAY DIAGNOSTIC
painless vaginal bleeding. She is currently on her having placenta previa and IE can only be done on a MACARAIG, MD EXAM - AUG
32nd week of gestation. The least you would do is? double set-up. You may induce bleeding that is very (TOP 8 - FEB 2013
A. Perform an internal examination hard to control. 2013 MED
B. Do an emergency ultrasound BOARDS;
C. Check the Fetal heart tone TOPNOTCH MD)
D. Non-stress test
E. No excemption. You should do all of the above
661 True of physiologic discharge of puberty? Physiologic discharge is grayish - white in color. HAZEL KAREN MIDTERM 2 -
A. Occurs 6-12 months before menarche RAZ, MD (TOP 6 - AUG 2013
B. Due to desquamation of vaginal epithelium FEB 2013 MED
C. Greenish white in color BOARDS;
D. A and b TOPNOTCH MD)
E. All of the above
662 True of nabothian cyst, except? A nabothian cyst (or nabothian follicle) is a mucus- HAZEL KAREN MIDTERM 2 -
A. Most common cervical cyst filled cyst on the surface of the cervix. They are RAZ, MD (TOP 6 - AUG 2013
B. Most common in postmenopausal women most often caused when stratified squamous FEB 2013 MED
C. Due to intermittent blockage of endicervical epithelium of the ectocervix (toward the vagina) BOARDS;
gland grows over the simple columnar epithelium of the TOPNOTCH MD)
D. Treatment is supportive endocervix (toward the uterus). This tissue growth
E. All of the above can block the cervical crypts (subdermal pockets
usually 2-10 mm in diameter), trapping cervical
mucus inside the crypts. The transformation of
tissue types is called metaplasia. Occurs most
comonly in menstruating women.
663 A 35 year old obese female presented with An endometrial polyp or uterine polyp is a mass in HAZEL KAREN MIDTERM 2 -
bleeding in between periods, what would be the the inner lining of the uterus.They may have a large RAZ, MD (TOP 6 - AUG 2013
initial diagnostic modality of choice? flat base (sessile) or be attached to the uterus by an FEB 2013 MED
A. Ultrasound elongated pedicle (pedunculated). Can be detected BOARDS;
B. Biopsy by vaginal ultrasound (sonohysterography), TOPNOTCH MD)
C. Laparotomy hysteroscopy and dilation and curettage.
D. Progesterone withdrawal test
E. None

TOPNOTCH MEDICAL BOARD PREP OBSTETRICS-GYNECOLOGY SUPEREXAM Page 78 of 84


For inquiries visit www.topnotchboardprep.com.ph or email us at topnotchmedicalboardprep@gmail.com
TOPNOTCH MEDICAL BOARD PREP OBSTETRICS-GYNECOLOGY SUPEREXAM
For inquiries visit www.topnotchboardprep.com.ph or email us at topnotchmedicalboardprep@gmail.com
Ite QUESTION EXPLANATION AUTHOR TOPNOTCH
m # EXAM
664 Triad of polycystic ovary syndrome includes which The principal features are anovulation, resulting in HAZEL KAREN MIDTERM 2 -
of the following? irregular menstruation, amenorrhea, ovulation- RAZ, MD (TOP 6 - AUG 2013
A. Hirsutism related infertility, and polycystic ovaries; excessive FEB 2013 MED
B. Amenorrhea amounts or effects of androgenic hormones, BOARDS;
C. Obesity resulting in acne and hirsutism; and insulin TOPNOTCH MD)
D. Abdominal Pain resistance, often associated with obesity, Type 2
E. A, b and c diabetes, and high cholesterol levels.
665 Normal semenalysis values? Concentration >20M/mL HAZEL KAREN MIDTERM 2 -
A. Concentration >20M/mL Volume 2 - 5 mL RAZ, MD (TOP 6 - AUG 2013
B. pH 8 - 9 pH 7.2 - 7.8 mL FEB 2013 MED
C. Motility>30% motility >50% BOARDS;
D. Normal forms>50% Normal forms > 30%
TOPNOTCH MD)
E. A and b only
666 Grade of pelvic organ prolapse wherein the Grade 1 - descend halfway to hymen HAZEL KAREN MIDTERM 2 -
prolapsed organ is halfway past the hymenal ring? Grade 2 - level of hymen
RAZ, MD (TOP 6 - AUG 2013
A. 1 Grade 3 - halfway past hymenal ring FEB 2013 MED
B. 2 Grade 4 - Prolapsed organ completely outside BOARDS;
C. 3 vagina TOPNOTCH MD)
D. 4
E. 5
667 The Paramesonephric duct if the female genital In males, the paramesonephric or mullerian duct HAZEL KAREN MIDTERM 2 -
tract corresponds to which organ in the males? regress to form the appendix of testes. RAZ, MD (TOP 6 - AUG 2013
A. Prostate gland FEB 2013 MED
B. Testicular appendix BOARDS;
C. Prostatic utricle TOPNOTCH MD)
D. Bulbourethral gland
E. Urinary bladder
668 Derivatives of the Urogenital Sinus in females The phallic part of the definitive SUG enlarges to HAZEL KAREN MIDTERM 2 -
except? become the vaginal vestibule that is caudally closed RAZ, MD (TOP 6 - AUG 2013
A. Lower 3/4 vagina off externally by the urogenital membrane. It then FEB 2013 MED
B. Vestibule tears in the course of the 7th week BOARDS;
C. Bladder TOPNOTCH MD)
D. Urethra
E. Ureter
669 Strongest ligament supporting the uterus? It attaches the cervix to the lateral pelvic wall by its HAZEL KAREN MIDTERM 2 -
A. Mesovarium attachment to the Obturator fascia of the Obturator RAZ, MD (TOP 6 - AUG 2013
B. Cardinal ligament of Mackenrodt internus muscle, and is continuous externally with FEB 2013 MED
C. Utersacral ligament the fibrous tissue that surrounds the pelvic blood BOARDS;
D. Suspensory ligament of the Ovary vessels. It thus provides support to the uterus TOPNOTCH MD)
E. Broad ligament
670 True of pelvic conjugates? OC - measured by substracting 1.5 to 2 cms from HAZEL KAREN MIDTERM 2 -
A. Obstetrical conjugate is the one which can be the diagonal conjugate which is the one that can be RAZ, MD (TOP 6 - AUG 2013
directly measured clinically measured clinically. True anatomic conjugate is the FEB 2013 MED
B. True anatomic conjugate is the shortest distance between between the promontory and the BOARDS;
distance between the midpoint of the promontory UPPER border of the inner surface of syphysis. TOPNOTCH MD)
and inner surface of the symphysis
C. Diagonal conjugate cannot be measured
clinically
D. The obstetrical conjugare is adequate if
distance > 10 cms.
E. all of the above
671 The fetal heart tones can be heard by Doppler by 6 - 7 weeks by Transvaginal UTZ HAZEL KAREN MIDTERM 2 -
how many weeks AOG? 10 weeks by UTZ RAZ, MD (TOP 6 - AUG 2013
A. 6 - 7 19 weeks by Stethoscope FEB 2013 MED
B. 10 BOARDS;
C. 12 TOPNOTCH MD)
D. 19
E. 20
672 This is the earliest histological evidence of At the time of ovulation the endometrium slows in HAZEL KAREN MIDTERM 2 -
progesterone action in the endometrial lining. its growth, and it ceases mitotic activity within days RAZ, MD (TOP 6 - AUG 2013
A. Glandular mitoses after ovulation, at which time the corpus luteum is FEB 2013 MED
B. Stromal edema producing progesterone in addition to estrogen. BOARDS;
C. Basal vacuolization The postovulatory endometrium is initially marked TOPNOTCH MD)
D. Secretion by secretory vacuoles beneath the nuclei in the
E. Pseudostratification of nuclei glandular epithelium . This secretory activity is
most prominent during the third week of the
menstrual cycle, when the basal vacuoles
progressively push past the nuclei.
673 For pregnancy to occur, implantation of what Blastocyst implants at around 7 days post - HAZEL KAREN MIDTERM 2 -
structure must occur on the 8th day of conception within the superior wall of the uterus. RAZ, MD (TOP 6 - AUG 2013
fertilization? FEB 2013 MED
A. zygote BOARDS;
B. blastomere TOPNOTCH MD)
C. morula
D. blastocyst
E. Fetus
674 Goodell sign? Goodell's sign is an indication of pregnancy. It is a HAZEL KAREN MIDTERM 2 -
A. Softening and compressibility of the isthmus significant softening of the vaginal portion of the RAZ, MD (TOP 6 - AUG 2013
occuring on the 6th - 8th week AOG cervix from increased vascularization. This FEB 2013 MED
B. Bluish/purplish discoloration of the vagina at 6 vascularization is a result of hypertrophy and BOARDS;
weeks AOG engorgement of the vessels below the growing TOPNOTCH MD)
C. Mask of pregnancy uterus.
D. Softening of the cervix
E. Ferning pattern

TOPNOTCH MEDICAL BOARD PREP OBSTETRICS-GYNECOLOGY SUPEREXAM Page 79 of 84


For inquiries visit www.topnotchboardprep.com.ph or email us at topnotchmedicalboardprep@gmail.com
TOPNOTCH MEDICAL BOARD PREP OBSTETRICS-GYNECOLOGY SUPEREXAM
For inquiries visit www.topnotchboardprep.com.ph or email us at topnotchmedicalboardprep@gmail.com
Ite QUESTION EXPLANATION AUTHOR TOPNOTCH
m # EXAM
675 Definitive evidence of pregnancy, EXCEPT? Pregnancy test is a probable evidence of pergnancy HAZEL KAREN MIDTERM 2 -
A. Fetal heart action identification RAZ, MD (TOP 6 - AUG 2013
B. Fetal movement perception by examiner FEB 2013 MED
C. Recognition of embryo or fetus by UTZ BOARDS;
D. Positive pregnancy test TOPNOTCH MD)
E. FHT by doppler at 10 weeks AOG
676 Definition of a reactive NST: Reactive NST - presence of two or more fetal heart HAZEL KAREN MIDTERM 2 -
A. 2 or more accelerations rate accelerations within a 20-minute period, with RAZ, MD (TOP 6 - AUG 2013
B. Peak at 15 bpm above baseline or without fetal movement discernible by the FEB 2013 MED
C. Each acceleration lasts more than 30 seconds woman. Accelerations are defined as 15 bpm above BOARDS;
D. A and b baselines for at least 15 seconds if beyond 32 weeks TOPNOTCH MD)
E. All of the above gestation, or 10 bpm for at least 10 seconds if at or
below 32 weeks.
677 Biophysical profile of a pregnant patient was done iHowever, this would be dependent on the AOG. HAZEL KAREN MIDTERM 2 -
which revealed BPS of 8 with decreased AF, According to our lecture, if AOG <37 weeks, repeat RAZ, MD (TOP 6 - AUG 2013
AOG<37 weeks, what is the management of choice? test per protocol. If AOG>37 weeks, deliver. FEB 2013 MED
A. Repeat test per protocol BOARDS;
B. Administer steroids TOPNOTCH MD)
C. Deliver
D. No fetal indication for intervention
E. None
678 Type of deceleration wherein a drop of heart rate Late deceleration may signal uteroplacental HAZEL KAREN MIDTERM 2 -
is seen to start at the peak of contraction but would insufficiency. RAZ, MD (TOP 6 - AUG 2013
return to baseline HR after the contraction ? FEB 2013 MED
A. Early deceleration BOARDS;
B. Head compression TOPNOTCH MD)
C. Late deceleration
D. Variable deceleration
E. Cord compression
679 An 18 year old female came to the ER presenting This is a case of incomplete abortion wherein there HAZEL KAREN MIDTERM 2 -
with vaginal bleeding. She stated to observe is already passage of parts of the fetus. The cervical RAZ, MD (TOP 6 - AUG 2013
passage of meaty material form the vagina. What os is open, bag of waters ruptured. FEB 2013 MED
would be the management of choice for this BOARDS;
patient? TOPNOTCH MD)
A. tocolysis
B. Completion curettage
C. bed rest
D. IV antibiotics
E. prostaglandins
680 type of version used for delivery of the second Podalic version is an obstetric procedure wherein HAZEL KAREN MIDTERM 2 -
twin? the fetus is turned within the womb such that one RAZ, MD (TOP 6 - AUG 2013
A. Internal podalic version or both feet present through FEB 2013 MED
B. External cephalic version the cervix during childbirth. It is used most often in BOARDS;
C. Internal cephalic version cases where the fetus lies transversely or in TOPNOTCH MD)
D. External podalic version another abnormal position in the womb.
E. None
681 A 25 yo, G3P4 comes to your office for a routine Answer: B. Molar pregnancy (pp. 19, High-Yield OB- MICHELLE JAY MIDTERM 1 -
prenatal check-up, complaining of vaginal bleeding. Gyne, 2nd edition) FRANCISCO, MD AUG 2013
Patient is 16 weeks AOG based on LMP. Upon PE, Notes: Molar pregnancy is the only time (TOP 9 - FEB
her uterus is at the level of the umbilicus and no preeclampsia is diagnosed prior to 20 weeks. 2013 MED
FHTs can be heard. BP is 150/90 mmHg with no * note that the uterine size is larger than the BOARDS;
prior history of hypertension. Urinalysis showed gestational age. TOPNOTCH MD)
2+ proteinuria on dipstick. Which of the following *SIMILAR TO PREVIOUS BOARD EXAM
condition should you rule out first? CONCEPT/PRINCIPLE
A. IUFD
B. Molar pregnancy
C. Preeclampsia
D. Gestational hypertension
682 What is the normal position of the uterus? Answer: A. Anteverted and anteflexed (must MICHELLE JAY MIDTERM 1 -
A. Anteverted and anteflexed know) FRANCISCO, MD AUG 2013
B. Anteverted and retroflexed (TOP 9 - FEB
C. Retroverted and anteflexed 2013 MED
D. Retroverted and retroflexed BOARDS;
TOPNOTCH MD)
683 What provides the major support of the uterus and Answer: D. Mackenrodt ligament (pp. 4, OB-Gyne, MICHELLE JAY MIDTERM 1 -
the cervix? Topnotch handouts) FRANCISCO, MD AUG 2013
A. Round ligament Notes: Cardinal ligament synonyms – transverse (TOP 9 - FEB
B. Uterosacral ligament cervical ligament or Mackenrodt ligament 2013 MED
C. Transverse sacral ligament *There is no such thing as transverse sacral BOARDS;
D. Mackenrodt ligament ligament. TOPNOTCH MD)
684 Proper recording and evaluation of the periodic Answer: D. Uteroplacental insufficiency (pp. 21, MICHELLE JAY MIDTERM 1 -
changes of the fetal heart rate (FHR) are needed OB-Gyne, Topnotch Handouts) FRANCISCO, MD AUG 2013
for proper intrapartum assessment. What is the * Kinds of deceleration were asked 4-5x during the (TOP 9 - FEB
probable etiology if there is gradual decrease in the Feb 2013 Boards. 2013 MED
FHR below the baseline with onset to nadir of at BOARDS;
least >30 secs? TOPNOTCH MD)
A. Normal
B. Head compression
C. Umbilical cord compression
D. Uteroplacental insufficiency
685 Which sequence of cardinal movements of labor Answer : B. Descent, Internal rotation, Flexion, MICHELLE JAY MIDTERM 1 -
best applies to a fetus delivered via face Restitution (pp. 27, OB-Gyne, Topnotch Handouts) FRANCISCO, MD AUG 2013
presentation? (TOP 9 - FEB
A. Descent , Engagement, Internal Rotation, Lateral 2013 MED
Flexion BOARDS;
B. Descent, Internal rotation, Flexion, Restitution TOPNOTCH MD)
TOPNOTCH MEDICAL BOARD PREP OBSTETRICS-GYNECOLOGY SUPEREXAM Page 80 of 84
For inquiries visit www.topnotchboardprep.com.ph or email us at topnotchmedicalboardprep@gmail.com
TOPNOTCH MEDICAL BOARD PREP OBSTETRICS-GYNECOLOGY SUPEREXAM
For inquiries visit www.topnotchboardprep.com.ph or email us at topnotchmedicalboardprep@gmail.com
Ite QUESTION EXPLANATION AUTHOR TOPNOTCH
m # EXAM
C. Engagement, Descent, Flexion, Internal Rotation
D. Engagement, Descent, Extension, External
Rotation
686 How many weeks post-partum does the uterus Answer: D. 4 weeks (pp.32, OB-Gyne, Topnotch MICHELLE JAY MIDTERM 1 -
regresses or involutes to its non-pregnant size? Handouts) FRANCISCO, MD AUG 2013
A. 2-3 days SIMILAR TO PREVIOUS BOARD EXAM (TOP 9 - FEB
B. 2 weeks CONCEPT/PRINCIPLE 2013 MED
C. 3 weeks BOARDS;
D. 4 weeks TOPNOTCH MD)
687 Benign GTD can be classified as complete and Answer: Incomplete means one haploid egg is MICHELLE JAY MIDTERM 1 -
incomplete mole. An incomplete mole has 3 sets of fertilized by 2 haploid eggs, so the outcome is FRANCISCO, MD AUG 2013
chromosomes due to which of the following diploid (TOP 9 - FEB
reasons? 2013 MED
A. A haploid egg is fertilized by 2 normal sperms BOARDS;
B. A haploid egg is fertilized by a diploid sperm TOPNOTCH MD)
C. A diploid egg is fertilized by a haploid sperm
D. All of these
688 What is the most frequent site of eccyesis? Answer: A. Ampulla (pp.35, OB-Gyne, Topnotch MICHELLE JAY MIDTERM 1 -
A. Ampulla Handouts) FRANCISCO, MD AUG 2013
B. Isthmus Notes: Eccyesis is another term for ectopic (TOP 9 - FEB
C. Interstitial pregnancy. 2013 MED
D. Supero-posterior part of uterus BOARDS;
TOPNOTCH MD)
689 A 26 yo, G1P1 (1001) complains of amenorrhea. Answer: C. Sheehan’s syndrome MICHELLE JAY MIDTERM 1 -
She had hypotensive episodes during her delivery Notes: Sheehan’s syndrome – destruction f pituitary FRANCISCO, MD AUG 2013
7 months ago and was transfused 10 units of fresh gland as a result of severe bleeding. (TOP 9 - FEB
whole blood. She was not able to breastfeed 2013 MED
because her breasts started to sag. She also lost BOARDS;
weight, became anorexic and weak. What explains TOPNOTCH MD)
the amenorrhea in this patient?
A. Simmond’s syndrome
B. Lactational amenorrhea
C. Sheehan’s syndrome
D. Anorexia nervosa
690 A 4 yo girl was brought to the clinic for slight Answer: B. A benign self-limiting condition that MICHELLE JAY MIDTERM 1 -
enlargement of the left breast with no other does not require treatment FRANCISCO, MD AUG 2013
associated signs and symptoms since two years (TOP 9 - FEB
ago. You will tell the parents that premature 2013 MED
thelarche is: BOARDS;
A. A condition that needs surgical removal of the TOPNOTCH MD)
ovaries
B. A benign self-limiting condition that does not
require treatment
C. A serious condition leading to central precocious
puberty
D. A condition that requires treatment with
hypothalamic suppressive therapy
691 A 40 yr-old G4P3 woman comes to the E.R. Answer: E MICHELLE JAY MIDTERM 1 -
complaining of vaginal bleeding, pelvic pain, flank Differentials include PID, miscarriage, DUB, cervical FRANCISCO, MD AUG 2013
pain, foul-smelling discharge and disorientation. lesions (including cervical CA), UTI leading to (TOP 9 - FEB
Her past medical history is significant for 3 NSVDs pyelonephritis. CBC & complete metabolic panel are 2013 MED
and 1 miscarriage. In addition, she did have a impt.because patient already has disorientation. BOARDS;
history of abnormal Papsmears approx.3 years TOPNOTCH MD)
earlier. What initial lab work-up must be done?
A. Complete metabolic panel
B. Complete blood count
C. B-BHCG levels
D. Urinalysis
E. All of the above
692 Potential sites for ureteral injury during abdominal Answer: A MICHELLE JAY MIDTERM 1 -
hysterectomy with bilateral salpigo-oophorectomy In general, ureters do not travel near the round FRANCISCO, MD AUG 2013
include all of the following except: ligament, in a pelvis with normal anatomy. (TOP 9 - FEB
A. Transaction of the round ligament 2013 MED
B. Transaction of uterine arteries BOARDS;
C. Transaction of cardinal ligaments TOPNOTCH MD)
D. Transition of infundibulopelivc ligaments
E. None
693 A 60 yr-old womans visits your clinic with Answer: B MICHELLE JAY MIDTERM 1 -
complaints of pelvic pressure and abdominal Histology suggests the CA is confined to the ovary FRANCISCO, MD AUG 2013
fullness. Her LMP was 3 yrs ago. BPE and IVP are but the 5-yr survival rate is only 80%. (TOP 9 - FEB
normal. Staging explore lap is performed. You fin 2013 MED
stage 1a ovarian CA. What is the best intervention BOARDS;
in this case? TOPNOTCH MD)
A. Start patient on chemotherapy immediately
B. Perform TAH-BSO only
C. Perform TAH-BSO and infracolic omentectomy
D. Cytoreductive debulking will suffice
694 A 19 yr-old primigravida at 32 weeks’ gestation Answer: E MICHELLE JAY MIDTERM 1 -
comes to the office for a routine prenatal visit. Her Antidote for magnesium toxicity is IV calcium FRANCISCO, MD AUG 2013
BP is 150/95 mmHg. Her previous BPs have been gluconate. (TOP 9 - FEB
120/7- mmHg range. 2 hours later: While receiving 2013 MED
IV MgSO4 therapy, her RR have decreased from 20- BOARDS;
5 rpm. Findings are consistent with TOPNOTCH MD)
A. Gestational HTN
B. Mild preeclampsia
C. Severe preeclampsia
TOPNOTCH MEDICAL BOARD PREP OBSTETRICS-GYNECOLOGY SUPEREXAM Page 81 of 84
For inquiries visit www.topnotchboardprep.com.ph or email us at topnotchmedicalboardprep@gmail.com
TOPNOTCH MEDICAL BOARD PREP OBSTETRICS-GYNECOLOGY SUPEREXAM
For inquiries visit www.topnotchboardprep.com.ph or email us at topnotchmedicalboardprep@gmail.com
Ite QUESTION EXPLANATION AUTHOR TOPNOTCH
m # EXAM
D. Eclampsia
E. Magnesium toxicity

695 A primigravida at 16 weeks by LMP has a fundal Answer: A MICHELLE JAY MIDTERM 1 -
height at umbilicus. She has abnormality elevated The combination of fundus larger than dates and FRANCISCO, MD AUG 2013
levels of MS-AFP and B-hCG. You: abnormally elevated levels of MS-AFP and B-hCG (TOP 9 - FEB
A. schedule a sonogram to rule out multiple suggests multiple gestation. 2013 MED
gestation BOARDS;
B. are confident it’s Down’s syndrome TOPNOTCH MD)
C. diagnose patient as having molar pregnancy
D. admit patient and watch out for variable
decelerations
696 A 43 yr-old woman comes to the office complaining Answer: C MICHELLE JAY MIDTERM 1 -
of involuntary urine loss. Loss of urine occurs Overflow incontinence occurs uniquely when FRANCISCO, MD AUG 2013
continuously day and night along with pelvic intravesical pressure from an overdistended (TOP 9 - FEB
pressure. Residual volume is 450 ml. hypotonic bladder exceeds urethral pressure. 2013 MED
a. Genuine incontinence BOARDS;
b. Bypass incontinence TOPNOTCH MD)
c. Overflow incontinence
d. Motor urge incontinence
697 How many weeks after abortion does ovulation MICHELLE JAY MIDTERM 1 -
usually occur? FRANCISCO, MD AUG 2013
A. 2 to 3 weeks (TOP 9 - FEB
B. 4 to 5 2013 MED
C. 5 to 6 BOARDS;
D. 6 to 7 TOPNOTCH MD)
698 Duration of pregnancy is most correctly measured MICHELLE JAY MIDTERM 1 -
clinically by which of the following units? FRANCISCO, MD AUG 2013
A. Number of weeks, rounded to the nearest whole (TOP 9 - FEB
week since the first day of LMP 2013 MED
B. Completed weeks since first day of LMP BOARDS;
C. Completed weeks since estimated date of TOPNOTCH MD)
conception
D. Numbers of weeks rounded to the nearest whole
week since the estimated date of conception
699 In which presentation is the fetal head partially MICHELLE JAY MIDTERM 1 -
flexed and a large anterior fontanel presenting? FRANCISCO, MD AUG 2013
A. Vertex (TOP 9 - FEB
B. Face 2013 MED
C. Brow BOARDS;
D. Sinciput TOPNOTCH MD)
700 Which of the following is not an indication of Answer: C. Creatinine > 1.2 mg/dl. MICHELLE JAY MIDTERM 1 -
severe pregnancy-induced hypertension? FRANCISCO, MD AUG 2013
A. Upper abdominal pain (TOP 9 - FEB
B. Oliguria 2013 MED
C. Creatinine 0.6 mg/dl BOARDS;
D. Fetal growth restriction TOPNOTCH MD)

TOPNOTCH MEDICAL BOARD PREP OBSTETRICS-GYNECOLOGY SUPEREXAM Page 82 of 84


For inquiries visit www.topnotchboardprep.com.ph or email us at topnotchmedicalboardprep@gmail.com
TOPNOTCH MEDICAL BOARD PREP OBSTETRICS-GYNECOLOGY SUPEREXAM
For inquiries visit www.topnotchboardprep.com.ph or email us at topnotchmedicalboardprep@gmail.com

Item # KEY 87 C 175 A 263 B 351 B


ANSWER 88 A 176 D 264 A 352 D
1 A 89 B 177 C 265 C 353 C
2 C 90 C 178 A 266 A 354 C
3 E 91 B 179 A 267 B 355 C
4 C 92 D 180 D 268 E 356 C
5 E 93 D 181 C 269 D 357 C
6 D 94 D 182 A 270 C 358 B
7 E 95 C 183 B 271 C 359 B
8 D 96 D 184 A 272 A 360 D
9 E 97 E 185 B 273 B 361 B
10 C 98 B 186 C 274 B 362 C
11 A 99 C 187 E 275 C 363 D
12 E 100 A 188 C 276 A 364 A
13 B 101 B 189 C 277 E 365 C
14 C 102 D 190 D 278 A 366 D
15 B 103 E 191 B 279 D 367 C
16 B 104 C 192 B 280 D 368 A
17 E 105 D 193 B 281 D 369 C
18 D 106 B 194 E 282 E 370 B
19 B 107 A 195 A 283 C 371 A
20 D 108 B 196 B 284 B 372 D
21 C 109 D 197 D 285 B 373 A
22 A 110 E 198 C 286 D 374 B
23 C 111 C 199 A 287 A 375 A
24 B 112 D 200 B 288 B 376 C
25 B 113 E 201 B 289 D 377 D
26 B 114 A 202 C 290 B 378 E
27 A 115 C 203 B 291 D 379 B
28 B 116 C 204 D 292 D 380 B
29 C 117 A 205 B 293 D 381 B
30 B 118 D 206 C 294 B 382 A
31 D 119 B 207 E 295 D 383 B
32 A 120 B 208 A 296 B 384 A
33 C 121 C 209 C 297 C 385 D
34 E 122 A 210 B 298 D 386 B
35 E 123 E 211 D 299 C 387 B
36 B 124 A 212 A 300 B 388 C
37 C 125 A 213 C 301 C 389 D
38 E 126 D 214 D 302 E 390 C
39 E 127 D 215 D 303 D 391 D
40 B 128 E 216 B 304 C 392 B
41 B 129 C 217 D 305 A 393 A
42 A 130 C 218 C 306 B 394 D
43 C 131 D 219 E 307 D 395 B
44 E 132 D 220 C 308 C 396 B
45 E 133 D 221 B 309 C 397 C
46 B 134 B 222 C 310 B 398 C
47 E 135 A 223 C 311 B 399 D
48 A 136 B 224 B 312 B 400 D
49 C 137 B 225 A 313 A 401 C
50 A 138 B 226 B 314 A 402 B
51 C 139 A 227 C 315 A 403 E
52 C 140 C 228 B 316 A 404 E
53 D 141 D 229 C 317 C 405 A
54 A 142 A 230 A 318 B 406 D
55 C 143 B 231 A 319 B 407 B
56 B 144 B 232 D 320 A 408 A
57 D 145 D 233 C 321 C 409 A
58 A 146 D 234 C 322 D 410 C
59 A 147 A 235 B 323 E 411 B
60 B 148 E 236 B 324 A 412 D
61 D 149 D 237 A 325 C 413 D
62 D 150 B 238 B 326 E 414 D
63 C 151 A 239 B 327 B 415 E
64 D 152 D 240 D 328 A 416 E
65 A 153 B 241 B 329 B 417 B
66 E 154 D 242 B 330 A 418 B
67 A 155 A 243 B 331 D 419 C
68 C 156 B 244 D 332 E 420 E
69 B 157 A or C 245 C 333 C 421 D
70 C 158 A 246 A 334 D 422 D
71 A 159 B 247 B 335 B 423 E
72 B 160 B 248 D 336 B 424 A
73 D 161 D 249 A 337 C 425 A
74 A 162 C 250 A 338 D 426 C
75 D 163 A 251 B 339 C 427 B
76 C 164 B 252 B 340 C 428 C
77 A 165 C 253 C 341 A 429 B
78 B 166 E 254 D 342 D 430 D
79 C 167 A 255 D 343 C 431 C
80 D 168 B 256 D 344 B 432 D
81 B 169 E 257 A 345 C 433 C
82 A 170 A 258 B 346 B 434 D
83 B 171 D 259 A 347 A 435 B
84 D 172 E 260 D 348 C 436 C
85 C 173 B, C 261 D 349 C 437 B
86 D 174 B 262 E 350 D 438 C
TOPNOTCH MEDICAL BOARD PREP OBSTETRICS-GYNECOLOGY SUPEREXAM Page 83 of 84
For inquiries visit www.topnotchboardprep.com.ph or email us at topnotchmedicalboardprep@gmail.com
TOPNOTCH MEDICAL BOARD PREP OBSTETRICS-GYNECOLOGY SUPEREXAM
For inquiries visit www.topnotchboardprep.com.ph or email us at topnotchmedicalboardprep@gmail.com
439 A 528 C 617 E
440 C 529 A 618 A
441 C 530 D 619 D
442 531 C 620 A
443 B 532 A 621 C
444 D 533 B 622 C
445 B 534 A 623 D
446 C 535 D 624 C
447 E 536 A 625 D
448 B 537 A 626 E
449 D 538 A 627 A
450 B 539 C 628 D
451 C 540 B 629 D
452 B 541 A 630 D
453 B 542 C 631 D
454 D 543 B 632 D
455 C 544 E 633 A
456 D 545 B 634 B
457 A 546 D 635 B
458 C 547 C 636 B
459 B 548 A 637 C
460 C 549 B 638 A
461 D 550 D 639 C
462 C 551 D 640 A
463 A 552 C 641 D
464 A 553 D 642 C
465 C 554 A 643 A
466 E 555 A 644 D
467 B 556 C 645 E
468 A 557 B 646 D
469 C 558 D 647 A
470 B 559 D 648 A
471 C 560 B 649 E
472 A 561 A 650 D
473 D 562 C 651 B
474 C 563 C 652 E
475 A 564 A 653 D
476 D 565 E 654 E
477 A 566 B 655 C
478 B 567 C 656 E
479 D 568 C 657 B
480 B 569 B 658 B
481 B 570 B 659 E
482 A 571 A 660 A
483 C 572 A 661 D
484 C 573 C 662 B
485 B 574 A 663 A
486 B 575 C 664 E
487 C 576 C 665 A
488 B 577 D 666 C
489 A 578 A 667 B
490 B 579 D 668 E
491 C 580 D 669 B
492 D 581 C 670 D
493 A 582 E 671 B
494 D 583 B 672 C
495 B 584 E 673 D
496 A 585 B 674 D
497 A 586 D 675 D
498 C 587 A 676 D
499 D 588 B 677 A
500 A 589 B 678 C
501 D 590 C 679 B
502 C 591 D 680 A
503 E 592 A 681 B
504 E 593 E 682 A
505 D 594 A 683 D
506 C 595 A 684 D
507 B 596 D 685 B
508 B 597 E 686 D
509 D 598 C 687 A
510 B 599 C 688 A
511 B 600 D 689 C
512 D 601 A 690 B
513 A 602 B 691 E
514 A 603 D 692 A
515 B 604 D 693 B
516 D 605 B 694 E
517 B 606 D 695 A
518 B 607 C 696 C
519 C 608 A 697 A
520 D 609 D 698 B
521 A 610 D 699 D
522 B 611 B 700 C
523 C 612 E
524 B 613 A
525 A 614 D
526 C 615 E
527 B 616 B
TOPNOTCH MEDICAL BOARD PREP OBSTETRICS-GYNECOLOGY SUPEREXAM Page 84 of 84
For inquiries visit www.topnotchboardprep.com.ph or email us at topnotchmedicalboardprep@gmail.com

Você também pode gostar